217
8/13/2019 Dãy s - Gi i h n Tác gi : Tr n Nam Dũng- Nguy n Văn M u, 2007 http://slidepdf.com/reader/full/day-so-gioi-han-tac-gia-tran-nam-dung-nguyen-van-mau 1/217 Mc lc 1 Dãy s các bài toán v dãy s 4 1.1 Gii thiu . . . . . . . . . . . . . . . . . . . . . . . . . . . . . . . . 4 1.2 Đnh nghĩa và các đnh lý cơ bn . . . . . . . . . . . . . . . . . . . 5 1.3 Mt s phương pháp gii bài toán v dãy s . . . . . . . . . . . . . 8 1.3.1 Dãy s thc: mt s dng dãy s đc bit . . . . . . . . . . 8 1.3.2 Dãy s nguyên . . . . . . . . . . . . . . . . . . . . . . . . . 12 1.3.3 Dãy s và phương trình . . . . . . . . . . . . . . . . . . . . 17 1.3.4 Mt vài th thut khác . . . . . . . . . . . . . . . . . . . . 18 1.4 Mt s phương pháp xây dng h thng bài tp . . . . . . . . . . . 23 1.4.1 Xây dng dãy hi t bng phương trình . . . . . . . . . . . 23 1.4.2 Xây dng dãy truy hi t cp nghim ca phương trình bc 2 24 1.4.3 Xây dng các dãy s nguyên t li gii các phương trình nghim nguyên . . . . . . . . . . . . . . . . . . . . . . . . . 25 1.4.4 Xây dng dãy s là nghim ca mt h phương trình ph thuc bin  n  . . . . . . . . . . . . . . . . . . . . . . . . . . 26 1.5 Lý thuyt dãy s dưi con mt toán cao cp . . . . . . . . . . . . . 27 1.5.1 Ri rc hóa các khái nim và đnh lý ca lý thuyt hàm bin s thc . . . . . . . . . . . . . . . . . . . . . . . . . . . 27 1.5.2 Phương pháp hàm sinh và bài toán tìm s hng tng quát . 29 1.5.3 Đi s tuyn tính và phương trình sai phân . . . . . . . . . 30 1.5.4 S dng xp x trong d đoán kt qu . . . . . . . . . . . . 31 1.6 Bài tp . . . . . . . . . . . . . . . . . . . . . . . . . . . . . . . . . 32 2 Phương trình sai phân 41 2.1 Sai phân . . . . . . . . . . . . . . . . . . . . . . . . . . . . . . . . . 41 2.1.1 Đnh nghĩa . . . . . . . . . . . . . . . . . . . . . . . . . . . 41 2.1.2 Tính cht . . . . . . . . . . . . . . . . . . . . . . . . . . . . 41 2.2 Phương trình sai phân tuyn tính . . . . . . . . . . . . . . . . . . . 43 2.2.1 Mt s khái nim chung v phương trình sai phân . . . . . 43 2.3 Phương trình sai phân tuyn tính bc nht . . . . . . . . . . . . . 44 1

Dãy số - Giới hạn Tác giả: Trần Nam Dũng- Nguyễn Văn Mậu, 2007

Embed Size (px)

Citation preview

Page 1: Dãy số - Giới hạn Tác giả: Trần Nam Dũng- Nguyễn Văn Mậu, 2007

8/13/2019 Dãy số - Giớ i hạn Tác giả: Trần Nam Dũng- Nguyễn Văn Mậu, 2007

http://slidepdf.com/reader/full/day-so-gioi-han-tac-gia-tran-nam-dung-nguyen-van-mau 1/217

Mc lc

1 Dãy s và các bài toán v dãy s 41.1 Gii thiu . . . . . . . . . . . . . . . . . . . . . . . . . . . . . . . . 41.2 Đnh nghĩa và các đnh lý cơ bn . . . . . . . . . . . . . . . . . . . 51.3 Mt s phương pháp gii bài toán v dãy s . . . . . . . . . . . . . 8

1.3.1 Dãy s thc: mt s dng dãy s đc bit . . . . . . . . . . 81.3.2 Dãy s nguyên . . . . . . . . . . . . . . . . . . . . . . . . . 121.3.3 Dãy s và phương trình . . . . . . . . . . . . . . . . . . . . 171.3.4 Mt vài th thut khác . . . . . . . . . . . . . . . . . . . . 18

1.4 Mt s phương pháp xây dng h thng bài tp . . . . . . . . . . . 231.4.1 Xây dng dãy hi t bng phương trình . . . . . . . . . . . 231.4.2 Xây dng dãy truy hi t cp nghim ca phương trình bc 2 241.4.3 Xây dng các dãy s nguyên t li gii các phương trình

nghim nguyên . . . . . . . . . . . . . . . . . . . . . . . . . 251.4.4 Xây dng dãy s là nghim ca mt h phương trình ph

thuc bin  n   . . . . . . . . . . . . . . . . . . . . . . . . . . 261.5 Lý thuyt dãy s dưi con mt toán cao cp . . . . . . . . . . . . . 27

1.5.1 Ri rc hóa các khái nim và đnh lý ca lý thuyt hàmbin s thc . . . . . . . . . . . . . . . . . . . . . . . . . . . 27

1.5.2 Phương pháp hàm sinh và bài toán tìm s hng tng quát . 291.5.3 Đi s tuyn tính và phương trình sai phân . . . . . . . . . 301.5.4 S dng xp x trong d đoán kt qu . . . . . . . . . . . . 31

1.6 Bài tp . . . . . . . . . . . . . . . . . . . . . . . . . . . . . . . . . 32

2 Phương trình sai phân 412.1 Sai phân . . . . . . . . . . . . . . . . . . . . . . . . . . . . . . . . . 41

2.1.1 Đnh nghĩa . . . . . . . . . . . . . . . . . . . . . . . . . . . 412.1.2 Tính cht . . . . . . . . . . . . . . . . . . . . . . . . . . . . 412.2 Phương trình sai phân tuyn tính . . . . . . . . . . . . . . . . . . . 43

2.2.1 Mt s khái nim chung v phương trình sai phân . . . . . 432.3 Phương trình sai phân tuyn tính bc nht . . . . . . . . . . . . . 44

1

Page 2: Dãy số - Giới hạn Tác giả: Trần Nam Dũng- Nguyễn Văn Mậu, 2007

8/13/2019 Dãy số - Giớ i hạn Tác giả: Trần Nam Dũng- Nguyễn Văn Mậu, 2007

http://slidepdf.com/reader/full/day-so-gioi-han-tac-gia-tran-nam-dung-nguyen-van-mau 2/217

MC LC    2

2.3.1 Đnh nghĩa . . . . . . . . . . . . . . . . . . . . . . . . . . . 442.3.2 Phương pháp gii . . . . . . . . . . . . . . . . . . . . . . . . 44

2.3.3 Phương pháp tìm nghim riêng ca phương trình sai phântuyn tính cp 1 không thun nht khi v phi   f (n)   códng đc bit . . . . . . . . . . . . . . . . . . . . . . . . . . 45

2.3.4 Bài tp . . . . . . . . . . . . . . . . . . . . . . . . . . . . . 472.4 Phương trình sai phân tuyn tính cp 2 . . . . . . . . . . . . . . . 47

2.4.1 Đnh nghĩa . . . . . . . . . . . . . . . . . . . . . . . . . . . 472.4.2 Cách gii . . . . . . . . . . . . . . . . . . . . . . . . . . . . 48

2.5 Phương trình sai phân tuyn tính cp 3 . . . . . . . . . . . . . . . 552.5.1 Đnh nghĩa . . . . . . . . . . . . . . . . . . . . . . . . . . . 552.5.2 Phương pháp gii . . . . . . . . . . . . . . . . . . . . . . . . 552.5.3 Ví d . . . . . . . . . . . . . . . . . . . . . . . . . . . . . . 56

2.5.4 Phương trình sai phân tuyn tính cp k   . . . . . . . . . . . 583 Xác đnh s hng tng quát ca mt dãy s 60

3.1 Tìm s hng tng quát ca dãy (dng đa thc) khi bit các shng đu tiên . . . . . . . . . . . . . . . . . . . . . . . . . . . . . . 61

3.2 Công thc truy hi là mt biu thc tuyn tính . . . . . . . . . . . 633.2.1 Ví d . . . . . . . . . . . . . . . . . . . . . . . . . . . . . . 64

3.3 Công thc truy hi là mt h biu thc tuyn tính . . . . . . . . . 703.3.1 Ví d . . . . . . . . . . . . . . . . . . . . . . . . . . . . . . 70

3.4 Công thc truy hi là biu thc tuyn tính vi h s bin thiên . . 723.5 Công thc truy hi dng phân tuyn tính vi h s hng . . . . . . 783.6 H thc truy hi phi tuyn . . . . . . . . . . . . . . . . . . . . . . 81

3.6.1 Quy trình tuyn tính hoá mt phương trình sai phân . . . . 823.6.2 Ví d . . . . . . . . . . . . . . . . . . . . . . . . . . . . . . 833.6.3 Mt s ví d khác . . . . . . . . . . . . . . . . . . . . . . . 873.6.4 Bài tp. . . . . . . . . . . . . . . . . . . . . . . . . . . . . . 96

4 Phương trình hàm sai phân bc hai 994.1 Hàm tun hoàn và phn tun hoàn cng tính . . . . . . . . . . . . 994.2 Phương trình hàm sai phân bc hai vi hàm tun hoàn và phn

tun hoàn . . . . . . . . . . . . . . . . . . . . . . . . . . . . . . . . 1004.3 Phương trình vi hàm s tun hoàn, phn tun hoàn nhân tính . . 108

4.3.1 Đnh nghĩa . . . . . . . . . . . . . . . . . . . . . . . . . . . 109

4.3.2 Mt s bài toán . . . . . . . . . . . . . . . . . . . . . . . . . 1094.3.3 Mt s ví d áp dng . . . . . . . . . . . . . . . . . . . . . 125

Page 3: Dãy số - Giới hạn Tác giả: Trần Nam Dũng- Nguyễn Văn Mậu, 2007

8/13/2019 Dãy số - Giớ i hạn Tác giả: Trần Nam Dũng- Nguyễn Văn Mậu, 2007

http://slidepdf.com/reader/full/day-so-gioi-han-tac-gia-tran-nam-dung-nguyen-van-mau 3/217

MC LC    3

5 Dãy s sinh bi hàm s 1285.1 Hàm s chuyn đi phép tính s hc và đi s . . . . . . . . . . . . 128

5.2 V các dãy s xác đnh bi dãy các phương trình . . . . . . . . . . 1355.3 Đnh lý v ba mnh đ tương đương . . . . . . . . . . . . . . . . . 1415.4 Mt s bài toán v ưc lưng tng và tích . . . . . . . . . . . . . . 1425.5 Bài tp . . . . . . . . . . . . . . . . . . . . . . . . . . . . . . . . . 144

6 Mt s lp hàm chuyn đi các cp s 1456.1 Cp s cng, cp s nhân và cp s điu hoà . . . . . . . . . . . . 1456.2 Dãy s tun hoàn . . . . . . . . . . . . . . . . . . . . . . . . . . . . 1466.3 Hàm s chuyn đi cp s cng . . . . . . . . . . . . . . . . . . . . 1526.4 Hàm s chuyn đi cp s cng vào cp s nhân . . . . . . . . . . . 1546.5 Hàm s chuyn đi cp s nhân vào cp s cng . . . . . . . . . . . 1556.6 Hàm s chuyn đi cp s nhân vào cp s điu hoà . . . . . . . . 156

7 Mt s lp hàm chuyn đi các cp s trong tp ri rc 1587.1 Hàm s chuyn đi cp s cng thành cp s cng . . . . . . . . . 1587.2 Hàm s chuyn đi cp s nhân thành cp s nhân . . . . . . . . . 161

8 Mt s bài toán xác đnh dãy s trong lp dãy tun hoàn cngtính và nhân tính. 1678.1 Mt s bài toán xác đnh dãy s trong lp dãy tun hoàn cng tính1678.2 Hàm s xác đnh trên tp các s nguyên . . . . . . . . . . . . . . . 170

8.2.1 Hàm s chuyn đi các phép tính s hc . . . . . . . . . . 1708.2.2 Hàm s chuyn tip các đi lưng trung bình . . . . . . . . 172

8.2.3 Phương trình trong hàm s vi cp bin t do . . . . . . . 1778.2.4 Mt s dng toán liên quan đn dãy truy hi . . . . . . . . 1808.3 Hàm s xác đnh trên tp các s hu t . . . . . . . . . . . . . . . 1848.4 Phương trình trong hàm s vi cp bin t do . . . . . . . . . . . . 1918.5 S dng gii hn đ gii phương trình hàm . . . . . . . . . . . . . 198Tài liu tham kho . . . . . . . . . . . . . . . . . . . . . . . . . . . . . 217

Page 4: Dãy số - Giới hạn Tác giả: Trần Nam Dũng- Nguyễn Văn Mậu, 2007

8/13/2019 Dãy số - Giớ i hạn Tác giả: Trần Nam Dũng- Nguyễn Văn Mậu, 2007

http://slidepdf.com/reader/full/day-so-gioi-han-tac-gia-tran-nam-dung-nguyen-van-mau 4/217

Chương 1

Dãy s và các bài toán v dãys

1.1 Gii thiu

Chn đ tài v dãy s, chúng tôi đã t trưc mình mt nhim v vô cùng khókhăn, bi đây là mt lĩnh vc rt khó và rt rng, s dng nhiu kin thc khácnhau ca toán hc. Hơn th, trưc đó đã có khá nhiu cun sách chuyên kho vđ tài này. Dù vy, chúng tôi vn mun c gng đóng góp mt s kinh nghim vàghi nhn ca mình thu lưm đưc trong quá trình ging dy nhng năm qua.

Tp tài liu này không phi là mt giáo trình v dãy s, li càng không philà mt cm nang hưng dn gii các bài toán dãy s. Tp tài liu này đúng hơn

ht là nhng cóp nht ca tác gi v nhng phương pháp gii các bài toán dãys cùng vi nhng nhn đnh đôi khi mang đy tính ch quan ca tác gi. Vì vy,hãy coi đây là mt tài liu m. Hãy tip tc trin khai, liên h và đúc kt kinhnghim, ghi nhn nhng cái hay và góp ý cho nhng cái chưa hay, thm chí chưachính xác.

Trong tài liu này, không phi tt c các vn đ ca dãy s đu đưc đ cpti. Ví d phn dãy s và bt đng thc ch đưc nói đn rt sơ sài, các bài toándãy s mà thc cht là các bài toán v đng dư cũng không đưc xét ti... Haimng ln mà tp tài liu này chú ý đn nht là bài toán tìm s hng tng quátca mt dãy s và bài toán tìm gii hn dãy s.

Trong tp tài liu này, các vn đ và các bài toán có mc đ khó d khác

nhau. Có nhng bài cơ bn, có nhng bài khó hơn và có nhng bài rt khó. Vìvy, cn phi la chn vn đ vi mc đ thích hp (ví d có mt s vn đ vàbài toán ch đng phi mc kỳ thi chn đi tuyn hoc quc t).

Vit tp tài liu này, tác gi đã s dng rt nhiu ngun tài liu khác nhau,tuy nhiên ch có mt s bài có ghi ngun gc, mt s bài không th xác đnh đưc.

4

Page 5: Dãy số - Giới hạn Tác giả: Trần Nam Dũng- Nguyễn Văn Mậu, 2007

8/13/2019 Dãy số - Giớ i hạn Tác giả: Trần Nam Dũng- Nguyễn Văn Mậu, 2007

http://slidepdf.com/reader/full/day-so-gioi-han-tac-gia-tran-nam-dung-nguyen-van-mau 5/217

1.2. Đnh nghĩa và các đnh lý cơ bn   5

Tác gi cũng đã s dng các bài ging ca các thy Phan Đc Chính, NguynVăn Mu, Lê Đình Thnh, Đng Hùng Thng, Nguyn Minh Đc... trong bài vit

ca mình.Cui cùng, tp tài liu này không khi có nhng nhm ln và thiu sót, tácgi rt mong nhn đưc s góp ý ca tt c các thy cô giáo. Và rt mong rng,vi n lc chung ca tt c chúng ta, tp tài liu s tip tc đưc hoàn thin vàb sung.

1.2 Đnh nghĩa và các đnh lý cơ bn

Đnh nghĩa 1.1.  Dãy s là mt hàm s t  N  vào mt tp hp s ( N,Q,R,C)hay mt tp con nào đó ca các tp hp trên). Các s hng ca dãy s thưng đưc ký hiu là  un, vn, xn, y n thay vì  u(n), v(n), x(n), v(n). Bn thân dãy s đưc 

ký hiu là  {xn}.Vì dãy s là mt trưng hp đc bit ca hàm s nên nó cũng có các tính

cht ca mt hàm s.

Đnh nghĩa 1.2.  Dãy s  {xn} đưc gi là dãy tăng (gim) nu vi mi  n ta có xn+1 ≤ xn(xn+1 ≤ xn). Dãy s tăng hoc dãy s gim đưc gi chung là dãy đơn điu.Dãy s  {xn} đưc gi là b chn trên nu tn ti s thc  M  sao cho vi mi  nta có  xn ≤ M .Dãy s  {xn} đưc gi là b chn dưi nu tn ti s thc   m sao cho vi mi   nta có  xn ≥ m.

Mt dãy s va b chn trên, va b chn dưi đưc gi là dãy b chn.Dãy s  xn đưc gi là tun hoàn vi chu kỳ  k  nu  xn+k  = xn vi mi  n ∈ N. Dãy s tun hoàn vi chu kỳ 1 gi là dãy hng.

Đnh nghĩa 1.3.  Ta nói dãy s  {xn}  có gii hn hu hn  a khi  n dn đn vô cùng nu vi mi   >  0, tn ti s t nhiên  N 0  (ph thuc vào dãy s  xn  và   )sao cho vi mi  n > N 0 ta có  |xn − a| nh hơn  .

limn→∞ xn  =  a ⇔ > 0∃N 0 ∈ N   : ∀ n > N 0|xn − a| <

Ta nói dãy s  {xn}  dn đn vô cùng khi  n  dn đn vô cùng nu vi mi s thc dương  M  ln tuỳ ý, tn ti s t nhiên  N 0 (ph thuc vào dãy s  xn và  M )

sao cho vi mi  n > N 0 ta có  |xn| ln hơn  M .limn→∞ xn = ∞ ⇔ ∀ M > 0∃N 0 ∈ N   : ∀ n > N 0 |x| > M.

Dãy s có gii hn hu hn đưc gi là dãy hi t. Dãy s không có gii hnhoc dn đn vô cùng khi  n  dn đn vô cùng gi là dãy phân kỳ.

Page 6: Dãy số - Giới hạn Tác giả: Trần Nam Dũng- Nguyễn Văn Mậu, 2007

8/13/2019 Dãy số - Giớ i hạn Tác giả: Trần Nam Dũng- Nguyễn Văn Mậu, 2007

http://slidepdf.com/reader/full/day-so-gioi-han-tac-gia-tran-nam-dung-nguyen-van-mau 6/217

1.2. Đnh nghĩa và các đnh lý cơ bn   6

Đnh lý 1.1 (Tng, hiu, tích, thương các dãy hi t).  Nu  {xn}, {y n} là các dãy hi t và có gii hn tương ng là  a, b thì các dãy s 

 {xn +  y n

},

{xn

−y n

},

{xny n} và  {xn/y n} cũng hi t và có gii hn tương ng là  a + b, a − b, a.b, a/b.(Trong trưng hp dãy s thương, ta gi s  y n  và  b khác không)

Đnh lý 1.2 (Chuyn qua gii hn trong bt đng thc).  Cho dãy s  {xn}  có gii hn hu hn  l, nu  ∃N 0 ∈ N : ∀ n > N 0  ta có  a ≤ xn ≤ b thì  a ≤ xn ≤ b.

Đnh lý 1.3 (Đnh lý kp).  Cho ba dãy s  {xn}, {y n}, {z n} trong đó  xn và  z n có cùng gii hn hu hn  1, và  N 0 ∈  N   : ∀ n > N 0   ta có  xn ≤  y n ≤  z n. Khi đó  y ncũng có gii hn là  1.

Đnh lý 1.4 (Dãy đơn điu).  Mt dãy tăng và b chn trên hay mt dãy gim và b chn dưi thì hi t. Nói ngn gn hơn, mt dãy s đơn điu và b chn thì 

hi t.Đnh lý 1.5 (V dãy các đon thng lng nhau).  Cho hai dãy s thc  {an}, {bn}sao cho

a) ∀ n ∈ N, an ≤ bn;b) ∀ nßN, [an+1, bn+1] ⊂ [an, bn];c)  bn − an → 0 khi  n → ∞.

Khi đó tn ti duy nht s thc   l sao cho ∩ [an, bn] = 1.

Đnh lý 1.6 (Bolzano Veierstrass).  T mt dãy b chn luôn có th trích ra mt dãy con hi t.

Đnh nghĩa 1.4.  Dãy  {

xn

}đưc gi là dãy Cauchy nu 

∀  > 0

∃N 0

∈N:

∀ m, n >

N 0|xm − xn| < .

Đnh nghĩa 1.5 (Tiêu chun Cauchy).  Dãy s  {xn} có gii hn hu hn khi và ch khi nó là dãy Cauchy.

Cp s cng. Dãy s {xn} đưc gi là mt cp s cng khi và ch khi tnti  d  sao cho

∀ n ∈ N, xn+1 =  xn + d.

d đưc gi là công sai ca cp s cng,  x0  là s hng đu,  xn  là s hng th  n.Ta có các công thc cơ bn sau:

xn =  x0 + ndS n =  x0 + x1 + · · · + xn−1

= nx0 + n(n − 1)d/2

= n(x0 + xn−1)/2

Page 7: Dãy số - Giới hạn Tác giả: Trần Nam Dũng- Nguyễn Văn Mậu, 2007

8/13/2019 Dãy số - Giớ i hạn Tác giả: Trần Nam Dũng- Nguyễn Văn Mậu, 2007

http://slidepdf.com/reader/full/day-so-gioi-han-tac-gia-tran-nam-dung-nguyen-van-mau 7/217

1.2. Đnh nghĩa và các đnh lý cơ bn   7

Cp s nhân.  Dãy s

 {xn

} đưc gi là mt cp s nhân khi và ch khi tn ti

q  sao cho ∀ n ∈ N, xn+1 =  qxn.

d đưc gi là công bi ca cp s nhân,  x0  là s hng đu,  xn là s hng th  n.Ta có các công thc cơ bn sau:

xn =  q nx0

S n =  x0 + x1 + · · · + xn−1  = (q n − 1)x0/(q − 1)

Nu |q |  <  1 thì {xn} đưc gi là cp s nhân lùi vô hn. Tng ca cp s nhânlùi vô hn đưc tính theo công thc

S  =  x0/(1

−q )

Dãy Fibonacci. Dãy s Fibonacci là dãy s đưc đnh nghĩa bi

f 0  = 0, f 1 = 1, ∀ n ∈ N, f n+2 =  f n+1 + f n.

Dãy s Fibonacci có rt nhiu tính cht thú v và xut hin mt cách t nhiêntrong nhiu lĩnh vc khác nhau. Chúng ta có công thc sau đây đ tìm s hngtng quát ca dãy s Fibonacci:Công thc Binet.

f n =

1+√ 5

2

n−1−√ 5

2

n

√ 5

.

Nói chung, các dãy s xác đnh bi công thc truy hi  f n+2   =   f n+1  + f n   (vif 0, f 1 bt kỳ) đưc gi là dãy Fibonacci m rng.Dãy Farey. Dãy Farey  F n  vi mi s nguyên dương  n  là tp hp các phân sti gin dng  a/b  vi 0 ≤ a ≤ b ≤ n và (a, b) = 1 xp theo th t tăng dn.

Ví d 1.1.

F 5 = {0/1, 1/5, 1/4, 1/3, 2/5, 1/2, 3/5, 2/3, 3/4, 4/5, 1/1}.

Ngoi tr  F 1,  F n  có s l các phn t và  1/2 luôn nm gia. Gi  p/q ,  p/q  và  p/q   là các s hng liên tip trong dãy Farey thì 

 pq 

−qp = 1,  và  p/q  = ( p + p)/(q  + q ).

S các s hng  N (n) trong dãy Farey đưc tính theo công thc 

N (n) = 1 +n

k=1

ϕ(k) = 1 + φ(n).

Page 8: Dãy số - Giới hạn Tác giả: Trần Nam Dũng- Nguyễn Văn Mậu, 2007

8/13/2019 Dãy số - Giớ i hạn Tác giả: Trần Nam Dũng- Nguyễn Văn Mậu, 2007

http://slidepdf.com/reader/full/day-so-gioi-han-tac-gia-tran-nam-dung-nguyen-van-mau 8/217

1.3. Mt s phương pháp gii bài toán v dãy s   8

1.3 Mt s phương pháp gii bài toán v dãy s

Phương pháp gii các bài toán dãy s rt đa dng như chính yêu cu cachúng. Đó có th là mt tính cht s hc, mt tính cht đi s hay mt tính chtgii tích. Dưi đây chúng ta s xem xét nhng phương pháp cơ bn nht.

Tuy nhiên, có th đưa ra hai nguyên lý chung đ gii các bài toán dãy s là- Đng ngi vit ra các s hng đu tiên ca dãy s- Đng ngi tng quát hóa bài toán

1.3.1 Dãy s thc: mt s dng dãy s đc bit

Dãy s dng  xn+1  =  f (xn)

Đây là dng dãy s thưng gp nht trong các bài toán v gii hn dãy s.

Dãy s này s hoàn toàn xác đnh khi bit  f  và giá tr ban đu  x0. Do vy s hit ca dãy s s ph thuc vào tính cht ca hàm s  f (x) và x0. Mt đc đimquan trng khác ca dãy s dng này là nu  a là gii hn ca dãy s thì a phi lànghim ca phưng trình  x  =  f (x). Chúng ta có mt s kt qu cơ bn như sau:

Đnh nghĩa 1.6.  Hàm s  f   : D → D  đưc gi là mt hàm s co trên  D  nu tn ti s thc  q, 0 < q < 1  sao cho |f (x) − f (y )| ≤ q |x − y | vi mi  x, y  thuc  D.

Đnh lý 1.7.  Nu  f (x)   là mt hàm s co trên  D  thì dãy s  {xn}  xác đnh bi x0  = a ∈ D, xn+1  = f (xn)  hi t. Gii hn ca dãy s là nghim duy nht trên D ca phương trình  x =  f (x).

Chng minh. Vi mi  n > m thì áp dng đnh nghĩa hàm s co, ta có|xn − xm| = |f (xn−1) − f (xm−1)| ≤ q |xn−1 − xm−1| ≤ · · · ≤ q m|xn−m − x0|   (1.1)

T đây |xn − x0| ≤ |xn − xn−1| + · · · + |x1 − x0| ≤ (q n−1 + · · · + 1)|x1 − x0|, suyra {xn} b chn. Xét   > 0. T (1.1), do  q < 1  và |xn−m − x0| b chn nên ta suyra tn ti  N  sao cho q N |xn−m − x0| < . Suy ra {xn} là dãy Cauchy và do đó hit.

Ví d 1.2 (Vit Nam, 2000).  Cho dãy s  {xn} xác đnh như sau 

x0  = 0, xn+1 =  c − √ c + xn.

Tìm tt c các giá tr ca  c đ vi mi giá tr  x0 ∈ (0, c), xn xác đnh vi mi  nvà tn ti gii hn hu hn  limn→∞ xn.

Page 9: Dãy số - Giới hạn Tác giả: Trần Nam Dũng- Nguyễn Văn Mậu, 2007

8/13/2019 Dãy số - Giớ i hạn Tác giả: Trần Nam Dũng- Nguyễn Văn Mậu, 2007

http://slidepdf.com/reader/full/day-so-gioi-han-tac-gia-tran-nam-dung-nguyen-van-mau 9/217

1.3. Mt s phương pháp gii bài toán v dãy s   9

Gii.   Đ x1 tn ti thì ta thì  c−√ c + xn ≥ 0 vi mi x0 ∈ (0, c) hay c(c−1) ≥ x0

vi mi  x0

 ∈ (0, c), suy ra  c

 ≥ 2. Vi  c

 ≥ 2  thì  0  < x1  <

 √ c. Nu  0 < xn  <

 √ c

thì c − √ c + xn > c − 2√ c, suy ra  xn+1 tn ti và ta cũng có  0  < xn+1  < √ c.Đt f (x) = 

c − √ c + x thì  f (x) = −1

4

√ x + x

 c − √ 

c + x.Vi mi   x ∈   (0,

√ c)  ta có   (c +  x)(c − √ 

c + x)  > c(c − 

c +√ 

c) ≥   2(2 − 2 +

√ 2)  >   1

4 . T đó suy ra |f (x)| ≤   q <  1  vi mi  x ∈   (0,√ 

c), tc  f (x)   làhàm s co trên (0,

√ c), suy ra dãy s đã cho hi t. Vy tt c các giá tr  c  cn

tìm là c ≥ 2.Mt trưng hp na cũng có th xét đưc s hi t ca dãy s {xn} là trưng

hp f  đơn điu. C th làNu  f  là hàm s tăng trên  D thì {xn} s là dãy đơn điu. Dãy s này tăng

hay gim tuỳ theo v trí ca  x0 so vi  x1.Nu f  là hàm gim trên  D thì các dãy con

{x2 p

},

{x2 p+1

}là các dãy đơn điu

(và ngưc chiu nhau).Ví d 1.3 (Vô đch sinh viên Moskva, 1982).  Cho dãy s  {xn}   xác đnh bi x0  = 1982, xn+1 = 1/(4 − 3xn). Hãy tìm  limn→∞ xn

Gii.   Tính toán trc tip ta thy 0  < x2  <  1, x3 > x2. Vì  f (x) = 1/(4 − 3x) làmt hàm s tăng t  [0, 1] vào [0, 1] nên t đây, {xn}n≥2 là mt dãy s tăng và bchn trên bi 1 do đó có gii hn. Gi s gii hn là  a thì ta có  a = 1/(4 − 3a)hay a  = 1 (giá tr  a  = 1/3 loi do dãy tăng).Câu hi: Vi nhng giá tr nào ca  x0  thì dãy s xác đnh vi mi  x  và có giihn? Khi nào thì gii hn là  1? Khi nào thì gii hn là  1/3?

Trong trưng hp f  là hàm gim, ta có th chng minh dãy hi t bng cách

chng minh hai dãy con trên cùng hi t v mt gii hn.Tuy nhiên, khó khăn nht là gp các hàm s không đơn điu. Trong trưng

hp này, ta phi xét tng khong đơn điu ca nó và s hi t ca hàm s s tùythuc vào giá tr ban đu.

Ví d 1.4.  Tìm tt  c các giá tr ca  a đ dãy s {xn} xác đnh bi  x0 =  a, xn+1  =2 − x2

n  có gii hn hu hn.

Gii.   Hàm s  f (x) = 2 − x2 tăng trên  (−∞, 0) và gim trên  (0, +∞). Phươngtrình  f (x) =  x có hai nghim là  x  = −2  và  x  = 1. Đó là nhng d kin quantrng trong li gii bài toán này.

Đu tiên, ta nhn xét rng nu a <

 −2 thì do  f   : (

−∞,

−2)

→ (

−∞,

−2) và

là hàm tăng,  x1  = 2 − a2 < x0 nên dãy s {xn} gim. Nu dãy {xn} b chn dưithì nó hi t v nghim ca phương trình  x  = 2 − x2, điu này mâu thun vì dãygim và  x0 < −2. Vy {xn} không b chn dưi, tc không có gii hn hu hn.Nu  a > 2  thì  x1  < −2 và ta cũng suy {xn} không có gii hn hu hn.

Page 10: Dãy số - Giới hạn Tác giả: Trần Nam Dũng- Nguyễn Văn Mậu, 2007

8/13/2019 Dãy số - Giớ i hạn Tác giả: Trần Nam Dũng- Nguyễn Văn Mậu, 2007

http://slidepdf.com/reader/full/day-so-gioi-han-tac-gia-tran-nam-dung-nguyen-van-mau 10/217

1.3. Mt s phương pháp gii bài toán v dãy s   10

Vi  a = −2, 1 thì dãy s có gii hn. Xét  x0 ∈ [−2, 2]. Ta chng minh dãy scó gii hn hu hn khi và ch khi tn ti  n  sao cho  xn =

−2 hoc xn = 1. Tht

vy, gi s  xn có gii hn hu hn là b và xn   /∈ {−2, 1} vi mi n. Khi đó  b  = −2hoc  b = 1. Gi s  b = −2 thì tn ti  N 0  sao cho  xn  nm trong lân cn −2 vimi n ≥ N 0. Nhưng nu xn = −2 +  thì xn+1  = −2 + 4− 2 > xn, suy ra dãy xn

tăng k t  N 0 và không th dn v  2. Nu  b = 1 k t  n ≥ N 0  nào đó  xn  thuclân cn  1. Xét

xn+2 − xn = 2 − (2 − x2n)2 − xn  = (2 − xn − x2

n)(x2n − xn − 1)

Ti lân cn  1 thì  x2n − xn − 1 <  0. Vì nu  xn  < 1  thì  xn+1  > 1  (và ngưc li

xn  >  1 thì  xn+1  <  1 - chúng ta đang xét trong lân cn đim  1!) nên có th gis  xn  > 1. Khi đó  2 − xn − x2

n  <  0 suy ra  xn+2  > xn. Tip tc như vy, suy ra1 < xn  < xn+2  <

 · · ·< xn+2k  <

 · · · mâu thun vi gi thit  b  = 1. Vy điu gi

s là 2, tc là dãy s ch có gii hn khi tn ti  n  sao cho xn  = −2 hoc xn  = 1.Sau khi thu đưc kt qu này, ta s dng hàm ngưc  f −1(x) = ±√ 

2 − x đxây dng tt c các giá tr  a tha mãn điu kin đu bài.

Trong ví d trên, ta đã s dng gi thit tn ti gii hn đ thu gn min  D,t đó mt hàm có bin thiên phc tp tr thành mt hàm đơn điu.

Dãy s dng  xn+1  =  xn ± (xn)α và đnh lý trung bình Cesaro

Đây là trưng hp đc bit ca dãy s dng   xn+1   =  f (xn). Tuy nhiên, vidãy s dng này vn đ hi t ca  xn thưng không đưc đt ra (vì quá đơn ginvà gii hn ch có th là  0  hoc ∞). đây, ta s có mt yêu cu cao hơn là tìm

bc tim cn ca  xn, c th là tìm  b sao cho xn  = O(nβ 

). Vi các dãy s có dngnày, đnh lý trung bình Cesaro s t ra rt hu hiu.

Đnh lý 1.8 (Trung bình Cesaro).  Nu dãy s  {xn}  có gii hn hu hn là   athì dãy s các trung bình  {x1 + x2 + · · · + xn)/n} cũng có gii hn là  a.

Đnh lý này có th phát biu dưi dng tương đương nhưư sau: Nu lim n → ∞(xn+1−xn) =  a  thì  limn→∞ xn/n =  a.

Ta chng minh đnh lý cách phát biu 2. Rõ ràng ch cn chng minh chotrưng hp  a = 0. Vì  limn→∞(xn+1 − xn) = 0 nên vi mi   > 0  tn ti,  N 0 saocho vi mi n ≥ N 0 ta có |xn+1 − xn| < . Khi đó, vi mi  n > N 0

|xn/n

| ≤[|xN 

0|+

|xN 

0+1

−xN 0

|+

· · ·+

|xn

−xn

−1

|]/n <

|xN 

0 |/n + (n

−N 0)/n.

Gi c đnh  N 0, ta có th tìm đưc  N 1  > N 0  sao cho |xN 0|/N 1  < . Khi đó vimi n > N 1 ta s có |xn/n| < 2. Vy  limn→∞ xn/n = 0.

Đnh lý trung bình Cesaro có nhiu ng dng quan trng trong vic tìm giihn dãy s và có th phát biu cho các trung bình khác như trung bình nhân,

Page 11: Dãy số - Giới hạn Tác giả: Trần Nam Dũng- Nguyễn Văn Mậu, 2007

8/13/2019 Dãy số - Giớ i hạn Tác giả: Trần Nam Dũng- Nguyễn Văn Mậu, 2007

http://slidepdf.com/reader/full/day-so-gioi-han-tac-gia-tran-nam-dung-nguyen-van-mau 11/217

1.3. Mt s phương pháp gii bài toán v dãy s   11

trung bình điu hòa, trung bình lũy tha. Tuy nhiên, đây ta ch khai thác cáchphát biu 2 ca đnh lý đ áp dng cho các dãy s có dng  xn+1  =  xn

±(xn)α. Đ

tìm s  β  sao cho  xn/nβ  có gii hn hu hn, theo đnh lý trung bình Cesaro, tach cn tìm g sao cho xγ n+1−xγ 

n có gii hn hu hn a. Khi đó, limn→∞ xγ n/n =  a,

suy ra lim xn/nγ 1  = aγ 

1 , tc là  β  = 1/γ .

Ví d 1.5.  Cho dãy s  {xn} đưc xác đnh bi  x0 = 1/2, xn+1 =  xn − x2n. Chng 

minh rng  limn→∞ nxn = 1.

Gii.  Trong bài này,  β  = −1 do đó ta s th vi  γ  = −1. D dàng chng minhđưc  limn→∞ xn = 0. Ta có

1/xn+1 − 1/xn = (xn − xn+1)/xn+1xn = x2n/(xn − x2

n)xn = 1/(1 − xn) → 1.

T đó áp dng đnh lý trung bình Cesaro, suy ra lim1/nxn = 1, suy ta lim nxn  =1.

Ví d 1.6.  Cho dãy s  {xn}  đưc xác đnh bi  x0  = 1, xn+1  = sin(xn). Chng minh rng  lim

√ nxn =

√ 3.

Gii.   Dãy s đã cho không có dng xn+1  = xn ± (xn)α (?) nhưng kt lun cabài toán gi cho chúng ta đn đnh lý trung bình Cesaro. Vì  β  = −1 nên ta sth vi γ  = −2. D dàng chng minh đưc rng  lim xn  = 0. Xét

1/x2n − 1/x2

n = [x2n − sin2(xn)]/x2

n sin2(xn) → 1/3

(Dùng quy tc L’Hopitale)

T đó, theo đnh lý trung bình Cesaro lim1/nx2n = 1/3, suy ra limlim √ n.xn =√ 3.

Như vy, ta có th tìm  γ  nu bit  β . Trong trưng hp không bit  β  thì taphi d đoán.

Ví d 1.7 (Chn đi tuyn Vit Nam, 1993).  Dãy s  {an}  đưc xác đnh bi a1  = 1 và  an+1  = an + 1/

√ an  . Hãy tìm tt c các s thc  β  đ dãy s  (an)β /n

có gii hn hu hn khác  0.

Gii.  Trưc ht ta chng minh  an dn ti vô cùng khi n  dn ti vô cùng. Thtvy, ta có   a2

n+1   =   a2n  + 2

√ an + 1/an   > a2

n  + 2. Suy ra  a2n+1   >   1 + 2n  suy ra

(đpcm). Tr li bài toán, xét

a3/2n+1 − a3/2

n   = (an + 1/√ 

an)3/2 − a3/2n   = (1 + 1/a3/2

n   )3/2/(1/a3/2n   )

Đt   x   = 1/a3/2n   thì   x →   0   khi   n → ∞. Do đó   limn→∞(a

3/2n+1 −  a

3/2n   ) =

limx→0(1 + x)3/2/x = 3/2 (Quy tc L’Hopitale) T đó suy ra lim  a3/2n   /n = 3/2.

Page 12: Dãy số - Giới hạn Tác giả: Trần Nam Dũng- Nguyễn Văn Mậu, 2007

8/13/2019 Dãy số - Giớ i hạn Tác giả: Trần Nam Dũng- Nguyễn Văn Mậu, 2007

http://slidepdf.com/reader/full/day-so-gioi-han-tac-gia-tran-nam-dung-nguyen-van-mau 12/217

1.3. Mt s phương pháp gii bài toán v dãy s   12

Vi  β >  3/2 suy ra gii hn bng ∞, vi  β <  3/2 suy ra gii hn bng  0. Vyβ  = 3/2 là giá tr duy nht tho mãn yêu cu bài toán.

Câu hi:1) Làm sao có th d đoán đưc giá tr  β ?2) α  và β  có mi quan h gì?

1.3.2 Dãy s nguyên

Dãy s nguyên là mt phn quan trng trong lý thuyt dãy s. Ngoài các vnđ chung như tìm s hng tng quát ca dãy s, tìm công thc tính tng  n  shng đu tiên... các bài toán v dãy s nguyên còn quan tâm đn tính cht shc ca dãy s như chia ht, đng dư, nguyên t, chính phương, nguyên t cùngnhau... Các bài toán v dãy s nguyên rt đa dng. Trong nhiu trưng hp, dãys ch là cái b ngoài, còn bn cht bài toán là mt bài toán s hc. Trong cácphn dưi đây, chúng ta s ít đ cp đn nhng bài toán như vy mà chuynchúng vào phn bài tp.

Nguyên lý Dirichlet và dãy s nguyên

Nguyên lý Dirichlet là mt nguyên lý ht sc đơn gin nhưng li vô cùng huhiu trong các bài toán chng minh, đc bit là chng minh s tn ti ca mtđi tưng tho mãn mt điu kin nào đó. S dng nguyên lý này, ngưi ta đãchng minh đưc nhiu kt qu rt mnh, ví d như đnh lý Fermat-Euler vtng hai bình phương, đnh lý Weil v phân b đu... đây ta nêu ra hai ktqu liên quan đn dãy s:

Đnh lý 1.9 (Weil, v phân b đu).  Nu  α  là s vô t thì dãy  {nα}n=1 phân b đu trên khong  (0, 1).

Đnh lý 1.10 (V s tun hoàn ca các s dư).  Cho dãy s nguyên  {xn}xác đnh bi công thc truy hi  xn+k  = a1xn+k−1 + · · · + akxn  và  k s hng đu tiên nguyên. Khi đó, vi mi s nguyên dương  N , dãy s dư ca  xn khi chia choN  s tun hoàn.

Tip theo ta xét mt vài ví d v vic s dng nguyên lý Dirichlet trong cácbài toán dãy s.

Ví d 1.8.  Chng minh rng nu  1≤

 a1, a2,...,an+1

≤ 2n thì tn ti   i < j  sao

cho  ai | a j.

Gii.  Mi s  ai có th vit dưi dng  ai  = 2siri vi  ri là s l. Các s  ri ch cóth nhn n giá tr t  1, 3, ...,2n− 1. Vì có  n + 1 s nên theo nguyên lý Dirichlet,tn ti i < j sao cho  ri =  r j  và tương ng ta có  ai | a j .

Page 13: Dãy số - Giới hạn Tác giả: Trần Nam Dũng- Nguyễn Văn Mậu, 2007

8/13/2019 Dãy số - Giớ i hạn Tác giả: Trần Nam Dũng- Nguyễn Văn Mậu, 2007

http://slidepdf.com/reader/full/day-so-gioi-han-tac-gia-tran-nam-dung-nguyen-van-mau 13/217

1.3. Mt s phương pháp gii bài toán v dãy s   13

Ví d 1.9 (Tp chi AMM).  Xét  n s nguyên dương  a1 < a2 < · · · < an ≤ 2nsao cho   [ai, a j ] >  2n vi mi   i

= j . Chng minh rng  a1  > 2n/3.

Gii.   Nu a1 ≤ 2n/3, ta xét  n + 1 s  2a1, 3a1, a2, . . . , an. Các s này đu khôngln hn 2n và không có s nào là bi ca s nào. Điu này mâu thun vi kt qabài toán trên.

Ví d 1.10.  (Canada, 2000) Cho  A  = (a1, a2,...,an) là dãy các s nguyên thuc đon  [−1000, 1000]. Gi s tng các s hng ca  A bng  1. Chng minh rng tn ti mt dãy con (cha ít nht 1 phn t) ca  A có tng bng  0.

Gii.   Ta có th gi s trong  A  không có phn t nào bng 0, vì nu ngưcli thì bài toán hin nhiên. Ta sp xp dãy  A  thành dãy  B  = (b1,...,b2000) bngcách chn dn t các s hng ca dãy  A theo quy tc sau:   b1  >  0, b2  <  0. Vimi   i

 ≥  3  chn   bi  là s có du ngưc vi du ca tng  si

−1   =   b1 +

· · ·+ bi

−1

(vì sao luôn thc hin đưc?). Bng cách xây dng như th, ta đưc   2000  ss1, s2,...,s2000  nm trong đon   [−999, 1000]. Nu trong s  si  có mt s bng 0thì bài toán đúng. Trong trưng hp ngưc li, theo nguyên lý Dirichlet tn tii < j sao cho  si =  s j . Khi đó  bi+1 + · · · + b j  = 0.

H đm cơ s và dãy s nguyên

H đm cơ s có th dùng đ xây dng nhiu dãy s có tính cht rt thúv. Nhìn trên phương din ca mt cơ s khác, có th rt khó nhn ra quy lut,nhưng nu chn đúng cơ s thì bài toán tr nên vô cùng đơn gin.

Xin nhc li là vi  b  là mt s nguyên dương ln hơn hay bng 2 thì mi snguyên dương  N  đu có th biu din mt cách duy nht dưi dng

N  = a1...ak(b) =  a1bk−1 + · · · + ak vi 1 ≤ a1 ≤ b − 1,   0 ≤ a2, . . . , a k ≤ b − 1.

Đó là đnh nghĩa h đm cơ s dng cơ bn nht. Tuy nhiên, có th ly mt dãys nguyên bt kỳ (có tr tuyt đi tăng nghiêm ngt) làm h đm cơ s ví d hđm cơ s  (−2), h đm cơ s Fibonacci (3 = 4 − 2 + 1, 17 = 13 + 3 + 1...)

Các h đm thưng s dng nht là h đm c s 2 và c s 3. Dưi đây ta xétmt vài vì d:

Ví d 1.11 (IMO 1983).  Chng minh hoc ph đnh mnh đ sau: T tp hp105 s nguyên dương đu tiên luôn có th chn ra mt tp con gm 1983 s saocho không có ba s nào lp thành mt cp s cng.

Gii.   Ta chng minh mnh đ tng quát: T  3n s t nhiên đu tiên luôn cóth chn ra  2n s sao cho không có ba s nào lp thành mt cp s cng. Thtvy, xét trong h đm cơ s 3 tp hp tt c các s có ≤ n  ch s. Chn các smà trong biu din tam phân ca nó ch cha ch s  2  và ch s 0. Khi đó có  2ns như vy và không có ba s nào trong chúng lp thành mt cp s cng.

Page 14: Dãy số - Giới hạn Tác giả: Trần Nam Dũng- Nguyễn Văn Mậu, 2007

8/13/2019 Dãy số - Giớ i hạn Tác giả: Trần Nam Dũng- Nguyễn Văn Mậu, 2007

http://slidepdf.com/reader/full/day-so-gioi-han-tac-gia-tran-nam-dung-nguyen-van-mau 14/217

1.3. Mt s phương pháp gii bài toán v dãy s   14

Ví d 1.12 (Singapore 1995).  Cho dãy s  {f n} xác đnh bi  f 1 = 1, f 2n =  f nvà  f 2n+1 = f 2n+1.

(i) Tính  M  = max{f 1,...,f 1994}(ii) Tìm tt c các giá tr  n,  1 ≤ n ≤ 1994 sao cho  f n = M .

Gii.   Kinh nghim mt chút ta thy ngay f n  chính là tng các ch s ca   ntrong h đm nh phân. T đây do  1994 <  2048 = 211  suy ra M  = 10.

Ví d 1.13.  Dãy s  {f n}  đưc xác đnh bi  f 1   = 1, f 2n  = 3f n, f 2n+1  =  f 2n+1.Hãy tính  f 100.

Gii.   f n  đưc xác đnh như sau: Xét biu din nh phân ca   n   ri tính giátr ca s nh phân này trong h tam phân. Vì  100 = 26 + 25 + 22  nên  f 100  =36 + 35 + 32 = 981.

Ví d 1.14.  Dãy s  {an} đưc xác đnh bi  0 ≤ a0 <  1, an = 2an−1 nu  2an−1  <1 và  an = 2an−1 − 1 nu  2an−1 ≥ 1. Hi có bao nhiêu giá tr  a0  đ  a5 =  a0.

Gii.  Phân tích: Khi tính  an   theo  an−1  ta có th la chn mt trong hai côngthc. Tt nhiên, vi a0 đã chn ri thì tt c các bưc tip theo đu xác đnh mtcách duy nht. Tuy nhiên, ta có th chn  a0  như th nào đó đ sau đó các côngthc tính theo đúng kch bn đã cho. Có 25 = 32 kch bn như vy. Ví d vi kchbn (1, 1, 2, 1, 2) ta có  x1  = 2x0, x2  = 2x1  = 4x0, x3  = 2x2 − 1 = 8x0 − 1, x4  =2x3  = 16x0 − 2, x5 = 2x4 − 1 = 32x0 − 3.

Gii phương trình  x0  =  x5 ta đưc  x0  = 3/31. Tt nhiên, đ có đưc mt ligii hoàn chnh, ta cn phi lp lun cht ch đ thy rng các  x0  thu đưc là

khác nhau và vi mi  x0  thu đưc, dãy s s "đi" đúng như kch bn đã đnh.Tuy nhiên, phân tích này gi chúng ta hưng đn h nh phân. Và ta có li giiđp mt sau:

Nu a0 = 0, d1d2d3 . . . là biu din nh phân ca a0 thì  a1 = 0, d2d3d4 . . . Thtvy, nu 2a0 <  1 thì d1 = 0 và a1 = 2a0  = 0, d2d3d4 . . . còn nu 2a0 ≥ 1 thì d1 = 1và  a1  = 2a0 − 1 = 0, d2d3d4 . . .

Hoàn toàn tương t, a2 = 0, d3d4d5 . . . , . . . , a5 = 0, d6d7d8 . . . Như vy a5  =  a0

khi và ch khi  a0 là phân s nh phân tun hoàn chu kỳ  5. Có 25 = 32 chu kỳ tunhoàn như vy, trong đó chu kỳ  11111 cho chúng ta  a0   = 1 (loi). Vy tt c có31 giá tr a0 tha mãn yêu cu đ bài. Đó là  0, (00000), 0, (00001), . . . , (0, 11110).Tính sang h thp phân đó là các giá tr  0, 1/31, 2/31, . . . , 30/31.

S phc và dãy s nguyên

S phc có nhng ng dng rt quan trng trong toán hc nói chung và tronglý thuyt dãy s nói chung. Nh s phc, chúng ta có th thy đưc mi quan hgia hàm lưng giác và hàm mũ. Nh s phc, mi đa thc bc  n đu có đ  n

Page 15: Dãy số - Giới hạn Tác giả: Trần Nam Dũng- Nguyễn Văn Mậu, 2007

8/13/2019 Dãy số - Giớ i hạn Tác giả: Trần Nam Dũng- Nguyễn Văn Mậu, 2007

http://slidepdf.com/reader/full/day-so-gioi-han-tac-gia-tran-nam-dung-nguyen-van-mau 15/217

1.3. Mt s phương pháp gii bài toán v dãy s   15

nghim và vì vy đnh lý Viét mi phát huy đưc tác dng. Dưi đây ta xét mts ví d v ng dng ca s phc trong các bài toán tính tng và dãy truy hi.

Ví d 1.15.  Vi s nguyên dương  n, hãy tính 

A(n) =  C 0n + C 3n + · · · + C 3[n/3]n   .

Gii.  Có th đt B (n) = C 1n + C 4n + · · ·+ C (n) = C 2n + C 5n + · · ·  ri s dng cáccông thc

A(n) + B (n) = B (n + 1), B (n) + C (n) = C (n + 1), C (n) + A(n) =  A(n + 1)

đ tìm công thc tính A(n). Tuy nhiên da theo cách tính C 0n +C 2n +· · ·+C 2n[n/2]bng cách thay x  = 1, y  = 1 và x  = 1, y  = −1 vào công thc nh thc Newton, tacó cách gii khác khá đp như sau: Gi  là s tha mãn phưng trình  2++1 = 0.

Do  3

= 1 nên ta có(1 + 1)n = A(n) + B (n) + C (n)

(1 + )n = A(n) + B (n) + 2C (n)

(1 + 2)n = A(n) + 2B (n) + C (n)

T đây suy ra  3A(n) = 2n + (1 + )n + (1 + 2)n. T đây, dùng công thcMoivre ta tìm đưc

A(n) = [2n + 2 cos(np/3)]/3.

Ví d 1.16.  Tính tng  S n(x) =  C 0n + C 1n cos x + · · · + C nn cos nx.

Gii.   Đt  T n(x) = 0 + C 1

n

 sin x +· · ·

+ C n

n

 sin nx thì  S n(x) +  iT n(x) =  C 0

n

 +C 1n(cos x+i sin x)+· · ·+C nn (cos x+i sin x)n = (1+cos x+i sin x)n = 2[cos(x/2)[cos(x/2)+i sin(x/2)]]n = 2n cosn(x/2)[cos(nx/2) + i sin(nx/2)].

T đó suy ra  S n(x) = 2n cosn(x/2) cos(nx/2).

Ví d 1.17 (AMM).  Cho dãy s  {un}  xác đnh bi   u0   = 3, u1   = 0, u2   =2, un+3  =  un+1 + un. Chng minh rng  u p luôn chia ht cho  p nu  p  là s nguyên t.

Gii.   Phương trình đc trưng ca dãy s có dng  x3 − x − 1 = 0. Nu phươngtrình đc trưng này có nghim nguyên thì ta có th s dng đnh lý nh Fermatđ chng minh kt lun ca bài toán. Tuy nhiên, các nghim này không nguyên,thm chí phưng trình ch có 1 nghim thc. Ta phi cu cu đn s tr giúp ca

s phc.Gi u,v,w là ba nghim ca phương trình thì u+v+w  = 0, uv+vw+wu = −1,

suy ra u2 + v2 + w2 = (u + v + w)2 − 2(uv + vw + wu) = 2. T đó ta có th ktlun

un  = un + vn + wn

Page 16: Dãy số - Giới hạn Tác giả: Trần Nam Dũng- Nguyễn Văn Mậu, 2007

8/13/2019 Dãy số - Giớ i hạn Tác giả: Trần Nam Dũng- Nguyễn Văn Mậu, 2007

http://slidepdf.com/reader/full/day-so-gioi-han-tac-gia-tran-nam-dung-nguyen-van-mau 16/217

1.3. Mt s phương pháp gii bài toán v dãy s   16

Vi  p là s nguyên t l thì  u p = −(v + w) p = −v p − w p −

 p−1i=1  C i pviw p−i.

Tương t v p =

−w p

−u p

− i = 1 p−1C i pwiu p−i, w p =

−u p

−v p

− p−1i=1  C i puiv p−i.

T đó suy ra  3(u p + v p + w p) = − p−1i=1  C i p(viw p−i + wiu p−i + uiv p−i)

Bây gi, chú ý rng  C i p chia ht cho p  vi 1 ≤ i ≤  p − 1i (vì p  là s nguyên t)và (viw p−i + wiu p−i + uiv p−i) là s nguyên (biu thc đi xng đi vi  u, v,w)nên v phi là mt s nguyên chia ht cho  p. Vy vi p nguyên t, p > 3 bài toánđã đưc chng minh. Cui cùng chú ý u2  = 2, u3 = 3 ta có bài toán đúng vi mi p.

Dãy s dng   [nα]

Dãy s dng   xn   = [nα]  có nhiu tính cht s hc thú v. Nu a > 1 thì{[nα]}n≥1 là dãy các s nguyên dương phân bit, có s bin thiên gn ging mt

cp s cng nhưng li không phi là mt cp s cng. Dãy s này đc bit thú vkhi a  là s vô t bc hai. Ta có mt kt qa quen thuc sau đây

Đnh lý 1.11.  Nu  a, b  là các s vô t dưng tho mãn điu kin  1/a + 1/b = 1thì hai dãy s  xn  = [nα], y n  = [nβ ], n = 1, 2, 3, ... lp thành mt phân hoch ca tp hp các s nguyên dương.

Chng minh. Xét hai dãy s  α, 2α, 3α, ...và β, 2β, 3β,... Không mt s hngnào trong các s hng trên là s nguyên. Vi mi s nguyên dương N , có [N/α] shng ca dãy th nht nm bên trái  N  và [N/β ] s hng ca dãy th hai. NhưngN/α + N/β  = N , vì  α, β  là các s vô t, phn l ca các s  N/α và N/β  là cács dương có tng bng 1 (do đng thc trên). Suy ra có  [N/α] + [N/β ] = N  − 1

s hng ca c hai dãy nm bên trái  N . Vì bên trái N  + 1 có  N  s hng ca chai dãy nên gia N  và  N  + 1 có đúng mt s hng ca mt trong hai dãy, t đósuy ra điu phi chng minh.Câu hi: Có th phát biu và chng minh đnh lý đo như th nào?

Hai dãy s trên vét ht tp hp các s nguyên dương. Điu này cho chúng tamt hưng suy nghĩ: nu hai dãy s vét ht tp hp các s nguyên dương thì cókh năng chúng s có dng trên. Và nhiu bài toán đã đưc xây dng theo hưngnày. Chúng ta xét mt ví d

Ví d 1.18 (AMM).  Gi s  {f n} và  {gn} là hai dãy s nguyên dương đưc xác đnh như sau 

1)  f 1  = 1

2)  gn = na − 1 − f n, trong đó  a là s nguyên ln hơn  4,3)  f n+1  là s nguyên dương nh nht khác các s  f 1, f 2,...,f n, g1, g2,...,gn.Chng minh rông tn ti các hng s  α, β  sao cho   f n   = [nα], gn  = [nβ ]  vi 

mi  n = 1, 2, 3,...

Page 17: Dãy số - Giới hạn Tác giả: Trần Nam Dũng- Nguyễn Văn Mậu, 2007

8/13/2019 Dãy số - Giớ i hạn Tác giả: Trần Nam Dũng- Nguyễn Văn Mậu, 2007

http://slidepdf.com/reader/full/day-so-gioi-han-tac-gia-tran-nam-dung-nguyen-van-mau 17/217

1.3. Mt s phương pháp gii bài toán v dãy s   17

Gii.  Theo cách xây dng {f n} và {gn} lp thành mt phân hoch ca  N ∗. Gis ta đã tìm đưc a, b tha mãn điu kin đu bài, khi đó, ta phi có  1/α+1/β  = 1.

Ngoài ra, khi  n đ ln thì  na − 1 = f n +  gn ∼  nα  + nβ , suy ra  α + β  = a. Vyα, β  phi là nghim ca phương trình  x2 − ax + a = 0.

Xét phương trình x2 − ax + a = 0 có hai nghim α < β . Vì a > 4,  α, β  là cács vô t. Dãy s {f n} và {gn} đưc xác đnh mt cách duy nht, do đó đ chngminh khng đnh ca bài toán, ta ch cn chng minh  {[nα]} và {[nβ ]} tha mãncác điu kin 1), 2), 3).

Rõ ràng  [a] = 1, [nβ ] = [n(a − α)] = nα  + [−nα)] = na − [nα] − 1 (do  nα  vôt).

Gi s  [nα] = [mβ ] =  k, đt nα  =  k  + r,  mβ  =  k  + s vi  0  < r,s <  1  thì

n + m =  k(1/α + 1/β ) + r/α + s/β  =  k  + r/α + s/β,

điu này không th xy ra vì  0  < r/α +  s/β <  1. Như vy vi mi  m, n ta có[nα] = [mβ ].

Tip theo,

[(n + 1)α] ≥ [nα] + 1,   [(n + 1)β ] ≥ [nβ ] + 2  >  [nα] + 1.

Cui cùng gi s  k  là mt s nguyên bt kỳ và  n  = [(k + 1)/α]. Nu  n > k/αthì  k < nα < α(k + 1)/α  =  k + 1  và   [nα] =   k. Nu   n < k/α  thì   (k − n)β >kβ − βk/α =  βk(1 − 1/α) =  k, (k − n)β < kβ − β ((k + 1)/α − 1) =  k + 1, suy ra[(k − n)β ] = k.

T các nhn xét trên ta suy ra mi s nguyên dương  k có mt trong dãy s

đúng mt ln và hai dãy s {[nα]} và {[nβ ]} tha mãn điu kin 3) (đpcm)Ghi chú: Trong li gii trên, ta đã không dùng đn kt qu ca đnh lý trênvà đó cũng chính là mt cách chng minh khác cho đnh lý.

Các bài toán v dãy s dng {[nα]} thưng liên quan đn phân hoch và cácdãy s gn tuyn tính (xm+n ∼ xm + xn). Xin xem thêm mt s ví d trong phnbài tp.

1.3.3 Dãy s và phương trình

Dãy s có mi quan h rt cht ch vi phương trình. Điu này có th thyrt rõ qua hai ví d cơ bn: phương trình sai phân tuyn tính đưc gii bng vicxét nghim ca phương trình đc trưng, gii hn ca dãy s cũng thưng đưc

gii ra t mt phương trình. V vn đ này, xin đc thêm các mc tương ngtrong bài này. Đây là mt trong nhng ni dung quan trng nht trong phn dãys.

Page 18: Dãy số - Giới hạn Tác giả: Trần Nam Dũng- Nguyễn Văn Mậu, 2007

8/13/2019 Dãy số - Giớ i hạn Tác giả: Trần Nam Dũng- Nguyễn Văn Mậu, 2007

http://slidepdf.com/reader/full/day-so-gioi-han-tac-gia-tran-nam-dung-nguyen-van-mau 18/217

1.3. Mt s phương pháp gii bài toán v dãy s   18

1.3.4 Mt vài th thut khác

Sp xp li th t 

Sp xp li th t là mt th thut thưng đưc áp dng trong các bài toánliên quan đn bt đng thc trong dãy s. Vic sp xp li th t các s trênđưng thng dn đn các tính cht đc bit mà mt dãy s bt kỳ không có,chng hn nu  a < b < c thì |c − a| = |c − b| + |b − a|. Cũng như các nguyên lýcơ bn khác, nguyên lý đơn gin này t ra khá hu hiu trong nhiu trưng hp.

Ví d 1.19 (Vit Nam 1998).  Tn ti hay không mt dãy s thc  {xn} tha mãn điu kin 

1) |xn| ≤ 0, 666 vi mi  n = 1, 2, 3,...2) |xm − xn| ≥ 1/n(n + 1) + 1/m(m + 1)  vi mi s nguyên dương  m n.

Gii.   Gi s tn ti dãy s như vy. Vi mi s nguyên dương  N , ta sp xp licác s x1,...,xN  theo th t tăng dn

xi1 ≤ xi2 ≤ · · · ≤ xiN 

Khi đó |xiN −xi1| = |xiN −xiN −1|+· · ·+|xi2−xi1|1/iN (iN +1)+1/iN −1(iN −1+1)+ · · ·+1/i2(i2+1)+1/i1(i1+1) = 2

1/ik(ik+1)−1/iN (iN +1)−1/i1(i1+1) =

A(N ).Vì  i1, i2,...,iN  ch là mt hoán v ca 1, 2,...,N  nên ta có

A(N ) = 2

1/k(k + 1) − 1/iN (iN  + 1) − 1/i1(i1 + 1)

= 2(1

−1/(N  + 1))

−1/iN (iN  + 1)

−1/i1(i1 + 1)

≥ 2(1 − 1/(N  + 1)) − 1/1.2 − 1/2.3 = 4/3 − 2/(N  + 1)

Bây gi chú ý rng |xiN  − xi1| ≤ 2x0, 666 <  4/3. Chn  N  đ ln sao cho  4/3 −2/(N  + 1) >  2x0, 666, ta suy ra mâu thun. Vy không tn ti dãy s tha mãnyêu cu đ bài.

Ví d 1.20 (Liên Xô 1986).  Gi s  a1, a2,...,an là các s dương tuỳ ý. Chng minh bt đng thc 

1/a1 + 2/(a1 + a2) + · · · + n/(a1 + · · · + an) <  4(1/a1 + 1/a2 + · · · + 1/an)

Gii.   V phi không thay đi nu ta thay đi th t ca  ai  do đó ta ch cn(và phi) chng minh bt đng thc đúng cho trưng hp tng bên trái lnnht. Điu này xy ra khi  ai  đưc sp theo th t tăng dn. Tht vy, gi s 0 < b1 ≤ b2 ≤ ... ≤ bn là các s  ai đưc sp xp li. Khi đó rõ ràng vi mi  k  tacó b1 + · · · + bk ≤ a1 + · · · + ak  và

1/a1+2/(a1+a2)+· · ·+n/(a1+· · ·+an) ≤ 1/b1+2/(b1+b2)+· · ·+n/(b1+· · ·+bn)

Page 19: Dãy số - Giới hạn Tác giả: Trần Nam Dũng- Nguyễn Văn Mậu, 2007

8/13/2019 Dãy số - Giớ i hạn Tác giả: Trần Nam Dũng- Nguyễn Văn Mậu, 2007

http://slidepdf.com/reader/full/day-so-gioi-han-tac-gia-tran-nam-dung-nguyen-van-mau 19/217

1.3. Mt s phương pháp gii bài toán v dãy s   19

Vi mi  k , ghép các s hng ca tng bên phi thành cp ta có đánh giá sau

(2k−1)/(b1+· · ·+b2k−1)+2k/(b1+· · ·+b2k−1) <  (2k−1)/kbk+2k/(k+1)bk  < 4/bk

T đó suy ra bt đng thc cn chng minh.

Phép th lưng giác

Nhiu dãy s đi s vi công thc phc tp có th tr thành các dãy s đơngin nh phép th lưng giác. Th thut này đc bit hiu quan trong các bàitoán chng minh mt dãy s là tun hoàn hay không tun hoàn. Đ áp dngđưc th thut này, điu cn thit là bit các công thc lưng giác và mt chútnhy cm toán hc.

Ví d 1.21 (Vit Nam, 1990). Cho{

xn

}là dãy s tha mãn điu kin 

|x1

|< 1,

xn+1  = (−xn +  3 − 3x2n)/2 (n ≥ 1)

a) x1 phi tha mãn điu kin gì đ tt c các s hng ca dãy s đu dương? b) Dãy s trên có tun hoàn không? 

Điu kin |x1|   <   1  và dng ca hàm s gi ngay cho chúng ta phép đtx1  = cos ϕ vi  ϕ  thuc  (0, π) khi đó  x2  = (− cos ϕ + 3 sin ϕ)/2 =  cos(ϕ − 2π/3).T đó suy ra  xn+1  = cos(ϕ − 2nπ/3). T đây có th d dàng tr li các câu hica đ bài.

Ví d 1.22 (KVANT).   Cho dãy s   un   xác đnh bi:   u1   = 2, un+1   = (2 +un)/(1 − 2un).

a) Chng minh rng  un

 = 0 vi mi  n nguyên dương 

b) Chng minh dãy không tun hoàn 

Gii.   Đt ϕ  = arctan2, tan = 2. Khi đó nu  un = tan x thì  un+1  = tan(ϕ + x),suy ra  un   = tan(nϕ). S dng công thc   tan2x  = 2tan x/(1 − tan2 x)  suy rau2n   = 2un/(1 − u2

n). T đây nu   u2n   = 0   thì   un   = 0. Nu tn ti   n   sao cho

un   = 0  thì s dng tính cht này, ta suy ra tn ti  s  sao cho  u2s + 1 = 0 hay(2 + u2s)/(1 − 2u2s) = 0 hay u2s = −2, 2us/(1 − us2) = −2. Suy ra us vô t. Điunày vô lý. Phn b) là h qu ca câu a).

Ví d 1.23.  Tìm công thc tng quát tính s hng ca dãy s  x0  =  a, xn+1  =2 − x2

n.

Gii.   Nu |a| ≤ 2 thì đt  a  = −2cos ϕ, ta đưc  xn = −2 cos(2nϕ). Nu |a| > 2,đt a  = −(a + 1/a) thì ta đưc  xn  = −(α2n + 1/α2n).

Ví d 1.24 (Th Nhĩ Kỳ 1997).  Hai dãy  {an}, {bn} đưc xác đnh bi  a1  =α, b1 =  β, an+1 =  αan −βbn, bn+1  = βan + αbn. Có bao nhiêu cp  (a, b) tha mãn a1997 =  b1,  b1997 =  a1? 

Page 20: Dãy số - Giới hạn Tác giả: Trần Nam Dũng- Nguyễn Văn Mậu, 2007

8/13/2019 Dãy số - Giớ i hạn Tác giả: Trần Nam Dũng- Nguyễn Văn Mậu, 2007

http://slidepdf.com/reader/full/day-so-gioi-han-tac-gia-tran-nam-dung-nguyen-van-mau 20/217

1.3. Mt s phương pháp gii bài toán v dãy s   20

Gii.   Ta có  a2n+1 +  b2n+1  = (a2 + b2)(a2

n +  b2n) nên yêu cu bài toán xy ra chkhi  α2 + β 2 = 1. Đt a  = cos ϕ, β  = sin ϕ thì  an  = cos(nϕ), bn = sin(nϕ). T đó

suy ra li gii ca bài toán.Phép th lưng giác thưng đưc áp dng trong các bài toán có công thc"gi nh" đn các công thc lưng giác hoc có kt qu ging tính cht hàmlưng giác (chng hn tính tun hoàn hoc tính b chn). Tuy nhiên, phép thlưng giác có th xut hin nhng trưng hp mà tưng chng không dính dánggì đn vi lưng giác.

Ví d 1.25.  Vi mi s t nhiên  n > 1  và  n s thc dương  x1, x2,...,xn đt 

f  = max{x1, 1/x1 + x2, ..., 1/xn−1 + xn, 1/xn}.

Hãy tìm  min f .

Gii.   Tưng chng như bài toán này không liên quan gì đn lưng giác. Và hơnth, cũng chng liên quan gì đn dãy s. Tuy nhiên, điu kin đt giá tr nh nhtca  f  s to ra mt dãy s! Ta chng minh rng nu  x1, x2,...,xn là  n  s thcmà ti đó  f  đt min thì ta phi có  x1  = 1/x1 + x2  = ...  = 1/xn−1 + xn  = 1/xn.Và bài toán dãy s đã xut hin: Vi mi s nguyên dương  n, xét dãy s {xk}nk=1

xác đnh bi   x1   =   a và  xk   =  x1 − 1/xk−1, vi  k   = 2,...,n. Hãy tìm  a  sao cho1/xn  = x1. Và bài toán cui cùng này có th gii như sau. Đt  x1  = 2 cosϕ thìx2  = 2cos ϕ − 1/2cos ϕ  = (4cos2 ϕ − 1)/2cos ϕ  = sin3 ϕ/ sin2 ϕ, x3  = 2cos ϕ −sin2ϕ/ sin3ϕ = sin 4ϕ/ sin3ϕ... Tip tc như vy suy ra xn  = sin(n+1)ϕ/ sin nϕ.T đó đng thc  1/xn  =  x1   sin nϕ/ sin(n + 1)ϕ = 2 cos ϕsin(n + 2)ϕ = 0. Đnđây, t điu kin  xk  dương ta suy ra ϕ  =  π/(n + 2) và min f  = 2 cos(π/(n + 2)).Câu hi:

1) Ti sao có th khng đnh khi f  đt min thì các giá tr trên đây phi bngnhau?

2) Ti sao có th đt  x1 = 2cos ϕ?3) Làm sao có th d đoán ra cách đt trên?4) Phép gii trên còn chưa cht ch đim nào?5) Mi s thc x đu có th biu din dưi dng x  = 2cos ϕ hoc, x =  a +1/a.

Điu đó có ý nghĩa gì?

Dãy s ph

Khi kho sát s hi t ca mt dãy s ta thưng đnh lý v dãy đn điu và bchn. Nu dãy không đơn điu thì có th th xét dãy vi ch s chn và dãy vich s l. Tuy nhiên, có nhng dãy s có "hành vi" phc tp hơn nhiu. Chúngtăng gim rt bt thưng. Trong mt s trưng hp như th, ta có th xây dngmt (hoc 2) dãy s ph đơn điu, chng minh các dãy s ph có gii hn và

Page 21: Dãy số - Giới hạn Tác giả: Trần Nam Dũng- Nguyễn Văn Mậu, 2007

8/13/2019 Dãy số - Giớ i hạn Tác giả: Trần Nam Dũng- Nguyễn Văn Mậu, 2007

http://slidepdf.com/reader/full/day-so-gioi-han-tac-gia-tran-nam-dung-nguyen-van-mau 21/217

1.3. Mt s phương pháp gii bài toán v dãy s   21

sau đó chng minh dãy s ban đu có cùng gii hn. Tt nhiên, dãy s ph phiđưc xây dng t dãy s chính.

Ví d 1.26.  Dãy s  {an}  đưc xác đnh bi  a1  >  0, a2  >  0 và  an+1  = 2/(an +an−1). Chng minh rng dãy s  {an} hi t và tìm gii hn ca dãy s đó.

Gii.  Xét hai dãy

M n  = max{an, an+1, an+2, an+3}mn  = min{an, an+1, an+2, an+3}

Ta chng minh  M n là dãy s gim và  mn  là dãy s tăng. Tht vy, ta s chngminh   an+4 ≤   max{an+1, an+3}. T đây suy ra   M n+1   =   an+1   hoc   an+2   hocan+3  và rõ ràng khi đó   M n   = max

{an, an+1, an+2, an+3

} ≥  M n+1. Tht vt

nu   an+4 ≥   an+3   thì   2/(an+3 +  an+2) ≥   an+3  suy ra   2 ≥   (an+3  +  an+2)an+3.Khi đó   an+1   = 2/an+3 − an+2   = 2/an+3 − 2/(an+2  +  an+3) − an+2  + an+4   =2an+2/(an+3 + an+2)an+3 − an+2 +  an+4 ≥  an+4  suy ra đpcm. Vy ta đã chngminh đưc M n gim. Tương t  mn tăng. Hai dãy s này đu b chn nên hi t.Cui cùng, ta ch còn cn chng minh hai gii hn bng nhau.

Ví d 1.27.  Dãy s  {an}  đưc xác đnh bi  a1   >  0, a2  >  0  và   an+1   = √ 

an  +√ an−1. Chng minh rng dãy s  {an} hi t và tìm gii hn ca dãy s đó.

Gii.  Xét dãy s M n = max{an, an+1, 4}.Nu  M n  = 4 thì  an, an+1 ≤ 4, suy ra  an+2 ≤ 4, t đó M n+1 = 4.Nu  M n  = an+1  thì  an+1

 ≥ an, 4. Khi đó

 √ an

−1  =  an+1

−√ 

an+1

 ≥ √ 

an+1,

suy ra an+2  = √ an+√ an+1 ≤ √ an+√ an−1  = an+1 suy ra M n+1  = max{an+1, an+2, 4} =an+1.

Nu M n  =  an thì  an ≥ an+1, 4. Khi đó  an+2  =√ 

an +√ 

an+1 ≤ 2√ 

an. Suy raM n+1 ≤ an = M n.

Vy trong mi trưng hp thì  M n+1 ≤ M n, tc là dãy {M n} là dãy s gim.Do  M n b chn dưi bi  4  nên dãy này có gii hn. Ta chng minh gii hn nàybng  4. Thc vy, gi s gii hn là  M > 4. Khi đó vi mi   > 0, tn ti N  saocho vi mi n ≥ N  thì M  − < M n  < M  + . Chn n ∈ N  sao cho  M n+2  =  an+2

(theo các lp lun trên và do  M > 4  thì tn ti ch s  n  như vy). Ta có

M  − < M n+2  = an+2  =√ 

an +√ 

an−1  <  2√ 

M  +

hay M (M  − 4) − (2M  + 4 − ) <  0Mâu thun vì  M > 4  và   có th chn nh tuỳ ý.

Page 22: Dãy số - Giới hạn Tác giả: Trần Nam Dũng- Nguyễn Văn Mậu, 2007

8/13/2019 Dãy số - Giớ i hạn Tác giả: Trần Nam Dũng- Nguyễn Văn Mậu, 2007

http://slidepdf.com/reader/full/day-so-gioi-han-tac-gia-tran-nam-dung-nguyen-van-mau 22/217

1.3. Mt s phương pháp gii bài toán v dãy s   22

Phương pháp sai phân

Đ tính tng  n s hng đu tiên ca mt dãy s, mt trong nhng phươngpháp hiu qu nht là phương pháp sai phân: Đ tính tng  n s hng đu tiênca dãy s {an}, ta tìm hàm s   f (n)  sao cho   an   =   f (n + 1) − f (n). Khi đóa0 + · · · + an−1  =  f (n) − f (0).

Mt trong nhng ví d kinh đin chính là phương pháp mà Bernoulli vàcác nhà toán hc th k 18 đã đưa ra đ tìm công thc tính tng   S (k, n) =1k + 2k + · · · + nk . Dùng phương pháp h s bt đnh, h tìm đa thc  f k(n) saocho nk = f k(n+ 1)−f k(n) và t đó tìm đưc S (k, n) =  f k(n + 1)−f k(n). Phươngpháp này hiu qu hơn phương pháp xây dng công thc truy hi, vì đ tính  S kta không cn phi dùng đn các công thc tính S k−1, S k−2

Khi d đoán các hàm  f , ta có th s dng tích phân ri tương t hóa qua.Ví d tích phân ca đa thc bc  k  là đa thc bc  k + 1. Vy thì  ∆f k  = nk  suy

ra  f k  phi có bc  k  + 1.Tuy nhiên, khác vi tích phân, đôi khi các hàm ri rc không có "nguyên

hàm". Trong trưng hp đó ta không tính đưc tng mà ch có th đánh giá tngbng các bt đng thc.

Ví d 1.28.  Tìm phn nguyên ca tng  S  = 1/1 + 1/√ 

2 + · · · + 1/√ 

100.

Gii.   Ta cn tìm mt đánh giá cho S . Nhn xét rng hàm 1/√ 

x có nguyên hàmlà 2

√ x, ta xét hàm s  f (n) = 2

√ n. Khi đó  f (n + 1) − f (n) = 2

√ n + 1 − 2

√ n =

2/(√ 

n + 1 +√ 

n).Suy ra,  1/

√ n + 1  < f (n + 1) − f (n)   <   1/

√ n. T đó,   2(

√ 101 − 1)  < S <

2(√ 

100

−1) + 1, suy ra  [S ] = 18.

Ví d 1.29 (Đ đ ngh Toán quc t 2001).   Cho   x1, x2,...,xn   là các s thc bt kỳ. Chng minh rng 

x1/(1 + x21) + x2/(1 + x2

1 + x22) + · · · + xn/(1 + x2

1 + · · · + x2n) <

√ n.

Gii.   Đt v trái ca bt đng là A. Áp dng bt đng thc Bunhiacopsky tacó

A2 ≤ n[x21/(1 + x2

1)2 + x22/(1 + x2

1 + x22)2 + · · · + x2

n/(1 + x21 + · · · + x2

n)2]

Đ chng minh bt đng thc đu bài, ta ch cn chng minh

x21/(1 + x2

1)2 + x22/(1 + x2

1 + x22)2 + · · · + x2

n/(1 + x21 + · · · + x2

n)2 < 1.

Nhưng điu này là hin nhiên do bt đng thc

x2k/(1 + x2

1 + · · · + x2k)2 ≤ 1/(1 + x2

1 + · · · + x2k−1) − 1/(1 + x2

1 + · · · + x2k).

Page 23: Dãy số - Giới hạn Tác giả: Trần Nam Dũng- Nguyễn Văn Mậu, 2007

8/13/2019 Dãy số - Giớ i hạn Tác giả: Trần Nam Dũng- Nguyễn Văn Mậu, 2007

http://slidepdf.com/reader/full/day-so-gioi-han-tac-gia-tran-nam-dung-nguyen-van-mau 23/217

1.4. Mt s phương pháp xây dng h thng bài tp    23

Ví d 1.30.  Xét dãy s  {xn}n=1  cho bi:  xn+2  = [(n − 1)xn+1 + xn]/n. Chng minh rng vi mi giá tr ban đu  x1, x2, dãy s đã cho hi t. Tìm gii hn ca 

dãy như mt hàm s theo  x1, x2.Gii.   Ta có t công thc ca dãy s  xn+2 − xn+1  = −(xn+1 − xn)/n  = (xn −xn−1)/n(n − 1) = · · · = (−1)n(x2− x1)/n!. T đó suy ra  xn+2  = (xn+2 − xn+1) +(xn+1 − xn) + · · ·+ (x2 − x1) + x1 =  x1 + (x2 − x1)K n, trong đó  K n  = 1 − 1/1! +1/2! − · · · + (−1)n/n!. T đây suy ra dãy s có gii hn và gii hn đó bngx1 + (x2 − x1)/e.Câu hi:

1) Có th tng quát hóa bài toán trên như th nào?2) Hãy tìm sai phân ca các hàm s  arctan(n). T đó đt ra bài toán tính

tng tưng ng.3) Tìm sai phân ca hàm s   ln(n). T đó tìm đánh giá cho tng  1 + 1/2 +

· · · + 1/n.4) T công thc  sin3x = 3 sin x − 4sin3x có th lp ra công thc tính tng

nào?

1.4 Mt s phương pháp xây dng h thng bài tp

1.4.1 Xây dng dãy hi t bng phương trình

Có th xây dng dãy s hi t v mt s a  xut phát t mt phương trình cónghim là  a  theo cách sau:

Ví d 1.31.  Xét  a =√ 

2, α là nghim ca phương trình  α2 = 2. Ta vit li dưi dng 

α = 2/α ⇔   2α =  α  + 2/α ⇔   α = (α + 2/α)/2

và ta thit lp dãy s  xn   tho mãn  x0  = a, xn+1  = (xn + 2/xn)/2. Nu dãy này hi t thì gii hn s là 

 √ 2. Tương t như vy, ta có th xây dng đưc dãy s 

tin v căn bc  k ca  m như sau:

x0  =  a, xn+1  = (xn + m/xk−1n   )/2

Cũng vi gii hn cn đn là √ 

2, ta có th xây dng mt dãy s khác theo"phong cách" như vy:

x0

 = a, xn+1

 = 1 + xn −

x2

n/2

Tt nhiên, trong tt c các ví d trên, ta ch có đưc phương trình vi nghimtheo ý mun khi đã chng minh đưc s hi t ca dãy s. Vì vy, cn cn thnvi cách thit lp bài toán kiu này. Ví d, vi dãy s  xn+1  = 1 + xn − x2

n/2 thìkhông phi vi  x0 nào dãy cũng hi t, và không phi lúc nào gii hn cũng là.

Page 24: Dãy số - Giới hạn Tác giả: Trần Nam Dũng- Nguyễn Văn Mậu, 2007

8/13/2019 Dãy số - Giớ i hạn Tác giả: Trần Nam Dũng- Nguyễn Văn Mậu, 2007

http://slidepdf.com/reader/full/day-so-gioi-han-tac-gia-tran-nam-dung-nguyen-van-mau 24/217

1.4. Mt s phương pháp xây dng h thng bài tp    24

Mt cách tng quát, ta có th dùng phương pháp tìm nghim xp x Newtonđ xây dng các dãy s. Đ tìm nghim ca phương trình F (x) = 0, phương pháp

Newton đ ngh chn x0 tương đi gn nghim đó và xây dng dãy truy hi

xn+1  =  xn − F (xn)/F (xn)

khi đó dãy xn  s dn đn nghim ca phương trình  F (x) = 0.

Ví d 1.32.  Xét hàm s   F (x) =  x2 − 2, thì   F (x)/F (x) = (x2 − 2)/2x và ta đưc dãy s  xn+1  = (xn + 2/xn)/2.Xét hàm s  F (x) = x3 − x thì  F (x)/F (x) = (x3 − x)/(3x2 − 1) và ta đưc dãy s 

xn+1  = 2x3n/(3x2

n − 1)

1.4.2 Xây dng dãy truy hi t cp nghim ca phương trìnhbc 2

Chúng ta thy, t hai nghim ca mt phương trình bc 2 có th xây dngra các dãy truy hi tuyn tính bc 2 (kiu dãy s Fibonacci). Tương t như th,có th xây dng các dãy truy hi tuyn tính bc cao t nghim ca các phươngtrình bc cao. Trong phn này, chúng ta s đi theo mt hưng khác: xây dngcác dãy truy hi phi tuyn bc nht t cp nghim ca phưng trình bc 2.

Xét phương trình bc 2:  x2 − mx ± 1 = 0 có hai nghim là  α  và β . Xét mt sthc  a bt kỳ. Xét dãy s xn =  a(α2n + β 2

n). Khi đó x2

n  =  a2(α2n+ + β 2n+1

+2) =axn+1 + 2a2, t đó suy ra dãy s  xn tho công thc truy hi:  xn+1  =  x2

n/a − 2a.Ví d chn a  = 1/2, m = 4, ta có bài toán: Tìm công thc tng quát ca dãy

s  xn đưc xác đnh bi  x0 = 2, xn+1  = 2x2n − 1.Tương t như vy, nu xét  xn   =  a(α3n + β 3

n)  thì  x3

n  =  a3(α3n+1+ β 3

n+1 ±3(α3n + β 3

n) =  a2(xn+1 ± 3xn). T đó suy ra dãy s  xn  tho công thc truy hi

xn+1  = x3n/a2 − (±3xn).

Ví d xét   α, β   là hai nghim ca phương trình  x2 − 4x − 1 = 0, a   = 1/4,ta đưc bài toán: Tìm công thc tng quát ca dãy s   xn  đưc xác đnh bix0  = 1, xn+1  = 16x3

n + 3xn. Hoàn toàn tương t, có th xây dng các dãy truyhi phi tuyn dng đa thc bc 4, 5. Bng phép di trc, ta có th thay đi dngca các phương trình này.

Ví d 1.33.  nu trong dãy  x0  = 2, xn+1  = 2x2n − 1 ta đt  xn  = y n − 1/2 thì ta 

đưc dãy  y n  tho:  y 0 = 5/2, y n+1 = 2(y 

2

n − y n).Nu α, β  là các s thc thì trong hai s có ít nht mt s có tr tuyt đi ln

hơn 1, vì vy dãy s không hi t (Tr trưng hp hai nghim đi nhau và dãylà dãy hng). Tuy nhiên, nu chn  α, β   là cp s phc liên hp có môđun nhhơn hay bng 1, ta có th to ra các dãy tun hoàn hoc dãy hi t. Chú ý rng

Page 25: Dãy số - Giới hạn Tác giả: Trần Nam Dũng- Nguyễn Văn Mậu, 2007

8/13/2019 Dãy số - Giớ i hạn Tác giả: Trần Nam Dũng- Nguyễn Văn Mậu, 2007

http://slidepdf.com/reader/full/day-so-gioi-han-tac-gia-tran-nam-dung-nguyen-van-mau 25/217

1.4. Mt s phương pháp xây dng h thng bài tp    25

chn  α, β   đây chính là chn  m và cũng chính là chn  x0. Do đó tính cht cadãy s s ph thuc rt nhiu vào  x0.

Ví d vi dãy s tho xn+1  = 2x2n − 1, nu x0  = 2 thì xn  = [(2 + √ 3)2n

+ (2 −√ 3)2

n]/2; nu  x0  = 1 thì  xn là dãy hng; nu  x0 = cos α thì  xn  = cos(2nα).

Câu hi:1) Xét xem vi nhng  a, b, c nào thì phương trình sai phân xn+1  =  ax2

n+bxn+cgii đưc bng phương pháp trên?

2) Hãy tìm dng ca các dãy truy hi to đưc bng cách xét xn  =  a(αkn+β kn

)vi k = 4, 5.

1.4.3 Xây dng các dãy s nguyên t li gii các phương trìnhnghim nguyên

Mt dãy truy hi tuyn tính vi h s nguyên và các s hng đu đu nguyêns cha toàn s nguyên. Đó là điu hin nhiên. Th nhưng có nhng dãy s màtrong công thc truy hi có phân s, thm chí có c căn thc nhưng tt c cács hng ca nó vn nguyên. Đy mi là điu bt ng. Tuy nhiên, nu xem xétk, ta có th thy chúng có mt mi quan h rt trc tip.

Chúng ta hãy bt đu t bài toán quen thuc sau: Chng minh rng mi shng ca dãy s {an} xác đnh bi  a0 = 1, an+1 = 2an +

 3a2

n − 2 đu nguyên.Chuyn v và bình phương công thc truy hi, ta đưc

a2n+1 − 4an+1an + 4a2

n = 3a2n − 2

⇔ a2n+1 − 4an+1an + a2

n + 2 = 0

Thay n  bng  n − 1, ta đưc

a2n − 4anan−1 + a2

n−1 + 2 = 0

T đây suy ra an−1, an+1 là hai nghim ca phương trình

x2 − 4anx + a2n + 2 = 0

Suy ra:  an+1 +  an−1  = 4an  hay  an+1  = 4an − an−1. T đây suy ra tt c các shng trong dãy đu nguyên.

C công thc ban đu ln công thc h qu  an+1  = 4an − an−1  đu gi chochúng ta đn vi phương trình Pell. Qu tht là có th xây dng hàng lot dãy

s tương t bng cách xét phương trình Pell.Xét phương trình  x2 − Dy 2 = k . Gi s phương trình có nghim không tm

thưng  (x0, y 0) và (α, β ) là nghim cơ s ca phương trình x2−Dy 2 = 1. Khi đó,nu xét hai dãy {xn}, {y n} xác đnh bi  xn+1  =  αxn + βDy n, y n+1  =  βxn + αy nthì xn, y n là nghim ca  x2 − Dy 2 = k.

Page 26: Dãy số - Giới hạn Tác giả: Trần Nam Dũng- Nguyễn Văn Mậu, 2007

8/13/2019 Dãy số - Giớ i hạn Tác giả: Trần Nam Dũng- Nguyễn Văn Mậu, 2007

http://slidepdf.com/reader/full/day-so-gioi-han-tac-gia-tran-nam-dung-nguyen-van-mau 26/217

1.4. Mt s phương pháp xây dng h thng bài tp    26

T h phương trình trên, ta có th tìm đưc

xn+1  = αxn + β  D(x2

n − k);   y n+1 = αy n + β  k + Dy 2

n

và như vy đã xut hin hai dãy s nguyên đưc cho bi mt công thc khôngnguyên.

Ví d, vi  D  = 4a(a + 1), k = 1 thì ta có  x0  =  α  = 2a + 1, y 0  =  β  = 1. Tađưc hai dãy s nguyên sau đây:

x0  = 2a + 1, xn+1  = 2a + 1 + 

4a(a + 1)(x2n − 1)

y 0  = 1, y n+1  = 2a + 1 + 

4a(a + 1)y 2n + 1

Cui cùng, chú ý rng ta có th to ra mt kiu dãy s khác t kt qu an−1, an+1

là hai nghim ca phương trình

x2 − 4anx + a2n + 2 = 0

trên đây: Theo đnh lý Viet thì  an+1an−1  = a2n + 2, suy ra

an+1  = (a2n + 2)/an−1

và ta có bài toán: Cho dãy s {an}  xác đnh bi   a0   = 1, a1   = 3   và   an+1   =(a2

n + 2)/an−1. Chng minh rng  an nguyên vi mi  n.

1.4.4 Xây dng dãy s là nghim ca mt h phương trình phthuc bin  n

Xét mt h phương trình F (n, x) = 0. Nu vi mi n, phương trình F (n, x) =0 có nghim duy nht trên mt min  D nào đó thì dãy s  xn  đã đưc xác đnh.T mi liên h gia các hàm  F (n, x), dãy s này có th có nhng tính cht rtthú v.

Ví d 1.34.  Vi mi s t nhiên  n ≥ 3, gi  xn   là nghim dương duy nht ca phương trình  xn−x2−x−1 = 0. Chng minh rng  lim xn  = 1 và tìm  lim n(xn−1).

Ví d 1.35.  Chng minh rng vi mi  n nguyên dương, phương trình 

1/x + 2/(x − 1)+ 2/(x − 4) + · · · + 2/(x − n2) = 0

có nghim duy nht  xn thuc khong  (0, 1). Tìm  limn

→∞ xn.

Ví d 1.36.  Chng minh rng vi mi  n nguyên dương, phương trình 

1/x + 2/(x − 1)+ 2/(x − 4) + · · · + 2/(x − n2) = 0

có nghim duy nht  xn thuc  (0, 1). Tìm  limn→∞ xn.

Page 27: Dãy số - Giới hạn Tác giả: Trần Nam Dũng- Nguyễn Văn Mậu, 2007

8/13/2019 Dãy số - Giớ i hạn Tác giả: Trần Nam Dũng- Nguyễn Văn Mậu, 2007

http://slidepdf.com/reader/full/day-so-gioi-han-tac-gia-tran-nam-dung-nguyen-van-mau 27/217

1.5. Lý thuyt dãy s dưi con mt toán cao cp    27

Đ to ra các phương trình có nghim duy nht trên mt khong nào đó, cóth s dng tng ca các hàm đơn điu. Riêng vi hàm đa thc ta có th s dng

quy tc Đ-các v s nghim dương ca phương trình: Nu dãy các h s caphương trình đi du  k ln thì phương trình có không quá  k  nghim dương.Ví d phương trình x4 − x2 − nx − 1 = 0 có nghim dương duy nht  x0, còn

phương trình  x4 − x2 + nx − 1 = 0 có nhiu nht hai nghim dương.Khi xây dng các hàm  F (n, x), có th s dng công thc truy hi. Như trong

ví d trên thì F (n + 1, x) =  F (n, x) + 1/(x − n − 1). Xây dng  F (n, x) kiu này,dãy nghim  xn s d có nhng quy lut thú v hơn. Ví d, vi dãy s trên, ta cóF (n + 1, xn) = F (n, xn) + 1/(xn − n − 1) <  0. T đây, do  F (n + 1, 0+) = ∞ tasuy ra xn+1  nm gia 0 và  xn, tc dãy  xn gim.Câu hi:

1) Có th xây dng dãy s nào vi h hàm s  F (x) = x(x − 1) . . .(x − n)?

2) Cho  0  < a1  < a2  < · · · < an < · · ·  là mt dãy s dương tăng nghiêm ngt.Xét h phương trình 1/x + 1/(x1−a1) + · · ·+ 1/(x−an) = 0 có nghim duy nhtxn thuc (0, a1). Khi nào thì  xn  dn v 0 khi  n dn đn vô cùng?

1.5 Lý thuyt dãy s dưi con mt toán cao cp

1.5.1 Ri rc hóa các khái nim và đnh lý ca lý thuyt hàmbin s thc

Dãy s là hàm s, do đó nó có đy đ các tính cht chung ca hàm s. Tuynhiên, do tính cht đc bit ca  N , mt s khái nim như đo hàm, tích phânkhông đưc đnh nghĩa cho các dãy s. Nhưng thc ra, dãy s cũng có các kháinim tương ng vi các khái nim này. Bng cách so sánh và phép tương t, tacó th tìm đưc nhng đnh lý thú v ca lý thuyt dãy s. Đó là quá trình rirc hóa.

Ri rc hóa ca đo hàm f (x) chính là sai phân ∆xn = xn−xn−1 ca dãy s.Cũng như đo hàm ca hàm bin s thc, sai phân dùng đ xét tính tăng gimca dãy s. Tương t như vy, ta đnh nghĩa sai phân cp 2 và dùng đ đo tínhli lõm ca dãy. Ri rc hóa ca khái nim tích phân chính là khái nim tng:S (xn) =  x0 + · · ·+ xn. Hai khái nim này ngưc nhau:  ∆(S (xn)) =  xn, S (∆xn) =xn.

Ví d 1.37 (Đnh lý Stolz).  Xét hai dãy s  {xn} và  {y n} trong đó  {y n} là dãy 

s dương tăng và dn đn vô cùng. Th thì  lim xn/y n = lim(xn−xn−1)/(y n−y n−1)vi gi thit là gii hn v phi tn ti. (So sánh vi quy tc L’Hopitale)

Chng minh:  Đt   lim(xn − xn−1)/(y n − y n−1) =   A. Vi mi   >   0  tn tiN 1   sao cho vi mi   n ≥   N 1   ta có |(xn − xn−1)/(y n − y n−1) − A|   < , suy ra

Page 28: Dãy số - Giới hạn Tác giả: Trần Nam Dũng- Nguyễn Văn Mậu, 2007

8/13/2019 Dãy số - Giớ i hạn Tác giả: Trần Nam Dũng- Nguyễn Văn Mậu, 2007

http://slidepdf.com/reader/full/day-so-gioi-han-tac-gia-tran-nam-dung-nguyen-van-mau 28/217

1.5. Lý thuyt dãy s dưi con mt toán cao cp    28

A − < (xn − xn−1)/(y n − y n−1) < A + . T đây, do  y n  là dãy tăng nên ta có

(A − )(y N 1 − y N 1−1) < xN 1 − xN 1−1  < (A + )(y N 1 − y N 1−1). . .

(A − )(y n − y n−1) < xn − xn−1  <  (A + )(y n − y n−1)

Cng các bt đng thc trên li, ta đưc

(A − )(y n − y N 1−1) < xn − xN 1−1  <  (A + )(y n − y N 1−1)

Chia hai v cho  y n, ta đưc

A −  + [xN 1 − (A − )y N 1−1]/y n  < xn/y n < A +  + [xN 1 − (A + )y N 1−1]/y n

Vì  y n  dn đn vô cùng nên tn ti  N 2 > N 1 sao cho

[xN 1 − (A − )y N 1−1]/y n > − và [xN 1 − (A + )y N 1−1]/y n  <

vi mi  n ≥ N 2. Khi đó vi mi  n ≥ N 2 ta có  A − 2 < xn/y n  < A + 2 và điunày có nghĩa là  lim xn/y n =  A.Câu hi: Điu kin  y n tăng và dn đn vô cùng có cn thit không?

Ví d 1.38.  Chng minh rng nu dãy s {xn} tho mãn điu kin  xn+1−2xn +xn−1 ≥ 0 và  k1, k2, . . . , kr là các s t nhiên tho mãn điu kin  k1+k2+· · ·+kr  =r.k thì 

xk1  + · · · + xkr ≥ r.xk

(So sánh vi bt đng thc Jensen)

Ví d 1.39.  Cho dãy s  {xn}   tho mãn điu kin  xk+1 − 2xk  +  xk−1 ≥  0  vi mi   k   = 1, . . . , n. Ngoài ra  x0   =   xn+1   = 0. Chng minh rng  xk ≤   0  vi mi k = 1, . . . , n.

(Đo hàm bc 2 không âm, suy ra đo hàm bc nht là hàm tăng và ch cónhiu nht 1 nghim, suy ra chiu bin thiên ca hàm s ch có th là 0 gim →cc tiu ri tăng → 0)

Ví d 1.40.  Cho dãy s dương  {an}. Bit rng tn ti gii hn 

limn→∞

nk=1

1ak

= A < ∞.

Đt  sn  = a1 + a2 + · · · + an. Chng minh rng tng  limn→∞n

k=1 k2ak/s2k  cũng có gii hn hu hn khi  n → ∞.

Page 29: Dãy số - Giới hạn Tác giả: Trần Nam Dũng- Nguyễn Văn Mậu, 2007

8/13/2019 Dãy số - Giớ i hạn Tác giả: Trần Nam Dũng- Nguyễn Văn Mậu, 2007

http://slidepdf.com/reader/full/day-so-gioi-han-tac-gia-tran-nam-dung-nguyen-van-mau 29/217

1.5. Lý thuyt dãy s dưi con mt toán cao cp    29

Gii.   Dch sang ngôn ng hàm s, ta có bài toán sau "Nu f (x) là hàm s tăng

t  R+

vào  R

+

và tn ti tích phân suy rng  ∞0 dx

f (x) thì cũng tn ti tích phân ∞0

x2f (x)dx

F 2(x)  trong đó F (x) là nguyên hàm ca  f (x)". Bài này có th gii bng

phương pháp tích phân tng phn như sau:   A0

xdx

F (x) =

1

2

   A0

d(x2)

F (x) =

1

2

  x2

F (x)

A0

+

   A0

x2f (x)dx

F 2(x)

như vy ch cn chng minh tn ti  ∞

0

xdx

F (x) và  limx→∞

x2

F (x).

Câu hi:1) Đnh lý Rolle có dng ri rc như th nào?2) Công thc tính tích phân tng phn có dng ri rc như th nào?

1.5.2 Phương pháp hàm sinh và bài toán tìm s hng tng quát

Cho dãy s a0, a1, . . . , an, . . . Hàm sinh F (x) ca dãy s này là biu thc hìnhthc

F (x) =  a0 + a1x + · · · + anxn + · · ·Các phép toán trên hàm sinh đưc thc hin mt cách t nhiên và chúng takhông quan tâm đn tính cht gii tích ca chúng (bán kính hi t ca chuitương ng có th bng 0). Phép toán đc bit nht ca hàm sinh là phép nhân:

Nu F (x), G(x) là hàm sinh ca các dãy {

an

},

{bn

} tương ng thì  F (x).G(x)

là hàm sinh ca dãy {cn} trong đó  cn = n0 aibn−i.Sơ đ ng dng ca hàm sinh vào bài toán tìm s hng tng quát ca dãy s

như sau: Gi s ta cn tìm s hng tng quát ca dãy s {an} cho bi mt côngthc truy hi nào đó. Ta thit lp hàm sinh   F (x)  ca {an}. Da vào h thctruy hi, ta tìm đưc mt phương trình cho  F (x), gii phương trình, ta tìm đưcF (x). Khai trin  F (x)  theo lu tha  x (Khai trin Taylor), ta tìm đưc  an  vimi n.

Ví d 1.41.  Tìm s hng tng quát ca dãy s  {an} xác đnh bi:  a0  = 3, a1 =2, an+2  = 5an+1 − 6an.

Gii.  Xét hàm sinh  F (x) =  a0 + a1x + a2x2 + · · · + an+2xn+2 + · · · . Vi mi  n

t nhiên, ta thay  an+2  bng  5an+1 − 6an thì đưcF (x) = a0 + a1x + (5a1 − 6a0)x2 + · · · + (5an+1 − 6an)xn+2 + · · ·

= a0 + a1x + 5x(a1x + · · · + an+1xn+1 + · · · ) − 6x2(a0 + a1x + · · · + anxn + · · · )

= a0 + a1x + 5x(F (x) − a0) − 6x2F (x)

Page 30: Dãy số - Giới hạn Tác giả: Trần Nam Dũng- Nguyễn Văn Mậu, 2007

8/13/2019 Dãy số - Giớ i hạn Tác giả: Trần Nam Dũng- Nguyễn Văn Mậu, 2007

http://slidepdf.com/reader/full/day-so-gioi-han-tac-gia-tran-nam-dung-nguyen-van-mau 30/217

1.5. Lý thuyt dãy s dưi con mt toán cao cp    30

Suy ra F (x) = (3 − 13x)/(6x2 − 5x + 1) = 7/(1 − 2x) − 4/(1 − 3x) = 7(1 + 2x +(2x)2 +

· · ·+ (2x)n +

· · ·)

−4(1 + 3x + (3x)2 +

· · ·+ (3x)n +

· · ·)

T đó  an  = 7.2n − 4.3n.Trên lý thuyt, khi tìm đưc  F (x), ta phi dùng công thc Taylor đ tìm khaitrin ca  F (x). Đây là mt bài toán phc tp. Tuy nhiên, trong nhiu trưnghp, công thc nh thc Newton tng quát dưi đây đã đ dùng:

(1 + x)α = 1 + αx + [α(α − 1)/2]x2 + · · · + [α(α − 1) . . .(α − n + 1)/n!]xn + · · ·Ví d 1.42.  Dãy s  {an}  xác đnh bi  a0  = 1, a0an +  a1an−1 + · · · + ana0  = 1vi mi  n. Hãy tìm công thc tng quát ca  an.

Gii.   Xét hàm sinh   F (x) =   a0  +  a1x +  a2x2 + · · · + anxn + · · · . T côngthc truy hi ta suy ra  F 2(x) = 1 + x  +  x2 + · · · + xn + · · ·  = (1 − x)−1. T đây F (x) = (1

−x)−1/2. Khai trin  F (x)  theo công thc Newton, ta tìm đưc

an  = C n2n/22n.

1.5.3 Đi s tuyn tính và phương trình sai phân

Trong phn trên, chúng ta đã s dng phương pháp hàm sinh đ gii bài toántìm công thc tính s hng tng quát ca mt dãy s. Trong phn này, ta s xemxét cu trúc nghim ca phương trình sai phân dưi góc đ đi s tuyn tính.

Xét phương trình sai phân thun nht:  xn+k   =  a1xn+k−1  + · · · + akxn. Dthy rng nu dãy s {xn}, {y n} tho mãn phương trình này thì {axn + by n} cũngtho mãn phương trình vi mi  a, b. Như vy tp hp tt c các dãy s tho mãnphương trình sai phân trên lp thành mt không gian véc-tơ. Hơn th, ta có đnhlý:

Đnh lý 1.12.  Tp hp tt c các dãy s tho mãn phương trình sai phân 

xn+k  = a1xn+k−1 + · · · + akxn.

là mt không gian véctơ  k chiu.

Chng minh đnh lý này khá đơn gin: Dãy s s hoàn toàn xác đnh nubit  k s hng đu tiên. Gi {xi

n}(i = 0, k − 1) là dãy s có  xi j  = 0 nu   i = j  và

xii  = 1. Khi đó có th chng minh d dàng rng các dãy {x1

n}, . . . , {xkn} đc lp

tuyn tính và vi mi dãy {xn} ta có

xn  =  x0x0n + · · · + xk−1xk−1

n

Như th, cu trúc nghim ca phương trình sai phân tuyn tính thun nhtlà đã rõ. Ta ch cn tìm mt cơ s nào đó ca không gian nghim là có th môt đưc tt c các nghim ca phương trình sai phân. Cơ s mà chúng ta đưa ra trên không có tính tưng minh, do đó khó có th s dng trong vic thit lpcông thc tng quát. Đ xây dng mt cơ s khác tt hơn, ta có đnh lý:

Page 31: Dãy số - Giới hạn Tác giả: Trần Nam Dũng- Nguyễn Văn Mậu, 2007

8/13/2019 Dãy số - Giớ i hạn Tác giả: Trần Nam Dũng- Nguyễn Văn Mậu, 2007

http://slidepdf.com/reader/full/day-so-gioi-han-tac-gia-tran-nam-dung-nguyen-van-mau 31/217

1.5. Lý thuyt dãy s dưi con mt toán cao cp    31

Đnh lý 1.13.  Nu  λ là nghim bi  r ca phương trình đc trưng 

xk

−a1

xk

−1

− · · · −ak

 = 0

thì các dãy s  {λn}, . . . , {nr−1λn}   tho mãn phương trình sai phân   xn+k   =a1xn+k−1 + · · · + akxn.

Vi đnh lý này, ta có th tìm đ  k  dãy s tưng minh to thành mt cơ sca không gian nghim.

Cui cùng, nu ta gp phương trình sai phân tuyn tính không thun nht

xn+k  = a1xn+k−1 + · · · + akxn + f (n).

thì nghim tng quát ca phương trình này s có dng là tng ca nghim tngquát ca phương trình sai phân tuyn tính thun nht tương ng vi mt nghimriêng ca phương trình không thun nht.

Đ tìm nghim riêng, ta vn dng phương pháp hoàn toàn tương t như trongphưng trình vi phân: Nu  f (n)  là đa thc thì ta  xn  tìm dưi dng đa thc, làhàm mũ thì tìm dưi dng hàm mũ... đây, trưng hp cơ s là nghim kép caphương trình đc trưng cũng đưc x lý tưng t như trong phương trình vi phân.

1.5.4 S dng xp x trong d đoán kt qu 

Trong nhiu trưng hp, d đoán đưc kt qu đã là mt na, thm chí 2/3li gii. Chúng ta đã gp nhiu tình hung là li gii đu tiên thu đưc mt cáchrt khó khăn, nhưng sau đó thì hàng lot li gii đp hơn, gn hơn xut hin. Vìsao chúng ta không nghĩ ngay đưc nhng li gii đp? Vì chúng ta chưa bit đáp

s. Khi bit ri thì có th đnh hưng d dàng hơn rt nhiu. Dưi đây, chúng tas xem xét mt s ng dng ca xp x trong vic d đoán kt qu.Trong ví d v dãy s  xn+1   = sin(xn), chúng ta đã áp dng đnh lý trung

bình Cesaro đ tìm gii hn √ 

nxn, mc dù dãy s không có dng quen thucxn+1   =  xn ± (xn)α. Th nhưng, nu đ ý rng  xn →  0 khi  n → ∞, mà ti lâncn 0 thì  sin x ∼ x − x3/6 thì ta s thy tính quy lut ca kt qu đã tìm đưc trên.

Vi phương pháp tương t, ta có th thy dãy dng  xn+1   =   xn ± (xn)α hàng lot các dãy s có b ngoài khác hn như:   xn+1   = ln(1 + xn), xn+1   =xn cos xn, xn+1   =   arctg(xn) . . . (Dĩ nhiên, phi kim tra điu kin   xn →   0  khin → ∞).

Ta cũng có th gii thích đưc vì sao trong bài toán  an+1

 =  an

+1/√ 

an  phn

trên, ta đã tìm đưc s 3/2. Ta có an+1  =  an+1/√ an  =  an(1+1/a3/2n   ). Vì an → ∞

khi   n → ∞  nên vi mi   β  ta có   aβ n+1   =   aβ 

n(1 + 1/a3/2n   )β  ∼   aβ 

n(1 +  β/a3/2n   ) =

aβ n + βa

β −3/2n   . Do đó đ hiu s này xp x hng s, ta chn b  = 3/2.

Ta xét mt ví d khác

Page 32: Dãy số - Giới hạn Tác giả: Trần Nam Dũng- Nguyễn Văn Mậu, 2007

8/13/2019 Dãy số - Giớ i hạn Tác giả: Trần Nam Dũng- Nguyễn Văn Mậu, 2007

http://slidepdf.com/reader/full/day-so-gioi-han-tac-gia-tran-nam-dung-nguyen-van-mau 32/217

1.6. Bài tp    32

Ví d 1.43 (ĐHSP, 2000).  Cho dãy s  {an} xác đnh bi: a1 =  a2 = 1, an+1  =an + an−1/n(n + 1). Chng minh rng dãy 

 {an

} có gii hn.

Gii.  D thy {an} là dãy tăng. Vì vy ta ch cn chng minh dãy {an} b chntrên. Ta có

an+1  = an + an−1/n(n + 1) < an[1 + 1/n(n + 1)]

T đây suy ra

an+1  < [1+ 1/n(n + 1)] . . .[1 + 1/2.3]a2 = [1 + 1/n(n + 1)] . . .[1 + 1/2.3]

Như vy ta ch cn chng minh tích  [1 + 1/n(n + 1)] . . . [1 + 1/2.3] b chn. Ktqu này không phc tp và có th chng minh hoàn toàn sơ cp. Tuy nhiên,nhng kinh nghim v dãy s  1/n(n + 1)  gi cho chúng ta ti mi quan h giatích trên và tng 1/2.3 + · · ·+ 1/n(n + 1). Theo hưng đó, chúng ta có th đưa ra

mt kt qu tng quát hơn và kt qu đó đưc d đoán t vic s dng xp x.Gi s rng {xn} là dãy s thc sao cho tng x1 + · · ·+xn có gii hn hu hnkhi n → ∞. Khi đó  xn → 0 khi  n → ∞. Vì vy, vi  n  đ ln thì  xn ∼ ln(1+ xn).Do đó tng ln(1 + x1) + · · ·+ln(1+ xn) cũng có gii hn hu hn khi  n → ∞ vàcó nghĩa là tích  (1 + x1) . . . (1 + xn) cũng vy. Ta có đnh lý

Đnh lý 1.14.  Cho dãy s thc  {xn}. Khi đó nu tng  x1 + · · · + xn có gii hn hu hn khi   n → ∞  thì tích   (1 +  x1) . . . (1 +  xn)  cũng có gii hn hu hn khi n → ∞.

Câu hi:1) Mnh đ đo ca đnh lý trên có đúng không?2) Cho  n > 3  và  xn  là nghim dương duy nht ca phương trình  xn

−x2

−x − 1 = 0. Có th d đoán đưc  limn→∞ n(xn − 1)?

1.6 Bài tp

Bài 1.1 (Canada 1998).  Cho m là s nguyên dương. Xác đnh dãy  a0, a1, a2, . . .như sau:  a0  = 0, a1  =  m và  am+1  =  m2an − an−1  vi  n  = 1, 2, . . . Chng minh rng vi mi cp sp th t các s t nhiên  (a, b) vi  a ≤ b là nghim ca phương trình  (a2 + b2)/(ab + 1) = m2 khi và ch khi  (a, b) = (an, an+1) vi  n là mt s t nhiên nào đó.

Bài 1.2 (Bulgari 1978).  Cho dãy s  {an} xác đnh bi  an+1  = (a2n + c)/an−1.

Chng minh rng nu  a0, a1 và  (a2

0 + a2

1 + c)/a0a1 là s nguyên thì  an nguyên vi mi  n.

Bài 1.3.  Trong mt dãy vô hn các s nguyên dương, mi mt s hng sau ln hơn s hng trưc đó hoc là 54 hoc là 77. Chng minh rng trong dãy này tn ti s hng có hai ch s tn cùng ging nhau.

Page 33: Dãy số - Giới hạn Tác giả: Trần Nam Dũng- Nguyễn Văn Mậu, 2007

8/13/2019 Dãy số - Giớ i hạn Tác giả: Trần Nam Dũng- Nguyễn Văn Mậu, 2007

http://slidepdf.com/reader/full/day-so-gioi-han-tac-gia-tran-nam-dung-nguyen-van-mau 33/217

1.6. Bài tp    33

Bài 1.4 (Séc-Slovakia 1997).  Chng minh rng tn ti dãy s tăng  {an}∞n=1

các s nguyên dương sao cho vi mi s t nhiên  k, dãy 

 {k + an

} cha hu hn 

s nguyên t.Hưng dn: Dùng đnh lý Trung hoa v s dư.

Bài 1.5 (Putnam 1995).   Đt   S (α) =  {[nα]|n   = 1, 2, 3, . . .}. Chng minh rng tp hp các s nguyên dương   N ∗  không th phân hoch thành 3 tp hpS (α), S (β ), S (γ ).

Bài 1.6 (Putnam 1999). Dãy s {an}n=1 đưc xác đnh bi  a1 = 1, a2 = 2, a3 =24 và vi  n ≥ 4.

an = (6a2n−1an−3 − 8an−1a2

n−2)/an−2an−3

Chng minh rng vi mi  n, an là s nguyên chia ht cho  n.

Bài 1.7.  Trong dãy s nguyên dương  {ak}k=1 tng ca 10 s hng đu tiên bng 100, còn t  a11, mi   an  bng s các ch s   i < n  sao cho  ai +  i ≥ n. Bit rng a11  = 10. Chng minh rng k t mt ch s nào đó, tt c các s hng ca dãy bng nhau.

Bài 1.8 (Balkan).  Cho  x0 ≤  x1 ≤  x2 ≤ · · · ≤  xn ≤ · · ·  là dãy s không gim các s t nhiên sao cho vi mi s t nhiên  k, s các s ca dãy này không vưt quá  k  là hu hn (và ký hiu là  y k). Chng minh rng vi mi  m, n

n0

xi +m0

y i ≥ (n + 1)(m + 1)

Bài 1.9 (Bulgari 87).   Xét dãy s  {xn}  xác đnh bi   x1   =   x2   = 1, xn+2   =14xn+1 − xn − 4. Chng minh rng vi mi   n, xn   là bình phương ca mt s nguyên.

Hưng dn:  Xét dãy   u1   =   u2   = 1, un+2   = 4un+1 − un. Chng minh rngun+2un − u2

n+1  = 2 sau đó chng minh rng  xn  = u2n. Có th dùng ý tưng bài

này đ xây dng các bài toán khác như th nào?

Bài 1.10 (Canada 1988).  Cho hai dãy s  {xn}, {y n}  xác đnh bi   xn+1   =4xn − xn−1, x0  = 0, x1  = 1  và   y n+1   = 4y n − y n−1, y 0   = 1, y 1  = 2. Chng minh rng vi mi  n, y 2n = 3x2

n + 1.

Bài 1.11 (Canada 1993).   Cho  y 1, y 2, y 3, . . . là dãy s xác đnh bi  y 1  = 1  và 

vi mi s nguyên dương  k

y 4k  = 2y 2k, y 4k+1 = 2y 2k + 1, y 4k+2 = 2y 2k+1 + 1, y 4k+3 = 2y 2k+1

Chng minh rng dãy s  y 1, y 2, y 3 . . . nhn tt c các giá tr nguyên dương, mi giá tr đúng mt ln.

Page 34: Dãy số - Giới hạn Tác giả: Trần Nam Dũng- Nguyễn Văn Mậu, 2007

8/13/2019 Dãy số - Giớ i hạn Tác giả: Trần Nam Dũng- Nguyễn Văn Mậu, 2007

http://slidepdf.com/reader/full/day-so-gioi-han-tac-gia-tran-nam-dung-nguyen-van-mau 34/217

1.6. Bài tp    34

Bài 1.12.   Gi s rng   sn   là dãy s nguyên dương tho mãn điu kin   0 ≤sn+m

 −sn

 −sm

 ≤ K   vi   K   là mt s nguyên dương cho trưc. Vi s nguyên 

dương  N  có tn ti các s thc  a1, a2, . . . , aK  sao cho

sn  = [a1n] + · · · + [aK n]  vi mi   n = 1, 2, ...N ?

Bài 1.13.  Cho  a1  = 1, b1 = 2, c1 = 3. Gi  S (n) là tp hp các s nguyên dương ai, bi, ci vi  i ≤ n. Xây dng  an, bn, cn  như sau:

an+1  = s nguyên dương nh nht không thuc  S (n);bn+1  = s nguyên dương nh nht không thuc  S (n) và khác  an+1;cn+1  =  an+1 + bn+1;

Gi  dk  là dãy tăng các ch s  n sao cho  bn  =  an + 2. Chng minh rng a)  dk/k → 6  khi  k dn đn vô cùng b) Nu  B  là s nguyên thì  (dk

−6k)/2 =  B  vi vô s các ch s  k.

Bài 1.14 (AMM).  Các dãy s  {an}, {bn}, {cn}  đưc xác đnh như sau:  a1   =1, b1 = 2, c1 = 4 và 

an = s nguyên dương nh nht không thuc {a1, . . . , an−1, b1, . . . , bn−1, c1, . . . , cn−1}bn = s nguyên dương nh nht không thuc {a1, . . . , an−1, an, b1, . . . , bn−1, c1, . . . , cn−1}cn = 2bn+n−an . Hãy chng minh hoc ph đnh rng  0  < n(1+

√ 3)−bn <  2

vi mi  n.

Bài 1.15 (AMM).  Cho  a1  = 1  và  an+1   =  an + [√ 

an]  vi   n  = 1, 2, . . . Chng minh rng  an là s chính phương khi và ch khi  n  = 2k + k − 2 vi  k  là s nguyên dương nào đó.

Bài 1.16 (Bulgari 1973). Cho dãy s {an}n=1 đưc xác đnh bi  a1 = 2, an+1  =a2n − an + 1.

a) Chng minh rng  (an, am) = 1 vi mi  m = n.b) Chng minh rng  lim

n1 1/ak  = 1.

Hưng dn:a) am − 1 = am−1 . . . an(an − 1)b)  1/ak  = 1/(ak − 1) − 1/(ak+1 − 1)

Bài 1.17 (Ba Lan 2002).  Cho trưc s nguyên dương  k. Dãy s  {an} đưc xác đnh bi  a1  =  k + 1, an+1  =  a2

n − kan +  k  vi mi  n ≥  1. Chng minh rng vi mi  m

= n  ta có  (am, an) = 1.

Bài 1.18 (KVANT).  Cho   1 ≤   a0   < a1   < · · ·  < an   là các s nguyên dương.Chng minh rng 

1/[a0, a1] + 1/[a1, a2] + · · · + 1/[an−1, an] ≤ 1 − 1/2n

Page 35: Dãy số - Giới hạn Tác giả: Trần Nam Dũng- Nguyễn Văn Mậu, 2007

8/13/2019 Dãy số - Giớ i hạn Tác giả: Trần Nam Dũng- Nguyễn Văn Mậu, 2007

http://slidepdf.com/reader/full/day-so-gioi-han-tac-gia-tran-nam-dung-nguyen-van-mau 35/217

1.6. Bài tp    35

Hưng dn: Vi a < b, 1/[a, b] = (a, b)/ab ≤ (b − a)/ab = 1/a − 1/b.

Bài 1.19 (Ba Lan 1997).  Dãy s  a1, a2, . . . xác đnh bi a1 = 0, an =  a[n/2] + (−1)n(n+1)/2

Vi mi s t nhiên  k, tìm s các ch s  n sao cho  2k ≤ n < 2k+1 và  an = 0.

Hưng dn: Dùng h đm cơ s.

Bài 1.20 (Vit Nam, 1998).  Cho dãy s  {an} đưc xác đnh bi  a0  = 20, a1 =100, an+2 = 4an+1 + 5an + 20 vi  n  = 0, 1, 2, . . . Tìm s nguyên dương  h  nh nht tho mãn điu kin  an+h − an  chia ht cho 1998 vi mi  n = 0, 1, 2, . . .

Bài 1.21 (Chn đi tuyn VN, 1993).  Gi  ϕ(n) là hàm Euler (nghĩa là  ϕ(n)

là s các ưc s nguyên dương không ln hơn  b và nguyên t cùng nhau vi  n).Tìm tt c các s nguyên dương  k > 1  tho mãn điu kin:

Vi  a là s nguyên >1 bt kỳ, đt  x0  =  a, xn+1 =  kϕ(xn) vi  n  = 0, 1, . . . thì (xn) luôn b chn.

Bài 1.22 (M 1997).  Cho dãy s t nhiên  a1, a2, . . . , a1997 tho 

ai + a j ≤ ai+ j ≤ ai + a j + 1

vi mi   i, j  nguyên dương tho   i + j ≤ 1997. Chng minh rng tn ti s thc  xsao cho  an = [nx] vi mi  n = 1, 2, ...,1997.

Hưng dn: Chng minh rng  an/n <  (am + 1)/m vi mi  m, n.Bài 1.23.  Cho dãy s  {an}

a) [Liên Xô 1977] Chng minh rng nu  lim(an+1−an/2) = 0 thì  lim an = 0.b) Tìm tt c các giá tr  a sao cho nu  lim(an+1 − αan) = 0 thì   lim an = 0.

Bài 1.24 (CRUX).  Tìm s hng tng quát ca dãy s  { pn} xác đnh bi  p0  =1, pn+1 = 5 pn(5 p4

n − 5 p2n + 1)

Bài 1.25.  Dãy s  {an} đưc xác đnh bi  a1 >  0, a2 >  0  và  an+1  =√ 

an +√ 

an−1.Chng minh dãy s  {an} hi t và tìm gii hn.

Bài 1.26 (LMO 1989).  Dãy s thc 

 {ak

}k=1 tho mãn điu kin  ak+1  = (kak +

1)/(k − ak). Chng minh rng dãy s cha vô hn s hng dương và vô hn s hng âm.

Bài 1.27 (LMO 1989).  Dãy s thc  {ak}k=1  tho mãn điu kin  |am +  an −am+n| ≤ 1/(m + n) vi mi  m, n. Chng minh rng  {ak} là cp s cng.

Page 36: Dãy số - Giới hạn Tác giả: Trần Nam Dũng- Nguyễn Văn Mậu, 2007

8/13/2019 Dãy số - Giớ i hạn Tác giả: Trần Nam Dũng- Nguyễn Văn Mậu, 2007

http://slidepdf.com/reader/full/day-so-gioi-han-tac-gia-tran-nam-dung-nguyen-van-mau 36/217

1.6. Bài tp    36

Bài 1.28.  Vi  n ≥ 2, gi  xn   là nghim dương duy nht ca phương trình  xn =xn−1 + xn−2 +

· · ·+ x + 1.

a) Chng minh rng  lim xn  = 2.b) Hãy tìm  lim(2 − xn)1/n.

Bài 1.29 (Bulgari 82).  Cho  x1, . . . , xn là các s thc thuc đon  [0, 2]. Chng minh rng 

ni=1

n j=1

|xi − x j | ≤ n2.

Du bng xy ra khi nào? 

Hưng dn: Sp li th t!

Bài 1.30 (Bulgari 86).  Cho dãy s thc  {an}∞n=1  tho mãn điu kin  an+1 ≤(1 + k/n)an − 1, n = 1, 2, . . . trong đó  0 < k < 1. Chng minh rng tn ti s t nhiên   t sao cho  at  <  0.

Hưng dn: an+1/(n + 1) < an/n − 1/(n + 1).

Bài 1.31.   Hai dãy s  {an}, {bn}   xác đnh bi   a1   >   0, b1   >   0, an+1   =   an  +1/bn, bn+1 =  bn + 1/an. Chng minh rng  a50 + b50 >  20.

Hưng dn: Xét  cn = (an + bn)2.

Bài 1.32 (Canada 1985).   Cho   1   < x1   <   2. Vi   n   = 1, 2, . . .  ta đnh nghĩa xn+1  = 1 + xn

−x2n/2. Chng minh rng vi mi  n

≥3 ta có 

 |xn

√ 2

|< 1/2n.

Bài 1.33 (PARABOLA).  Cho   a,b >   0. Hai dãy s  {an}, {bn}  xác đnh bi a1  =

√ ab, b1  = (a + b)/2, an+1  =

√ anbn, bn+1  = (an +  bn)/2. Chng minh rng 

vi mi  n nguyên dương ta có  |bn − an| ≤ |b − a|/2n.

Bài 1.34 (IMO 1978).  Cho {an} là dãy các s nguyên dương phân bit. Chng minh rng vi mi  n ta có 

nk=1

ak/k2 ≥n

k=1

1/k.

Bài 1.35 (Putnam 2001).  Gi s  {

an

}n=1  là dãy s tăng các s thc dương 

sao cho   lim an/n   = 0. Có th tn ti vô s các s nguyên dương   n   sao choan−i + an+i < 2an  vi mi   i = 1, 2, . . . , n − 1 hay không? 

Bài 1.36 (Áo - Ba Lan 2001).  Cho  a1, a2, . . . , a2010 là dãy s tho mãn điu kin 

Page 37: Dãy số - Giới hạn Tác giả: Trần Nam Dũng- Nguyễn Văn Mậu, 2007

8/13/2019 Dãy số - Giớ i hạn Tác giả: Trần Nam Dũng- Nguyễn Văn Mậu, 2007

http://slidepdf.com/reader/full/day-so-gioi-han-tac-gia-tran-nam-dung-nguyen-van-mau 37/217

1.6. Bài tp    37

1. Tng 20 s hng liên tip ca dãy s là không âm.

2. |aiai+1| ≤ 1 vi mi   i = 1, 2, . . . , 2009.Hãy tìm  min

2001i=1   ai.

Bài 1.37 (Ba Lan 2001).  Cho dãy s  {an} xác đnh bi  a0  = 1, an =  a[7n/9] +

a[n/9], n = 1, 2, . . . Chng minh rng tn ti  k  sao cho  ak  < k/2001!.

Bài 1.38 (Trung Quc 1997).   Cho   a1, a2, . . .  là dãy s thc tho mãn điu kin  an+m ≤ an + am  vi mi  m, n. Chng minh rng  an ≤ ma1 + (n/m − 1)am

vi mi  n ≥ m.

Bài 1.39 (Singapore 1997).  Cho dãy s  {an}  xác đnh bi  a0  = 1/2, ak+1  =ak + a2

k/n,k = 1, 2, . . . , n

−1. Chng minh rng  1

−1/n < an  < 1.

Hưng dn:  Chng minh bng quy np rng   (n + 1)/(2n − k + 2)  < ak   <n/(2n − k).

Bài 1.40 (Baltic Way).  Gi s  a1, a2, . . . , a9 là các s không âm sao cho  a1  =a9  = 0 và ít nht có mt s khác 0. Chng minh rng tn ti ch s   i, 2 ≤ i ≤ 8sao cho  ai−1 + ai+1 <  2ai. Khng đnh có còn đúng không nu thay 2 bt đng thc cui cùng bng  1.9? 

Bài 1.41.  Dãy s  an  đưc xác đnh bi công thc truy hi 

a0 = 1, an+1  =an

1 + nan

, n = 0, 1, 2, . . .

Hãy tìm công thc tng quát cho an.

Bài 1.42 (Vit Nam, 1984).  Dãy s  u1, u2, . . . đưc xác đnh bi:  u1 = 1, u2 =2, un+1  = 3un − un−1  vi  n = 2, 3, . . . Đt  vn  =

 1≤k≤n arcotguk.

Hãy tìm gii hn  vn  khi  n dn đn vô cùng.

Hưng dn: Dùng sai phân.

Bài 1.43 (PTNK, 1999).  Cho  a >  1  và dãy s  {xn} đưc xác đnh như sau 

x1  =  a, xn+1 = nax vi mi   n

≥1.

Hãy xác đnh tt c các giá tr ca  a đ dãy  {xn} hi t.

Page 38: Dãy số - Giới hạn Tác giả: Trần Nam Dũng- Nguyễn Văn Mậu, 2007

8/13/2019 Dãy số - Giớ i hạn Tác giả: Trần Nam Dũng- Nguyễn Văn Mậu, 2007

http://slidepdf.com/reader/full/day-so-gioi-han-tac-gia-tran-nam-dung-nguyen-van-mau 38/217

1.6. Bài tp    38

Bài 1.44.  Cho dãy s dương  {an}. Bit rng tn ti gii hn 

limn→∞

nk=1

1

ak= A < ∞

Đt  sn  = a1 + a2 + · · · + an. Chng minh rng tng n

k=1

k2ak

(sk)2

cũng có gii hn hu hn khi  n → ∞.

Hưng dn: Dùng công thc tính tng tng phn

Bài 1.45. Cho  f   : N → R tho điu kin  f (a+b) ≤ f (a)+ f (b) vi mi  |b−a| ≤ k( k là s nguyên dương c đnh). Hi có tn ti gii hn  f (n)/n khi  n dn đn vô cùng không? 

Bài 1.46.  Các phn t ca dãy s   a1, a2, a3, . . ., là các s nguyên dương khác nhau. Chng minh rng vi mi  k tn ti  n sao cho tn ti  an ≥ n.

Bài 1.47.  Chng minh rng nu  a1 >  2  và  an =  a2n−1 − 2 thì 

1

a1+

1

a1a2+

1

a1a2a3 + · · · =

1

2[a1 −

 a21 − 4].

Hưng dn: Dùng lưng giác.

Bài 1.48.  Dãy s dương  an  tho mãn điu kin  an  < an+1 +  a2n. Có th khng 

đnh tng  n

i=1 ai  dn đn vô cùng khi  n dn đn vô cùng hay không? Bài 1.49 (THTT).  Cho s thc  r > 2. Cho dãy s thc dương  {an} tho mãn điu kin  ar

n  = a1 + · · · + an−1  vi mi  n ≥ 2. Chng minh rng dãy  {an/n} có gii hn hu hn khi  n → ∞ và tìm gii hn đó.

Bài 1.50 (Chn đi tuyn Vit Nam, 1985).  Dãy s thc  {xn}  đưc xác đnh bi:

x1 = 29/10, xn+1 = (xn/ 

x2n − 1) +

√ 3, n = 1, 2, 3 . . .

Hãy tìm s thc nh hơn  x2k−1  và ln hơn  x2k  vi mi  k = 1, 2, . . .

Bài 1.51 (Chn đi tuyn Vit Nam, 1996).  Tìm tt c các giá tr ca  a

đ dãy s  {xn} đưc xác đnh bi x0 =

√ 1996

xn+1 = a/(1 + x2n)

có gii hn hu hn khi  n dn ti vô cùng.

Page 39: Dãy số - Giới hạn Tác giả: Trần Nam Dũng- Nguyễn Văn Mậu, 2007

8/13/2019 Dãy số - Giớ i hạn Tác giả: Trần Nam Dũng- Nguyễn Văn Mậu, 2007

http://slidepdf.com/reader/full/day-so-gioi-han-tac-gia-tran-nam-dung-nguyen-van-mau 39/217

1.6. Bài tp    39

Hưng dn: Chuyn v dng  xn+1  =  f (xn), x0 =  b.

Bài 1.52 (Vit Nam, 1997).  Cho n  là s nguyên >1, không chia ht cho 1997.Đt 

ai =  i + ni/1997  vi mi   i = 1, 2, . . . , 1996,

b j  = j  + 1997 j/n  vi mi   j  = 1, 2, . . . , n − 1.

Ta sp xp các s  {ai} và  {bi} theo th t tăng dn:

c1 ≤ c2 ≤ · · · ≤ c1995+n

Chng minh rng  ck+1 − ck  <  2  vi mi  k = 1, 2, ..1994+ n.

Bài 1.53 (Vit Nam, 1998).  Cho  a là mt s thc không nh hơn 1. Đt 

x1 =  a, xn+1  = 1 + ln(x2n/(1 + ln(xn))   vi   n = 1, 2, . . .

Chng minh rng dãy s  {xn} có gii hn và tìm gii hn đó.

Bài 1.54.  Cho dãy s  {xn} xác đnh bi,  x1 =  a, xn+1  = (2x3n)/(3x2

n−1) vi mi n ≥ 1. Tìm tt c các giá tr ca  a đ dãy s xác đnh và có gii hn hu hn.

Bài 1.55.  Chng minh rng dãy s xác đnh bi điu kin  xn+1   =  xn +  x2n/n2

vi  n ≥ 1, trong đó  0 < x1  <  1  là dãy b chn.

Bài 1.56.  Cho dãy s 

an = 

1 + 2 

1 + · · · 1 + (n − 1)√ 1 + n

Chng minh rng  limn→∞ an  = 3.

Bài 1.57.   Dãy   a1  + 2a2, a2  + 2a3, a3  + 2a4, . . .  hi t. Chng minh rng dãy a1, a2, a3, . . . cũng hi t.

Bài 1.58. Cho dãy  A(n), n = 1, 2, . . . tho mãn: vi mi  x thc thì  limn→∞ A([xn]) =0. Chng minh rng  lim A(n) = 0 khi  n tin ti vô cùng.

Bài 1.59.  Cho hàm s 

f (x) = x + A sin x + B cos x   vi   A2 + B 2 < 1.

Xét dãy s a0  =  a, a1 =  f (a0), . . . , an+1  = f (an), . . .

Chng minh rng vi mi  a, dãy s  {an} có gii hn và hãy tìm gii hn đó.

Page 40: Dãy số - Giới hạn Tác giả: Trần Nam Dũng- Nguyễn Văn Mậu, 2007

8/13/2019 Dãy số - Giớ i hạn Tác giả: Trần Nam Dũng- Nguyễn Văn Mậu, 2007

http://slidepdf.com/reader/full/day-so-gioi-han-tac-gia-tran-nam-dung-nguyen-van-mau 40/217

1.6. Bài tp    40

Bài 1.60.   Cho dãy s  {an}, đưc xác đnh như sau:   a0   =   a, a1   =   b, an+1   =an + (an

−an−1)/2n. Tìm   limn→∞ an.

Bài 1.61 (AMM).  Cho {H n}   là dãy s Fibonacci tng quát, tc là  H 1, H 2   là các s nguyên bt kỳ và vi  n > 2  thì  H n =  H n−1 + H n−2.

a) Hãy tìm  T , ph thuc vào H 1 và  H 2 sao cho các s  H 2nH 2n+2+T, H 2nH 2n+4+T, H 2n−1H 2n+1 − T, H 2n−1H 2n+3 − T  đu là các s chính phương.

b) Chng minh  T  là duy nht.

Bài 1.62.  Cho   r   là s thc. Xác đnh dãy s  {xn}  bi  x0   = 0, x1  = 1, xn+2   =rxn+1 − xn  vi  n ≥ 0. Chng minh rng  x1 + x3 + · · · + x2m−1  =  x2

m.

Bài 1.63 (IMO 1977).  Trong mt dãy s hu hn các s thc, tng 7 s hng liên tip ca dãy luôn âm, còn tng 11 s hng liên tip luôn dương. Hi dãy s đó có th có nhiu nht bao nhiêu s hng.

Tài liu tham kho

1. Jean-Marie Monier, Gii tích 1, 2, 3, 4, NXBGD 1999-2000.

2. Lê Hi Châu: Tuyn tp các đ thi toán quc t.

3. Titu Andreescu, Razvan Gelca: Mathematical Olympiad Challenges, Birkhauser2000.

4. A. Gardiner, The Mathematical Olympiad Hanbook, Oxford, 1997.

5. Titu Andreescu, Zuming Feng: Mathematical Olympiads 1998-1999, 1999-2000, 2000-2001, MAA, 2000-2002.

6. Arthur Engel: Problem-Solving Strategies, Springer 1997. 7

7. G.Polya, G.Szego: Các bài tp và đnh lý ca gii tích, Nauka 1977 (TingNga).

8. Cupsov, Nesterenko   . . .: Thi vô đch toán toàn Liên Xô, Prosvesenie, 1999(Ting Nga).

9. 400 bài toán t American Mathematical monthly, Mir, 1977 (Ting Nga).

10. Đ thi toán ca Vit Nam, các nưc và khu vc.

11. Tp chí Toán hc và Tui tr (THTT), Parabola, Kvant, American Math-ematical monthly (AMM).

Trn Nam Dũng - ĐHKHTN TP H Chí Minh227 Nguyn Văn C, Qun 5, TP H Chí MinhEmail: [email protected], [email protected]

Page 41: Dãy số - Giới hạn Tác giả: Trần Nam Dũng- Nguyễn Văn Mậu, 2007

8/13/2019 Dãy số - Giớ i hạn Tác giả: Trần Nam Dũng- Nguyễn Văn Mậu, 2007

http://slidepdf.com/reader/full/day-so-gioi-han-tac-gia-tran-nam-dung-nguyen-van-mau 41/217

Chương 2

Phương trình sai phân

2.1 Sai phân2.1.1 Đnh nghĩa 

Cho hàm s  y  =  f (x) xác đnh trên  R, đt  xk  = x0 + kh   (k ∈ N∗) vi  x0 ∈R, h ∈ R, bt kỳ, cho trưc. Gi  y k  = f (xk) là giá tr ca hàm s  f (x) ti x  =  xk.Khi đó, Hiu s ∆y k   := y k+1−y k   (k ∈ N∗) đưc gi là sai phân cp 1 ca hàm sf (x). Hiu s ∆2 y k  := ∆y k+1−∆y k  = ∆(∆y k) (k ∈ N∗) đưc gi là sai phân cp2 ca hàm s  f (x). Tng quát,  ∆iy k  := ∆i−1y k+1 − ∆i−1y k  = ∆(∆i−1 y k) (k ∈N∗) đưc gi là sai phân cp  i ca hàm s  f (x) (i = 1; 2; · · · ; n; · · ·).

2.1.2 Tính cht

Mnh đ 2.1 (Biu din sai phân theo giá tr ca hàm s).  Sai phân mi cpđu có th biu din theo các giá tr ca hàm s:

y 0; y 1; y 2; · · · ; y n; · · ·Chng minh. Tht vy, ta có

∆y k  = y k+1 − y k

∆2y k  = ∆y k+1 − ∆y k

= y k+2 − y k+1 − (y k+1 − y k)

= y k+2

−2y k+1 + y k .

Tương t, bng quy np ta có th chng minh đưc.

∆iy k  =

is=1

(−1)sC si y k+i−s ,   (đpcm).

41

Page 42: Dãy số - Giới hạn Tác giả: Trần Nam Dũng- Nguyễn Văn Mậu, 2007

8/13/2019 Dãy số - Giớ i hạn Tác giả: Trần Nam Dũng- Nguyễn Văn Mậu, 2007

http://slidepdf.com/reader/full/day-so-gioi-han-tac-gia-tran-nam-dung-nguyen-van-mau 42/217

2.1. Sai phân   42

Mnh đ 2.2 (Sai phân ca hng s).  Sai phân ca hng s bng 0.

Chng minh. Tht vy, vi  y  =  f (x) = C  = const ta có:  ∆f (x) = C  − C  = 0.Hơn th na, sai phân mi cp ca hng s đu bng 0.

Mnh đ 2.3 (Tính cht tuyn tính ca sai phân ).  Sai phân mi cp là mt toán t tuyn tính trên tp các hàm s. Tc là.

∀ i ∈ N∗, ∀ α; β  ∈ R, ∀ f (x); g(x) : R → R,  ta luôn có:

∆i(αf (x) + βg(x)) =  α∆if (x) + β ∆ig(x).

Chng minh.  Tht vy, đt  f k  = f (xk) ;   gk  = g(xk), ta thu đưc

∆i

(αf k + βgk) =

i

s=0(−1)

s

C s

i   [αf k+i−s +  βgk+i−s]

= α

is=0

(−1)sC si f k+i−s + β 

is=0

(−1)sC si gk+i−s

= α∆if k + ∆igk .

Vy nên

∆i(αf (x) + βg(x)) =  α∆if (x) + β ∆ig(x)   vi mi i ∈ N∗ (đpcm).

Mnh đ 2.4 (Sai phân ca đa thc).  Sai phân cp   i ca mt đa thc bc  n.

+) Là mt đa thc bc  n − i khi   i < n. +) Là hng s khi   i =  n. +) Bng 0 khi i > n.

Chng minh. Do sai phân mi cp là toán t tuyn tính nên ta ch cn chngminh tính cht cho đa thc  y  =  P n(x) =  xn.

+) Khi  i < n ta có1o) Vi i = 1 thì:  ∆xn = (x + h)n − xn = P n−1(x) là đa thc bc  n − 1 đi

vi x. Vy khng đnh đúng vi  i = 1.2o) Gi s khng đnh đúng vi  i  =  k < n, tc là  ∆kxn = P n−k(x) là đa thc

bc n − k đi vi x. Khi đó

∆k+1xn = ∆(∆kxn) = ∆k ((x + h)n) − ∆k(xn)

= P n−k(x + h) − P n−k(x) =  P n−k−1(x)là đa thc bc  n − k − 1 = n − (k + 1)  đi vi  x.

Vy khng đnh cũng đúng vi   i  =  k + 1. T đó, theo nguyên lý quy np toánhc suy ra khng đnh đúng vi mi  i ∈ N∗ (đpcm).

Page 43: Dãy số - Giới hạn Tác giả: Trần Nam Dũng- Nguyễn Văn Mậu, 2007

8/13/2019 Dãy số - Giớ i hạn Tác giả: Trần Nam Dũng- Nguyễn Văn Mậu, 2007

http://slidepdf.com/reader/full/day-so-gioi-han-tac-gia-tran-nam-dung-nguyen-van-mau 43/217

2.2. Phương trình sai phân tuyn tính   43

+) Khi  i  =  n  thì theo trên,  ∆n(xn) là đa thc cp  n − n = 0 đi vi  x  nên làhng s.

+) Khi  i > n thì

∆i(xn) = ∆i−n (∆n(xn)) = ∆i−nC    (C  = const) = 0.

Vy tính cht đã đưc chng minh trn vn.

Mnh đ 2.5 (Công thc sai phân tng phn).

∆(f kgk) = f k∆gk + gk+1∆f k.

Chng minh. Ta có

∆(f kgk) =  f k+1gk+1

−f kgk

= f k+1gk+1 − f kgk+1 + f kgk+1 − f kgk

= gk+1(f k+1 − f k) + f k(gk+1 − gk)

= f k∆gk + gk+1∆f k .

Mnh đ 2.6 (Tng các sai phân).

nk=1

∆y k  = y n+1 − y 1.

Chng minh.

nk=1

∆y k  = ∆y 1 + ∆y 2 + · · · + ∆y n−1 + ∆y n

= y 2 − y 1 + y 3 − y 2 + · · · + y n − y n−1 + y n+1 − y n

= y n+1 − y 1.

2.2 Phương trình sai phân tuyn tính

2.2.1 Mt s khái nim chung v phương trình sai phân

Đnh nghĩa 2.1.  Phương trình sai phân (cp  k) là mt h thc tuyn tính cha 

sai phân các cp ti  k.

f (y n; ∆y n; ∆2y n; · · · ; ∆ky n) = 0.   (1)

Page 44: Dãy số - Giới hạn Tác giả: Trần Nam Dũng- Nguyễn Văn Mậu, 2007

8/13/2019 Dãy số - Giớ i hạn Tác giả: Trần Nam Dũng- Nguyễn Văn Mậu, 2007

http://slidepdf.com/reader/full/day-so-gioi-han-tac-gia-tran-nam-dung-nguyen-van-mau 44/217

2.3. Phương trình sai phân tuyn tính bc nht   44

Vì sai phân các cp đu có th biu din theo giá tr ca hàm s nên (1) códng:

a0y n+k  +  a1y n+k−1 + · · · + aky n =  f (n).   (2)trong đó  a0; a1; · · · ; ak, f (n) đã bit, còn  y n, y n+1, · · · , y n+k  là các giá tr chưabit.

•  Phương trình (2) đưc gi là phương trình sai phân tuyn tính cp  k.

•  Nu  f (n) = 0 thì phương trình (2) có dng

a0y n+k  + a1y n+k−1 + · · · + aky n = 0.   (3)

và đưc gi là phương trình sai phân tuyn tính thun nht cp  k.

•  Nu  f (n) = 0 thì (2) đưc gi là phương trình sai phân tuyn tính khôngthun nht.

b. Nghim.

•  Hàm s  y n  bin  n  tho mãn (2) đưc gi là nghim ca phương trình saiphân tuyn tính (2).

•  Hàm s  y n ph thuc k  tham s tho mãn (3) đưc gi là nghim tng quátca (3).

•  Mt nghim  y ∗n tho mãn (2) đưc gi là mt nghim riêng ca (2).

2.3 Phương trình sai phân tuyn tính bc nht

2.3.1 Đnh nghĩa 

Phương trình sai phân tuyn tính bc nht (cp mt) là phương trình saiphân dng:

u1  =  α, aun+1 + bun  =  f (n)   n ∈ N∗   (1)

trong đó  α  ;   a = 0 ;   b = 0 là các hng s và  f (n) là biu thc ca  n  cho trưc.

2.3.2 Phương pháp gii

A. Gii phương trình sai phân thun nht tương ng.1+) Gii phương trình đc trưng:  aλ + b = 0 đ tìm λ.2+) Tìm nghim ca phương trình sai phân tuyn tính thun nht tương

ng:  aun+1 + bun = 0 dưi dng  un  =  cλn (c là hng s ).B. Tìm mt nghim riêng  u∗n  ca phương trình không thun nht.C. Tìm nghim tng quát ca phương trình (1):

un  =  u∗n + un.

Page 45: Dãy số - Giới hạn Tác giả: Trần Nam Dũng- Nguyễn Văn Mậu, 2007

8/13/2019 Dãy số - Giớ i hạn Tác giả: Trần Nam Dũng- Nguyễn Văn Mậu, 2007

http://slidepdf.com/reader/full/day-so-gioi-han-tac-gia-tran-nam-dung-nguyen-van-mau 45/217

2.3. Phương trình sai phân tuyn tính bc nht   45

Ví d 2.1.  Phương trình  un+1  = 3un+1 có  un  = C.3n ;   u∗n  = −1

2 nên có nghim 

tng quát là  un  = C.3n − 1

2  vi  C  là hng s bt kỳ.

Sau đây ta trình bày phương pháp tìm nghim riêng.

2.3.3 Phương pháp tìm nghim riêng ca phương trình sai phântuyn tính cp 1 không thun nht khi v phi  f (n) có dngđc bit

Trưng hp 1. Nu  f (n) =  P m(n)  là đa thc bc  m đi vi  n. Khi đó: +)Nu   λ = 1  thì ta chn   u∗n   =  Qm(n)  cũng là đa thc bc  m đi vi  n. +) Nuλ = 1 thì ta chn  u∗n  = nQm(n) trong đó  Qm(n) cũng là đa thc bc  m đi vin.

Ví d 2.2.  Gii phương trình sai phân:x0 = 7

xn+1  = 15xn − 14n + 1

Gii.  Ta có f (n) = −14n+1 là đa thc bc nht, λ = 15 = 1 ⇒ chn x∗n  =  an+b.Thay vào phương trình ta đưc

a(n + 1) + b = 15(an + b) − 14n + 1.

Suy ra   a   = 1 ;   b   = 0.  Vy   x∗n

  =   n   còn   xn

  =   C.15n và nghim tng quátlà:   xn   =   C.15n + n. Mà   x0   = 7  nên   C   = 7. Vy phương trình có nghim:xn = 7.15n + n.

Ví d 2.3.  Gii phương trình sai phân:

x0  = 99

xn+1  = xn − 2n − 1

Gii.   f (n) = −2n − 1 là đa thc bc nht, λ  = 1 ⇒ chn x∗n  =  n(an + b). Thayvào (1.2) đưc:

(n + 1)[a(n + 1) + b] =  n(an + b) − 2n − 1 ⇒ a = −1 ;   b = 0 ⇒ x∗n = −n2.

Còn  xn =  C.1n

= C  ⇒ xn  = C  − n2

, mà  x0 = 99 ⇒ C  − 02

= 99 ⇔ C  = 99. Vyphương trình (1.2) có nghim:  xn = 99 − n2.Trưng hp 2.   f (n) =  p.β n ( p; β = 0). Khi đó:+) Nu  λ = β  thì ta chn  x∗n  =  d.β n (d ∈ R).+) Nu  λ =  β  thì ta chn  x∗n  =  d.n.β n (d ∈ R).

Page 46: Dãy số - Giới hạn Tác giả: Trần Nam Dũng- Nguyễn Văn Mậu, 2007

8/13/2019 Dãy số - Giớ i hạn Tác giả: Trần Nam Dũng- Nguyễn Văn Mậu, 2007

http://slidepdf.com/reader/full/day-so-gioi-han-tac-gia-tran-nam-dung-nguyen-van-mau 46/217

2.3. Phương trình sai phân tuyn tính bc nht   46

Ví d 2.4.  Gii phương trình sai phân:

x0  = 8

xn+1  = 2xn + 3n (1.3).

Gii.   Do  λ  = 2 = 3 =  β  nên ta chn  x∗n  =  d.3n. Thay vào phương trình (1.3)đưc  d = 1 ⇒ x∗n  = 3n. Còn  xn =  C.2n. Vy xn =  C.2n + 3n. Thay vào điu kinbiên đưc  C  = 7.Tr li: phương trình đã cho có nghim  xn  = 7.2n + 3n.

Ví d 2.5.  Gii phương trình sai phân:

x0  = 101

xn+1  = 7.xn + 7n+1 (1.4).

Gii.   Do λ  = 7 = β  nên ta chn  x∗n =  d.n.3n. Thay vào phương trình (1.4) đưcd = 1

⇒ x∗n  = n.7n. Còn  xn  =  C.7n. Vy  xn  = C.7n + n.7n. Thay vào điu kin

biên đưc  C  = 101.Tr li: phương trình đã cho có nghim  xn  = (101 + n).7n.

Trưng hp 3.   f (n) =  α. sin nx + β. cos nx   (α + β  = 0; x = kπ ; k ∈ Z). Khiđó, ta chn u∗n  =  A. sin nx + B. cos nx vi  A; B  ∈ R là các hng s.

Ví d 2.6.  Gii phương trình sai phân:

x0 = 1√ 

2.xn+1  =  xn − sin nπ

4  (1.5)

  .

Gii.   Có λ  =  1√ 

2;   f (n) = sin

 nπ

4  nên ta chn x∗n = A. cos

 nπ

4  + B. sin

 nπ

4  .

Thay x∗n  vào (1.5), bin đi và so sánh các h s ta đưc  A = 1 ;   B  = 0 ⇒ x∗n  =

cos nπ4   . Còn  xn  =  C.(   1√ 2

)n ⇒ xn = C.(   1√ 2

)n + cos nπ4  . Thay vào điu kin biên

x0  = 1 ta đưc  C  = 0. Vy phương trình đã cho có nghim  xn = cos nπ

4  .

Trưng hp 4.

f (n) =m

k=1

f k(n).

Khi đó ta chn nghim riêng  x∗n ca (1) dưi dng:  x∗n  =m

k=1 x∗nk  trong đó x∗nktương ng là nghim riêng ca phương trình sai phân (1) vi  V P   = f k(n).

Ví d 2.7. Gii phương trình sai phân: x0 = 17

xn+1  = 2xn − n2

+ 2n + 1 + 6.2n

(1.6)

.

Gii.   Có λ  = 2 ;   f 1(n) = −n2 + 2n + 1 ;   f 2(n) = 6.2n ⇒

⇒  xn  =  C.2n ;   x∗n1  = an2 + bn + c ;   x∗n2  =  d.n.2n.

Page 47: Dãy số - Giới hạn Tác giả: Trần Nam Dũng- Nguyễn Văn Mậu, 2007

8/13/2019 Dãy số - Giớ i hạn Tác giả: Trần Nam Dũng- Nguyễn Văn Mậu, 2007

http://slidepdf.com/reader/full/day-so-gioi-han-tac-gia-tran-nam-dung-nguyen-van-mau 47/217

2.4. Phương trình sai phân tuyn tính cp 2    47

Vy ta chn x∗n =  an2 + bn + c + d.n.2n. Thay vào (1.6) và so sánh các h s đưc:a = 1 ;   b =  c  = 0 ;   d = 3. Vy:  xn  =  C.2n + n2 + 3n.2n. Thay vào điu kin biên

x0  = 17 ta đưc  C  = 17 và do đó nghim ca phương trình sai phân đã cho là:

xn  = 17.2n + n2 + 3n.2n.

2.3.4 Bài tp

Gii các phương trình sai phân tuyn tính sau:

1.   un+1  = 3un − 6n + 1 ;   u1 = 1.Đáp s:  un = 3n + 1 − 3n.

2.   un+1  = un + 2n2 ;   u1  = 1.

Đáp s:  un =

 1

3 (2n3

− 3n2

+ n + 3).

3.   un+1  = 5un − 3n ;   u0 = 1.

Đáp s:  un = 1

2(5n + 3n).

4.   un+1  = 2un + 6.2n ;   u0  = 1.Đáp s:  un = (3n + 1).2n.

5.   un+1  = un + 2n.3n ;   u0  = 0.

Đáp s:  un = 1

2[(2n − 3).3n + 3].

6.   un+1 −

2un

 = (n2 + 1).2n ;   u0

 = 1.

Đáp s:  un =

n(2n2 − 3n + 7)

6  + 2

.2n.

7.   un+1 − 2un  =  n + 3n ;   u0  = 1.Đáp s:  un = 2n + 3n − n − 1.

2.4 Phương trình sai phân tuyn tính cp 2

2.4.1 Đnh nghĩa 

Phương trình sai phân tuyn tính cp hai là phương trình sai phân dng:

u1  =  α, u2  =  β, aun+2 + bun+1 + cun =  f (n), n ∈ N∗   (1)

trong đó   a, b, c, α, β  là các hng s,  a = 0, c = 0 và f (n) là biu thc cha  ncho trưc.

Page 48: Dãy số - Giới hạn Tác giả: Trần Nam Dũng- Nguyễn Văn Mậu, 2007

8/13/2019 Dãy số - Giớ i hạn Tác giả: Trần Nam Dũng- Nguyễn Văn Mậu, 2007

http://slidepdf.com/reader/full/day-so-gioi-han-tac-gia-tran-nam-dung-nguyen-van-mau 48/217

2.4. Phương trình sai phân tuyn tính cp 2    48

2.4.2 Cách gii

+) Gii phương trình thun nht tương ng. +) Tìm nghim riêng ca phươngtrình không thun nht. +) Tìm nghim tng quát ca phương trình (1) dưidng:

un  = un + u∗n.

A- Gii phương trình thun nht tương ng 

aun+2 + bun+1 + cun = 0 (2)

1+) Gii phương trình đc trưng:

a.λ2 + b.λ + c = 0.   (3)   đ tìm λ.

2+) Tìm nghim tng quát ca phương trình thun nht tương ng.

Trưng hp 1: Nu (3) có hai nghim phân bit:  λ  =  λ1  ;   λ =  λ2 thì:

un  = A.λn1 +  B.λn

2 ,

trong đó  A  và B  đưc xác đnh khi bit  u1 và u2.

Trưng hp 2: Nu (3) có nghim kép:  λ1 =  λ2  =  λ  thì:

un  = (A + Bn).λn,

trong đó  A  và B  đưc xác đnh khi bit  u1 và u2.

Trưng hp 3: Nu  λ là nghim phc,  λ =  x + i.y  thì ta đt

r = |λ| = 

x2 + y 2,   tan θ  = y 

x, θ ∈

−  π

2; π

2

.

lúc đó  λ  =  r(cos θ + i. sin θ) và

un =  rn(A. cos nθ + B. sin nθ),

trong đó  A  và B  đưc xác đnh khi bit  u1 và u2. Ví d 

Ví d 2.8.  Gii phương trình sai phân:x0 = 2 ;   x1 = −8

xn+2  = −8xn+1 + 9xn

.

Page 49: Dãy số - Giới hạn Tác giả: Trần Nam Dũng- Nguyễn Văn Mậu, 2007

8/13/2019 Dãy số - Giớ i hạn Tác giả: Trần Nam Dũng- Nguyễn Văn Mậu, 2007

http://slidepdf.com/reader/full/day-so-gioi-han-tac-gia-tran-nam-dung-nguyen-van-mau 49/217

2.4. Phương trình sai phân tuyn tính cp 2    49

Gii.  Phương trình đc trưng:

λ

2

+ 8λ − 9 = 0 ⇔ λ = 1   hoc   λ = −9.   (Hai nghim phân bit.)Vy:  xn =  xn  =  A.1n + B.(−9)n = A + B.(−9)n.Gii điu kin biên:

x0  = 2

x1  = −8⇔

A + B  = 2

A − 9B  = −8⇔

A = 1

B  = 1.

Vy phương trình đã cho có nghim:  xn  = 1 + (−9)n.

Ví d 2.9.  Gii phương trình sai phân:

x0 = 1 ;   x1 = 16

xn+2  = 8xn+1 − 16xn

.

Gii.  Phương trình đc trưng:

λ2 − 8λ + 16 = 0   ⇔ λ1  =  λ2 = 4 (có nghim kép).

Vy:  xn =  xn  = (A + Bn).4n.Gii điu kin biên:

x0  = 1

x1  = 16⇔

A = 1

(A + B ).4 = 16⇔

A = 1

B  = 3.

Vy phương trình đã cho có nghim:  xn  = (1 + 3n).4n.

Ví d 2.10.  Gii phương trình sai phân:x0 = 1 ;   x1  =

 1

2xn+2  =  xn+1 − x − n

.

Gii.  Phương trình đc trưng:

λ2 − λ + 1 = 0 ⇔ λ = 1 + i

√ 3

2  hoc   λ =

 1 − i√ 

3

2  .   (Hai nghim phc.)

Có:

λ = 1 + i

√ 3

2  = cos

 π

3 + i. sin

 π

3 ⇒ r = 1 ;   θ =

 π

3.

Nghim tng quát:  xn

 =  xn

 =  A. cos nπ

3  + B. sin

 nπ

3  .

Gii điu kin biên:x0 = 1

x1 = 1

2

⇔A = 1

A. cos π

3 + B. sin

 π

3  =

 1

2

A = 1

B  = 0.

Page 50: Dãy số - Giới hạn Tác giả: Trần Nam Dũng- Nguyễn Văn Mậu, 2007

8/13/2019 Dãy số - Giớ i hạn Tác giả: Trần Nam Dũng- Nguyễn Văn Mậu, 2007

http://slidepdf.com/reader/full/day-so-gioi-han-tac-gia-tran-nam-dung-nguyen-van-mau 50/217

2.4. Phương trình sai phân tuyn tính cp 2    50

Vy phương trình đã cho có nghim:  xn  = cos nπ

3  . Bài tp Gii các phương trình

sai phân sau:

1.   y n+2  = 4y n+1 − 3y n   vi   y 0  = 1 ;   y 1  = 1.

2.   y n+2  = 8y n+1 − 16y n   vi   y 0  = 1 ;   y 1  = 1.

3.   y n+2  = 2y n+1 − 2y n   vi   y 0  = 1

2 ;   y 1  = 2.

B- Các phương pháp tìm nghim riêng ca phương trình sai phân tuyn tính cphai không thun nht 

aun+2 + bun+1 + cun  =  f (n)

vi v phi có dng đc bit Trưng hp 1.

f (n) =  P k(n)   là đa thc bc k  đi vi  n.

Khi đó: +) Nu phương trình đc trưng (3) không có nghim  λ  = 1 thì ta chn

x∗n =  Qk(n)

trong đó Qk(n) là đa thc bc k nào đó đi vi n. +) Nu phương trình đc trưng(3) có nghim đơn  λ  = 1 thì ta chn

x∗n = nQk(n)

trong đó Qk(n) là đa thc bc k nào đó đi vi n. +) Nu phương trình đc trưng

(3) có nghim kép λ  = 1 thì ta chn

x∗n  =  n2Qk(n)

trong đó  Qk(n) là đa thc bc  k  nào đó đi vi  n.

Ví d 2.11.  Tìm mt nghim riêng  x∗n ca phương trình sai phân:

xn+2  = −4xn+1 + 5xn + 12n + 8.

Gii.  Phương trình đc trưng:

λ2 + 4λ − 5 = 0 ⇔ λ = 1   hoc   λ = −5 ;   f (n) = 12n + 8.

Chn x∗n  =  n(an + b). Thay vào phương trình đã cho và so sánh các h s ta đưca = 1 ;   b = 0.Vy phương trình đã cho có mt nghim riêng là  x∗n  = n2.

Page 51: Dãy số - Giới hạn Tác giả: Trần Nam Dũng- Nguyễn Văn Mậu, 2007

8/13/2019 Dãy số - Giớ i hạn Tác giả: Trần Nam Dũng- Nguyễn Văn Mậu, 2007

http://slidepdf.com/reader/full/day-so-gioi-han-tac-gia-tran-nam-dung-nguyen-van-mau 51/217

2.4. Phương trình sai phân tuyn tính cp 2    51

Ví d 2.12.  Gii phương trình sai phân:

2xn+2 − 5xn+1 + 2xn = n2

− 2n + 3   vi    x0 = 1 ;   x1 = 3.

Gii.  Phương trình đc trưng:

2λ2 − 5λ + 2 = 0 ⇔ λ = 2   hoc   λ = 1

2  ;   f (n) =  n2 − 2n + 3.

Vy phương trình thun nht có nghim tng quát  xn = A.2n + B.(1

2)n.

Chn  x∗n  = an2 + bn + c. Thay vào phương trình đã cho và so sánh các h s tađưc  a = −1 ;   b = 4 ;   c = −10.Vy phương trình đã cho có mt nghim riêng là  x∗n  = −n2 + 4n − 10.Do đó phương trình đã cho có nghim tng quát là:

xn =  A.2n + B.(1

2)n − n2 + 4n − 10.

Thay vào các điu kin biên ta tìm đưc  A  = 3 ;   B  = 8. Vy nghim ca phươngtrình đã cho là:

xn = 3.2n + 8.(1

2)n − n2 + 4n − 10.

Trưng hp 2.

f (n) = P k(n).β n trong đó  P k(n) là mt đa thc bc  k  đi vi  n.

Khi đó: +) Nu  β  không phi là nghim ca phương trình đc trưng (3) thì tachn:

x∗n =  Qk(n)

trong đó  Qk(n)   là mt đa thc bc  k  nào đó đi vi  n  vi h s cn đưc xácđnh. +) Nu  β  là mt nghim đơn ca phương trình đc trưng (3) thì ta chn:

x∗n  =  n.Qk(n)

trong đó  Qk(n) là mt đa thc bc k  nào đó đi vi  n  . +) Nu  β  là nghim képca phương trình đc trưng (3) thì ta chn:

x∗n = n2.Qk(n),

trong đó  Qk(n) là mt đa thc bc  k nào đó đi vi n  .

Ví d 2.13.  Tìm mt nghim riêng ca phương trình sai phân sau:

2xn+2 + 5xn+1 + 2xn = (35n + 51).3n.

Page 52: Dãy số - Giới hạn Tác giả: Trần Nam Dũng- Nguyễn Văn Mậu, 2007

8/13/2019 Dãy số - Giớ i hạn Tác giả: Trần Nam Dũng- Nguyễn Văn Mậu, 2007

http://slidepdf.com/reader/full/day-so-gioi-han-tac-gia-tran-nam-dung-nguyen-van-mau 52/217

2.4. Phương trình sai phân tuyn tính cp 2    52

Gii.   Ta có β  = 3 ;   P k(n) = 35n + 51  là đa thc bc nht.Phương trình đc trưng:

2λ2 + 5λ + 2 = 0 ⇔ λ = −2 := λ1   hoc   λ = −1

2  := λ2   (λ1; λ2 = β ).

Chn  x∗n  = (an + b).3n. Thay vào phương trình đã cho và so sánh các h s tađưc:  a = 1 ;   b = 0.Vy phương trình đã cho có mt nghim riêng là:  x∗n =  n.3n.

Ví d 2.14.  Tìm mt nghim riêng ca phương trình sai phân sau:

xn+2 − 5xn+1 + 6xn = (8n + 11).2n.

Gii.   Ta có β  = 2 ;   P k(n) = 8n + 11 là đa thc bc nht.

Phương trình đc trưng:λ2 − 5λ + 6 = 0 ⇔ λ = 2 := λ1   hoc   λ = 3 := λ2   (λ1 =  β  ;   λ2 = β ).

Chn  x∗n  = n(an + b).2n. Thay vào phương trình đã cho và so sánh các h s tađưc:  a = −4 ;   b = −23.Vy phương trình đã cho có mt nghim riêng là:  x∗n = −(4n2 + 23n).2n.

Ví d 2.15.  Tìm mt nghim riêng ca phương trình sai phân sau:

xn+2 − 10xn+1 + 25xn = (n + 2).5n+1

Gii.   Ta có β  = 5 ;   P k(n) = 5n + 10 là đa thc bc nht.

Phương trình đc trưng:

λ2 − 10λ + 25 = 0 ⇔ λ = 5 := λ0   (nghim kép)   (λ1 =  λ2 =  β )

Chn  x∗n =  n2(an + b).5n. Thay vào phương trình đã cho và so sánh các h s ta

đưc:  a =  4

50 ;   b =

  7

50.

Vy phương trình đã cho có mt nghim riêng là:  x∗n = n2

50(4n + 7).5n.

Trưng hp 3.

f (n) = P m(n). cos nβ  + P l(n). sin nβ 

trong đó  P m(n) ;   P l(n) ln lưt là các đa thc bc  m  ;   l đi vi  n.Ký hiu k =  M ax{m; l} và gi  ρ  = cos β  + i sin β    (i2 = −1). Khi đó:+) Nu  ρ  không là nghim ca phương trình đc trưng (3) thì ta chn:

x∗n =  T k(n). cos nβ  + Rk(n). sin nβ 

Page 53: Dãy số - Giới hạn Tác giả: Trần Nam Dũng- Nguyễn Văn Mậu, 2007

8/13/2019 Dãy số - Giớ i hạn Tác giả: Trần Nam Dũng- Nguyễn Văn Mậu, 2007

http://slidepdf.com/reader/full/day-so-gioi-han-tac-gia-tran-nam-dung-nguyen-van-mau 53/217

2.4. Phương trình sai phân tuyn tính cp 2    53

trong đó,  T k(n) ;   Rk(n) là các đa thc bc  k đi vi  n  nào đó.+) Nu  ρ  là nghim ca phương trình đc trưng (3) thì ta chn:

x∗n =  nT k(n). cos nβ  + Rk(n). sin nβ 

trong đó,  T k(n) ;   Rk(n) là các đa thc bc  k đi vi  n  nào đó.

Ví d 2.16.  Gii phương trình sai phân:

x0  = 1 ;   x1 = 0

xn+2  = 2xn+1 − xn + sin nπ

2

Gii.   Ta có P m(n) ≡ 0 ;   P l(n) ≡ 1 ;   β  = π

2 ⇒ ρ =  i.

Phương trình đc trưng: λ2 − 2λ + 1 = 0 có nghim kép λ = 1 (= i  =  ρ).Nghim tng quát ca phương trình thun nht :  xn =  an + b.

Nghim riêng ca phương trình đã cho có dng:  x∗n =  c. cos

 nπ

2   + d. sin

 nπ

2   .Thay x∗n  vào phương trình đã cho, rút gn và so sánh các h s ta đưc:

c = 1

2 ;   d = 0 ⇒ x∗n =

 1

2 cos

 nπ

2

Phương trình đã cho có nghim tng quát là:

xn = an + b + 1

2 cos

 nπ

2

Gii các điu kin biên:

x0 =  b + 1

2 = 1

x1 =  a + b = 0⇔

a = −12

b = 1

2

Vy phương trình đã cho có nghim:

xn = 1

2(1 − n + cos

 nπ

2  )

Ví d 2.17.  Tìm mt nghim riêng ca phương trình sai phân sau:

xn+2 − 3xn+1 + 2xn = (n − 2). cos nπ

2  + (3n + 1). sin

 nπ

2  .

Gii.   Ta có P m(n) = n − 2 ;   P l(n) = 3n + 1 ;   β  = π

2 ⇒ ρ =  i.

Phương trình đc trưng:

λ2 − 3λ + 2 = 0 ⇔ λ = 1 := λ1 ;   λ = 2 := λ2   (λ1; λ2 = ρ).

Page 54: Dãy số - Giới hạn Tác giả: Trần Nam Dũng- Nguyễn Văn Mậu, 2007

8/13/2019 Dãy số - Giớ i hạn Tác giả: Trần Nam Dũng- Nguyễn Văn Mậu, 2007

http://slidepdf.com/reader/full/day-so-gioi-han-tac-gia-tran-nam-dung-nguyen-van-mau 54/217

Page 55: Dãy số - Giới hạn Tác giả: Trần Nam Dũng- Nguyễn Văn Mậu, 2007

8/13/2019 Dãy số - Giớ i hạn Tác giả: Trần Nam Dũng- Nguyễn Văn Mậu, 2007

http://slidepdf.com/reader/full/day-so-gioi-han-tac-gia-tran-nam-dung-nguyen-van-mau 55/217

2.5. Phương trình sai phân tuyn tính cp 3    55

1.   y n+2 − 5y n+1 + 6y n  = 0

2.   8y n+2 − 6y n+1 + y n  = 2n

3.   y n+2 − 3y n+1 + 2y n  = 5n + 2n3 + 3n + 1

4.   y n+2 − y n+1 + 2y n =  n2

5.   y n+2 + y n = sin nπ

2

6.   4y n+2 + 4y n+1 + y n  = 2

7.   y n+2 − 2y n+1 + y n = 5 + 3n

8.   y n+2 − 2y n+1 + y n = 2n(n − 1)

9.   y n+2 − 3y n+1 + 2y n  = 3n

10.   8y n+2 − 6y n+1 + y n  = 5 sin nπ

2

2.5 Phương trình sai phân tuyn tính cp 3

2.5.1 Đnh nghĩa 

Cho a,b, c, d, α,β, γ  là các hng s ∈ R ;   a = 0 ;   d = 0 còn  f (n) là mt hàms bin s  n. Phương trình:

u1  =  α  ;   u2 =  β  ;   u3 =  γ aun+3 + bun+2 + cun+1 + dun =  f (n)   quad(1)

đưc gi là phương trình sai phân tuyn tính cp (bc) ba.

2.5.2 Phương pháp gii

Phương trình sai phân tuyn tính cp ba luôn gii đưc. Nghim tng quátca nó có dng:

un = un + u∗n

trong đó,  un  là nghim tng quát ca phương trình sai phân tuyn tính thun

nht, còn  u∗n là mt nghim riêng nào đó ca phương trình đã cho. Cách tìm  un

Xét phương trình đc trưng:

aλ3 + bλ2 + cλ + d = 0 (2)

Page 56: Dãy số - Giới hạn Tác giả: Trần Nam Dũng- Nguyễn Văn Mậu, 2007

8/13/2019 Dãy số - Giớ i hạn Tác giả: Trần Nam Dũng- Nguyễn Văn Mậu, 2007

http://slidepdf.com/reader/full/day-so-gioi-han-tac-gia-tran-nam-dung-nguyen-van-mau 56/217

2.5. Phương trình sai phân tuyn tính cp 3    56

+) Nu (3) có ba nghim thc phân bit:  λ1 = λ2 = λ3 = λ1 thì:

un  =  C 1λ

n

1  +  C 2λ

n

2  + C 3λ

n

3

+) Nu (3) có mt nghim thc bi 2 và mt nghim đơn:  λ1 =  λ2 = λ3 thì:

un  = (C 1 + C 2n)λn1  +  C 3λn

3

+) Nu (3) có mt nghim thc bi 3:  λ1 =  λ2 =  λ3 :=  λ0 thì:

un  = (C 1 + C 2n + C 3n2)λn0

+) Nu (3) có mt nghim thc  λ1 và hai nghim phc liên hp:λ2;3 =  r(cos θ ± i sin θ) thì

un =  C 1λn1  +  rn(C 2 cos nθ + C 3 sin nθ)

Trên đây ta ký hiu   C 1; C 2; C 3  là các hng s mà s đưc xác đnh bng cáchthay  un vào các điu kin biên và gii h phương trình thu đưc.  Cách tìm  u∗n

Trưng hp 1. Nu  f (n) =  P m(n) là đa thc bc  m đi vi  n thì: +) Khi(3) không có nghim  λ  = 1 thì ta chn:  u∗n = Qm(n) trong đó,  Qm(n) là đa thcbc   m đi vi  n. +) Khi (3) có nghim đơn   λ  = 1  thì ta chn:   u∗n   =   nQm(n)trong đó, Qm(n) là đa thc bc  m  đi vi n. +) Khi (3) có nghim bi hai  λ  = 1thì ta chn:  u∗n  = n2Qm(n) trong đó,  Qm(n) là đa thc bc  m đi vi  n. +) Khi(3) có nghim bi ba  λ  = 1 thì ta chn:  u∗n  =  n3Qm(n)  trong đó,  Qm(n) là đathc bc m đi vi n.

Trưng hp 2. Nu  f (n) = A.µ

n

(  A ;   µ là các hng s cho trưc) thì: +)Khi µ  không là nghim ca (3) thì ta chn:  u∗n  =  B.µn vi B  là hng s đưc xácđnh bng cách thay  u∗n  vào phương trình đã cho. +) Khi  µ là nghim đơn ca(3) thì ta chn:  u∗n   =  B.n.µn. +) Khi  µ  là nghim bi hai ca (3) thì ta chn:u∗n  = B.n2.µn. +) Khi  µ là nghim bi ba (3) thì ta chn:  u∗n = B.n3.µn.

2.5.3 Ví d

Ví d 2.19.  Gii phương trình sai phân:

x1 = 0 ;   x2 = 1 ;   x3  = 3

xn = 7xn−1 − 11xn−2 + 5xn−3

.

Gii.  Phương trình đc trưng:

λ3 − 7λ2 + 11λ − 5 = 0 :   có ba nghim:  λ1 =  λ2 = 1 ;   λ3 = 5.

Vy phương trình có nghim tng quát

xn  = (C 1 + C 2n).1n + C 3.5n = C 1 + C 2n + C 3.5n.

Page 57: Dãy số - Giới hạn Tác giả: Trần Nam Dũng- Nguyễn Văn Mậu, 2007

8/13/2019 Dãy số - Giớ i hạn Tác giả: Trần Nam Dũng- Nguyễn Văn Mậu, 2007

http://slidepdf.com/reader/full/day-so-gioi-han-tac-gia-tran-nam-dung-nguyen-van-mau 57/217

2.5. Phương trình sai phân tuyn tính cp 3    57

Thay vào điu kin biên ta đưc h phương trình:

C 1 + C 2 + 5C 3  = 0

C 1 + 2C 2 + 25C 3  = 1

C 1 + 3C 2 + 125C 3 = 3

C 1  = −1316

C 2  = 3

4

C 3  =  1

80

.

Vy phương trình đã cho có nghim:

xn =  1

16(5n−1 + 12n − 13).

Ví d 2.20.  Gii phương trình sai phân: x0 = 1 ;   x1 = 2 ;   x2  = 3

xn+3 − 3xn+2 + 3xn+1 − xn  = 1

Gii.  Phương trình đc trưng:

λ3 − 3λ2 + 3λ − 1 = 0 :   có nghim bi ba: λ1 =  λ2 =  λ3  = 1.

Vy phương trình thun nht có nghim tng quát

xn  = (C 1 + C 2n + C 3n2).1n = C 1 + C 2n + C 3n2.

Do  f (n) = 1 = 1.1n nên ta chn nghim riêng  x∗n  = B .n3.1n = B .n3. Thay vàophương trình đã cho ri so sánh các h s ta đưc

B  = 1

6 ⇒ x∗n  =

 1

6n3.

Vy nghim tng quát ca phương trình đã cho là:

xn =  x∗n + xn =  C 1 + C 2n + C 3n2 + 1

6n3.

Thay vào các điu kin biên và gii h phương trình thu đưc ta có:

C 1  = 1 ;   C 2  = 4

3 ;   C 3  = −1

2. Vy phương trình đã cho có nghim là

xn = 1 +

 4

3 n − 1

2n2

+

 1

6 n3

.

Page 58: Dãy số - Giới hạn Tác giả: Trần Nam Dũng- Nguyễn Văn Mậu, 2007

8/13/2019 Dãy số - Giớ i hạn Tác giả: Trần Nam Dũng- Nguyễn Văn Mậu, 2007

http://slidepdf.com/reader/full/day-so-gioi-han-tac-gia-tran-nam-dung-nguyen-van-mau 58/217

2.5. Phương trình sai phân tuyn tính cp 3    58

2.5.4 Phương trình sai phân tuyn tính cp  k

Đnh nghĩa  Phương trình

a0y n+k  +  a1y n+k−1 + · · · + aky n =  f (n)   quad(1)

đưc gi là phương trình sai phân tuyn tính cp k. Cách gii A. Gii phương trình sai phâ1o) Gii phương trình đc trưng

a0λk + a1λk−1 + · · · + ak−1λ + ak  = 0 (2) đ tìm  k.

2o) Tìm nghim tng quát ca phương trình thun nht tương ng.

•  Nu (2) có  k  nghim thc khác nhau là  λ1, λ2, · · · , λk  thì nghim tngquát là

y n  = c1λ

n

1  +  c2λ

n

2  + · · · + ckλ

n

k   (3).trong đó  c1, c1, · · · , ck  là các hng s tuỳ ý.

•  Nu (2) có nghim thc  λ j  bi  s  thì nghim tng quát là:

y n  = s−1

i=1

c j+ini

λn j   +

ki=1;i= j

ciλni .

•  Nu phương trình đc trưng (2) có nghim phc đơn  λ j  = r(cos θ + i. sin θ)thì  λ j   = r(cos θ − i. sin θ) cũng là nghim ca (2). Đt  λ j+1  = λ j . Đ thuđưc công thc nghim tng quát, trong công thc (3) ta thay b phn

c jλn j   + c j+1λn

 j+1

bi b phn tương ng:

c jrn cos nθ + c j+1rn sin nθ.

•  Nu phương trình đc trưng (2) có nghim phc bi  s

λ j  = λ j+1 = · · · = λ j+s−1  = r(cos θ + i. sin θ)

thì (2) cũng có nghim phc bi  s  liên hp vi  λ j   là  λ j  mà ta đt là

λ j+s  =  λ j+s+1  = · · · = λ j+2s−1  =  r(cos θ − i. sin θ).

Trong trưng hp này, đ thu đưc công thc nghim tng quát, trong côngthc (3) ta thay b phn

c jλn j   + c j+1λn

 j+1 + · · · + c j+2s−1λn j+2s−1

Page 59: Dãy số - Giới hạn Tác giả: Trần Nam Dũng- Nguyễn Văn Mậu, 2007

8/13/2019 Dãy số - Giớ i hạn Tác giả: Trần Nam Dũng- Nguyễn Văn Mậu, 2007

http://slidepdf.com/reader/full/day-so-gioi-han-tac-gia-tran-nam-dung-nguyen-van-mau 59/217

2.5. Phương trình sai phân tuyn tính cp 3    59

bi b phn tương ng

s−1

i=0

c j+inirn cos nθ +

s−1

i=0

c j+s+inirn sin nθ.

B. Tìm nghim riêng ca phương trình sai phân tuyn tínhkhông thun nht.   Vic tìm nghim riêng ca phương trình sai phân

tuyn tính không thun nht cp   k   làm tương t như tìm nghim riêng caphương trình sai phân tuyn tính không thun nht cp hai và cp ba.

C. Tìm nghim tng quát ca phương trình sai phân tuyn tính cp  k.Nghim tng quát có dng

y n  = y n + y ∗n,trong đó: +)  y n là nghim ca phương trình sai phân tuyn tính cp  k. +)  y n  lànghim ca phương trình thun nht tương ng. +)  y ∗n là mt nghim riêng caphương trình không thun nht.

Page 60: Dãy số - Giới hạn Tác giả: Trần Nam Dũng- Nguyễn Văn Mậu, 2007

8/13/2019 Dãy số - Giớ i hạn Tác giả: Trần Nam Dũng- Nguyễn Văn Mậu, 2007

http://slidepdf.com/reader/full/day-so-gioi-han-tac-gia-tran-nam-dung-nguyen-van-mau 60/217

Chương 3

Xác đnh s hng tng quátca mt dãy s

Vic tính gii hn ca mt dãy s đưc cho bi công thc truy hi thưngphi qua giai đon chng minh s tn ti gii hn ca dãy đã cho và sau đó s dng h thc   lim

n→∞ xn+1   = limn→∞xn  đi vi dãy hi t bt kỳ. Điu đó thưng

đưc thc hin bng cách s dng nguyên lý Weierstrass ( điu kin đ đ dãyhi t ) hoc nguyên lý hi t Bolzano - Cauchy. Quá trình đó gp không ít khókhăn. Mt trong nhng phương thc khc phc khó khăn đó là chuyn t cáchcho dãy bng công thc truy hi sang cho dãy bng phương pháp gii tích, tclà xác đnh dãy bng công thc s hng tng quát ca nó. Bài toán xác đnh shng tng quát ca mt dãy s đưc cho bi h thc truy hi là bài toán thưnggp trong chương trình ph thông. Bài toán đó đưc phát biu như sau. Xác đnhs hng tng quát ca dãy s  (xn) đưc cho bi h thc truy hi.

x1 =  α1  ;   x2 =  α2 ; · · ·   ;   xk  = αk   (∗)

f (xn+k ; xn+k−1; · · · ; xn+1; xn; n) = 0 (1)quad(I )

trong đó  α1; α2; · · · ; αk   là các s ∈  R, cho trưc, còn  f   là mt biu thc chak + 2 bin, cho trưc. Thc cht bài toán đang xét là bài toán xác đnh hàm sxn =  x(n) tho mãn phương trình sai phân  (I ) vi các điu kin biên  (∗). Do đó,đôi khi ta cũng gi bài toán xác đnh dãy s đưc cho bi h thc truy hi  (I ) làbài toán gii phương trình sai phân  (I ).

60

Page 61: Dãy số - Giới hạn Tác giả: Trần Nam Dũng- Nguyễn Văn Mậu, 2007

8/13/2019 Dãy số - Giớ i hạn Tác giả: Trần Nam Dũng- Nguyễn Văn Mậu, 2007

http://slidepdf.com/reader/full/day-so-gioi-han-tac-gia-tran-nam-dung-nguyen-van-mau 61/217

3.1. Tìm s hng tng quát ca dãy (dng đa thc) khi bit các s hng đu tiên   61

3.1 Tìm s hng tng quát ca dãy (dng đa thc)

khi bit các s hng đu tiênVí d 3.1.  Cho dãy s:

1; −1; −1;1;5;11;19;29;41;55; · · · .

Hãy tìm quy lut biu din ca dãy s đó và tìm s tip theo.

Gii.   Lp bng mt s sai phân ban đu:

y  =   1 -1 -1 1 5 11 19 29 41 55∆y    -2 0 2 4 6 8 10 12 14

∆2

y    2 2 2 2 2 2 2 2Ta thy sai phân cp hai không đi nên dãy s là dãy các giá tr ca đa thc bchai:

y  =  an2 + bn + c   (a = 0)

trong đó  n  là s th t ca các s trong dãy s. Cho  n = 0 ; 1 ; 2  (Đánh s cács bt đu t 0) ta nhn đưc h phương trình:

c = 1

a + b + c = −1

4a + 2b + c =

−1

a = 1

b = −3

c = 1

.

Vy dãy s tuân theo quy lut sau:

y n  =  n2 − 3n + 1

S hng đu tiên là  y 0  = 1, s hng tip theo s hng 55 s ng vi  n  = 10 nêns là:

y 10  = 102 − 3.10 + 1 = 71.

Ví d 3.2.  Cho dãy s:

−5; −3; 11; 43; 99; 185; 307; 471; · · · .

Hãy tìm quy lut biu din ca dãy s đó và tìm hai s hng tip theo.

Gii.   Lp bng mt s sai phân ban đu:

Page 62: Dãy số - Giới hạn Tác giả: Trần Nam Dũng- Nguyễn Văn Mậu, 2007

8/13/2019 Dãy số - Giớ i hạn Tác giả: Trần Nam Dũng- Nguyễn Văn Mậu, 2007

http://slidepdf.com/reader/full/day-so-gioi-han-tac-gia-tran-nam-dung-nguyen-van-mau 62/217

3.1. Tìm s hng tng quát ca dãy (dng đa thc) khi bit các s hng đu tiên   62

y  =   -5 -3 11 43 99 185 307 471

∆y    2 14 32 56 86 122 164∆2y    12 18 24 30 36 42∆3y    6 6 6 6 6

Ta thy sai phân cp ba không đi nên dãy s là dãy các giá tr ca đa thc bcba:

y  =  an3 + bn2 + cn + d   (a = 0)

trong đó  n là s th t ca các s trong dãy s. Cho  n = 0 ; 1 ; 2 ; 3 (đánh sth t các s hng bt đu t 0) ta nhn đưc h phương trình:

d = −5

a + b + c + d = −38a + 4b + 2c + d = 11

27a + 9b + 3c + d = 43

⇔ a = 1

b = 3c = −2

d = −5

.

Vy dãy s tuân theo quy lut sau:

y n = n3 + 3n2 − 2n − 5.

S hng đu tiên là y 0  = −5, hai s hng tip theo s hng 471 s ng vi n  = 8; 9nên s là:

y 8

 = 83 + 3.82

−2.8

−5 = 683 ;   y 

9 = 93 + 3.92

−2.9

−5 = 949.

Chú ý: 1) Quy lut tìm đưc trên là không duy nht vì hin nhiên, các s hngđã cho cũng tho mãn, chng hn quy lut:

y n  = n3+3n2−2n−5+P (n).(n+5)(n+3)(n−11)(n−43)(n−99)(n−185)(n−307)(n−471)

trong đó P (x) là mt đa thc bt kỳ. Vy thc cht trên đây ta mi ch tìm đưcmt quy lut mà dãy các s đã cho tho mãn mà không tìm đưc tt c các quylut mà dãy các s đã cho tho mãn. 2) Nh rng  ∆2(ax2 + bx + c) =  Const,nhưng nu ∆2y  = Const thì chưa chc là (không th suy ra đưc) y  =  ax2+bx+c.

Bài tp tương t 

Bài toán 3.1.  1 Vi mi dãy s sau đây hãy: a) Tìm mt quy lut biu din ca dãy s. b) Vit hai s hng tip theo ca mi dãy s theo quy lut va tìm đưc đó:

1. :  1; −2; −2; 1; 7; 16;28;43; 61; · · · .

Page 63: Dãy số - Giới hạn Tác giả: Trần Nam Dũng- Nguyễn Văn Mậu, 2007

8/13/2019 Dãy số - Giớ i hạn Tác giả: Trần Nam Dũng- Nguyễn Văn Mậu, 2007

http://slidepdf.com/reader/full/day-so-gioi-han-tac-gia-tran-nam-dung-nguyen-van-mau 63/217

3.2. Công thc truy hi là mt biu thc tuyn tính   63

2. :  1; 6; 17; 34;57;86; 121; · · · .

3. :  2;3; 7;14; 24; 37;· · · .4. :   3; 5; 10; 18;29;· · · .

5. :  5; 1; 5; 14; 28; 47; 71; 100; 134; 173; 217; · · ·.Bài toán 3.2.  2 Tìm quy lut ca các dãy s sau:

1.   2; 2; 8; 26; 62;122; 212; 338 ; · · · .

2.   1; 6; 17; 34; 57; 86; 121; 162;209; 262 ; · · · .

3.   −5; −3; 11; 43; 99; 185;307; 471; 683;949 ; · · · .

Bài toán 3.3.  3 Tìm công thc s hng tng quát ca dãy s khi bit các s hng đu tiên.

1.   {8; 14;20; 26;32 ; · · · }.

2.   {−0, 5 ; 1, 5 ;  −4, 5 ; 13, 5 ; −40, 5 ; · · · }.

3.   {2; 3

2; 4

3; 5

4; 6

5 ; · · · }.

4.   {1 ;3 ;1 ;3 ; · · · }.

5.   {5;7;11;19;35; · · · }.

6.   {1;2;6;24;120; · · · }.

7.   {−2 ; − 1

2 ; − 4

3  ; −3

4  ; − 6

5  ; · · · }.

8.   {0, 3 ; 0, 33 ; 0, 333 ; · · · }.

9.   {1

2 ;

  1

2 ;

  3

8  ;

  1

4  ;

  5

32 ; · · · }.

3.2 Công thc truy hi là mt biu thc tuyn tính

Trưng hp h thc truy hi đã cho là h thc tuyn tính.

a0xn+k +  a1xn+k−1 + · · · + akxn =  f (n).

vi  a0; a1; · · · ; ak   (a0 = 0 ;   ak = 0) là các hng s thì bài toán có th đưc xem như mt phương trình sai phân tuyn tính và đưc gii như trong chương trưc.Tuy nhiên, cũng có th gii bng các phương pháp khác.

Page 64: Dãy số - Giới hạn Tác giả: Trần Nam Dũng- Nguyễn Văn Mậu, 2007

8/13/2019 Dãy số - Giớ i hạn Tác giả: Trần Nam Dũng- Nguyễn Văn Mậu, 2007

http://slidepdf.com/reader/full/day-so-gioi-han-tac-gia-tran-nam-dung-nguyen-van-mau 64/217

3.2. Công thc truy hi là mt biu thc tuyn tính   64

3.2.1 Ví d

Ví d 3.3.  Tìm s hng tng quát ca dãy s   (xn

)  đưc cho bi h thc truy hi.  

x0  = 99

xn+1  =  xn − 2n − 1 (1).

Gii.   Coi  (1) là phương trình sai phân tuyn tính cp 1. Do  f (n) = −2n − 1 là đa thc bc nht,  λ = 1 ⇒ nên ta chn  x∗n  = n(an + b). Thay vào (1) đưc.

(n + 1)[a(n + 1) + b] =  n(an + b) − 2n − 1 ⇒ a = −1 ;   b = 0 ⇒ x∗n = −n2.

Còn  xn  =  C.1n = C  ⇒ xn  = C − n2, mà  x0 = 99 ⇒ C − 02 = 99 ⇔ C  = 99. Vy phương trình (1) có nghim.  xn  = 99 − n2.Gii (Cách 2).  T h thc đã cho ta có.

x0 = 99

x1 =  x0 − 1

x2 =  x1 − 3

· · · · · · · · ·xn−1  = xn−2 − (2n − 3)

xn = xn−1 − (2n − 1)

Cng tng v các đng thc trên, ta đưc 

xn = 99−

[1 + 3 + 5 +· · ·

+ (2n−

1)] = 99−

n2.

Vy công thc s hng tng quát ca dãy s cn tìm là  xn = 99 − n2.

Ví d 3.4.  Tìm s hng tng quát ca dãy s   (xn)  đưc cho bi h thc truy hi.  

x0 = 8

xn+1  = 2xn + 3n (2).

Gii.   Do   λ  = 2 = 3 =  β  nên ta chn   x∗n   =  d.3n. Thay vào phương trình (2)đưc  d = 1 ⇒ x∗n  = 3n. Còn  xn = C.2n. Vy  xn  = C.2n + 3n. Thay vào điu kin biên đưc  C  = 7.

Tr li: phương trình đã cho có nghim.  xn = 7.2n + 3n.

Gii (Cách khác).   Đt  y n =  xn − 3n, ta đưc y 0 = 8 − 1 = 7

y n+1 + 3.3n = 2(y n + 3n) + 3n  ⇔

y 0 = 7

y n+1  = 2y n

Page 65: Dãy số - Giới hạn Tác giả: Trần Nam Dũng- Nguyễn Văn Mậu, 2007

8/13/2019 Dãy số - Giớ i hạn Tác giả: Trần Nam Dũng- Nguyễn Văn Mậu, 2007

http://slidepdf.com/reader/full/day-so-gioi-han-tac-gia-tran-nam-dung-nguyen-van-mau 65/217

3.2. Công thc truy hi là mt biu thc tuyn tính   65

T đó có  (y n) là cp s nhân  ⇒ y n  = 7.2n ⇒ xn  = 7.2n + 3n là công thc s hng tng quát cn tìm .

Ví d 3.5.  Tìm tt c các dãy s  (an) tho mãn  an+1 = 2n − 3an và  (an) là mt dãy s tăng.

Gii.  Xét phương trình sai phân.  an+1  = 2n − 3an   (3).Đt  an =  un.2n. Thay vào (3) đưc.

un+1.2n+1 = −3.un.2n + 2n ⇔ un+1  = −3

2un +

 1

2.   (3.1)

Phương trình này có nghim tng quát là.

un  = C.(−3

2)n +

 1

5 ⇔ an  =  C.(−3)n +

 1

5.2n.

Ta có.

(an) ↑ ⇔ an+1 > an

⇔ −3C.(−3)n + 2

5.2n > C.(−3)n +

 1

5.2n vi mi  n ∈ N

⇔ 4C.(−3)n < 1

5.2n vi mi  n ∈ N.   (∗)

+) Vi  C > 0  thì  (∗) ⇔   1

20C   > (−3

2)n vi mi  n ∈ N. Ta không chn đưc  C  vì 

khi  n chn thì  (−3

2)n → +∞.

+) Vi  C < 0  thì  (∗) ⇔   120C 

  < (−32

)n vi mi  n ∈ N. Ta cũng không chn đưc 

C  vì khi  n l thì  (−3

2)n → −∞.

+) Vi  C  = 0 thì  an  = 1

5.2n là dãy s tăng.

Vy dãy s cn tìm là.  an  = 1

5.2n.

Ví d 3.6.  Cho  a ;   q   ;   d là các s  ∈  R, cho trưc. Hãy xác đnh s hng tng quát ca dãy s  (un) đưc cho bi công thc truy hi.

u1 =  a

un+1  =  qun + d   (n ≥ 1) .   (4)

Hãy xét tt c các trưng hp có th xy ra đi vi các tham s  a ;   q  ;   d.

(Dãy s đưc cho bi công thc trên còn đưc gi là cp s nhân - cng)

Page 66: Dãy số - Giới hạn Tác giả: Trần Nam Dũng- Nguyễn Văn Mậu, 2007

8/13/2019 Dãy số - Giớ i hạn Tác giả: Trần Nam Dũng- Nguyễn Văn Mậu, 2007

http://slidepdf.com/reader/full/day-so-gioi-han-tac-gia-tran-nam-dung-nguyen-van-mau 66/217

3.2. Công thc truy hi là mt biu thc tuyn tính   66

Gii.+) Nu  q  = 0 thì (4) xác đnh dãy s có công thc s hng tng quát 

u1  =  a  ;   un =  d, ∀ n ∈ N∗, n ≥ 2.

+) Nu  q  = 1 thì (4) xác đnh mt cp s cng có công thc s hng tng quát 

un =  a + (n − 1)d.

+) Nu  d = 0 thì (4) xác đnh mt cp s nhân có công thc s hng tng quát 

un  =  a · q n−1.

+) Ta xét trưng hp  d = 0 ;   q  = 0 ;   q  = 1. Đt  un  := vn + α  (vi  α chn sau)ta đưc 

(4) ⇔ v1

 =  a−

α

vn+1 + α =  q (vn + α) + d   (n ≥ 1) .   (4.1)

Chn  α =  d

1 − q  ta đưc (4.1) là h thc truy hi xác đnh mt cp s nhân vi 

công bi  q  và do đó 

vn =  v1 · q n−1 ⇒ un =

a −   d

1 − q 

q n−1 +

  d

1 − q .

Ví d 3.7.  Cho   a   ;   b   ;   p   ;   q   là các s  ∈  R, cho trưc. Hãy xác đnh s hng tng quát ca dãy s  (un) đưc cho bi công thc truy hi.

u0 =  a  ;   u1 =  bun+1 = ( p + q )un − pqun−1,   (n ≥ 1) (5)

.

Hãy xét tt c các trưng hp có th xy ra đi vi các tham s  a ;   b ;   p ;   q .

Gii.   Đt  vn =  un − pun−1  ta có đưc 

v1  =  u1 − pu0 =  a − pb ;   vn+1  =  qvn ⇒ vn =  v1 · q n−1 (α).

Ap dng liên tip  (α) ta có.

u1 − pu0 =  v1

u2 − pu1 =  v1q u3 − pu2 =  v1q 2

· · · · · · · · ·un − pun−1  =  v1q n−1.

Page 67: Dãy số - Giới hạn Tác giả: Trần Nam Dũng- Nguyễn Văn Mậu, 2007

8/13/2019 Dãy số - Giớ i hạn Tác giả: Trần Nam Dũng- Nguyễn Văn Mậu, 2007

http://slidepdf.com/reader/full/day-so-gioi-han-tac-gia-tran-nam-dung-nguyen-van-mau 67/217

3.2. Công thc truy hi là mt biu thc tuyn tính   67

T các dng thc trên d dàng nhn đưc kt qu sau.

un  =

 pn

−q n

 p − q   b − pq  pn

−1

−q n−1

 p − q    a   nu  p = q 

npn−1b − (n − 1) pna   nu  p =  q .

Ví d 3.8.  Cho  a   ;   b   ;   p   ;   q   ;   r  là các s  ∈  R, cho trưc,   pr = 0. Bit rng phương trình 

 pt2 + qt + r = 0 (α)

có hai nghim thc   t  =  t1   ;   t  =  t2. Hãy xác đnh s hng tng quát ca dãy s (un) đưc cho bi công thc truy hi.

u1 =  a  ;   u2  =  b

 pun+2 + qun+1 + run  = 0 (n≥

1).   (6)

HDG. Chia hai v ca phương trình cho p ri s dng đnh lý Vieete, đưa v ví d 5.

Ví d 3.9.  Xác đnh s hng tng quát ca dãy s đưc cho bi h thc truy hi:x0  = 101

xn+1  = 7.xn + 7n+1 (1.4).

Gii.   Coi (1.4) là phương trình sai phân tuyn tính (cp 1), không thun nht,vi h s hng s. Do   λ  = 7 =   β  nên ta chn   x∗n   =  d.n.7n. Thay vào phương 

trình (1.4) đưc   d  = 1 ⇒  x∗n   =   n.7n. Còn  xn   =  C.7n. Vy   xn   =  C.7n + n.7n.Thay vào điu kin biên đưc  C  = 101.Tr li. phương trình đã cho có nghim.  xn = (101 + n).7n.

Gii (Cách khác).   Đt  xn  = y n.7n. Ta thu đưc h thc truy hi đi vi dãy s  (y n).

x0   = y 0.70

y n+1.7n+1 = 7.y n.7n + 7n+1 ⇔

y 0   = 101

y n+1   = y n + 1

T đó ta thy  (y n) là dãy s cng vi s hng đu  y 0  = 101, công sai  d  = 1. Theocông thc s hng tng quát ca dãy s cng ta đưc 

y n  =  y 0 + n.d ⇔ y n  = 101 + n.

Bi vy xn  = (101 + n).7n

là công thc s hng tng quát ca dãy s cn tìm.

Page 68: Dãy số - Giới hạn Tác giả: Trần Nam Dũng- Nguyễn Văn Mậu, 2007

8/13/2019 Dãy số - Giớ i hạn Tác giả: Trần Nam Dũng- Nguyễn Văn Mậu, 2007

http://slidepdf.com/reader/full/day-so-gioi-han-tac-gia-tran-nam-dung-nguyen-van-mau 68/217

3.2. Công thc truy hi là mt biu thc tuyn tính   68

Ví d 3.10.  Xác đnh công thc s hng tng quát ca dãy s  (xn) đưc cho bi h thc sau .   x0 = 1 ;   x1 = 16

xn+2  = 8xn+1 − 16xn

.

Gii.   Coi h thc đã cho là phương trình sai phân tuyn tính (cp 2) thun nht, vi h s hng s. Ta có phương trình đc trưng.

λ2 − 8λ + 16 = 0   ⇔ λ1 =  λ2 = 4 (có nghim kép).

Vy.  xn  =  xn = (A + Bn).4n.Gii điu kin biên.

x0  = 1

x1  = 16 ⇔ A = 1

(A + B ).4 = 16 ⇔ A = 1

B  = 3.

Vy phương trình đã cho có nghim.  xn = (1 + 3n).4n.Gii.   Đt  xn = y n.4n. Ta thu đưc h thc truy hi đi vi dãy s  (y n).

y 0.40 = x0

y 1.41 = x1

y n+2.4n+2 = 8.y n.4n+1 − 16.y n.4n

y 0   = 1

y 1   = 4

y n+2   = 2y n+1 − y n

Đt tip  z n =  y n+1 − y n   (n ≥ 0) ta đưc 

z 0

 = 3

z n+1  = z n  ∀ n ≥ 0 ⇔ z n  = 3 ∀ n ≥ 0

Như vy, ta đưc h thc truy hi đi vi dãy s  (y n) là.

y 0   = 1

y 1   = 4

y n+1   = y n + 3 (n ≥ 0)

T đó ta thy   (y n)  là dãy s cng vi s hng đu  y 0  = 1, công sai  d = 3. Theocông thc s hng tng quát ca dãy s cng ta đưc 

y n = y 0 + n.d ⇔ y n = 1 + 3n.

Bi vy xn = (1 + 3n).4n

là công thc s hng tng quát ca dãy s cn tìm.

Page 69: Dãy số - Giới hạn Tác giả: Trần Nam Dũng- Nguyễn Văn Mậu, 2007

8/13/2019 Dãy số - Giớ i hạn Tác giả: Trần Nam Dũng- Nguyễn Văn Mậu, 2007

http://slidepdf.com/reader/full/day-so-gioi-han-tac-gia-tran-nam-dung-nguyen-van-mau 69/217

3.2. Công thc truy hi là mt biu thc tuyn tính   69

Bài tp

1. Tìm s hng tng quát ca dãy s  (un) đưc cho bi.un+1 = 3un − 6n + 1 ;   u1 = 1.

Đáp s:  un  = 3n + 1 − 3n.

2. Tìm s hng tng quát ca dãy s  (un) đưc cho bi.

un+1  =  un + 2n2 ;   u1  = 1.

Đáp s:  un  = 1

3(2n3 − 3n2 + n + 3).

3. Tìm s hng tng quát ca dãy s  (un) đưc cho bi.un+1  = 5un − 3n ;   u0 = 1.

Đáp s:  un  = 1

2(5n + 3n).

4. Tìm s hng tng quát ca dãy s  (un) đưc cho bi.

un+1  = 2un + 6.2n ;   u0 = 1.

Đáp s:  un  = (3n + 1).2n.

5. Tìm s hng tng quát ca dãy s  (un) đưc cho bi.un+1  = un + 2n.3n ;   u0  = 0.

Đáp s:  un  = 1

2[(2n − 3).3n + 3].

6. Tìm s hng tng quát ca dãy s  (un) đưc cho bi.

un+1 − 2un = (n2 + 1).2n ;   u0 = 1.

Đáp s:  un  =

n(2n2 − 3n + 7)

6  + 2

.2n.

7. Tìm s hng tng quát ca dãy s  (un) đưc cho bi.

un+1 − 2un  =  n + 3n ;   u0  = 1.

Đáp s.  un  = 2n + 3n − n − 1.

Page 70: Dãy số - Giới hạn Tác giả: Trần Nam Dũng- Nguyễn Văn Mậu, 2007

8/13/2019 Dãy số - Giớ i hạn Tác giả: Trần Nam Dũng- Nguyễn Văn Mậu, 2007

http://slidepdf.com/reader/full/day-so-gioi-han-tac-gia-tran-nam-dung-nguyen-van-mau 70/217

3.3. Công thc truy hi là mt h biu thc tuyn tính   70

3.3 Công thc truy hi là mt h biu thc tuyn tính

Xét bài toán sau: Xác đnh s hng tng quát ca các dãy s  (xn) ; (y n) tho mãn h thc truy hi dng.

x1 =  a  ;   y 1  =  b   (∗)

xn+1  =  pxn + qy n   (1)

y n+1  =  rxn + sy n   (2)

(I )   vi  a; b; p; q ; r; s là các hng s  ∈ R

Phương pháp gii.  Trong (1) thay  n bi  n + 1  và bin đi ta đưc.

xn+2 = pxn+1 + qy n+1

= pxn+1 + q (rxn + sy n)

= pxn+1 + qrxn + s(xn+1 − pxn)

⇒xn+2 − ( p + s)xn+1 + ( ps − qr)xn = 0

T (1) ta cũng có  x2  =  px1 +  qy 1  =  pa + qb. Vy ta thu đưc phương trình sai phân tuyn tính cp hai thun nht:

x1 =  a  ;   x2 =  pa + qb

xn+2 − ( p + s)xn+1 + ( ps − qr)xn  = 0

Mà ta đã bit cách gii chương trưc. Gii phương trình này ta tìm đưc  xn.Thay vào (1) ta tìm đưc  y n.

3.3.1 Ví d

Ví d 3.11.  Tìm  xn ;   y n  tho mãn.

x1  = 1 ;   y 1  = 1

xn+1  = 4xn − 2y n   (1)

y n+1  = xn + y n   quad(2)

.

Gii.  Trong (1) thay  n bi  n + 1  ta đưc 

xn+2

 = 4xn+1 −

2y n+1

= 4xn+1 − 2(xn + y n) = 4xn+1 − 2xn − 2y n

= 4xn+1 − 2xn + xn+1 − 4xn = 5xn+1 − 6xn

⇒ xn+2 − 5xn+1 + 6xn = 0.

Page 71: Dãy số - Giới hạn Tác giả: Trần Nam Dũng- Nguyễn Văn Mậu, 2007

8/13/2019 Dãy số - Giớ i hạn Tác giả: Trần Nam Dũng- Nguyễn Văn Mậu, 2007

http://slidepdf.com/reader/full/day-so-gioi-han-tac-gia-tran-nam-dung-nguyen-van-mau 71/217

3.3. Công thc truy hi là mt h biu thc tuyn tính   71

T (1) ta có:  x2  = 4x1 − 2y 1  = 4.1 − 2.1 = 2. Vy ta có phương trình sai phân tuyn tính thun nht:

x1 = 1 ;  x2  = 2

xn+2 − 5xn+1 + 6xn = 0.

Gii phương trình này ta đưc  xn  = 2n−1. Thay  xn vào (1) đưc  y n  = 2n−1. Vy 

h đã cho có nghim.

xn = 2n−1

y n  = 2n−1  .

Ví d 3.12.  Tìm  xn ;   y n  tho mãn.

x0  = 2 ;   y 0  = 0

4xn+1  = 2xn − 3y n   (1)2y n+1  = 2xn + y n   quad(2)

.

Gii.   Lp lun tương t ví d trên ta đưc phương trình sai phân tuyn tính thun nht:  

x0 = 2 ;   x1 = 1

xn+2 − xn+1 + xn = 0.

Phương trình đc trưng.

λ2 − λ + 1 = 0 ⇔ λ = cos π

3 ± i sin

 π

3.

Vy phương trình trên có nghim tng quát.

xn = A. cos nπ

3  + B. sin

 nπ

3  .

Thay vào điu kin biên đưc  A = 2 ;   B  = 0 ⇒ xn = 2 cos nπ

3  . Thay tip vào (1)

đưc  y n  =  4√ 

3sin

 nπ

3  .

Vy h đã cho có nghim:

xn = 2cos nπ

3

y n  =  4√ 

3sin

 nπ

3

. Bài tp Tìm  xn, y n  tho mãn:

1.

x0   = 2; y 0 = 2

xn+1   = 1

2xn − 3

4y n

y n+1   = xn + 1

2y n

.

Page 72: Dãy số - Giới hạn Tác giả: Trần Nam Dũng- Nguyễn Văn Mậu, 2007

8/13/2019 Dãy số - Giớ i hạn Tác giả: Trần Nam Dũng- Nguyễn Văn Mậu, 2007

http://slidepdf.com/reader/full/day-so-gioi-han-tac-gia-tran-nam-dung-nguyen-van-mau 72/217

3.4. Công thc truy hi là biu thc tuyn tính vi h s bin thiên   72

2.

x0   = 0; y 0 = 6

xn+1   = 3xn + y n

y n+1   = 5xn − y n

.

3.

x0   = 2; y 0 = 1

xn+1   = 2xn − y n

y n+1   = xn + 4y n

.

4.

x0   = −1; y 0 = 2

xn+1   = 2xn − 8y n

y n+1   = 2xn − 6y n

.

5.x0   = 1; y 0 = 1

xn+1   = 4xn − 2y n + 9n − 3

y n+1   = xn + y n + 3n

.

6.

x0   = 1; y 0 = 1 −   1√ 2

xn+1   = xn − y n

y n+1   = xn + y n

.

3.4 Công thc truy hi là biu thc tuyn tính vi h

s bin thiênLý thuyt v phương trình sai phân tuyn tính vi các h s bin thiên cho

đn nay vn chưa hoàn chnh. Vic gii các phương trình sai phân tuyn tính vi các h s bin thiên là rt phc tp. Trong phn này ta s ch xét mt s dng đc bit, đơn gin ca các phương trình sai phân tuyn tính vi các h s bin thiên ch yu bng phương pháp đt dãy s ph, đưa v phương trình sai phân tuyn tính.

Ví d 3.13.  Tìm  un  bit rng.

u1 = 0 ;   un+1  =  n

n + 1(un + 1)  vi mi  n ≥ 1.

Gii.  T gi thit có:  (n + 1)un+1 =  nun + n. Đt  xn  =  nun, ta có 

x1  = 0 ;   xn+1  =  xn + n.

Page 73: Dãy số - Giới hạn Tác giả: Trần Nam Dũng- Nguyễn Văn Mậu, 2007

8/13/2019 Dãy số - Giớ i hạn Tác giả: Trần Nam Dũng- Nguyễn Văn Mậu, 2007

http://slidepdf.com/reader/full/day-so-gioi-han-tac-gia-tran-nam-dung-nguyen-van-mau 73/217

3.4. Công thc truy hi là biu thc tuyn tính vi h s bin thiên   73

Gii phương trình này ta đưc:

xn =  n(n − 1)2   . Vy ta có:  un =  n − 12   .

Ví d 3.14.  Tìm  un  bit rng.

u1 = 0 ;   un+1  =  n(n + 1)

(n + 2)(n + 3)(un + 1)  vi mi  n ≥ 1.

Gii.  T gi thit có:

(n + 1)(n + 2)2(n + 3)un+1  =  n(n + 1)2(n + 2)un + n(n + 1)2(n + 2).

Đt  xn  =  n(n + 1)2(n + 2)un, ta có 

x1 = 0 ;   xn+1  =  xn + n(n + 1)2(n + 2).

Gii phương trình này ta đưc:

xn  = (n − 1)n(n + 1)(n + 2)(2n + 1)

10  .

Vy ta có đáp s:

un = (n − 1)(2n + 1)

10(n + 1)  .

Ví d 3.15.  Tìm:

J n  = π2

0sinn x dx.

Gii.  S dng công thc tích phân tng phn ta có.

J n = − cos x. sinn−1 xπ

20

+

 π

2

0

(n − 1) sinn−2 x cos2 x dx

=

 π

2

0(n − 1) sinn−2 x(1 − sin2 x) dx

= (n − 1)  π

20

sinn−2 x − sinn x   dx

= (n − 1)(J n−2 − J n).

⇒ J n+2 = n + 1

n + 2J n   (1).

Page 74: Dãy số - Giới hạn Tác giả: Trần Nam Dũng- Nguyễn Văn Mậu, 2007

8/13/2019 Dãy số - Giớ i hạn Tác giả: Trần Nam Dũng- Nguyễn Văn Mậu, 2007

http://slidepdf.com/reader/full/day-so-gioi-han-tac-gia-tran-nam-dung-nguyen-van-mau 74/217

3.4. Công thc truy hi là biu thc tuyn tính vi h s bin thiên   74

D thy:  J 0 = π

2  ;   J 1  = 1. T đó và t (1) ta có 

+) Khi  n chn ( n = 2k) thì 

J 2 = 1

2J 0

J 4 = 3

4J 2

J 6 = 5

6J 4

· · · · · · · · ·J 2k  =

 2k − 1

2k  J 2k−2.

Nhân tng v các đng thc trên và rút gn ta đưc.

J 2k  = 1.3. · · · .(2k − 1)

2.4. · · · .(2k)  .

π

2  =

 (2k − 1)!!

(2k)!!  .

π

2.

+) Khi  n l ( n = 2k + 1)  thì.

J 3  = 2

3J 1

J 5  = 4

5J 3

J 7  = 6

7J 5

· · · · · · · · ·J 2k+1  =

  2k

2k + 1J 2k−1.

Nhân tng v các đng thc trên và rút gn ta đưc.

J 2k+1  =  2.4. · · · .(2k)

1.3. · · · .(2k + 1).1 =

  (2k)!!

(2k + 1)!!.

Đáp s.

J 2k  = (2k − 1)!!

(2k)!!  .

π

2  ;   J 2k+1 =

  (2k)!!

(2k + 1)!!.

Ví d 3.16.  Tìm  xn  bit rng.x1 =  a > 0 ;   xn+1  =  g(n).xk

n   (1)  vi mi  n ≥ 1,

trong đó  g(n) >  0  vi mi  n ∈ N∗ ;   k ∈ R+.

Page 75: Dãy số - Giới hạn Tác giả: Trần Nam Dũng- Nguyễn Văn Mậu, 2007

8/13/2019 Dãy số - Giớ i hạn Tác giả: Trần Nam Dũng- Nguyễn Văn Mậu, 2007

http://slidepdf.com/reader/full/day-so-gioi-han-tac-gia-tran-nam-dung-nguyen-van-mau 75/217

3.4. Công thc truy hi là biu thc tuyn tính vi h s bin thiên   75

Gii.  T gi thit suy ra  xn  > 0  vi mi  n ∈ N∗. Ly logarit Neper hai v ca (1) ta đưc 

ln xn+1  = ln g(n) + k. ln xn.   (2)

Đt  y n  = ln xn, khi đó (2) có dng.

y n+1 − ky n  = ln g(n).   (3)

Đt tip  y n  =  kn−1un khi đó (3) có dng.

un+1 − un =  ln g(n)

kn  ⇒ un = u1 +

n−1i=1

ln g(i)

ki  .

T gi thit  x1  =  a > 0 ⇒ u1 = ln a. Vy.

un  = ln a +

n

−1

i=1

ln g(i)ki

  ⇒ y n  = kn−1ln a +

n

−1

i=1

ln g(i)ki

.

Cui cùng, ta có:

xn =  ekn−1(lna+   n−1

i=1ln g(i)

ki  )

= exp

kn−1(ln a +

n−1i=1

ln g(i)

ki  )

.

Ví d 3.17.  Tìm  xn  bit rng.

x1  =  a > 0 ;   xn+1 = f (n + 1)

f k(n)  .xk

n   (1)  vi mi  n ≥ 1.

Trong đó  f (n) >  0  vi mi  n ∈ N∗  còn  k ∈ N∗, cho trưc.

Gii.  T (1) ta có:xn+1

f (n + 1) =

  xkn

f k(n).   (2)

Đt dãy ph:   vn   =  xn

f (n), khi đó (2) có dng:   vn+1   =  vk

n.   (3). Đt tip dãy s 

ph:  un  = ln vn, khi đó (3) có dng.

un+1  =  kun ⇒ un =  C.kn (Vi  C  là hng s).

Mà  x1 =  a ⇒ v1  =   af (1)

 ⇒ u1 = ln   af (1)

 = C.k ⇒ un = ln   af (1)

kn−1. Vy ta có.

vn = ekn−1 ln

a

f (1) =

  a

f (1)

kn−1

.  Hay là.   xn =  f (n)(  a

f (1))k

n−1.

Page 76: Dãy số - Giới hạn Tác giả: Trần Nam Dũng- Nguyễn Văn Mậu, 2007

8/13/2019 Dãy số - Giớ i hạn Tác giả: Trần Nam Dũng- Nguyễn Văn Mậu, 2007

http://slidepdf.com/reader/full/day-so-gioi-han-tac-gia-tran-nam-dung-nguyen-van-mau 76/217

3.4. Công thc truy hi là biu thc tuyn tính vi h s bin thiên   76

Ví d 3.18.  Tìm s hng tng quát ca dãy s  (xn) bit rng  x1 =  a  và 

xn+1  = a(n)xn + b(n)   quad(6)

trong đó  a(n) ;   b(n) là các hàm s đi vi  n ∈ N,  a(n) = 0, ∀ n ∈ N.

Gii.  Đt dãy s ph 

xn = y n.n−1k=0

a(k)

Khi đó ta có  y 1  =  a

a(0) và 

(6)

⇔y n+1

−y n  =

  b(n)

n

k=0a(k)

 := g(n). quad(6.1)

T đng thc  (6.1) ta d dàng nhn đưc 

y n  = y 1 +n−1k=1

g(k) =  a

a(0) +

n−1k=1

g(k).

Vy nên ta có 

xn  =   a

a(0) +

n−1k=1

b(k)k j=0 a( j)

n−1k=0

a(k).

là công thc s hng tng quát cn tìm.

Ví d 3.19.  Tìm s hng tng quát ca dãy s  (xn) bit rng  x1  =  a ;   x2  = bvà 

xn+2  =  a(n)xn+1 + b(n)xn + f (n)   quad(7)

trong đó  a(n) ;   b(n) ;   f (n) là các hàm s đi vi  n ∈ N,  b(n) = 0, ∀ n ∈ N và tn ti s  p = 0, tn ti hàm s  q (n) = 0 ∀ n ∈ N sao cho:

 p + q (n) =  a(n) ;   p.q (n) = −b(n).   (∗)

Gii.  S dng điu kin  (∗) ta có th vit li (7) dưi dng 

(xn+2 − pxn+1) − q (n)(xn+1 − pxn) =  f (n)   quad(7.1)

Đt  y n  = xn+1 − pxn   (∗1). Khi đó, (7.1) có dng.

y n+1  = q (n)y n + f (n) ;   y 1  =  b − pa :=  α.

Page 77: Dãy số - Giới hạn Tác giả: Trần Nam Dũng- Nguyễn Văn Mậu, 2007

8/13/2019 Dãy số - Giớ i hạn Tác giả: Trần Nam Dũng- Nguyễn Văn Mậu, 2007

http://slidepdf.com/reader/full/day-so-gioi-han-tac-gia-tran-nam-dung-nguyen-van-mau 77/217

3.4. Công thc truy hi là biu thc tuyn tính vi h s bin thiên   77

Theo ví d 6, ta tìm đưc 

y n  =   α

q (0) +

n−1

k=1

f (k)k j=0 q ( j)

n−1

k=0

q (k) :=  h(n), n > 1.

Thay   y n  vào   (∗1), gii phương trình sai phân tuyn tính cp 1 thu đưc ta tìm đưc công thc s hng tng quát cn tìm là.

xn  = a.pn−1 + pn.n−1k=1

h(k)

 pk+1, n >  1.

Ví d 3.20.  Tìm s hng tng quát ca dãy s  (xn) nu bit 

x1  = 1 ;   x2  = 2 ;   xn+2  = (n + 1)(xn+1 + xn), n ≥ 1.   (7)

Gii.   Đt  xn = n!y n. Khi đó, (7) có dng 

n!y n = (n − 1)[(n − 1)!xn−1 + (n − 2)!xn−2]

hay là 

ny n − (n − 1)y n−1 − y n−2  = 0 ⇔ (y n − y n−1  = − 1

n(y n−1 − y n−2), ∀ n ≥ 2.

T đó có 

y n − y n−1  =

 (

−1)n−1

n!   , y 3 − y 2 =

 1

6 , y 2 − y 1  = −1

2 .

Vit các đng thc trên ri cng li ta đưc 

y n  = y 1 +n−1k=2

(−1)k−1

k!  = 1 +

n−1k=2

(−1)k−1

k!  , n ≥ 3.

Vy công thc s hng tng quát cn tìm là 

xn  =  n!

1 +

n−1k=2

(−1)k−1

k!

, n ≥ 3.

Bài tp

Tìm  xn  bit rng:

Page 78: Dãy số - Giới hạn Tác giả: Trần Nam Dũng- Nguyễn Văn Mậu, 2007

8/13/2019 Dãy số - Giớ i hạn Tác giả: Trần Nam Dũng- Nguyễn Văn Mậu, 2007

http://slidepdf.com/reader/full/day-so-gioi-han-tac-gia-tran-nam-dung-nguyen-van-mau 78/217

3.5. Công thc truy hi dng phân tuyn tính vi h s hng    78

1.

x1   = 0

xn+1   =  n

n + 1(xn + 1)

 .

2.

x1   = 0

xn+1   =  n(n + 1)

(n + 2)(n + 3)(xn + 1)

 .

3.

x1   = 1

2

xn+1   =  (n + 1)2

n(n + 2)xn +

 n(n + 1)

n + 2  · n!

.

4. x1   = a

xn+1   = (n + 1)(xn + 1).

5.

x1   = 2

xn+1   =  n

n + 2xn +

  4.3n

(n + 1)(n + 2)

.

6.

x1 = 8 ;   x2  = 33

2

xn+2 − 2(n + 1)(n + 3)

(n + 2)2  xn+1 − 3

  n(n + 2)

(n + 1)(n + 3)xn = −4

n(n + 3)

n + 2

.

3.5 Công thc truy hi dng phân tuyn tính vi h

s hngTrong phn này ta s tìm s hng tng quát ca dãy s đưc cho dưi dng 

công thc truy hi dng phân tuyn tính vi h s hng thông qua các ví d c th.

Ví d 3.21.  Tìm dãy s  (xn) tho mãn các điu kin sau.

x1 =  a > 0 ;   xn+1  =  xn

xn + 2 vi mi  n ≥ 1.

Gii.  T gi thit suy ra  xn  >  0  vi mi  n ∈ N∗. Mà.

xn+1  =  xn

xn + 2 ⇔  1

xn+1

= 1 +  2

xn

.

Đt  y n  =  1

xn, khi đó ta đưc.

y n+1  = 2y n + 1 ⇔ y n+1 − 2y n − 1 = 0 ;   y 1  =  1

a.

Page 79: Dãy số - Giới hạn Tác giả: Trần Nam Dũng- Nguyễn Văn Mậu, 2007

8/13/2019 Dãy số - Giớ i hạn Tác giả: Trần Nam Dũng- Nguyễn Văn Mậu, 2007

http://slidepdf.com/reader/full/day-so-gioi-han-tac-gia-tran-nam-dung-nguyen-van-mau 79/217

3.5. Công thc truy hi dng phân tuyn tính vi h s hng    79

Gii phương trình sai phân này ta đưc:

y n  = (a + 1)2n

−1

− aa

  .   Hay là.   xn  =   a(a + 1)2n−1 − a

Ví d 3.22.  Tìm dãy s  (xn) tho mãn các điu kin sau.

x0  =  a  ;   xn+1 = pxn + q 

rxn + s  vi mi  n ∈ N.   (1)

trong đó  a, p, q,r,s ∈ R, cho trưc.

Gii.  Li gii ca ví d này thu đưc trc tip t B đ sau.

B đ 3.1.  Nu  y n  và  z n  là nghim ca h phương trình sai phân.y n+1 = py n + qz n   ;   y 0 =  a

z n+1  =  ry n + sz n   ;   z 0 = 1.   (2)

thì  xn = y nz n

là nghim ca phương trình.

x0  =  a  ;   xn+1  = pxn + q 

rxn + s.

Chng minh.  Tht vy, ta có:  x0 = y 0

z 0

= a

1

 = a. Ngoài ra.

xn+1  = y n+1

z n+1=

 py n + qz nry n + sz n

= p

y nz n

+ q 

ry nz n

+ s=

 pxn + q 

rxn + s.

T đó suy ra đpcm. T B đ trên ta có đưc cách gii ca phương trình sai phân dng phân tuyn tính (1) bng cách lp và gii h phương trình (2). T đó thu đưc nghim ca (1) theo B đ. Ví d c th xem trong li gii ca ví d 3 sau đây.

Ví d 3.23.  Tìm dãy s  (xn) tho mãn các điu kin sau.

x0 = 0 ;   xn+1  =  xn + 1

−xn + 1  vi mi  n ≥ 1.

Page 80: Dãy số - Giới hạn Tác giả: Trần Nam Dũng- Nguyễn Văn Mậu, 2007

8/13/2019 Dãy số - Giớ i hạn Tác giả: Trần Nam Dũng- Nguyễn Văn Mậu, 2007

http://slidepdf.com/reader/full/day-so-gioi-han-tac-gia-tran-nam-dung-nguyen-van-mau 80/217

3.5. Công thc truy hi dng phân tuyn tính vi h s hng    80

Gii.  Xét h phương trình 

y 0  = 0 ;   z 0 = 1y n+1  = y n + z n

z n+1  = −y n + z n

.

Gii h này ta đưc.

y n = (√ 

2)n sin nπ

4  ;   z n = (

√ 2)n cos

 nπ

4  .

T đó, theo B đ đã chng minh trên ta đưc nghim ca phương trình đã cholà.

xn  =

 y nz n =

(√ 

2)n sin nπ

4

(√ 2)n cos nπ4

= tg

4   .

Ví d 3.24.  Tìm công thc s hng tng quát ca dãy s  (xn) đưc cho bi h thc truy hi sau 

x1  =  a  ;   xn+1  =  bxn

cxn + d  quad(4).

Trong đó,  a, b, c ∈ R∗, d ∈ R.

Gii.   Đt  y n :=  1

xn;

  c

b  := p  ;

  d

b  := q  ta đưc 

(4) ⇔ y n+1  = pxn + q  ;   y 1  =  1a .

Đó là h thc xác đnh dãy s nhân - cng. Công thc s hng tng quát ca dãy s này đã đưc xác đnh trong phn trưc (xem ví d 4 mc 2.1 ca chương này).

Ví d 3.25.  Xác đnh s hng tng quát ca dãy s cho bi h thc truy hi sau.

x1 = 1

2 ;   xn+1  =

  1

2 − xn.

Gii.  Ta xác đnh mt s s hng đu tiên.

x1  = 1

2 ;   x2 = 2

3 ;   x3  = 3

4 .

Ta s chng minh dãy s đã cho có s hng tng quát 

xn =  n

n + 1  quad(∗)

Page 81: Dãy số - Giới hạn Tác giả: Trần Nam Dũng- Nguyễn Văn Mậu, 2007

8/13/2019 Dãy số - Giớ i hạn Tác giả: Trần Nam Dũng- Nguyễn Văn Mậu, 2007

http://slidepdf.com/reader/full/day-so-gioi-han-tac-gia-tran-nam-dung-nguyen-van-mau 81/217

3.6. H thc truy hi phi tuyn   81

bng phương pháp quy np. Tht vy, theo trên,   (∗) đã đúng ti  n  = 3. Gi s (

∗) đúng ti  n, khi đó 

xn+1 =  1

2 − xn=

  1

2 −   n

n + 1

= n + 1

n + 2.

Vy  (∗) cũng đúng ti  n + 1  nên theo nguyên lý quy np toán hc,  (∗) đúng vi mi  n ∈ N∗. Đó chính là điu phi chng minh.

Bài tp

Tìm dãy s tho mãn điu kin sau:

1.   xn+1  = 1 − 4xn1 − 6xn

;   x0 = 1.

2.   xn+1  = 2xn − 2

3xn − 4  ;   x0 = −1.

3.   xn+1  =  xn + 1

−xn + 1 ;   x0 = 0.

4.   xn+1  = xn − 2

xn + 4 ;   x0 = 0.

5.   xn+1  = xn − 1

xn + 3 ;   x0 = 1.

6.   xn+1  = xn − 3

xn + 1 ;   x0 = 0.

7.   xn+1  =  xn

2xn + 1  ;   x0 = 0.

3.6 H thc truy hi phi tuyn

Trong phn này ta xét các ví d gii các phương trình sai phân phi tuyn. Lý thuyt tng quát gii các phương trình dng này cho đn nay còn chưa xây dng đưc. Trong phn này ch yu ta s xét các phương trình có th đưc tuyn tính 

hoá bng phép đt hàm ph hoc bng phương pháp quy np toán hc.

Page 82: Dãy số - Giới hạn Tác giả: Trần Nam Dũng- Nguyễn Văn Mậu, 2007

8/13/2019 Dãy số - Giớ i hạn Tác giả: Trần Nam Dũng- Nguyễn Văn Mậu, 2007

http://slidepdf.com/reader/full/day-so-gioi-han-tac-gia-tran-nam-dung-nguyen-van-mau 82/217

3.6. H thc truy hi phi tuyn   82

3.6.1 Quy trình tuyn tính hoá mt phương trình sai phân

Tuyn tính hoá mt phương trình sai phân nghĩa là đưa mt phương trình sai phân dng phi tuyn v dng tuyn tính. Gi s dãy s   (un)   tho mãn điu kin.  

u1  =  α1  ;   u2  = α2 · · ·  uk  = αk

un  =  f (un−1, un−2, · · · , un−k)   vi   n; k ∈ N∗  ;   n > k.

Trong đó  f  là mt đa thc đi s bc  m  hoc là phân thc, hoc là biu thc siêu vit. Gi s hàm s  f (un−1, un−2, · · · , un−k)  có th tuyn tính hoá đưc, khi đó tn ti các giá tr  x1; x2; · · · ; xk  sao cho.

un = x1un−1 + x2un−2 + · · · + xkun−k   (1)

Đ tìm  x1; x2; · · · ; xk  trưc ht ta xác đnh  uk+1; uk+2; · · · ; u2k.T công thc lp đã cho ta có.

uk+1  =  f (αk; αk−1; · · · ; α2; α1) :=  αk+1

uk+2  =  f (αk+1; αk; · · · ; α3; α2) :=  αk+2

· · ·   quad · · ·   quad · · ·   quad

u2k  =  f (α2k−1; α2k−2; · · · ; αk+1; αk) := α2k

.

Thay các giá tr  u1; u2; · · · ; uk  đã cho và các giá tr  uk+1; uk+2 · · · ; u2k  va tìm đưc trên vào (1) ta đưc h phương trình tuyn tính gm  k  phương trình vi k n  x1; x2; · · · ; xk.

uk+1  =  x1αk + x2αk−1 + · · · + xkα1

uk+2  =  x1αk+1 + x2αk + · · · + xkα2

· · ·   quad · · ·   quad · · ·   quad

u2k  = x1α2k−1 + x2α2k−2 + · · · + xkαk

.   (∗)

Gii h phương trình này ta thu đưc nghim: x1; x2; · · · ; xk. Thay vào (1) ta s đưc biu din tuyn tính cn tìm.

un =  f (un−1, un−2, · · · , un−k) =  x1un−1 + x2un−2 + · · · + xkun−k

Sau đó ta chng minh công thc biu din trên bng phương pháp quy np toán hc.Chú ý. Nu h (*) vô nghim thì hàm  f  không th tuyn tính hoá đưc.

Page 83: Dãy số - Giới hạn Tác giả: Trần Nam Dũng- Nguyễn Văn Mậu, 2007

8/13/2019 Dãy số - Giớ i hạn Tác giả: Trần Nam Dũng- Nguyễn Văn Mậu, 2007

http://slidepdf.com/reader/full/day-so-gioi-han-tac-gia-tran-nam-dung-nguyen-van-mau 83/217

3.6. H thc truy hi phi tuyn   83

3.6.2 Ví d

Ví d 3.26.  Cho dãy s  (an

) tho mãn.

a1 =  a2 = 1 ;   an = a2

n−1 + 2

an−2(∗)  vi mi  n ≥ 3

Hãy tuyn tính hoá, tìm s hng tng quát. Chng minh rng  an nguyên vi mi n ∈ N∗.

Gii.   Gi s  an  có biu din tuyn tính là:

an = αan−1 + βan−2 + γ.   (1)

Ta có.

a3 = a2

2 + 2

a1=

 1 + 2

1  = 3.

a4 = a2

3 + 2

a2=

 9 + 2

1  = 11.

a5 = a2

4 + 2

a3=

 121 + 2

3  = 41.

Thay  a3 = 3 ;   a4  = 11 ;   a5  = 41 vào (1) ta thu đưc h phương trình.

αa2 + βa1 + γ  = a3αa3 + βa2 + γ  = a4

αa4 + βa3 + γ  = a5

⇔α + β  + γ  = 3

3α + β  + γ  = 11

11α + 3β  + γ  = 41

⇔α = 4

β  = −1

γ  = 0

.

Vy ta có:an = 4an−1 − an−2   (2)

Ta s chng minh dãy s  (an) tho mãn (*) có biu din tuyn tính là.

a1 =  a2  = 1 ;   an = 4an−1 − an−2  vi mi  n ≥ 3.   (3)

Tht vy, vi  n = 3 ta có:  a3  = 4.a2 − a1 = 4.1 − 1 ⇒ (2) đúng vi  n = 3.

Page 84: Dãy số - Giới hạn Tác giả: Trần Nam Dũng- Nguyễn Văn Mậu, 2007

8/13/2019 Dãy số - Giớ i hạn Tác giả: Trần Nam Dũng- Nguyễn Văn Mậu, 2007

http://slidepdf.com/reader/full/day-so-gioi-han-tac-gia-tran-nam-dung-nguyen-van-mau 84/217

3.6. H thc truy hi phi tuyn   84

Gi s (2) đúng ti  n =  k  tc là: ak  = 4ak−1 − ak−2   (k ≥ 3). Ta có.

ak+1  =  a2

k + 2ak−1

= (4ak−1 − ak−2)2

+ 2ak−1

=16a2

k−1 − 8ak−1ak−2 + a2k−2 + 2

ak−1

=15a2

k−1 − 4ak−1ak−2 + a2k−1 − 4ak−1ak−2 + ak−1ak−3

ak−1

( Nh rng:  a2k−2 + 2 = ak−1ak−3)

=15a2

k−1 − 4ak−1ak−2 + ak−1(ak−1 − 4ak−2 + ak−3)

ak−1

=

15a2k

−1

−4ak−1ak−2

ak−1(Do  ak−1 − 4ak−2 + ak−3  = 0)

= 15ak−1 − 4ak−2  = 4(ak−1 − ak−2) − ak−1

= 4ak − ak−1.

Vy (2) cũng đúng ti  n  =  k  + 1. Theo nguyên lý quy np ta đưc (2) đúng vi mi  n ∈ N ;   n ≥ 3.T (3) ta thy ngay  ∀ n ∈ N∗ :  an ∈ Z. Ngoài ra, ta đã chng minh đưc.

a1  =  a2  = 1

an  =

 a2n−1 + 2

an−2 (n ≥ 3)

(

∗)

⇔ a1  = a2 = 1

an  = 4an−1 − an−2   (n ≥ 3)

.   (

∗∗)

Đ tìm s hng tng quát ta gii phương trình (**). Có phương trình đc trưng:

λ2 − 4λ + 1 = 0 ⇔ λ = 2 +√ 

3   hoc    λ = 2 − √ 3.

Do đó:an = a.(2 +

√ 3)n + b.(2 −

√ 3)n.   (4)

Thay vào điu kin biên ta tìm đưc 

a = 1

2(3 −   5√ 

3

) ;   b = 1

2(3 +

  5√ 3

).

Vy ta có s hng tng quát cn tìm là.

an  = 1

2

(3 −   5√ 

3)(2 +

√ 3)n + (3 +

  5√ 3

)(2 −√ 

3)n

.

Page 85: Dãy số - Giới hạn Tác giả: Trần Nam Dũng- Nguyễn Văn Mậu, 2007

8/13/2019 Dãy số - Giớ i hạn Tác giả: Trần Nam Dũng- Nguyễn Văn Mậu, 2007

http://slidepdf.com/reader/full/day-so-gioi-han-tac-gia-tran-nam-dung-nguyen-van-mau 85/217

3.6. H thc truy hi phi tuyn   85

Ví d 3.27.  Cho dãy s  (un) tho mãn.

u1  =  α  ;   un+1  =  aun +  bu

2

n + c   vi  a2

− b = 1 ;   α > 0 ;   a > 1 (∗)Hãy tuyn tính hoá dãy s trên.

Gii.

un+1  = aun + 

bu2n + c ⇔ un+1 − aun =

 bu2

n + c

⇒ (un+1 − aun)2 = bu2n + c

⇒ u2n+1 + (a2 − b)u2

n = 2aun+1un + c

⇒ u2n+1 + u2

n = 2aun+1un + c   (1)

⇒u2n

−u2n

−1  = 2aunun

−1 + c   (2)

Tr tng v (1) và (2) ta đưc .

u2n+1 − u2

n−1  = 2aun(un+1 − un−1)

Mà  un+1 − un−1  > 0  nên suy ra.

un+1 − 2aun + un−1  = 0

Nói cách khác:

(∗) ⇔

u1 =  α  ;   u2 =  aα +√ 

bα2 + c

un+1 − 2aun + un−1  = 0

Như vy vic tuyn tính hoá đã thc hin xong.

Ví d 3.28.  Cho dãy s  (xn) tho mãn.

x1  =  α  ;   xn+1  =  xn

a + 

x2n + b

vi  a2 − b = 1 ;   α > 0 ;   a > 1 (∗∗)

Hãy tuyn tính hoá dãy s trên.

Gii.

xn+1  =  xn

a +

 x2n + b

⇔   1

xn+1=

  a

xn+

 1 +

  b

xn(3)

Đt  un  =

  1

xn . Khi đó ta có th vit (2) dưi dng.

u1  =  1

α  ;   un+1  = aun +

 bu2

n + 1   vi  a2 − b = 1 ;   α >  0 ;   a >  1

Đó chính là phương trình sai phân mà ta đã tuyn tính hoá trong ví d 2 trên.

Page 86: Dãy số - Giới hạn Tác giả: Trần Nam Dũng- Nguyễn Văn Mậu, 2007

8/13/2019 Dãy số - Giớ i hạn Tác giả: Trần Nam Dũng- Nguyễn Văn Mậu, 2007

http://slidepdf.com/reader/full/day-so-gioi-han-tac-gia-tran-nam-dung-nguyen-van-mau 86/217

3.6. H thc truy hi phi tuyn   86

Ví d 3.29.  Cho dãy s  (un) tho mãn.

u1  =  α  ;   u2  =  β  ;   un+1 = a + u2

n

un−1 vi  a ;   α ;   β  ∈ R   (∗)

Hãy tuyn tính hoá dãy s trên.

Gii.

un+1  = a + u2

n

un−1⇔ un+1un−1  = u2

n + a   (1)

⇒ unun−2  =  u2n−1 + a   (2)

Tr tng v ca (1) và (2) ta đưc.

quadun+1un−1

−unun−2  =  u2

n

−u2n

−1

⇔ un+1un−1 + u2n−1  =  u2

n + unun−2

⇔   un

un+1 + un−1=

  un−1

un + un−2

⇒   un

un+1 + un−1=

  un−1

un + un−2= · · · =

  u2

u3 + u1=

  αβ 

α2 + α + β 2  := k

Do đó  un = k(un+1 + un−1) hay là.

(∗) ⇔

u1 =  α  ;   u2 =  β 

kun+1 − un + kun−1  = 0

Như vy vic tuyn tính hoá đã thc hin xong.Ví d 3.30.  Cho dãy s  (xn) tho mãn.

x1  = α  ;   x2  = β  ;   xn+1  = x2

n + 2bxn − bxn−1 + c

b + xn−1vi  α; β  ∈ R ;   n ≥ 2 (1)

Hãy tuyn tính hoá dãy s trên.

Gii.

(1) ⇔ xn+1 + b = x2

n + 2bxn − bxn−1 + c

b + xn−1+ b =

 (xn + b)2 + c

xn−1 + b  (2)

Đt  y n  = xn + b ta đưc phương trình sai phân.

y 1  = α + b ;   y 2  =  β  + b ;   y n+1  =  c + y 2n

y n−1vi  c ;   α ;   β  ∈ R   (∗)

Đó chính là phương trình sai phân mà ta đã tuyn tính hoá trong ví d 4 trên.

Page 87: Dãy số - Giới hạn Tác giả: Trần Nam Dũng- Nguyễn Văn Mậu, 2007

8/13/2019 Dãy số - Giớ i hạn Tác giả: Trần Nam Dũng- Nguyễn Văn Mậu, 2007

http://slidepdf.com/reader/full/day-so-gioi-han-tac-gia-tran-nam-dung-nguyen-van-mau 87/217

3.6. H thc truy hi phi tuyn   87

3.6.3 Mt s ví d khác

Ví d 3.31.  Xác đnh s hng tng quát ca dãy s  {

f (n)}

 (n ∈ N)  đưc cho

bi.f (0) = 2 ;   f (n + 1) = 3f (n) +

 8f 2(n) + 1 (1)  vi mi  n ≥ 0

Gii.  T gi thit ta có:  f (n + 1) − 3f (n) = 

8f 2(n) + 1 ≥ 0 nên.

(f (n + 1) − 3f (n))2 = 8f 2(n) + 1 ⇒ f 2(n + 1) + f 2(n) = 6f (n)f (n + 1) + 1 (∗)

Thay  n bi  n − 1 ta đưc:

f 2(n) + f 2(n − 1) = 6f (n − 1)f (n) + 1 (∗∗)

Tr tng v (*) và (**) ta đưc 

f 2(n + 1) − f 2(n − 1) = 6f (n)(f (n + 1) − f (n − 1)) (∗ ∗ ∗)

T gi thit ta còn có  f (n) >  0  vi mi  n (chng minh bng quy np). Ngoài ra.

f (n+1) >  3f (n) = 9f (n−1)+3 

8f 2(n − 1) + 1 > f (n−1) ⇒ f (n+1)−f (n−1) >  0

nên:  (∗ ∗ ∗) ⇔ f (n + 1) + f (n − 1) = 6f (n). Vy ta đưc phương trình sai phân tuyn tính.

f (0) = 2 ;   f (1) = 6 +√ 

33

f (n + 2) − 6f (n + 1) + f (n) = 0

Gii phương trình này ta đưc.

f (n) = (8 +

√ 66)(3 +

√ 8)n

8  +

 (8 − √ 66)(3 − √ 

8)n

8

D thy  f (n) xác đnh như trên tho mãn (1).

Ví d 3.32.  Xác đnh s hng tng quát ca dãy s  {f (n)} (n ∈ N∗) đưc chobi.

f (1) = α  ;   f (2) = β  ;   f (n + 1) = f 2(n) + a

f (n − 1)  (∗)  vi mi  n ≥ 2

Gii.  Khi đó, (*) có th đưc vit li dưi dng.

f (n + 1)f (n − 1) =  f 2(n) + a  vi mi  n ≥ 2

Trong đng thc trên thay  n bi  n − 1 ta đưc .

f (n)f (n − 2) =  f 2(n − 1) + a  vi mi  n ≥ 3

Page 88: Dãy số - Giới hạn Tác giả: Trần Nam Dũng- Nguyễn Văn Mậu, 2007

8/13/2019 Dãy số - Giớ i hạn Tác giả: Trần Nam Dũng- Nguyễn Văn Mậu, 2007

http://slidepdf.com/reader/full/day-so-gioi-han-tac-gia-tran-nam-dung-nguyen-van-mau 88/217

3.6. H thc truy hi phi tuyn   88

Tr tng v hai đng thc sau ta đưc.

f (n + 1)f (n − 1) − f (n)f (n − 2) =  f 2

(n) − f 2

(n − 1)  vi mi  n ≥ 3

Hay là.

f (n + 1)f (n − 1) + f 2(n − 1) =  f (n)f (n − 2) + f 2(n)  vi mi  n ≥ 3

T đó có.

f (n)

f (n + 1) + f (n − 1) =

  f (n − 1)

f (n) + f (n − 2)  vi mi  n ≥ 3

Đt  g(n) :=  f (n)

f (n + 1) + f (n

−1)

  (n ∈ N \ {0; 1}) ta có.

g(n) =  g(n − 1)  vi mi  n ∈ N \ {0; 1}

Do đó.

g(n) =  g(n − 1) = · · · = g(2) =  f (2)

f (3) + f (1) =

  αβ 

α2 + β 2 + a  := k

Ta đưc phương trình sai phân tuyn tính 

f (1) = α  ;   f (2) = β  ;   kf (n + 2) − f (n + 1) + kf (n) = 0 (n ≥ 1)

Gii phương trình này ta đưc biu thc ca  f (n) cn tìm.Chú ý: Các phương trình dng.

1)   f (n + 2) = f 2(n + 1) + 2bf (n + 1) − bf (n) + c

f (n) + b  ,   (n ∈ N∗) (1)

(Trong đó:  f (1) = α, f (2) = β )

2)   f (n + 1) =  f 2(n)

(1 + af 2(n))f (n − 1),   (n ∈ N, n ≥ 2) (2)

(Trong đó:   a > 0, f (1) = α = 0, f (2) = β  = 0)

Page 89: Dãy số - Giới hạn Tác giả: Trần Nam Dũng- Nguyễn Văn Mậu, 2007

8/13/2019 Dãy số - Giớ i hạn Tác giả: Trần Nam Dũng- Nguyễn Văn Mậu, 2007

http://slidepdf.com/reader/full/day-so-gioi-han-tac-gia-tran-nam-dung-nguyen-van-mau 89/217

3.6. H thc truy hi phi tuyn   89

có th đưa đưc v dng ca ví d trên. Tht vy, ta có.

(1) ⇔ f (n + 1) =  f 2

(n) + 2bf (n) + b2

− bf (n − 1) − b2

+ cf (n − 1) + b

= [f (n) + b]2 − b[f (n − 1) + b] + c

f (n − 1) + b

= [f (n) + b]2 + c

f (n − 1) + b  − b

⇔ f (n + 1) + b =  [f (n) + b]2 + c

f (n − 1) + b  vi mi  n ≥ 1

Đt  g(n) =  f (n + 1) + b  Ta đưc.

g(n + 1) = g2(n) + c

g(n − 1)

 ,

 ∀ n

∈N∗.

Đó là phương trình có dng đã xét ví d 2 trên. T các điu kin ca phương trình (2) ta có  f (n) = 0 vi mi  n ∈ N∗, do đó.

(2) ⇔   1

f (n + 1) =

 1 + af 2(n)

f 2(n)  .f (n − 1)

=

  1

f 2(n) +  a

  1

1

f (n − 1)

⇔g(n + 1) =

 g2(n) + a

g(n − 1)

 ,

 ∀ n

∈N∗.

Trong đó,  g(n) =  1

f (n), ∀ n ∈ N∗, g(1) =

  1

α, g(1) =

  1

β 

Đó là phương trình có dng đã xét ví d 2.

Ví d 3.33.  Xác đnh s hng tng quát ca dãy s  {f (n)} (n ∈ N∗) đưc chobi.

f (1) = 9

8 ;   f (n + 1) = nf (n) + n.n!  vi mi  n ∈ N∗

Gii.   Ta có nghim tng quát ca phương trình  f (n + 1) − nf (n) = 0 là.

ˆf (n) =  C.1.2. · · · .(n − 1) =  C.(n − 1)!

Ta s tìm nghim riêng ca phương trình đã cho dưi dng  f ∗(n) =  C (n).(n−1)!.Thay vào phương trình đã cho đưc:

C (n+ 1).n! = nC (n).(n−1)!+n.n! ⇔ ∆C  = C (n +1)−C (n) = n  vi mi  n ∈ N

Page 90: Dãy số - Giới hạn Tác giả: Trần Nam Dũng- Nguyễn Văn Mậu, 2007

8/13/2019 Dãy số - Giớ i hạn Tác giả: Trần Nam Dũng- Nguyễn Văn Mậu, 2007

http://slidepdf.com/reader/full/day-so-gioi-han-tac-gia-tran-nam-dung-nguyen-van-mau 90/217

3.6. H thc truy hi phi tuyn   90

T đây d dàng có.

C (n) = 12 (n2 − n) ⇒ f ∗ =  12 (n2 − n)(n − 1)!

Do đó nghim tng quát ca phương trình đã cho là.

f (n) = C.(n − 1)! + 1

2(n − 1)2(n − 1)!

Thay vào điu kin biên đưc  C   = 1. Vy  f (n) = (n − 1)! + 1

2(n − 1)2(n − 1)!.

D thy  f (n) xác đnh như trên tho mãn bài ra.

Ví d 3.34.  Xác đnh s hng tng quát ca dãy s  {f (n)}  (n ∈  N)  đưc cho

bi. f (0) = β  ;   f (n + 1) = 2f 2(n) − 1  vi mi  n ∈ N

Gii.  Ta có. Nu  β  = 1 thì.

f (0) = 1

f (1) = 2f 2(0) − 1 = 1

f (2) = 2f 2(1) − 1 = 1

f (3) = 2f 2(2) − 1 = 1

Bng phương pháp quy np ta chng minh đưc: ∀ n ∈ N, f (n) = 1. Nu  β  = −1

thì tương t ta cũng chng minh đưc: ∀ n ∈N

, f (n) = 1. Nu  |β | < 1  thì tn ti θ sao cho  cos θ =  β  ⇔ θ = arccos β . Khi đó, ta có:

f (0) = cos θ = cos 20θ

f (1) = 2f 2(0) − 1 = 2 cos2 θ − 1 = cos 21θ

f (2) = 2f 2(1) − 1 = 2 cos2 2θ − 1 = cos 22θ

f (3) = 2f 2(2) − 1 = 2 cos2 22θ − 1 = cos 23θ

Bng phương pháp quy np ta chng minh đưc: ∀ n ∈  N, f (n) = cos2nθ. Nu |β | > 1  thì tn ti  θ sao cho  cosh θ =  β . Khi đó, ta có:

f (0) = cosh θ = cosh 20

θf (1) = 2f 2(0) − 1 = 2 cosh2 θ − 1 = cosh 21θ

f (2) = 2f 2(1) − 1 = 2 cosh2 2θ − 1 = cosh22θ

f (3) = 2f 2(2) − 1 = 2 cosh2 22θ − 1 = cosh 23θ

Page 91: Dãy số - Giới hạn Tác giả: Trần Nam Dũng- Nguyễn Văn Mậu, 2007

8/13/2019 Dãy số - Giớ i hạn Tác giả: Trần Nam Dũng- Nguyễn Văn Mậu, 2007

http://slidepdf.com/reader/full/day-so-gioi-han-tac-gia-tran-nam-dung-nguyen-van-mau 91/217

3.6. H thc truy hi phi tuyn   91

Bng phương pháp quy np ta chng minh đưc: ∀ n ∈  N, f (n) = cosh2nθ. Vì 

cosh θ =  β  nên:  eθ + e−θ

2  = β . Gii phương trình này ta đưc.

eθ = β − 

β 2 − 1   hoc    eθ = β  + 

β 2 − 1

Suy ra.

f (n) = 1

2

(eθ)2

n

+  1

(eθ)2n

 =

 1

2

(β −

 β 2 − 1)2

n

+ (β  + 

β 2 − 1)2n

D thy các dãy s xác đnh như trên tho mãn phương trình đã cho. Vy ta có.

f (n) =

1   khi  β  = 1

−1 (n = 0)

1 (n ≥ 1) khi  β  = −1

cos2n arccos β   khi  |β | < 11

2

(β −

 β 2 − 1)2

n

+ (β  + 

β 2 − 1)2n

  khi  |β | > 1

Ví d 3.35.  Xác đnh s hng tng quát ca dãy s  {f (n)}  (n ∈  N)  đưc chobi.

f (0) = α  ;   f (n + 1) = af 2(n) + b   (n ∈ N)   vi  ab = −2

HDG:   Đt  f (n) = −bg(n), n ∈ N∗, ta đưc.

g(0) =−

α

b  := β  ;   g(n + 1) = 2g2(n)

−1 (n

∈N).

Đó chính là ví d 4.

Ví d 3.36.  Xác đnh s hng tng quát ca dãy s  {f (n)}  (n ∈  N)  đưc chobi.

f (1) = α  ;   f (n + 1) =  f 2(n) − 2a2n (n ∈ N∗)   vi  a > 0.

HDG:   Đt  f (n) = 2a2n−1g(n), n ∈ N∗, ta đưc.

g(1) = − α

2a  := β  ;   g(n + 1) = 2g2(n) − 1 (n ∈ N∗).

Đó chính là ví d 4.

Ví d 3.37.  Xác đnh s hng tng quát ca dãy s  {f (n)}  (n ∈  N)  đưc chobi.

f (n + 1) = af 2(n) + bf (n) + c   (n ∈ N) (1)   vi  a = 0 ;   c =  b2 − 2b

4a  .

Page 92: Dãy số - Giới hạn Tác giả: Trần Nam Dũng- Nguyễn Văn Mậu, 2007

8/13/2019 Dãy số - Giớ i hạn Tác giả: Trần Nam Dũng- Nguyễn Văn Mậu, 2007

http://slidepdf.com/reader/full/day-so-gioi-han-tac-gia-tran-nam-dung-nguyen-van-mau 92/217

3.6. H thc truy hi phi tuyn   92

Gii.  Khi đó ta có.

(1) ⇔ f (n + 1) +   b2a

 = a f (n) +   b2a2 +  b

2

− 4ac4a

  +   b2a

= a

f (n) +

  b

2a

2+

 b2 − (b2 − 2b) + 2b

4a

= a

f (n) +

  b

2a

2⇔ g(n + 1) = ag2(n)   (vi  g(n) =  f (n) +

  b

2a)

⇒ g(n + 1) = a[g(n)]2 = a[ag2(n − 1)]2 = a3[g(n − 1)]22

=

= · · ·= a2n−1[g(1)]2n = a2n−1[α +   b

2a]2n

⇒ g(n) =  a2n−1−1[α +  b

2a]2n−1

⇒ f (n) = a2n−1−1[α +  b

2a]2n−1

+  b

2a

Bng phép quy np ta d dàng chng t đưc   f (n) xác đnh như trên tho mãn phương trình đã cho. Vy ta đưc đáp s.

f (n) =  a2n−1−1[α +  b

2a]2n−1

+  b

2a  (n ∈ N∗).

Ví d 3.38.  Tìm dãy s  (xn) tho mãn các điu kin sau.

x1  =  a  ;   xn+1  = x2

n + d

2xnvi mi  n ≥ 1.   (1).

Gii.+) Nu  d = 0 thì ta có ngay.

xn+1  = 1

2xn ⇒ xn =  a.(

1

2)n−1.

+)  Xét trưng hp  d >  0.  Gi s   un; vn   là mt nghim ca h phương trình 

sai phân.

u1 =  a  ;   v1  = 1

un+1  =  u2n + dv2

n

vn+1  = 2unvn

,   (2)

Page 93: Dãy số - Giới hạn Tác giả: Trần Nam Dũng- Nguyễn Văn Mậu, 2007

8/13/2019 Dãy số - Giớ i hạn Tác giả: Trần Nam Dũng- Nguyễn Văn Mậu, 2007

http://slidepdf.com/reader/full/day-so-gioi-han-tac-gia-tran-nam-dung-nguyen-van-mau 93/217

3.6. H thc truy hi phi tuyn   93

khi đó,  xn = un

vnlà nghim ca phương trình (1). Tht vy, ta có:

x1  =  u1

v1=  a

1 = a ⇒  Khng đnh đúng vi  n = 1.

Gi s khng đnh đúng ti  n, tc là:  xn  = un

vnlà nghim ca (1). Khi đó.

xn+1  = un+1

vn+1=

 u2n + dv2

n

2unvn=

u2n

v2n

+ d

2un

vn

= x2

n + d

2xn.

Vy  xn+1  cũng là nghim ca (1). Tc là khng đnh cũng đúng ti  n + 1. Theonguyên lý quy np toán hc, khng đnh trên đúng vi mi  n

 ∈ N∗. Vy, đ gii 

(1) ta đi gii (2). Vit li (2) dưi dng.

u1  =  a  ;   v1  = 1

un+1  = u2n + dv2

n   (∗)√ dvn+1  = 2

√ dunvn   (∗∗)

.   (3)

Cng tng v (*) và (**) ta đưc.

un+1 +√ 

dvn+1  = (un +√ 

dvn)2 vi mi  n ≥ 1.

T đó có.un+1 +

√ dvn+1  = (u1 +

√ dv1)2

n

= (a +√ 

d)2n

.   (4)

Tr tng v (*) và (**) ta đưc.

un+1 −√ 

dvn+1  = (un −√ 

dvn)2 vi mi  n ≥ 1.

T đó có.un+1 −

√ dvn+1  = (u1 −

√ dv1)2

n

= (a −√ 

d)2n

.   (5)

T (4) và (5) ta có.

un+1  = 1

2

(a +

√ d)2

n

+ (a −√ 

d)2n

vn+1  =  1

2√ 

d (a +

√ d)2

n − (a −√ 

d)2n

  .   (6)

Do  xn = un

vnnên t (6) ta có.

xn =√ 

d(a +

√ d)2

n−1+ (a − √ 

d)2n−1

(a +√ 

d)2n−1 − (a − √ 

d)2n−1

.

Page 94: Dãy số - Giới hạn Tác giả: Trần Nam Dũng- Nguyễn Văn Mậu, 2007

8/13/2019 Dãy số - Giớ i hạn Tác giả: Trần Nam Dũng- Nguyễn Văn Mậu, 2007

http://slidepdf.com/reader/full/day-so-gioi-han-tac-gia-tran-nam-dung-nguyen-van-mau 94/217

3.6. H thc truy hi phi tuyn   94

Có th kim tra nghim này tho mãn bng cách th vào (1).+)Xét trưng hp  d <  0.  Đt   d   =

 −q   (q >   0). Tương t trên, ta s chng 

minh.Gi s  un; vn là mt nghim ca h phương trình sai phân.

u1 =  a  ;   v1 = 1

un+1  =  u2n − qv2

n

vn+1  = 2unvn

,   (7)

khi đó,  xn = un

vnlà nghim ca phương trình (1). Tht vy, ta có:

x1  = u1

v1=

 a

1 = a ⇒  Khng đnh đúng vi  n = 1.

Gi s khng đnh đúng ti  n, tc là:  xn  =  un

vnlà nghim ca (1). Khi đó.

xn+1  = un+1

vn+1=

 u2n − qv2

n

2unvn=

u2n

v2n

− q 

2un

vn

= x2

n + d

2xn.

Vy  xn+1  cũng là nghim ca (1). Tc là khng đnh cũng đúng ti  n + 1. Theonguyên lý quy np toán hc, khng đnh trên đúng vi mi  n ∈ N∗. Vy, đ gii phương trình (1) ta đi gii h (7). Vit li (7) dưi dng.

u1  =  a  ;   v1  = 1un+1  = u2

n − qv2n   (∗ ∗ ∗)

i√ 

qvn+1  = 2i√ 

qunvn   (∗∗∗∗)

.   (8)

Trong đó   i là đơn v o ( i2 = −1). Cng tng v (***) và (****) ta đưc.

un+1 + i√ 

qvn+1  = (un + i√ 

qvn)2 vi mi  n ≥ 1.

T đó có.

un+1 + i√ 

qvn+1  = (u1 + i√ 

qv1)2n

= (a + i√ 

q )2n

.   (9)

Tr tng v (***) và (****) ta đưc.

un+1 − i√ 

qvn+1  = (un − i√ 

qvn)2 vi mi  n ≥ 1.

T đó có.

un+1 − i√ 

qvn+1  = (u1 − i√ 

qv1)2n

= (a − i√ 

q )2n

.   (10)

Page 95: Dãy số - Giới hạn Tác giả: Trần Nam Dũng- Nguyễn Văn Mậu, 2007

8/13/2019 Dãy số - Giớ i hạn Tác giả: Trần Nam Dũng- Nguyễn Văn Mậu, 2007

http://slidepdf.com/reader/full/day-so-gioi-han-tac-gia-tran-nam-dung-nguyen-van-mau 95/217

3.6. H thc truy hi phi tuyn   95

T (9) và (10) ta có.

un+1  = 1

2(a + i√ q )

2n

+ (a − i√ q )2n

vn+1  =  1

2i√ 

(a + i

√ q )2

n − (a − i√ 

q )2n   .   (11)

Do  xn = un

vnnên t (6) ta có.

xn  =  i√ 

q (a + i

√ q )2

n−1+ (a − i

√ q )2

n−1

(a + i√ 

q )2n−1 − (a − i

√ q )2

n−1 .

Cũng có th kim tra nghim này tho mãn bng cách th vào (1).

Ví d 3.39.  Bit rng dãy s  (xn) có dng  xn  =  f (n), trong đó  f (x) là đa thc bc không quá 2. Hãy xác đnh công thc tng quát ca dãy s bit ba s hng đu:  x1, x2, x3.

Gii.   Gi s  f (x) =  ax2 + bx + c. Theo gi thit ta có 

a + b + c   = x1

4a + 2b + c   = x2

8a + 4b + c   = x3

Gii h này ta đưc 

a   =

 x1 − 2x2 + x3

2b   = −5x1 − 8x2 + 3x3

2c   = 3x1 − 3x2 + x3

Vy 

f (n) = x1 − 2x2 + x3

2  n2 −  5x1 − 8x2 + 3x3

2  n + 3x1 − 3x2 + x3.

là hàm s cn tìm.Gii.   Vit  f (n) dưi dng.

f (n) = a(n − 2)(n − 3) + b(n − 1)(n − 3) + c(n − 1)(n − 2).

Ln lưt cho  n = 1,   2,   3 ta đưc 

x1  =  f (1) = a.(1 − 2)(1 − 3) = 2a   ⇒ a = x1

2x2  =  f (2) = b.(2 − 1)(2 − 3) = −b   ⇒ b = −x2

x3  =  f (3) = c.(3 − 1)(3 − 2) = 2c   ⇒ c = x3

2

Page 96: Dãy số - Giới hạn Tác giả: Trần Nam Dũng- Nguyễn Văn Mậu, 2007

8/13/2019 Dãy số - Giớ i hạn Tác giả: Trần Nam Dũng- Nguyễn Văn Mậu, 2007

http://slidepdf.com/reader/full/day-so-gioi-han-tac-gia-tran-nam-dung-nguyen-van-mau 96/217

3.6. H thc truy hi phi tuyn   96

Vy công thc cn tìm có dng 

xn =  f (n) =

 x1

2 (n − 2)(n − 3) − x2(n − 1)(n − 3) +

 x3

2 (n − 1)(n − 2).

Ví d 3.40.  Xác đnh s hng tng quát ca dãy s cho bi h thc truy hi sau.

x1 = 0 ;   xn+1 = xn + 1

n + 1  .

Gii.  Vit li điu kin đã cho dưi dng.

x1 = 0   quad   (1)

2x2 =  x1 + 1   quad(2)

3x3 =  x2 + 1   quad(3)

· · · · · · · · ·(n − 1)xn =  xn−2 + 1   quad(n − 1)

nxn  = xn−1 + 1   quad(n)

Nhân hai v ca đng thc th  k  trên vi  (k − 1)!, cng tng v các đng thc thu đưc và rút gn các s hng đng dng hai v ta đưc.

n!.xn = 1! + 2! + · · · + (n − 1)! ⇒ xn =  1

n!.

n−1k=1

k!

là công thc s hng tng quát cn tìm.

3.6.4 Bài tp.

1. Xác đnh s hng tng quát ca dãy s  {f (n)}  (n ∈  N∗)  đưc cho bi h thc truy hi.

f (1) = α  ;   f (n + 1) = 2a2nf 2(n) − a(n+1)2n (n ∈ N∗)   vi  a >  0.

Đáp s.

f (n) =

an2n−1(n ∈ N∗)   nu  α =  a

−a quadkhi  n = 1

an2n−1 khi  n≥

2nu  α = −a

an2n−1cos2n−1θ   (n ∈ N∗)   nu  |α| < a

1

2a(n−1)2n−1

α +

 α2 − a2

2n−1

+

α − 

α2 − a22n−1

(n ∈ N∗)   nu  |α| > a

Page 97: Dãy số - Giới hạn Tác giả: Trần Nam Dũng- Nguyễn Văn Mậu, 2007

8/13/2019 Dãy số - Giớ i hạn Tác giả: Trần Nam Dũng- Nguyễn Văn Mậu, 2007

http://slidepdf.com/reader/full/day-so-gioi-han-tac-gia-tran-nam-dung-nguyen-van-mau 97/217

3.6. H thc truy hi phi tuyn   97

2. Xác đnh s hng tng quát ca dãy s  {f (n)} (n ∈ N) đưc cho bi h thc truy hi.

f (0) = α  ;   f (n + 1) = af 3(n) − 3f (n) (n ∈ N)   vi  a >  0.

Đáp s.

f (n) =

2√ a

 cos3n arccos α

√ a

2  (n ∈ N)  nu  |α| ≤   2√ 

a1

23n

α√ 

a − 

α2a − 43n

+

α√ 

a + 

α2a − 43n

(n ∈ N)   nu  |α| >  2√ 

a

3. Xác đnh s hng tng quát ca dãy s  {f (n)} (n ∈ N) đưc cho bi h thc 

truy hi.f (0) = α  ;   f (n + 1) = f 3(n) − 3a3nf (n) (n ∈ N)   vi  a > 0.

Đáp s.

f (n) =

2√ 

a3n

cos3n arccos  α

2√ 

a  (n ∈ N)   nu  |α| ≤ 2

√ a

1

23n

α −

 α2 − 4a

3n+

α + 

α2 − 4a3n

(n ∈ N)   nu  |α| >  2√ 

a

4. Xác đnh s hng tng quát ca dãy s  {f (n)} (n ∈ N) đưc cho bi h thc 

truy hi:f (0) = α  ;   f (n + 1) =  af 3(n) + 3f (n) (n ∈ N)   vi  a >  0 ; |α| >

  2√ a

.

Đáp s:

f (n) =  1√ 

a

α

√ a

2  −

 α2a

4  + 1

3n

+

α√ 

a

2  +

 α2a

4  + 1

3n   (n ∈ N∗)

5. Xác đnh s hng tng quát ca dãy s  {f (n)} (n ∈ N) đưc cho bi h thc truy hi:

f (0) = α  ;   f (n + 1) = f 3(n) + 3a3n

f (n) (n ∈ N)   vi  |α| > 2√ a.

Đáp s:

f (n) =  1

23n

α +

 α2 + 4a

3n+

α − 

α2 + 4a3n

  (n ∈ N).

Page 98: Dãy số - Giới hạn Tác giả: Trần Nam Dũng- Nguyễn Văn Mậu, 2007

8/13/2019 Dãy số - Giớ i hạn Tác giả: Trần Nam Dũng- Nguyễn Văn Mậu, 2007

http://slidepdf.com/reader/full/day-so-gioi-han-tac-gia-tran-nam-dung-nguyen-van-mau 98/217

3.6. H thc truy hi phi tuyn   98

6. Xác đnh s hng tng quát ca dãy s  {f (n)}  (n ∈  N∗)  đưc cho bi h thc truy hi:

f (1) = α  ;   f (n + 1) = af 3(n) + bf 2(n) + cf (n) + d   (n ∈ N∗)

vi  a > 0 ;   c =  b2

3a  ;   d =

  b(c − 3)

9a  ;   α > −   b

3a.

Đáp s:

f (n) =

α +

  b

3a

3n−1

(√ 

a)3n−1−1 −   b

3a  (n ∈ N∗).

7. Xác đnh s hng tng quát ca dãy s  {f (n)} (n ∈ N) đưc cho bi h thc truy hi:

f (0) = α  ;   f (n + 1) = af 3(n) + bf 2(n) + cf (n) + d   (n ∈ N).

vi  a > 0 ;   c =  b2 + 9a

3a  ;   d =

  b3 + 18ab)

27a2  ;   α >

  2√ a −   b

3a.

Đáp s:

f (n) =  1

23n√ 

a

γ √ 

a − 

γ 2a + 43n

+

γ √ 

a + 

γ 2a + 43n−   b

3a (n ∈ N∗).

( Vi  γ  =  α +  b

3a).

8. Xác đnh s hng tng quát ca dãy s  (xn) (n ∈ N∗) đưc cho bi h thc truy hi:

x1  = a  ;   xn+1  =  n + 1

xn + 1  (n ≥ 1).

Đáp s:

xn =  n!

a +n−1k=1

1

k!

.

9. Xác đnh s hng tng quát ca dãy s  (xn) (n ∈ N∗) đưc cho bi h thc truy hi:

x1

 = 1

2 ;   x

n+1 =

  2

3 − xn(n

≥1).

Đáp s:

xn = 3.2n−1 − 2

3.2n−1 − 1.

Page 99: Dãy số - Giới hạn Tác giả: Trần Nam Dũng- Nguyễn Văn Mậu, 2007

8/13/2019 Dãy số - Giớ i hạn Tác giả: Trần Nam Dũng- Nguyễn Văn Mậu, 2007

http://slidepdf.com/reader/full/day-so-gioi-han-tac-gia-tran-nam-dung-nguyen-van-mau 99/217

Chương 4

Phương trình hàm sai phânbc hai

Trong chương này, ta gii hai bài toán v phương trình hàm tuyn tính thun nht bc hai đi vi hàm tun hoàn và phn tun hoàn cng tính; phương trình hàm tuyn tính thun nht bc hai đi vi hàm tun hoàn và phn tun hoàn nhân tính. Ta s gii quyt hai bài toán trên da vào kt qu ca các bài toán v phương trình hàm tuyn tính bc nht đã có trong tài liu tham kho [3].

4.1 Hàm tun hoàn và phn tun hoàn cng tính

Đnh nghĩa 4.1.  Cho hàm s  f (x) và tp  M   ( M 

 ⊂ D(f )) Hàm  f (x) đưc gi 

là hàm tun hoàn trên  M  nu tn ti s dương  a sao cho ∀ x ∈ M  ta đu có  x ± a ∈ M f (x + a) = f (x), ∀ x ∈ M 

a đưc gi là chu kỳ ca hàm tun hoàn  f (x).

Chu kỳ nh nht (nu có) trong các chu kỳ ca  f (x) đưc gi là chu kỳ cơ s ca hàm tun hoàn  f (x).

Đnh nghĩa 4.2.  Cho hàm s  f (x) và tp  M   ( M  ⊂  D(f )) Hàm  f (x) đưc gi 

là hàm tun hoàn trên  M  nu tn ti s dương  a sao cho ∀ x ∈ M  ta đu có  x ± a ∈ M f (x + a) = −f (x), ∀ x ∈ M 

a đưc gi là chu kỳ ca hàm tun hoàn  f (x).

99

Page 100: Dãy số - Giới hạn Tác giả: Trần Nam Dũng- Nguyễn Văn Mậu, 2007

8/13/2019 Dãy số - Giớ i hạn Tác giả: Trần Nam Dũng- Nguyễn Văn Mậu, 2007

http://slidepdf.com/reader/full/day-so-gioi-han-tac-gia-tran-nam-dung-nguyen-van-mau 100/217

4.2. Phương trình hàm sai phân bc hai vi hàm tun hoàn và phn tun hoàn   100

Chu kỳ nh nht (nu có) trong các chu kỳ ca  f (x) đưc gi là chu kỳ cơ s ca hàm tun hoàn  f (x).

Đnh nghĩa 4.3.   f (x)   đưc gi là hàm tun hoàn nhân tính chu kỳ   a   ( a /  ∈{0, 1, −1}) trên  M   nu  M  ⊂ D(f ) và  ∀ x ∈ M  ⇒ a±1x ∈ M 

f (ax) =  f (x), ∀ x ∈ M 

Đnh nghĩa 4.4.   f (x)   đưc gi là hàm phn tun hoàn nhân tính chu kỳ   a( a /∈ {0, 1, −1}) trên  M  nu  M  ⊂ D(f ) và  ∀ x ∈ M  ⇒ a±1x ∈ M 

f (ax) = −f (x), ∀ x ∈ M 

4.2 Phương trình hàm sai phân bc hai vi hàm tunhoàn và phn tun hoàn

Bài toán 4.1.  Cho  a ∈ R\{0};   α, β  ∈ R, β = 0. Tìm tt c các hàm  f   :   R → R

tho mãn điu kin:

f (x + 2a) + αf (x + a) + βf (x) = 0   (4.1)

Phương trình có dng (1.2.1) đưc gi là phương trình tuyn tính thun nht bc hai 

Gii. Xét phương trình λ2 + αλ + β  = 0   (4.2)

(gi là phương trình đc trưng ca phương trình (1.2.1))Có 

= α2 − 4β 

a) Trưng hp  > 0

Khi đó phương trình (4.2) có hai nghim thc  λ1 = λ2. Áp dng đnh lý Viete:λ1 + λ2 = −α

λ1λ2 =  β 

thay vào (4.1)

(4.1) ⇔   f (x + 2a) − (λ1 + λ2)f (x + a) + λ1λ2f (x) = 0

⇔   f (x + 2a) − λ1f (x + a) =  λ2[f (x + a) − λ1f (x)]   (4.3)

Page 101: Dãy số - Giới hạn Tác giả: Trần Nam Dũng- Nguyễn Văn Mậu, 2007

8/13/2019 Dãy số - Giớ i hạn Tác giả: Trần Nam Dũng- Nguyễn Văn Mậu, 2007

http://slidepdf.com/reader/full/day-so-gioi-han-tac-gia-tran-nam-dung-nguyen-van-mau 101/217

4.2. Phương trình hàm sai phân bc hai vi hàm tun hoàn và phn tun hoàn   101

Đt  g1(x) =  f (x + a) − λ1f (x), (1.2.3) tr thành 

g1(x + a) = λ2g1(x)   (1.2.3*)

Đt  g1(x) = |λ2|xa .h(x). Khi đó ta có  h(x + a) =

h(x)   nu  λ2 >  0

−h(x)   nu  λ2 <  0Khi đó ta 

có f (x + a) − λ1f (x) = |λ2|xa h1(x)   (4.4)

Đi vai trò  λ2 cho  λ1 và bin đi tương t ta đưc 

f (x + a) − λ2f (x) = |λ1|xa h2(x)   (4.5)

Tr (4.4) cho (1.2.5) ta đưc 

(λ2 − λ1)f (x) = |λ2|xa

h1(x) − |λ1|xa

h2(x)Vy 

f (x) =1

λ2 − λ1

|λ2|xa h1(x) − |λ1|xa h2(x)

trong đó  h1(x) và  h2(x) là hai hàm tuỳ ý tho mãn:

h1(x + a) =

h1(x)   nu   λ2 >  0

−h1(x)   nu   λ2 <  0;   h2(x + a) =

h2(x)   nu   λ1 >  0

−h2(x)   nu   λ1 <  0

b) Trưng hp   = 0Tc là 

α2 − 4β  = 0 hay  β  =  α2

4

Khi đó phương trình (1.2.2) có nghim kép  λ1 =  λ2 = −α

2. Do đó 

(4.1) ⇔   f (x + 2a) + αf (x + a) + α2

4 f (x) = 0

⇔   f (x + 2a) +α

2

f (x + a) = −α

2

f (x + a) + (

α

2)f (x)

  (4.6)

Đt  f (x + a) +   α2 f (x) =  g(x)

b1) Trưng hp −α

2 = 1 hay α = −2.Khi đó (1.2.6) tr thành 

f (x + 2a) − f (x + a) = f (x + a) − f (x)

⇔   f (x + a) − f (x) =  g(x),   (1.2.6*)

Page 102: Dãy số - Giới hạn Tác giả: Trần Nam Dũng- Nguyễn Văn Mậu, 2007

8/13/2019 Dãy số - Giớ i hạn Tác giả: Trần Nam Dũng- Nguyễn Văn Mậu, 2007

http://slidepdf.com/reader/full/day-so-gioi-han-tac-gia-tran-nam-dung-nguyen-van-mau 102/217

4.2. Phương trình hàm sai phân bc hai vi hàm tun hoàn và phn tun hoàn   102

vi  g(x + a) = g(x)

Ta có  g(x) =   (x+a)−xa   g(x) =   x+a

a   g(x + a)

−xag(x), phương trình (1.2.6*) tr thành 

f (x + a) −   (x+a)a   g(x + a) =  f (x) −   x

ag(x)Đt  f (x) −   x

ag(x) =  h(x) ta có 

f (x) =  h(x) + xg (x)

a

trong đó  h(x) là hàm tuỳ ý sao cho  h(x + a) =  h(x).

b2) Trưng hp  0 < −α

2 = 1 hay  2 = α <  0  Bài toán quy v vic gii phương 

trình dng 

f (x + a) + (α

2)f (x) =  g(x)

vi  g(x + a) = −α

2g(x)

Tương t vic gii (1.2.3*) ta có 

g(x) = (−α

2)xa h1(x),  vi  h1(x + a) =  h1(x)

Suy ra 

f (x + a) − (−α

2)f (x) = (−α

2)xa h1(x)

⇔  f (x + a)

(−α

2)xa

− (−α2

)   f (x)

(−α

2)xa

= h1(x)

⇔ f (x + a)

(−α

2)x+aa

− f (x)

(−α

2)xa

=h1(x)

−α

2

(4.7)

Đt f (x)

(−α2 )

xa

= I 1(x);  h1(x)

−α2

= h2(x)

(4.7) ⇔

  I 1(x + a)−

I 1(x) =  h2(x),  vi  h2(x + a) = h2(x)

Page 103: Dãy số - Giới hạn Tác giả: Trần Nam Dũng- Nguyễn Văn Mậu, 2007

8/13/2019 Dãy số - Giớ i hạn Tác giả: Trần Nam Dũng- Nguyễn Văn Mậu, 2007

http://slidepdf.com/reader/full/day-so-gioi-han-tac-gia-tran-nam-dung-nguyen-van-mau 103/217

4.2. Phương trình hàm sai phân bc hai vi hàm tun hoàn và phn tun hoàn   103

Tương t cách gii (1.2.6*) ta có 

I 1(x) =  k1(x) +  xh2(x)a

= k1(x) −  2xh1(x)

αa

⇒   f (x) = (−α

2)xa

k1(x) − 2xh1(x)

αa

trong đó  k1(x) là hàm tuỳ ý tho mãn  k1(x + a) =  k1(x).b3) Trưng hp  −α

2   < 0  hay  α > 0.Bài toán quy v vic gii phương trình 

f (x + a) + α

2f (x) = g(x)   (4.8)

vi  g(x + a) = (−α2 )g(x).

Tương t vic gii (1.2.3*) ta có 

g(x) =−  α

2

xa h3(x) =α

2

xa

h3(x)   (4.9)

vi  h3(x + a) = −h3(x) (vì  −α2   < 0)

T (4.8) và (4.9) ta có 

f (x + a) + α

2f (x) =

α

2

xa

h3(x)

⇔ f (x + a)

α2

xa

+ α

2

f (x)α2

xa

= h3(x)

⇔ f (x + a)α2

x+aa

+f (x)α2

xa

=h3(x)

α2

Đt f (x)α2

xa

= I 2(x);2h3(x)

α  = h4(x)

vi  h4(x) là hàm tuỳ ý tho mãn: h4(x+a) = −h4(x), (vì: h4(x+a) =

2h3(x + a)

α   =2h3(x)

α  = h4(x))

Khi đó ta có I 2(x + a) + I 2(x) =  h4(x).   (1.2.9*)

Page 104: Dãy số - Giới hạn Tác giả: Trần Nam Dũng- Nguyễn Văn Mậu, 2007

8/13/2019 Dãy số - Giớ i hạn Tác giả: Trần Nam Dũng- Nguyễn Văn Mậu, 2007

http://slidepdf.com/reader/full/day-so-gioi-han-tac-gia-tran-nam-dung-nguyen-van-mau 104/217

Page 105: Dãy số - Giới hạn Tác giả: Trần Nam Dũng- Nguyễn Văn Mậu, 2007

8/13/2019 Dãy số - Giớ i hạn Tác giả: Trần Nam Dũng- Nguyễn Văn Mậu, 2007

http://slidepdf.com/reader/full/day-so-gioi-han-tac-gia-tran-nam-dung-nguyen-van-mau 105/217

4.2. Phương trình hàm sai phân bc hai vi hàm tun hoàn và phn tun hoàn   105

T (4.10) và (4.11) ta có 

g1(x) =  e

x

a

 ln λ2

h1(x)g2(x) =  e

xa ln λ1h2(x)

(i)

Ta chng minh  h1(x) =  h2(x)Tht vy, trưc ht ta chng minh  g1(x) =  g2(x).

Ta có  g1(x) =  f (x + a) − λ1f (x),Ly  x0  bt kỳ,  x0 ∈ R.Ta có  g1(x0) =  f (x0 + a) − λ1f (x0) =  f (x0+a)−λ1f (x0) =  f (x0+a)−λ2f (x0) =g2(x0).Vì  x0  bt kỳ nên  ∀ x ∈ R ta có 

g1(x) =  g2(x)   (4.12)

Tip theo ta chng minh exa ln λ2 = e

xa ln λ1

Tht vy 

exa ln λ2 =   e

xa(ln |λ2|+iargλ2+2kπi)

=   exa ln |λ2|.eiarrgλ2

xa ei2kπ 

xa

=   exa ln |λ2|

cos ϕx

a  + i sin

 ϕx

a

cos

 2kπx

a  + i sin

 2kπx

a

=   e

xa ln

|λ2

|cos ϕx

a  + i sin

 ϕx

a cos 2kπx

a  + i sin

 2kπx

a =   e

xa ln |λ2|

cos

 ϕx

a − i sin

 ϕx

a

cos

 2kπx

a  − i sin

 2kπx

a

=   e

xa ln |λ2|

cos

−ϕx

a

+i sin

−ϕx

a

cos

−2kπx

a

+i sin

−2kπx

a

=   e

xa ln |λ2|ei(−

ϕxa  )ei(−

2kπxx  )

=   exa ln |λ1|ei

argλ1xa ei

2kπxa

=   exa(ln |λ1|+iargλ1+2kπi) = e

xa ln λ1 (4.13)

đây: ( argλ1 = −ϕ;   argλ2 =  ϕ; −k =  k)

T (4.12) và (4.13):   g1(x)

exa ln λ1

 =

  g2(x)

exa ln λ1

⇔   h1(x) =  h2(x)

Theo trên ta có các hàm  h1  :   R→ C;   h2 :   R → C

Page 106: Dãy số - Giới hạn Tác giả: Trần Nam Dũng- Nguyễn Văn Mậu, 2007

8/13/2019 Dãy số - Giớ i hạn Tác giả: Trần Nam Dũng- Nguyễn Văn Mậu, 2007

http://slidepdf.com/reader/full/day-so-gioi-han-tac-gia-tran-nam-dung-nguyen-van-mau 106/217

4.2. Phương trình hàm sai phân bc hai vi hàm tun hoàn và phn tun hoàn   106

Như vy ta s đt:  h1(x) =  m(x) + in(x); trong đó các hàm:

m :   R→ R;   n :   R → R

Theo chng minh trên:  h1(x) =  h2(x) ⇒   h2(x) =  m(x) − in(x)Quay tr li bài toán ban đu ta có 

f (x + a) − λ1f (x) =  exa ln λ2h1(x)

f (x + a) − λ2f (x) =  exa ln λ1h2(x)

(4.14)

Tr ( ??) cho ( ??)

f (x) =  1

λ2 − λ1

exa ln λ2h1(x) − e

xa ln λ1h2(x)

=   1

λ2 − λ1

exa  ln λ2h1(x) − exa  ln λ2h2(x)=

  1

λ2 − λ1

exa(ln |λ2|+iargλ2+2kπi)h1(x) − e

xa(ln |λ2|+iargλ2+2kπi)h2(x)

Vì hàm  e

xa ln λ2 là hàm đa tr, ta s chn mt nhánh liên tc bng cách chn 

k = 0, nên ta có:

f (x) =  1

λ2 − λ1

exa(ln |λ2|+iargλ2)h1(x) − e

xa(ln |λ2|+iargλ2)h2(x)

=

  1

λ2 − λ1exa ln |λ2|

cos

 ϕx

a  + i sin

 ϕx

a h1(x) − e

xa ln |λ2|

cos

 ϕx

a − i sin

 ϕx

a h2(x)

=  e

x

a  ln |λ0|2iq 

cos ϕx

a  (h1(x) − h2(x)) + i sin ϕx

a  (h1(x) + h2(x))

=

 exa ln |λ0|

2iq 

2i cos

 ϕx

a  .n(x) + 2i sin

 ϕx

a  .m(x)

=

 exa ln |λ0|

cos

 ϕx

a  .n(x) + sin

 ϕx

a  .m(x)

=

 |λ0|xaq 

cos

 ϕx

a  .n(x) + sin

 ϕx

a  .m(x)

Trong đó  n(x) và  m(x) là hai hàm s bt kỳ tho mãn 

n(x + a) = n(x); m(x + a) =  m(x)

n : R→  R; m : R → R

λ1 =  p − iq λ2 =  p + iq 

⇒ |λ1| = |λ2| =

  p2 + q 2;   argλ2 =  ϕ  =  argλ1

Page 107: Dãy số - Giới hạn Tác giả: Trần Nam Dũng- Nguyễn Văn Mậu, 2007

8/13/2019 Dãy số - Giớ i hạn Tác giả: Trần Nam Dũng- Nguyễn Văn Mậu, 2007

http://slidepdf.com/reader/full/day-so-gioi-han-tac-gia-tran-nam-dung-nguyen-van-mau 107/217

4.2. Phương trình hàm sai phân bc hai vi hàm tun hoàn và phn tun hoàn   107

tan ϕ =  q 

 p;   argλ2 = −ϕ

Kt lun:+)  > 0

⇒   f (x) =  1

λ2 − λ1

|λ2|

xa .h1(x) − |λ1|

xa .h2(x)

h1(x) và  (h2(x) là hai hàm tuỳ ý tho mãn:

h1(x + a) =

h1(x)   nu  λ2 >  0

−h1(x)   nu  λ2 <  0

h2(x + a) = h2(x)   nu  λ1 >  0

−h2(x)   nu  λ1 <  0

+)  = 0

Trưng hp 1:  α = −2 ⇒   f (x) =  h(x) +   xg(x)a

h(x) và  g(x) là hai hàm tuỳ ý tho mãn:

h(x + a) =  h(x)

g(x + a) =  g(x)

Trưng hp 2: −2 = α < 0

⇒   f (x) =

− α

2

xa

k1(x) −  2xh1(x)

αa

k1(x) và  h1(x) là hai hàm tuỳ ý tho mãn:h1(x + a) =  h1(x)

k1(x + a) = k1(x)

Trưng hp 3:  α > 0

⇒   f (x) =α

2

xa

k2(x) −  2(x − a)h2(x)

αa

k2(x) và  h2(x) là hai hàm tuỳ ý tho mãn:

h2(x + a) = −h2(x)

k2(x + a) = −k2(x)

+)  < 0

f (x) = |λ0|xa

q  cos ϕx

a

  .n(x) + sin ϕx

a

  .m(x)m(x) và  n(x) là hai hàm tuỳ ý tho mãn 

m(x + a) = m(x);   m :   R → R

n(x + a) =  n(x);   n :   R → R

Page 108: Dãy số - Giới hạn Tác giả: Trần Nam Dũng- Nguyễn Văn Mậu, 2007

8/13/2019 Dãy số - Giớ i hạn Tác giả: Trần Nam Dũng- Nguyễn Văn Mậu, 2007

http://slidepdf.com/reader/full/day-so-gioi-han-tac-gia-tran-nam-dung-nguyen-van-mau 108/217

4.3. Phương trình vi hàm s tun hoàn, phn tun hoàn nhân tính   108

λ1 =  p − iq ;   λ2 =  p + iq 

|λ1

|=

|λ2

|=

|λ0

|=  q 2 + q 2;  argλ2 =  argλ1 =  ϕ

Các ví d áp dng

Ví d 4.1.  Tìm tt c các hàm  f   :   R → R tho mãn điu kin:

f (x + 2) − 8f (x + 1) + 15f (x) = 16   (1)

Li gii. Đt  f (x) =  g(x) + C ; ( C : xác đnh sau)

(1) ⇔   g(x + 2) − 8g(x + 1) + 15g(x) + 8C  = 16.

Chn  C  = 2

⇒  g(x + 2)

−8g(x + 1) + 15g(x) = 0.

Xét phương trình đc trưng:

λ2 − 8λ + 15 = 0;   = 1 >  0

⇒ λ1 = 3;   λ2 = 5

Áp dng công thc nghim vi   > 0  ta có 

g(x) = 1

2[5xh1(x) − 3xh2(x)] ⇒   f (x) =

 1

2[5xh1(x) − 3xh2(x)] + 2

h1(x) và  h2(x) là hai hàm tuỳ ý tho mãn h1(x + 1) = h1(x);   h1 :   R → R

h2(x + 1) = h2(x);   h2 :   R → R

Ví d 4.2.  Tìm tt c các hàm  f   :   R → R tho mãn điu kin 

4.3 Phương trình vi hàm s tun hoàn, phn tunhoàn nhân tính

Phương trình hàm là mt chuyên đ cơ bn ca chương trình toán chocác trưng THPT Chuyên. Các bài toán v phương trình hàm cũng là nhng bài 

tp thưng gp trong các kỳ thi hc sinh gii toán cp Quc gia, thi Olympic khu vc hay Olympic Quc t. Phương trình hàm tuyn tính bc hai là mt vn đ quan trng trong lp phương trình hàm nói chung. Trong chương trình toán chocác trưng THPT chuyên,phương trình hàm tuyn tính bc hai đưc đ cp trong trưng hp  ∆ >  0  ca phương trình đc trưng:  λ2 + αλ + β  = 0(∗) đi vi hàm 

Page 109: Dãy số - Giới hạn Tác giả: Trần Nam Dũng- Nguyễn Văn Mậu, 2007

8/13/2019 Dãy số - Giớ i hạn Tác giả: Trần Nam Dũng- Nguyễn Văn Mậu, 2007

http://slidepdf.com/reader/full/day-so-gioi-han-tac-gia-tran-nam-dung-nguyen-van-mau 109/217

4.3. Phương trình vi hàm s tun hoàn, phn tun hoàn nhân tính   109

tun hoàn cng tính; các trưng hp  ∆ = 0 và  ∆ <  0  ca phương trình ( ∗) chưa đưc đ cp đn. Ngoài ra, phương trình hàm tuyn tính bc hai đi vi hàm tun 

hoàn nhân tính chưa đưc đ cp đn c ba trưng hp:  ∆  >  0;∆ = 0và  ∆ < 0ca phương trình ( ∗). Hơn th na, phương trình hàm tuyn tính bc hai đi vi hàm tun hoàn cng tính và nhân tính cũng chưa đưc đ cp đn. Báo cáo này đưa ra ba bài ca phương trình hàm tuyn tính bc hai vi v phi là hàm s đi vi hàm tun hoàn và phn tun hoàn nhân tính.

4.3.1 Đnh nghĩa 

Cho   a ∈  R\{0; 1;−1};   α, β  ∈  R.  Tìm tt c các hàm:   f   :  R\{0} →  R  tha mãn điu kin:

f (a2x) + αf (ax) + βf (x) =  g(x).

trong đó  g(x) là hàm cho trưc.

4.3.2 Mt s bài toán

Bài toán 4.2.  Cho  h(x)  là hàm tun hoàn nhân tính chu kỳ  a trên  R(h(ax) =h(x));   a ∈ R\{0;1; −1};   α, β  ∈ R. Tìm tt c các hàm:  f   : R\{0} → R tha mãn điu kin:

f (a2x) + αf (ax) + βf (x) = h(x).   (4.15)

Li gii:Xét phương trình đc trưng:

λ2 + αλ + β  = 0; ∆ = α2 − 4β.   (4.16)

a) Trưng hp  ∆ >  0 :

Phương trình (2.2.40) có hai nghim thc:  λ1 = λ2. Áp dng đnh lý Viete ta đưc:

λ1 + λ2 = −α

λ1λ2 =  β Thay vào (2.2.39):

(2.2.39) ⇔ f (a2x) − (λ1 + λ2)f (ax) + λ1λ2f (x) =  h(x)⇔ f (a2x) − λ1f (ax) − λ2[f (ax) − λ1f (x)] =  h(x)   (4.17)

Page 110: Dãy số - Giới hạn Tác giả: Trần Nam Dũng- Nguyễn Văn Mậu, 2007

8/13/2019 Dãy số - Giớ i hạn Tác giả: Trần Nam Dũng- Nguyễn Văn Mậu, 2007

http://slidepdf.com/reader/full/day-so-gioi-han-tac-gia-tran-nam-dung-nguyen-van-mau 110/217

4.3. Phương trình vi hàm s tun hoàn, phn tun hoàn nhân tính   110

Đt  g1(x) =  f (ax) − λ1f (x) ta có:

(2.2.41) ⇔ g1(ax) − λ2g1(x) =  h(x)   (4.18)a1) Trưng hp 1:λ1 = 1 hoc  λ2 = 1.

Không mt tính tng quát ta gi s:  λ2 = 1. Khi đó:(2.2.42) ⇔ g1(ax) − g1(x) =  h(x) (theo Bài toán  1.1∗- chương 1)

⇒ g1(x) = g(x) + ln |x|h(x)

ln |a|   trong đó  g(x) là hàm tùy ý sao cho:

g(ax) =  g(x). Hay:

f (ax) − λ1f (x) =  g(x) + ln |x|h(x)

ln |a|   .   (4.19)

Đi vai trò  λ2 cho  λ1 và bin đi tương t ta có:

g2(ax) − λ1g2(x) =  h(x)

trong đó:g2(x) =  f (ax) − f (x).Vì  λ2 = 1 và  λ1 = λ2 ⇒ λ1 = 1. Theo Bài toán  1.3∗ chương 1 ta có:

g2(x) =  h(x)

1 − λ1+ |x|log|a| |λ1|.q (x)

trong đó  q (x) là hàm tùy ý sao cho:

q (ax) = q (x)   nu  λ1 >  0

−q (x)   nu  λ1 <  0

Hay:

f (ax) − f (x) =  h(x)

1 − λ1+ |x|log|a| |λ1|.q (x)   (4.20)

Tr (2.2.43) cho (2.2.44) ta đưc:

f (x) = ln |x|(1 − λ1) − ln |a|

ln |a|(1 − λ1)2  h(x) +

  1

1 − λ1

g(x) + |x|log|a| |λ1|q (x)

.

a2) Trưng hp 2:  λ1

 = 1 và  λ2

 = 1

⇒1 + α + β 

 = 0.

(vì nu  1 + α + β  = 0 ⇒ λ1  = 1 hoc  λ2 = 1 điu này mâu thun vi gi thit.)Ta có:

h(x) = 1 + α + β 

1 + α + β h(x) =

 h(a2x) + αh(ax) + βh(x)

1 + α + β 

Page 111: Dãy số - Giới hạn Tác giả: Trần Nam Dũng- Nguyễn Văn Mậu, 2007

8/13/2019 Dãy số - Giớ i hạn Tác giả: Trần Nam Dũng- Nguyễn Văn Mậu, 2007

http://slidepdf.com/reader/full/day-so-gioi-han-tac-gia-tran-nam-dung-nguyen-van-mau 111/217

4.3. Phương trình vi hàm s tun hoàn, phn tun hoàn nhân tính   111

(2.2.41) ⇔ f (a2x) −   h(a2x)

1 + α + β  + α

f (ax) −   h(ax)

1 + α + β +

+ β f (x) −   h(x)1 + α + β 

= 0

(4.21)

Đt  g(x) = f (x) −   h(x)

1 + α + β   ta có:

(2.2.45) ⇔ g(a2x) + αg(ax) + βg(x) = 0.

Theo bài toán 1.2- chương 1- trưng hp  ∆ >  0  ta có:

g(x) =  1

λ2 − λ1

|x|log|a| |λ2|h1(x) − |x|log|a| |λ1|h2(x)

trong đó:  λ1, λ2 là nghim ca phương trình:  λ2 + αλ + β  = 0.h1(x), h2(x) là hai hàm tùy ý sao cho:

h1(ax) =

h1(x)   nu  λ2 >  0

−h1(x)   nu  λ2 <  0;   h2(ax) =

h2(x)   nu  λ1 >  0

−h2(x)   nu  λ1 <  0

T đó ta có:

f (x) =  h(x)

1 + α + β  +

  1

λ2 − λ1

|x|log|a| |λ2|h1(x) − |x|log|a| |λ1|h2(x)

b) Trưng hp  ∆ = 0.

Phương trình đc trưng (2.2.40) có nghim: λ1 =  λ2 = −α2 ⇒ β  =  α2

4

(2.2.41) ⇔ f (a2x) + αf (ax) + α2

4 f (x) =  h(x))

⇔ f (a2x) + α

2f (ax) +

 α

2

f (ax) +

 α

2f (x)

 =  h(x)   (4.22)

Đt  g(x) = f (ax) + α

2f (x) thì:

(2.2.46) ⇔ g(ax) + α

2g(x) =  h(x)   (4.23)

b1) Trưng hp 1:  α  = 2 : (2.2.47) ⇔ g(ax) − g(x) = h(x). Theo Bài toán 

1.1∗- chương 1 ta có:   g(x) =  k(x) + ln |x|h(x)

ln |a|   trong đó   k(x)  là hàm tùy ý sao

Page 112: Dãy số - Giới hạn Tác giả: Trần Nam Dũng- Nguyễn Văn Mậu, 2007

8/13/2019 Dãy số - Giớ i hạn Tác giả: Trần Nam Dũng- Nguyễn Văn Mậu, 2007

http://slidepdf.com/reader/full/day-so-gioi-han-tac-gia-tran-nam-dung-nguyen-van-mau 112/217

4.3. Phương trình vi hàm s tun hoàn, phn tun hoàn nhân tính   112

cho:k(ax) =  k(x). Khi đó:

f (ax) − f (x) =  k(x) + ln |x|h(x)

ln |a|⇔ f (ax) − f (x) −   ln |x|h(x)

ln |a|   = k(x)

⇔ f (ax) − f (x) −   1

ln |a|(ln |ax|)2h(ax) − (ln |x|)2h(x) − (ln |a|)2h(x)

2 ln |a|

 =  k(x)

⇔ f (ax) − f (x) − (ln |ax|)2h(ax)

2(ln |a|)2   −

− (ln |x|)2h(x)

2(ln|a|)2

  − ln |ax|h(ax)

2 ln|a|

  −  ln |x|h(x)

2 ln|a| = k(x)

⇔ f (ax) − f (x) − (ln |ax|)2 − ln |a| ln |ax|2(ln |a|)2   h(ax)−

−  (ln |x|)2 − ln |a| ln |x|2(ln |a|)2

 =  k(x)

⇔ f (ax) − (ln |ax|)2 − ln |a| ln |ax|2(ln |a|)2   h(ax)−

− f (x) −  (ln |x|)2 − ln |a| ln |x|

2(ln |a|)2   h(x)

= k(x)   (4.24)

Đt  p(x) =  f (x) − (ln |x|)2

− ln |a| ln |x|2(ln |a|)2   h(x), khi đó:

(2.2.48) ⇔  p(ax) − p(x) =  k(x).

Theo Bài toán   1.1∗  chương 1 ta có:  p(x) =   I (x) + ln |x|k(x)

ln |a|   , trong đó   I (x)   là 

hàm tùy ý sao cho:  I (ax) =  I (x).

f (x) =  I (x) + ln |x|k(x)

ln |a|   + (ln |x|)2 − ln |a| ln |x|

2(ln |a|)2   h(x).

b2) Trưng hp 2:  α = −2; α <  0 :

(2.2.47)| ⇔ g(ax) +

 α

2 g(x) =  h(x).

Theo Bài toán  1.3∗  chương 1 ta có:

g(x) =  h(x)α2  + 1

 + |x|log|a| |α2 |q 1(x) = 2h(x)

α + 2 + |x|log|a|(−α

2 )q 1(x)

Page 113: Dãy số - Giới hạn Tác giả: Trần Nam Dũng- Nguyễn Văn Mậu, 2007

8/13/2019 Dãy số - Giớ i hạn Tác giả: Trần Nam Dũng- Nguyễn Văn Mậu, 2007

http://slidepdf.com/reader/full/day-so-gioi-han-tac-gia-tran-nam-dung-nguyen-van-mau 113/217

4.3. Phương trình vi hàm s tun hoàn, phn tun hoàn nhân tính   113

trong đó  q 1(x) là hàm tùy ý sao cho:  q 1(ax) =  q (x) (vì  α < 0). Ta có:

f (ax) +  α2 f (x) =  2h(x)α + 2  + |x|log|a|(−α

2 )q 1(x)

⇔ f (ax) + α

2f (x) −  2h(x)

α + 2  = |x|log|a|(−α

2 )q 1(x)

(4.25)

⇔ f (ax) + α

2f (x) −   2

α + 2

h(ax)α2  + 1

 + dfracα2 h(x)α2  + 1

 = |x|   log|a|(−α

2 )q 1(x)

(4.26)

⇔ f (ax) +  α2

f (x) −   4h(ax)(α + 2)2

 + α2

4h(x)(α + 2)2

 = |x|log|a|(−α2 )q 1(x)

⇔ f (ax) −   4h(ax)

(α + 2)2 +

 α

2

f (x) −   4h(x)

(α + 2)2

 = |x|log|a|(−α2 )q 1(x)   (4.27)

Đt  f 1(x) =  f (x) −   4h(x)

(α + 2)2 ta có:

(2.2.49) ⇔ f 1(ax) + α

2f 1(x) = |x|log|a|(−α

2 )q 1(x).

Áp dng kt qu Bài toán 1.2-chương 1 (trưng hp   ∆ = 0, α <  0, α = −2) ta đưc:

f 1(x) = |x|log|a|(−α2 )

k1(x) − 2 ln |x|q 1(x)

α ln |a|

trong đó:  k1(x) là hàm tùy ý sao cho:  k1(ax) =  k1(x). Do đó:

f (x) =  4h(x)

(α + 2)2 + |x|log|a|(−α

2 )k1(x) −  2 ln |x|q 1(x)

α ln |a|

b2) Trưng hp 3:  α >  0, khi đó  (2.2.47) ⇔ g(ax) + α

2g(x) =  h(x)

Theo Bài toán  1.3∗- chương 1 ta có:

g(x) =  h(x)

α2  + 1 +

|x|log|a| |α2 |q 

2(x) =

 2h(x)

α + 2 +

|x|log|a|

α2 q 

2(x)

trong đó  q 2(x) là hàm tùy ý sao cho:  q 2(ax) = −q 2(x) (vì  α > 0). T đó có:

f (ax) + α

2f (x) =

 2h(x)

α + 2 + |x|log|a| α2 q 2(x) (∗)

Page 114: Dãy số - Giới hạn Tác giả: Trần Nam Dũng- Nguyễn Văn Mậu, 2007

8/13/2019 Dãy số - Giớ i hạn Tác giả: Trần Nam Dũng- Nguyễn Văn Mậu, 2007

http://slidepdf.com/reader/full/day-so-gioi-han-tac-gia-tran-nam-dung-nguyen-van-mau 114/217

4.3. Phương trình vi hàm s tun hoàn, phn tun hoàn nhân tính   114

Bin đi tương t như trưng hp 2 trên ta có:

(∗) ⇔ f (ax) −   4h(ax)(α + 2)2  +  α2f (x) −   4h(x)(α + 2)2

= |x|log|a|α

2 q 2(x)   (4.28)

Đt  f 2(x) =  f (x) −   4h(x)

(α + 2)2 ta có:

(2.2.50) ⇔ f 2(ax) + α

2f 2(x) = |x|log|a| α2 q 2(x)

Áp dng kt qu Bài toán 1.2- chương 1 (trưng hp  ∆ = 0, α >  0) ta đưc:

f 2(x) = |x|log|a| α2

k2(x) −  2 ln |xa|q 2(x)

α ln

|a

| trong đó  k2(x) là hàm tùy ý sao cho:  k2(ax) = −k2(x). Vây:

f (x) =  4h(x)

(α + 2)2 + |x|log|a| α2

k2(x) −  2 ln |xa |q 2(x)

α ln |a|

c) Trưng hp  ∆ <  0 ⇔ 1 + α + β  = 0. (Chng minh hoàn toàn tương t như Bài toán 2.1.) ta có:

h(x) = 1 + α + β 

1 + α + β h(x) =

 h(a2x) + αh(ax) + βh(x)

1 + α + β 

(Vì:  h(a2x) = h(ax) =  h(x))

(2.2.39) ⇔ f (a2x)−   h(a2)

1 + α + β  + α

f (ax)−   h(ax)

1 + α + β 

+β 

f (x)−   h(x)

1 + α + β 

= 0

(4.29)

Đt  g(x) = f (x) −   h(x)

1 + α + β   ta có:

(2.2.51) ⇔ g(a2x) + αg(ax) + βg(x) = 0.

Phương trình đc trưng:  λ2 + αλ + β  = 0,   ∆  <  0  nên có hai nghim phc liên hp  λ1, λ2. Theo Bài toán (1.2)- chương 1 (trưng hp  ∆ <  0) ta có:

g(x) = |λ2|

ln |x|ln |a|

cos

 ϕ ln |x|ln |a|   n(x) + sin

 ϕ ln |x|ln |a|   m(x)

trong đó:

λ1 =  p − iq ;   λ2 =  p + iq  ⇔ |λ1| = |λ2| = 

 p2 + q 2

Page 115: Dãy số - Giới hạn Tác giả: Trần Nam Dũng- Nguyễn Văn Mậu, 2007

8/13/2019 Dãy số - Giớ i hạn Tác giả: Trần Nam Dũng- Nguyễn Văn Mậu, 2007

http://slidepdf.com/reader/full/day-so-gioi-han-tac-gia-tran-nam-dung-nguyen-van-mau 115/217

4.3. Phương trình vi hàm s tun hoàn, phn tun hoàn nhân tính   115

tan ϕ =  q 

 p; arg λ2 = arg λ1 =  ϕ

m(x), n(x)  là hai hàm tùy ý tha mãn:  m(ax) = m(x);   n(ax) = n(x). T đó ta có:

f (x) =  h(x)

1 + α + β  +

 |λ2|ln |x|ln |a|

cos

 ϕ ln |x|ln |a|   n(x) + sin

 ϕ ln |x|ln |a|   m(x)

Bài toán 4.3.   Cho   h(x)   là hàm phn tun hoàn nhân tính chu kỳ   a   trên   R( h(ax) = −h(x));   a ∈   R\{0;1; −1}; α, β  ∈   R.  Xác đnh tt c các hàm   f   :R\{0} → R tha mãn điu kin:

f (a2x) + αf (ax) + βf (x) = h(x).   (4.30)

Li gii:

Xét phương trình đc trưng:

λ2 + αλ + β  = 0   (4.31)

có  ∆ = α2 − 4β.

a) Trưng hp  ∆ >  0 : Phương trình (2.2.53) có hai nghim thc  λ1 = λ2. Ápdng đnh lí Viete ta có:

λ1 + λ2 = −α

λ1λ2 =  β thay vào (2.2.52) ta có:

(2.2.52) ⇔ f (a2

x) − (λ1 + λ2)f (ax) + λ1λ2f (x) =  h(x)⇔ f (a2x) − λ1f (ax) − λ2[f (ax) − λ1f (x)] =  h(x)

(4.32)

Đt  g1(x) =  f (ax) − λ1f (x), khi đó:

(2.2.54) ⇔ g1(ax) − λ2g1(x) =  h(x).   (4.33)

a1) Trưng hp 1:  λ1 = ±1 và  λ2 = 1.

(2.2.52) ⇔ f (a2x) − (λ1 + 1)f (ax) + λ1f (x) =  h(x).

(2.2.55) ⇔  g1(ax) − g1(x) =  h(x).  Theo Bài toán   1.2∗  - chương 1 thì:   g1(x) =

k(x) − h(x)

2   . trong đó  k(x) là hàm tùy ý sao cho:  k(ax) =  k(x).Hay 

f (ax) − λ1f (x) = k(x) − h(x)

2  .   (4.34)

Page 116: Dãy số - Giới hạn Tác giả: Trần Nam Dũng- Nguyễn Văn Mậu, 2007

8/13/2019 Dãy số - Giớ i hạn Tác giả: Trần Nam Dũng- Nguyễn Văn Mậu, 2007

http://slidepdf.com/reader/full/day-so-gioi-han-tac-gia-tran-nam-dung-nguyen-van-mau 116/217

4.3. Phương trình vi hàm s tun hoàn, phn tun hoàn nhân tính   116

Đi vai trò  λ2 cho  λ1 và bin đi tương t ta đưc:g2(ax)

−λ1g2(x) =  h(x) trong đó:  g2(x) =  f (ax)

−f (x).

Vì  λ1 = ±1 nên theo Bài toán  1.4∗(ii)- chương 1, ta có:

g2(x) = |x|log|a| |λ1|q (x) +  h(x)

−λ1 − 1 = |x|log|a| |λ1|q (x) −   h(x)

λ1 + 1

trong đó  q (x) là hàm tùy ý sao cho:

q (ax) =

q (x)   nu  λ1 >  0

−q (x)   nu  λ1 <  0

Khi đó:

f (ax) − f (x) = |x|log|a|

|λ1

|q (x) −  h(x)

λ1 + 1   (4.35)Tr (2.2.56) cho (2.2.57) ta đưc:

f (x) =  h(x)

2(λ1 + 1) +

  1

1 − λ1

k(x) − |x|log|a| |λ1|q (x)

a2) Trưng hp 2:  λ1 = −1 và  λ2 = 1(2.2.52) ⇔ f (a2x) − f (x) = h(x).(2.2.55) ⇔ g1(ax) − g1(x) =  h(x). Theo Bài toán  1.2∗- chương 1 ta có:

g1(x) =  k(x) −  h(x)

2  , trong đó  k(x) là hàm tùy ý tha mãn:  k(ax) =  k(x).

Vy:f (ax) + f (x) =  k(x) − h(x)

2  (4.36)

Đi vai trò ca  λ1 và  λ2 và bin đi tương t ta đưc:g2(ax) + g2(x) =  h(x) trong đó  g2(x) =  f (ax) − f (x).

Theo Bài toán  1.4∗(i)- chương 1 ta có:  g2(x) =  q (x) − ln |xa |h(x)

ln |a|   vi  q (x) là hàm 

tùy ý sao cho:  q (ax) = −q (x).Hay:

f (ax) − f (x) =  q (x) −  ln |xa |h(x)

ln |a|   (4.37)

Tr (2.2.58) cho (2.2.59) đưc:  f (x) =  k(x) − q (x)2

  + ln |xa3 |

4 ln |a|h(x).

a3) Trưng hp 3:  λ1 = ±1; λ2 = −1.

(2.2.52) ⇔ f (a2x) − (λ1 − 1)f (ax) − λ1f (x) =  h(x).

Page 117: Dãy số - Giới hạn Tác giả: Trần Nam Dũng- Nguyễn Văn Mậu, 2007

8/13/2019 Dãy số - Giớ i hạn Tác giả: Trần Nam Dũng- Nguyễn Văn Mậu, 2007

http://slidepdf.com/reader/full/day-so-gioi-han-tac-gia-tran-nam-dung-nguyen-van-mau 117/217

4.3. Phương trình vi hàm s tun hoàn, phn tun hoàn nhân tính   117

(2.2.55) ⇔ g1(ax) + g1(x) =  h(x), theo Bài toán  1.4∗(i)- chương 1 ta có:

g1

(x) =  q 1

(x)−

ln |xa |h(x)

ln |a|  , trong đó  q 

1(x) là hàm tùy ý sao cho:q 

1(ax) =

−q 1

(x).

Vy:

f (ax) − λf (x) =  q 1(x) −  ln |xa |h(x)

ln |a|   (4.38)

Đi vai trò  λ1 cho  λ2 và bin đi tương t ta có:g2(ax) − λ1g2(x) =  h(x), trong đó  g2(x) = f (ax) + f (x).Vì  λ1 = ±1 nên theo Bài toán  1.4∗(ii)- chương 1 có:

g2(x) = |x|log|a| |λ1|q 2(x) −   h(x)

λ1 + 1

trong đó  q 2(x) là hàm tùy ý sao cho:

q 2(ax) =

q 2(x)   nu  λ1 >  0

−q 2(x)   nu  λ1 <  0

hay:

f (ax) + f (x) = |x|log|a| |λ1|q 2(x) −   h(x)

λ1 + 1.   (4.39)

Tr (2.2.61) cho (2.2.60) ta đưc:

f (x) =

ln

|

xλ1+1

a

λ1+2

|ln |a|(λ1 + 1)2h(x) +

  1

λ1 + 1 |x|log

|a| |λ1|q 2(x) − q 1(x).

a4) Trưng hp 4:  λ1 = ±1, λ2 = ±1 ⇒ 1 + α + β  = 0.

Ta có:  h(x) = 1 + α + β 

1 + α + β h(x) =

 h(a2x) + αh(ax) + βh(x)

1 + α + β (Vì:  h(a2x) = h(ax) =  h(x).)

(2.2.52) ⇔ f (a2x) −   h(a2x)

1 + α + β  + α

f (ax) −   h(ax)

1 + α + β 

+

+ β 

f (x) −   h(x)

1 + α + β = 0

(4.40)

Đt  g(x) = f (x) −   h(x)1 + α + β 

  ta có:  (2.2.62) ⇔ g(a2x) + αg(ax) + βg(x) = 0.

Phương trình đc trưng:  λ2 + αλ + β  = 0  ta có:  ∆ >  0 nên có hai nghim thc 

Page 118: Dãy số - Giới hạn Tác giả: Trần Nam Dũng- Nguyễn Văn Mậu, 2007

8/13/2019 Dãy số - Giớ i hạn Tác giả: Trần Nam Dũng- Nguyễn Văn Mậu, 2007

http://slidepdf.com/reader/full/day-so-gioi-han-tac-gia-tran-nam-dung-nguyen-van-mau 118/217

4.3. Phương trình vi hàm s tun hoàn, phn tun hoàn nhân tính   118

phân bit  λ1, λ2.

Áp dng kt qu Bài toán 1.2- chương 1 (trưng hp  ∆ >  0) ta có:

g(x) =  1

λ2 − λ1

|x|log|a| |λ2|h1(x) − |x|log|a| |λ1|h2(x)

trong đó  h1(x), h2(x) là hai hàm tùy ý tha mãn:

h1(ax) =

h1(x)   nu  λ2 >  0

−h1(x)   nu  λ2 <  0, h2(ax) =

h2(x)   nu  λ1  >  0

−h2(x)   nu  λ1  <  0

Vy:

f (x) =  h(x)

1 + α + β 

 +  1

λ2 − λ1|x|log|a| |λ2|h1(x)

− |x

|log|a| |λ1|h2(x)

b) Trưng hp  ∆ = 0

Phương trình (2.2.53) có nghim:  λ1 =  λ2 = −α

2 ⇒ β  =

 α2

4  .

(2.2.52) ⇔ f (a2x) + αf (ax) + α2

4 f (x) =  h(x)

⇔ f (a2x) + α

2f (ax) +

 α

2[f (ax) +

 α

2f (x)] =  h(x)   (4.41)

Đt  g(x) = f (ax) + α

2f (x) thì:

(2.2.63) ⇔ g(ax) +  α2

g(x) =  h(x).   (4.42)

b1) Trưng hp 1:  α = −2, khi đó:  (2.2.64) ⇔ g(ax) − g(x) =  h(x).

Theo Bài toán  1.2∗- chương 1 ta có:  g(x) =  k(x) −  h(x)

2  trong đó   k(x)  là hàm 

tùy ý tha mãn:  k(ax) =  k(x). Ta có:

f (ax) − f (x) =  k(x) −  h(x)

2  ⇔ f (ax) − f (x) +

 h(x)

2  = k(x)

⇔ f (ax) − f (x) + 1

2[h(x)

2  −  h(ax)

2  ] = k(x)

⇔ f (ax) − h(ax)4

  − [f (x) − h(x)4

  ] = k(x)   (4.43)

Đt   p(x) =  f (x) −  h(x)

4  ,  khi đó:   (2.2.65) ⇔ p(ax) − p(x) =  k(x).  Theo Bài 

toán  1.1∗- chương 1 ta có:

Page 119: Dãy số - Giới hạn Tác giả: Trần Nam Dũng- Nguyễn Văn Mậu, 2007

8/13/2019 Dãy số - Giớ i hạn Tác giả: Trần Nam Dũng- Nguyễn Văn Mậu, 2007

http://slidepdf.com/reader/full/day-so-gioi-han-tac-gia-tran-nam-dung-nguyen-van-mau 119/217

4.3. Phương trình vi hàm s tun hoàn, phn tun hoàn nhân tính   119

 p(x) =  q (x) + ln |x|k(x)

ln |a|   trong đó  q (x) là hàm tùy ý tha mãn:  q (ax) =  q (x). Vy:

f (x) =  q (x) + h(x)

4  +

 ln |x|k(x)

ln |a|   .

b2) Trưng hp 2:α = 2.Khi đó:  (2.2.64) ⇔ g(ax) + g(x) =  h(x). Theo Bài toán  1.4∗(i)- chương 1 ta có:

g(x) =  g1(x) −  ln |xa |h(x)

ln |a|   vi  g1(x) là hàm tùy ý:  g1(ax) = −g1(x). Ta có:

f (ax) + f (x) =  g1(x) − ln |xa |h(x)

ln |a|

⇔ f (ax) + f (x) +

 ln

|xa

|h(x)

ln |a|   = g1(x)

⇔ f (ax) + f (x) + ln |xa |h(x)

ln |a|   − h(x) =  g1(x)

⇔ f (ax) + f (x) +  1

ln |a|(ln |ax|)2h(ax) + (ln |x|)2h(x) + (ln |a|)2h(x)

−2 ln |a|− −  ln |x|h(ax)

ln |a|   −   ln |xa|h(x)

ln |a|

= g1(x)

⇔ f (ax) + f (x) −  (ln |ax|)2h(ax)

2(ln |a|)2   − (ln |x|)2h(x)

2(ln |a|)2   −

− h(x)

2   +

 ln

|x

|h(ax)

ln |a|   +

 ln

|xa

|h(x)

ln |a|   = g1(x).

⇔ f (ax) + f (x) −  (ln |ax|)2h(ax)

2(ln |a|)2   − (ln |x|)2h(x)

2(ln |a|)2   + ln |x|h(ax)

ln |a|   + ln |xa |h(x)

ln |a|   −

− 1

2

−  ln |x|h(ax)

ln |a|   − ln |xa |h(x)

ln |a|

 =  g1(x)

⇔ f (ax) + 3 ln |a| ln |x| − (ln |ax|)2

2(ln |a|)2   h(ax)+

+ f (x) + 3 ln |a| ln |xa | − (ln |x|)2

2(ln |a|)2

  h(x) =  g1(x).   (4.44)

Đt  f 1(x) =  f (x) + 3 ln |a| ln |xa | − (ln |x|)2

2(ln |a|)2   h(x) ta có:

(2.2.66) ⇔ f 1(ax) + f 1(x) =  g1(x). Theo Bài toán  1.4∗(ii)- chương 1 ta có:

Page 120: Dãy số - Giới hạn Tác giả: Trần Nam Dũng- Nguyễn Văn Mậu, 2007

8/13/2019 Dãy số - Giớ i hạn Tác giả: Trần Nam Dũng- Nguyễn Văn Mậu, 2007

http://slidepdf.com/reader/full/day-so-gioi-han-tac-gia-tran-nam-dung-nguyen-van-mau 120/217

4.3. Phương trình vi hàm s tun hoàn, phn tun hoàn nhân tính   120

f 1(x) =  g2(x) −  ln |xa |g1(x)

ln |a|   , g1(x) tha mãn:  g2(ax) = −g2(x). Vy:

f (x) =  g2(x) −  ln |xa |g1(x)

ln |a|   − 3 ln |a| ln |xa | − (ln |x|)22(ln |a|)2   h(x).

b3) Trưng hp 3:  α <  0, α = −2 ⇒  α

2 = ±1.

g(ax) + α

2g(x) =  h(x). Theo Bài toán  1.4∗(ii)- chương 1 ta có:

g(x) =  h(x)α2 − 1

 + |x|log|a| |α2 |q 1(x)

= 2h(x)

α − 2 + |x|log|a|(−α

2 )q 1(x)

trong đó  q 1(x) là hàm tùy ý tha mãn:  q 1(ax) =  q 1(x).(Vì: α < 0). Như vy:

f (ax) + α

2f (x) =

 2h(x)

α − 2 + |x|log|a|(−α

2 )q 1(x)

⇔ f (ax) + α

2f (x) −  2h(x)

α − 2 = |x|log|a|(−α

2 )q 1(x)

⇔ f (ax) + α

2f (x) −   2

α − 2

h(ax)α2 − 1

 + α

2

h(x)α2 − 1

 = |x|log|a|(−α

2 )q 1(x)

⇔ f (ax) −   4h(x)

−2)2

 + α

2 f (x) −   4h(x)

−2)2

= |x|log|a|(−α2 )q 1(x)   (4.45)

Đt  g1(x) =  f (x) −   4h(x)

(α − 2)2,  (2.2.67) ⇔ g1(ax) +

 α

2g1(x) = |x|log|a|(−α

2 )q 1(x).

Theo Bài toán 1.2- chương 1 (trưng hp  ∆ = 0; α <  0; α = −2) ta có:

g1(x) = |x|log|a|(−α2 )q 1(x)

 p1(x) − 2 ln |x|q 1(x)

α ln |a|

trong đó  p1(x) là hàm tùy ý:  p1(ax) = p1(x). Vy:

f (x) =  4h(x)

(α − 2)2 + |x|log|a|(−α

2 ) p1(x) − 2 ln |x|q 1(x)

α ln |a|

.

b4) Trưng hp 4:  α >  0; α = 2 ⇔ α

2 = ±1 Khi đó:g(ax) +

 α

2g(x) =  h(x). Theo Bài toán  1.4∗(ii)- chương 1:

g(x) =  h(x)α2 − 1

 + |x|log|a| |α2 |q 2(x) = 2h(x)

α − 2 + |x|log|a| α2 q 2(x)

Page 121: Dãy số - Giới hạn Tác giả: Trần Nam Dũng- Nguyễn Văn Mậu, 2007

8/13/2019 Dãy số - Giớ i hạn Tác giả: Trần Nam Dũng- Nguyễn Văn Mậu, 2007

http://slidepdf.com/reader/full/day-so-gioi-han-tac-gia-tran-nam-dung-nguyen-van-mau 121/217

4.3. Phương trình vi hàm s tun hoàn, phn tun hoàn nhân tính   121

vi  q 2(x) là hàm tùy ý tha mãn:  q 2(ax) = −q 2(x) (Vì:  α > 0). Do đó:

f (ax) + α

2 f (x) = 2h(x)

α − 2 + |x|log|a| |α

2 |q 2(x) (∗)

Bin đi tương t như trưng hp 3 ta đưc:

(∗) ⇔ f (ax) −   4h(ax)

(α − 2)2 +

 α

2

f (x) −   4h(x)

(α − 2)2

= |x|log|a| α2 q 2(x)   (4.46)

Đt  g2(x) =  f (x) −   4h(x)

(α − 2)2  ta có:

(2.2.68) ⇔ g2(x) + α

2g2(x) = |x|log|a| α2 q 2(x).

Theo bài toán 1.2- chương 1(trưng hp  ∆ = 0; α >  0; α = 2):

g2(x) = |x|log|a| α2  p2(x) −  2 ln |xa |q 2(x)

α ln |a|

,

trong đó  p2(x) là hàm tùy ý tha mãn:  p2(ax) =  p2(x). T đó ta có:

f (x) =  4h(x)

(α − 2)2 + |x|log|a| α2  p2(x) −  2 ln |xa |q 2(x)

α ln |a|

.

c) Trưng hp:  ∆ <  0  D dàng chng minh đưc  1 − α + β  = 0. Ta có:

h(x) = 1 − α + β 

1−

α + β h(x) =

 h(a2x) + αh(ax) + βh(x)

1−

α + β 

(Vì:  h(a2x) = −h(ax) =  h(x))

(2.2.52) ⇔ f (a2x) −   h(a2)

1 − α + β  + α

f (ax) −   h(ax)

1 − α + β 

+

+ β 

f (x) −   h(x)

1 − α + β 

= 0

(4.47)

Đt  g(x) = f (x) −   h(x)

1 − α + β   ta có:

(2.2.69) ⇔ g(a2x) + αg(ax) + βg(x) = 0.

Phương trình đc trưng:  λ2 + αλ + β  = 0,   ∆  <  0  nên có hai nghim phc liên hp  λ1, λ2. Theo Bài toán (1.2)- chương 1 (trưng hp  ∆ <  0) ta có:

g(x) = |λo|

ln |x|ln |a|

cos

 ϕ ln |x|ln |a|   n(x) + sin

 ϕ ln |x|ln |a|   m(x)

Page 122: Dãy số - Giới hạn Tác giả: Trần Nam Dũng- Nguyễn Văn Mậu, 2007

8/13/2019 Dãy số - Giớ i hạn Tác giả: Trần Nam Dũng- Nguyễn Văn Mậu, 2007

http://slidepdf.com/reader/full/day-so-gioi-han-tac-gia-tran-nam-dung-nguyen-van-mau 122/217

4.3. Phương trình vi hàm s tun hoàn, phn tun hoàn nhân tính   122

trong đó:

λ1

 =  p−

iq ;   λ2

 =  p + iq  ⇔ |

λo|

=|λ1|

=|λ2|

=   p2 + q 2

tan ϕ =  q 

 p; arg λ2 = arg λ1 =  ϕ

m(x), n(x)  là hai hàm tùy ý tha mãn:  m(ax) = m(x);   n(ax) = n(x). T đó ta có:

f (x) =  h(x)

1 − α + β  +

 |λo|ln |x|ln |a|

cos

 ϕ ln |x|ln |a|   n(x) + sin

 ϕ ln |x|ln |a|   m(x)

.

Nhn xét 2.2  Sau khi th li hai bài toán trên ta nhn thy: Trong biu thc nghim ca tt c các trưng hp, phn biu thc có cha  h(x) là nghim riêng ca phương trình: f (a2x) + αf (ax) + βf (x) =  h(x).

Bài toán 4.4. Cho  g(x) là hàm tun hoàn nhân tính chu kỳ  a, (g(ax) = g(x)); h(x)là hàm phn tun hoàn nhân tính chu kỳ  a, (h(ax) = −h(x)); a ∈ R\{0, 1, −1}; α, β ∈R. Xác đnh tt c các hàm:  f   : R\{0} → R tha mãn điu kin:

f (a2x) + αf (ax) + βf (x) = g(x) + h(x).   (4.48)

Li gii:Xét phương trình đc trưng:

λ2 + αλ + β  = 0, ∆ =  α2 − 4β.   (4.49)

a) Trưng hp  ∆ >  0: Phương trình đc trưng có hai nghim thc:  λ1 = λ2.a1) Trưng hp 1:  λ1 = ±1; λ2 = 1.

Áp dng đnh lý Viete: 1 + λ1 = −α

λ1 =  β 

(2.2.70) ⇔ f (a2x) − (λ1 + 1)f (ax) + λ1f (x) =  g(x) + h(x)   (4.50)

Xét phương trình:

f 1(a2x) − (λ1 + 1)f 1(ax) + λ1f 1(x) =  g(x).   (4.51)

Áp dng kt qu ca Bài toán 2.4 (trưng hp a1) và nhn xét 2.2 ta có biu 

thc:  ln |x|(1 − λ1) − ln |a|

ln |a|(1 − λ1)2

  g(x) là nghim riêng ca (2.2.73). Thay vào ta đưc:

ln |a2x|(1 − λ1) − ln |a|ln |a|(1 − λ1)2

  g(a2x) − (λ1 + 1)ln |ax|(1 − λ1) − ln |a|

ln |a|(1 − λ1)2  g(ax)+

+ λ1ln |x|(1 − λ1) − ln |a|

ln |a|(1 − λ1)2  g(x) =  g(x)

(4.52)

Page 123: Dãy số - Giới hạn Tác giả: Trần Nam Dũng- Nguyễn Văn Mậu, 2007

8/13/2019 Dãy số - Giớ i hạn Tác giả: Trần Nam Dũng- Nguyễn Văn Mậu, 2007

http://slidepdf.com/reader/full/day-so-gioi-han-tac-gia-tran-nam-dung-nguyen-van-mau 123/217

4.3. Phương trình vi hàm s tun hoàn, phn tun hoàn nhân tính   123

Thay (2.2.74) vào (2.2.72):

(2.2.72) ⇔ f (a2x) − ln |a2

x|(1 − λ1) − ln |a|ln |a|(1 − λ1)2

  g(a2x)−

− (λ1 + 1)f (ax) −  ln |ax|(1 − λ1) − ln |a|

ln |a|(1 − λ1)2  g(ax)

+

+ λ1

f (x) − ln |x|(1 − λ1) − ln |a|

ln |a|(1 − λ1)2  g(x)

= g(x)

(4.53)

Đt  f 2(x) =  f (x) − ln |x|(1 − λ1) − ln |a|ln |a|(1 − λ1)2

  g(x) (i) ta có:

(2.2.75) ⇔ f (a2x) − (λ1 + 1)f 2(ax) + λ1f 2(x) = h(x).

Áp dng kt qu Bài toán 2.5 (trưng hp a1) ta đưc:

f 2(x) =  h(x)

2(λ1 + 1) +

  1

1 − λ1

k(x) − |x|log|a| |λ1|q (x)

  (ii)

trong đó  k(x); q (x) là hai hàm tùy ý tha mãn:

k(ax) =  k(x); q (ax) =

q (x)   nu  λ1 >  0

−q (x)   nu  λ1 <  0

T  (i) và  (ii) ta có:

f (x) =  h(x)

2(λ1 + 1) +

 ln |x|(1 − λ1) − ln |a|ln |a|(1 − λ1)2

  g(x) +  1

1 − λ1

k(x) − |x|log|a| |λ1|q (x)

.

a2) Trưng hp 2:  λ1 = −1; λ2 = 1.Bin đi tương t như trưng hp a1) ta đưc:

f (x) =ln | x

a3|

4 ln |a|h(x) + ln |x2a |

ln a4  g(x) +

 k(x) − q (x)

2

trong đó  k(x), q (x) là hai hàm tùy ý tha mãn:  k(ax) =  k(x); q (ax) = −q (x).a3) Trưng hp 3:  λ1

 =

±1; λ2 =

−1.

Bin đi tương t như trưng hp a1) ta có:

f (x) =ln |xλ1+1

aλ1+2 |h(x)

ln |a|(λ1 + 1)2 +

  g(x)

2(1 − λ1) +

|x|log|a||λ1| q 2(x) − q 1(x)

Page 124: Dãy số - Giới hạn Tác giả: Trần Nam Dũng- Nguyễn Văn Mậu, 2007

8/13/2019 Dãy số - Giớ i hạn Tác giả: Trần Nam Dũng- Nguyễn Văn Mậu, 2007

http://slidepdf.com/reader/full/day-so-gioi-han-tac-gia-tran-nam-dung-nguyen-van-mau 124/217

4.3. Phương trình vi hàm s tun hoàn, phn tun hoàn nhân tính   124

vi  q 1(x), q 2(x) là hai hàm tùy ý tha mãn:

q 1(ax) = −q 1(x);   q 2(ax) =q 2(x)   nu  λ1  >  0

−q 2(x)   nu  λ1  <  0

a4) Trưng hp 4:  λ1 = ±1; λ2 = ±1.Bin đi tương t như trưng hp a1) ta có:

f (x) =  h(x)

1 − α + β  +

  g(x)

1 + α + β  +

  1

λ2 − λ1

|x|log|a| |λ2|h1(x) − |x|log|a| |λ1|h2(x)

trong đó  h1(x), h2(x) là hai hàm tùy ý tha mãn:

h1(x) = h1(x)   nu  λ2  >  0

−h1(x)   nu  λ2  <  0 h2(x) = h2(x)   nu  λ1 >  0

−h2(x)   nu  λ1 <  0

b) Trưng hp   ∆ = 0b1) Trưng hp 1:  α = −2Bin đi tương t như trưng hp a1) ta có:

f (x) = h(x)

4  +

 (ln |x|)2 − ln |a| ln |x|2(ln |a|)2   h(x) + q (x) +

 ln |x|k(x)

ln |a|

Vi  k(x), q (x) là hai hàm tùy ý tha mãn:  k(ax) =  k(x), q (ax) = q (x).b2) Trưng hp 2:  α = 2

Bin đi tương t như trưng hp a1) ta có:

f (x) = −3 ln |a| ln |xa | − (ln |x|)22(ln |a|)2   h(x) +

 g(x)

4  + g2(x) − ln |xa |g1(x)

ln |a|

vi  g1(x), g2(x) là hai hàm tùy ý tha mãn:  g1(ax) = −g1(x); g2(ax) = −g2(x).b3) Trưng hp 3:  α <  0; α = −2Bin đi tương t như trưng hp a1) ta có:

4h(x)

(α − 2)2 +

  4g(x)

(α + 2)2 + |x|log|a|(−α

2 ) p1(x) −  2 ln |x|q 1(x)

α ln |a|

,

trong đó:  p1(x), q 1(x) là hai hàm tùy ý:  p1(ax) =  p1(x); q 1(ax) =  q 1(x)b4) Trưng hp 4:  α >  0; α = 2Bin đi tương t như trưng hp a1) ta có:

4h(x)

(α − 2)2 +

  4g(x)

(α + 2)2 + |x|log|a| α2  p2(x) −  2 ln |xa |q 2(x)

α ln |a|

Page 125: Dãy số - Giới hạn Tác giả: Trần Nam Dũng- Nguyễn Văn Mậu, 2007

8/13/2019 Dãy số - Giớ i hạn Tác giả: Trần Nam Dũng- Nguyễn Văn Mậu, 2007

http://slidepdf.com/reader/full/day-so-gioi-han-tac-gia-tran-nam-dung-nguyen-van-mau 125/217

4.3. Phương trình vi hàm s tun hoàn, phn tun hoàn nhân tính   125

trong đó:  p2(x), q 2(x) là hai hàm tùy ý:  p2(ax) = − p2(x); q 2(ax) = −q 2(x)c) Trưng hp   ∆ <  0  Bin đi tương t như trưng hp a1) ta có:

f (x) =  h(x)

1 − α + β  +

  g(x)

1 + α + β  +

 |λo|ln |x|ln |a|

cos

 ϕ ln |x|ln |a|   n(x) + sin

 ϕ ln |x|ln |a|   m(x)

trong đó:λ1 =  p − iq ;   λ2  =  p + iq  là nghim phương trình:  λ2 + αλ + β  = 0.⇔ |λo| = |λ1| = |λ2| =

  p2 + q 2

tan ϕ =  q 

 p; arg λ2  = arg λ1 =  ϕ; m(x), n(x) là hai hàm tùy ý tha mãn:  m(ax) =

m(x);   n(ax) =  n(x).

4.3.3 Mt s ví d áp dng

Ví d 4.3.  Cho  g(x) là hàm tun hoàn nhân tính chu kỳ  3, (g(3x) = g(x)); h(x)là hàm phn tun hoàn nhân tính chu kỳ  3, (h(3x) = −h(x)). Xác đnh tt c các hàm  f   : R∗ → R  sao cho:

f (9x) − 7f (3x) + 10f (x) = 5g(x) + 21h(x).

Li gii:Xét phương trình đc trưng: λ2 − 7λ + 10 = 0 ⇔ λ1 = 2; λ2 = 5.áp dng Bài toán 2.6 (trưng hp a4) ta có:

f (x) = h(x)

18  +

 g(x)

4  +

 1

3

|x|log3 5h1x − |x|log3 2h2x

trong đó  h1, h2 là hai hàm tùy ý:  h1(3x) =  h1(x); h2(3x) =  h2(x)

Ví d 4.4.   Cho   g(x)   là hàm tun hoàn nhân tính chu kỳ  −1

3, (g(−1

3x) =

g(x)); h(x)  là hàm phn tun hoàn nhân tính chu kỳ −1

3, (h(−1

3x) =  h(x)). Xác 

đnh tt c các hàm:  f   : R\{0} →  R  sao cho:

f (1

9x) + 2f (−1

3x) + f (x) = 4g(x) − 7h(x) + 13.

Li gii:Đt  g1(x) = 4g(x) + 13 ⇒ g1(−1

3x) = g1(x).Xét phương trình đc trưng:  λ2 + 2λ + 1 = 0, ∆ = 0

 ⇒ λ1  =  λ2  =

 −1  hơn na 

α = 2, áp dng bài toán 2.6 trưng hp b2) ta có:

f (x) = 4g(x) + 13

4  − ln 9 ln |3x| − (ln |x|)2

2(ln3)2  7h(x) +

 ln |3x|g1(x)

ln 3  + g2(x),

g1(x), g2(x) là các hàm tùy ý tha mãn:  g1(−13x) = −g1(x); g2(−1

3x) = −g2(x).

Page 126: Dãy số - Giới hạn Tác giả: Trần Nam Dũng- Nguyễn Văn Mậu, 2007

8/13/2019 Dãy số - Giớ i hạn Tác giả: Trần Nam Dũng- Nguyễn Văn Mậu, 2007

http://slidepdf.com/reader/full/day-so-gioi-han-tac-gia-tran-nam-dung-nguyen-van-mau 126/217

4.3. Phương trình vi hàm s tun hoàn, phn tun hoàn nhân tính   126

Ví d 4.5.  Cho hàm   g(x)   là hàm tun hoàn nhân tính chu kỳ  −e, (g(−ex) =g(x)); h(x) là hàm phn tun hoàn nhân tính chu kỳ 

 −e, (h(

−ex) =

−h(x)). Xác 

đnh tt c các hàm:  f   : R\{0} →  R  sao cho:

f (e2x) − 2√ 

3f (−ex) + 4f (x) =  h(x) − 3g(x).

Li gii:Xét phương trình đc trưng: λ2 − 2

√ 3λ + 4 = 0, ∆ = −1 <  0  phương trình có các 

nghim:

λ1 =√ 

3 − i; λ2 =√ 

3 + i; r = |λ1| = |λ2| = 2; q  = 1; cos ϕ =

√ 3

2  ⇒ ϕ =

 π

6.

áp dng Bài toán 2.6 (phn c) ) ta có:

f (x) =  h(x)

5 + 2√ 3 −  3g(x)

5 − 2√ 3+ 2ln |x| cos

 π ln |x|6

  n(x) + sin π ln |x|

6  m(x),

trong đó  m(x), n(x) là hai hàm tha mãn:  m(−ex) =  m(x); n(−ex) =  n(x).

Page 127: Dãy số - Giới hạn Tác giả: Trần Nam Dũng- Nguyễn Văn Mậu, 2007

8/13/2019 Dãy số - Giớ i hạn Tác giả: Trần Nam Dũng- Nguyễn Văn Mậu, 2007

http://slidepdf.com/reader/full/day-so-gioi-han-tac-gia-tran-nam-dung-nguyen-van-mau 127/217

Tài liu tham kho

[1] Nguyn Thy Thanh (1990), Lý thuyt hàm bin phc mt bin, NXB Đi hc và Trung hc Chuyên nghip.

[2] Nguyn Văn Mu (2004, 2006), Đa thc đi s và phân thc hu t, NXB Giáo dc.

[3] Nguyn Văn Mu (2003), Phương trình hàm, NXB Giáo dc.

[4] Lê Đình Thnh và các tác gi khác, Phương trình sai phân và mt s ng dng, NXB Giáo dc.

[5] Nguyn Trng Tun, Bài toán hàm s qua các kỳ thi Olimpic, NXB Giáodc.

[6] Kuczma Marek (1968), Functional equations in a single variable, PWN- Pol-ish scientific publishers.

127

Page 128: Dãy số - Giới hạn Tác giả: Trần Nam Dũng- Nguyễn Văn Mậu, 2007

8/13/2019 Dãy số - Giớ i hạn Tác giả: Trần Nam Dũng- Nguyễn Văn Mậu, 2007

http://slidepdf.com/reader/full/day-so-gioi-han-tac-gia-tran-nam-dung-nguyen-van-mau 128/217

Chương 5

Dãy s sinh bi hàm s

5.1 Hàm s chuyn đi phép tính s hc và đi sTrong mc này, ta kho sát mt s tính cht cơ bn ca mt s dng hàm s 

thông qua các h thc hàm đơn gin. Ta cũng kho sát mt s dng hàm bo toàn và chuyn đi các tính cht cơ bn ca phép tính đi s như giao hoán, phân b và kt hp.

Bài toán 1. Xác đnh các hàm s   f (x)  xác đnh và liên tc trên  R  tho mãn điu kin 

f (x + y ) = f (x) + f (y ) + f (x)f (y ),   ∀ x, y  ∈ R.   (1)

Gii.Đt  f (x) =  g(x)

−1, ta thu đưc 

g(x + y ) − 1 =  g(x) − 1 + g(y ) − 1 + [g(x) − 1][g(y ) − 1],   ∀ x, y  ∈ Rhay 

g(x + y ) = g(x)g(y ),   ∀ x, y  ∈ R.   (2)

Do   f (x)   liên tc trên  R  nên   g(x)  cũng là hàm liên tc trên  R. Suy ra (2) có nghim  g(x) = eax, a ∈ R và (1) có nghim 

f (x) =  eax − 1, a ∈ R.

Bài toán 2. Cho hàm s  F (u, v) ( u, v ∈ R). Gi s phương trình hàm:

f (x + y ) =  F [f (x), f (y )],   ∀ x, y  ∈ R   (3)

có nghim  f (x) xác đnh và liên tc trên  R. Chng minh rng  F (u, v) là hàm đi xng ( F (u, v) =  F (v, u) và có tính kt hp

F [F (u, v), w] = F [u, F (v, w)],   ∀ u,v,w ∈ f.   (4)

128

Page 129: Dãy số - Giới hạn Tác giả: Trần Nam Dũng- Nguyễn Văn Mậu, 2007

8/13/2019 Dãy số - Giớ i hạn Tác giả: Trần Nam Dũng- Nguyễn Văn Mậu, 2007

http://slidepdf.com/reader/full/day-so-gioi-han-tac-gia-tran-nam-dung-nguyen-van-mau 129/217

5.1. Hàm s chuyn đi phép tính s hc và đi s   129

Gii.Nhn xét rng tính đi xng ca  F (u, v) đưc suy trc tip t (3). Mt khác,

theo (3), ta có f (x + y  + z ) =  f [(x + y ) + z ] =  F {F [f (x), f (y )], f (z )},  ∀ x,y,z  ∈ R   (5)

và 

f (x + y  + z ) =  f [x + (y  + z )] =  f [(y  + z ) + x] =  F {F [f (y ), f (z )], f (x)}= F {f (x), F [f (y ), f (z )]},  ∀ x,y,z  ∈ R.   (6)

T (5) và (6) suy ra (4):

F [F (u, v), w] = F [u, F (v, w)],   ∀ u,v,w ∈ f.

Bài toán 3. Gi s phương trình hàm:

f (x + y ) =  F [f (x), f (y )],   ∀ x, y  ∈ Rvi hàm s  F (u, v) ( u, v ∈  R) là mt đa thc (khác hng), có nghim  f (x)  xác đnh và liên tc (khác hng) trên  R. Chng minh rng  F (u, v) có dng 

F (u, v) =  auv  + bu + bv + c.   (7)

Gii.Gi s  F (u, v)  là đa thc bc  m theo  u và bc  n   theo  v. Khi đó, do  F (u, v)

đi xng nên  m =  n. Theo (4) thì 

F [F (u, v), w] = F [u, F (v, w)],   ∀ u,v,w ∈ f 

nên v trái là mt đa thc bc  n  theo   w  còn v phi là đa thc bc  n2 theo   w.Suy ra  n2 = n  hay  n = 1. Vy  F (u, v) có dng 

F (u, v) =  auv  + b1u + b2v + c.

Do  F (u, v) là đa thc đi xng nên  b1 =  b2 và 

F (u, v) =  auv  + bu + bv + c.

Nhn xét rng, vi  F (u, v) =  auv + bu + bv + c và  F (u, v) tho mãn điu kin 

(4) thì ac =  b2 − b.

Vy vi  a = 0 thì 

ac =  b2 − b ⇔ c =  b2 − b

a  , a = 0.   (8)

Page 130: Dãy số - Giới hạn Tác giả: Trần Nam Dũng- Nguyễn Văn Mậu, 2007

8/13/2019 Dãy số - Giớ i hạn Tác giả: Trần Nam Dũng- Nguyễn Văn Mậu, 2007

http://slidepdf.com/reader/full/day-so-gioi-han-tac-gia-tran-nam-dung-nguyen-van-mau 130/217

5.1. Hàm s chuyn đi phép tính s hc và đi s   130

Bây gi, ta chuyn sang xét các dng đc bit ca (7).

Bài toán 4. Cho đa thc  F (u, v) =  bu + bv + c, b= 0. Xác đnh các hàm s  f (x)

xác đnh và liên tc trên  R tho mãn điu kin 

f (x + y ) =  F [f (x), f (y )],   ∀ x, y  ∈ Rtc là 

f (x + y ) = bf (x) + bf (y ) + c,   ∀ x, y  ∈ R.   (9)

Gii.Nhn xét rng, nu  b = 1 thì t (9) vi  y  = 0, ta có ngay  f (x) = const. Khi 

b = 1

2 và  c  = 0 thì mi hàm hng đu tho mãn (8). Khi  b  =

 1

2 và  c = 0 thì (9) vô 

nghim. Các trưng hp khác ( b = 1,  b = 1

2 thì nghim ca (9) là  f (x) =

  c

1−

2b.

Xét trưng hp  b = 1. Khi đó (9) có dng 

f (x + y ) =  f (x) + f (y ) + c,   ∀ x, y  ∈ Rvà phương trình hàm này có nghim  f (x) =  αx − c.

Bài toán 5. Cho đa thc  F (u, v) =  auv + bu + bv + b2 − b

a  , a = 0. Xác đnh các 

hàm s  f (x) xác đnh và liên tc trên  R tho mãn điu kin 

f (x + y ) =  F [f (x), f (y )],   ∀ x, y  ∈ Rtc là 

f (x + y ) =  af (x)f (y ) + bf (x) + bf (y ) +  b2

− ba

  ,   ∀ x, y  ∈ R.   (10)

Gii.Nhn xét rng, nu đt 

f (x) = h(x) − b

a

thì thì t (10) ta nhn đưc 

h(x + y ) =  h(x)h(y ),   ∀ x, y  ∈ Rvà phương trình hàm này có nghim  h(x) =  eαx. Suy ra nghim ca (10) có dng 

f (x) =  eαx

− ba

  .

Bài toán 6. Gi s  f (x) là nghim ca phương trình hàm:

f (ax + by  + c) =  Af (x) + Bf (y ) + C  (abAB  = 0),   ∀ x, y  ∈ R   (11)

Page 131: Dãy số - Giới hạn Tác giả: Trần Nam Dũng- Nguyễn Văn Mậu, 2007

8/13/2019 Dãy số - Giớ i hạn Tác giả: Trần Nam Dũng- Nguyễn Văn Mậu, 2007

http://slidepdf.com/reader/full/day-so-gioi-han-tac-gia-tran-nam-dung-nguyen-van-mau 131/217

Page 132: Dãy số - Giới hạn Tác giả: Trần Nam Dũng- Nguyễn Văn Mậu, 2007

8/13/2019 Dãy số - Giớ i hạn Tác giả: Trần Nam Dũng- Nguyễn Văn Mậu, 2007

http://slidepdf.com/reader/full/day-so-gioi-han-tac-gia-tran-nam-dung-nguyen-van-mau 132/217

5.1. Hàm s chuyn đi phép tính s hc và đi s   132

Theo Bài toán 7, thì điu kin cn đ phương trình hàm (11) có nghim là a =  A,  b =  B .

Gi s điu kin này đưc tho mãn. Theo (12), ta chia các trưng hp riêng đ kho sát.Xét các trưng hp sau:

Trưng hp  b + a = 1,   c = 0.

Khi đó, (11) có dng 

f (ax + (1 − a)y ) =  af (x) + (1 − a)f (y ) (abAB  = 0),   ∀ x, y  ∈ R.   (13)

Ta thu đưc (13) thuc lp hàm chuyn tip các đi lưng trung bình cng. Vì vy (13) có nghim  f (x) =  αx + β ,  α, β  ∈ R.

Trưng hp  b + a = 1,   c = 0.Khi đó, (11) có dng 

f (ax + (1 − a)y  + c) =  af (x) + (1 − a)f (y ) + C  (abAB  = 0),   ∀ x, y  ∈ R.   (13)

Đt  f (x) = C 

c x + h(x). Ta thu đưc (13) dưi dng 

h(ax + (1 − a)y  + c) =  ah(x) + (1 − a)h(y ),   ∀ x, y  ∈ R.   (14)

D kim tra, phương trình (14) ch có nghim hng tuỳ ý (xem (12)) và vì vy,

(13) có nghim  f (x) = C 

c x + β ,  β  ∈ R.

Trưng hp   b + a = 1. Theo Bài toán 6 thì nghim ca (13) có dng  f (x) =αx + β . Theo (12) thì  αc − C  = (a + b − 1)β.  Vy nu cho  α ∈ R giá tr tuỳ ý thì 

β  =  αc − C 

a + b − 1.

Chú ýNu không đòi hi nghim ca (11) là hàm s liên tc trên  R  thì các đng 

thc  a =  A,  b =  B  và (12) có th không tho mãn. Tuy nhiên, ta vn có các tính cht đi s sau đây.

Bài toán 9. Gi s phương trình hàm 

f (ax + y ) =  Af (x) + f (y ) (aA = 0),   ∀ x, y  ∈R

  (15)có nghim khác hng. Chng minh rng nu  a (hoc  A) là s đi s vi đa thc ti tiu  P a(t)(tương ng  P A(t)) thì  A (tương ng  a) là s đi s và 

P a(t) ≡ P A(t).   (16)

Page 133: Dãy số - Giới hạn Tác giả: Trần Nam Dũng- Nguyễn Văn Mậu, 2007

8/13/2019 Dãy số - Giớ i hạn Tác giả: Trần Nam Dũng- Nguyễn Văn Mậu, 2007

http://slidepdf.com/reader/full/day-so-gioi-han-tac-gia-tran-nam-dung-nguyen-van-mau 133/217

5.1. Hàm s chuyn đi phép tính s hc và đi s   133

Gii.Ta thy  f (0) = 0 nên  f (ax) =  af (x) và bng quy np toán hc, d dàng chng 

minh f (akx) =  Akf (x), k ∈ N.   (17)

Gi s 

P a(t) = tn +n−1i=0

riti, r0, . . . , rn−1 ∈ Q.

Khi đó, theo (17) thì 

(an +n−1i=0

ria)x

= f (anx) +n−1i=0

rif (aix)

An +

n−1i=0

riA

f (x).

Vì  f (x) khác hng nên 

An +

n−1i=0

riA  = 0 (18)

và vì vy  A  là s đi s. Suy ra  P a(t)   là ưc ca   P A(t)  và do   P A(t)  là đa thc ti tiu nên có (16).

Ngưc li, nu  A  là s đi s tho mãn (18) thì thc hin quy trình ngưc li,ta thu đưc 

an +n

−1

i=0

ria = 0 (19)

và t đó suy ra (16).

Bài toán 10. Gi s phương trình hàm 

f (ax + y ) = Af (x) + f (y ) (aA = 0, a ∈ Q),   ∀ x, y  ∈ R   (20)

có nghim khác hng. Chng minh rng khi đó  a =  A.Gii.

Tht vy, theo Bài toán 9 thì  P a(t) là đa thc bc nht và vì vy  P A(t) cũng là đa thc bc nht (vi h s bc cao nht đu bng 1) nên  a =  A.

Bài toán 11. Gii phương trình hàm sau trong lp các hàm s  f (x) liên tc trên R:

f (x + y ) =  axy f (x)f (y ) (a > 0),   ∀ x, y  ∈ R.   (21)

Gii.

Page 134: Dãy số - Giới hạn Tác giả: Trần Nam Dũng- Nguyễn Văn Mậu, 2007

8/13/2019 Dãy số - Giớ i hạn Tác giả: Trần Nam Dũng- Nguyễn Văn Mậu, 2007

http://slidepdf.com/reader/full/day-so-gioi-han-tac-gia-tran-nam-dung-nguyen-van-mau 134/217

5.1. Hàm s chuyn đi phép tính s hc và đi s   134

D thy  f (1) 0. Nu  f (1) = 0  thì t (21) ta có ngay  f (x) ≡ 0. Xét trưng hp  f (1) >  0. Bng quy np, d dàng kim chng h thc 

f (nx) = a(n2−n)x2

2   [f (x)]n,   ∀ n ∈ N∗.

Vy vi  x = 1 thì 

f (n) = an2−n

2   [f (1)]n,   ∀ n ∈ N∗.

Vi  x = m

n , ta thu đưc 

f (m) =  a(n2−n)(mn  )

2

2

f (

m

n )n

,   ∀ m, n ∈ N∗.

và 

f (m) =  a

m2−m

2   [f (1)]m

,   ∀ m ∈ N∗.Suy ra 

f m

n

 =  a

12(mn )2

a−12 f (1)

mn

.   (22)

Do  f (1) >  0  nên có th vit 

c = −1

2 + loga f (1).

T (21) suy ra f (x) =  a

12x2+cx,   ∀ x ∈ Q+.   (23)

Do  f (x) liên tc nên (16) tho mãn vi mi  x ∈ R+. Vi  x < 0, ta đt 

 −x =  y 

và do  f (0) = 1 nên t gi thit (21) ta nhn đưc 

1 =  a−x2f (x)a(x2/2)−cx,

hay f (x) =  a

12x2+cx,   ∀ x ∈ R.

Nhn xét.Bng cách đt 

f (x) = ax2/2g(x)

ta đưa (15) v dng quen bit 

g(x + y ) = g(x)g(y ),  ∀ x ∈ R.

Bài toán 12. Xác đnh các hàm s  f  xác đnh và liên tc trên  R tho mãn điu kin 

f (x + y ) + f (z ) =  f (x) + f (y  + z ),   ∀ x,y,z  ∈ R.   (1)

Page 135: Dãy số - Giới hạn Tác giả: Trần Nam Dũng- Nguyễn Văn Mậu, 2007

8/13/2019 Dãy số - Giớ i hạn Tác giả: Trần Nam Dũng- Nguyễn Văn Mậu, 2007

http://slidepdf.com/reader/full/day-so-gioi-han-tac-gia-tran-nam-dung-nguyen-van-mau 135/217

5.2. V các dãy s xác đnh bi dãy các phương trình   135

Gii.Đt  f (0) = a  thì vi  z  = 0 trong (1) ta thu đưc 

f (x + y ) + a =  f (x) + f (y ),   ∀ x, y  ∈ R.   (2)

Đt  f (x) =  g(x) + a. T (2) ta nhn đưc 

g(x + y ) = g(x) + g(y ),   ∀ x, y  ∈ R.   (3)

Phương trình (3) có nghim  g(x) =  αx,  α ∈ R.Suy ra phương trình (1) có nghim 

f (x) =  αx + β , α, β   ∈ R.

Th li, ta thy hàm  f (x) =  αx + β  tho mãn điu kin bài ra.

Bài toán 13. Xác đnh các hàm s  f  xác đnh và liên tc trên  R tho mãn điu kin 

f (x + y )f (z ) =  f (x)[f (y ) + f (z )],   ∀ x,y,z   ∈ R.   (4)

Gii.Thay  y  =  z  = 0 trong (4), ta thu đưc  f (0)f (x) = 0. Vy  f (0) = 0. Vi  z  = 0

thì f (x + y )f (0) = f (x)[f (y ) + f (0)],   ∀ x, y  ∈ R

hay f (x)f (y ) = 0,   ∀ x, y  ∈ R.

Suy ra  f (x) ≡ 0.

5.2 V các dãy s xác đnh bi dãy các phương trình

Trong toán hc, có rt nhiu trưng hp ta không xác đnh đưc giá tr c th đi tưng mà chúng ta đang xét (ví d s, hàm s) nhưng vn có th thc hin các phép toán trên các đi tưng đó. Ví d ta có th không bit giá tr các nghim ca mt phương trình, nhưng vn bit đưc tng ca chúng:

Ví d 5.1. Tìm tng các nghim ca phương trình  cos5 x−5cos3 x+3cos x−1 = 0trên đon   [0, 2].

Đôi khi ta cn tính tích phân ca mt hàm mà ta không có biu thc tưng minh:

Ví d 5.2.  Chng minh rng vi mi   t ≥ 0, phương trình  x3 + tx − 8 = 0 luôn có 1 nghim dương duy nht, ký hiu là  x(t). Tính 

  70 [x(t)]2dt.

Page 136: Dãy số - Giới hạn Tác giả: Trần Nam Dũng- Nguyễn Văn Mậu, 2007

8/13/2019 Dãy số - Giớ i hạn Tác giả: Trần Nam Dũng- Nguyễn Văn Mậu, 2007

http://slidepdf.com/reader/full/day-so-gioi-han-tac-gia-tran-nam-dung-nguyen-van-mau 136/217

5.2. V các dãy s xác đnh bi dãy các phương trình   136

Trong bài vit nh này, chúng ta s đ cp đn mt tình hung căn bn khác,đó là kho sát nhng dãy s xác đnh bi dãy các phương trình.

Bài toán 5.1.  Cho dãy các hàm s   f n(x)  xác đnh bi công thc tưng minh hoc truy hi tho mãn điu kin: các phương trình   f n(x) = 0  có nghim duy nht  xn ∈ D. Cn kho sát các tính cht ca  xn  như kho sát s hi t, tìm gii hn...

Chúng ta bt đu t mt bài toán thi tuyn sinh vào khoa Toán trưng Đi hc Đc lp Matxcơva năm 2000.

Bài toán 5.2.  Ký hiu  xn  là nghim ca phương trình 

1

x

+1

x − 1

 +

· · ·+

1

x − n

 = 0

thuc khong  (0, 1)a) Chng minh dãy  {xn} hi t;b) Hãy tìm gii hn đó.

Bình lun: Dãy  xn  đưc xác đnh duy nht vì hàm s 

f n(x) =  1

x+

  1

x − 1+ · · · +

  1

x − n

liên tc và đơn điu trên  (0, 1). Tuy nhiên, ta không th xác đnh đưc giá tr c th ca  xn. Rt may mn, đ chng minh tính hi t ca  xn, ta không cn đn điu đó. Ch cn chng minh tính đơn điu và b chn là đ. Vi tính b chn,mi th đu n vì  0  < xn < 1. Vi tính đơn điu, ta chú ý mt chút đn mi liên h gia  f n(x) và  f n+1(x) trong đó 

f n+1(x) =  f n(x) + f n+1(x) =  f n(x) +  1

x − n − 1.

Đây chính là chìa khoá đ chng minh tính đơn điu ca  xn.Li gii.   Rõ ràng  xn đưc xác đnh mt cách duy nht, 0 < xn < 1. Ta có 

f n+1(xn) =  f n(xn) + 1/(xn − n − 1) = 1/(xn − n − 1) <  0,

trong khi đó  f n+1(0+) >  0. Theo tính cht ca hàm liên tc, trên khong  (0, xn)có ít nht mt nghim ca   f n+1(x). Nghim đó chính là   xn+1. Như th ta đã chng minh đưc   xn+1  < xn, tc là dãy s  {xn}  đơn điu gim. Do dãy này b chn dưi bi 0 nên dãy s đã cho có gii hn.

Page 137: Dãy số - Giới hạn Tác giả: Trần Nam Dũng- Nguyễn Văn Mậu, 2007

8/13/2019 Dãy số - Giớ i hạn Tác giả: Trần Nam Dũng- Nguyễn Văn Mậu, 2007

http://slidepdf.com/reader/full/day-so-gioi-han-tac-gia-tran-nam-dung-nguyen-van-mau 137/217

5.2. V các dãy s xác đnh bi dãy các phương trình   137

Ta s chng minh gii hn nói trên bng 0. Đ chng minh điu này, ta cn đn kt qu quen thuc sau:

1 +1

2 +

1

3+ · · · +

1

n >  ln n

(Có th chng minh d dàng bng cách s dng đánh giá  ln

1 +1

n

 <

1

n ).

Tht vy, gi s   lim xn  =  a >  0. Khi đó, do dãy s gim nên ta có  xn ≥  avi mi  n.

Do  1 +1

2+

1

3+ · · ·+

1

n→ ∞ khi  n → ∞ nên tn ti  N  sao cho vi mi  n → N 

ta có  1 + 1

2 +

 1

3+ · · · +

 1

n >

  1

a.

Khi đó vi  n ≥ N   ta có 

0 =1

xn+

1

xn − 1 + · · · +

1

xn − n <

1

xn+

1

−1 +

1

−2 + · · · +

1

xn<

1

a+

1

a = 0,

mâu thun. Vy ta phi có  lim xn = 0.

Bài toán 5.3.  Cho n  là mt s nguyên dương  (n > 1). Chng minh rng phương trình  xn = x + 1 có mt nghim dương duy nht, ký hiu là  xn. Chng minh rng xn  dn v 1 khi  n dn đn vô cùng và tìm 

limn→∞n(xn) − 1).

Li gii.   Rõ ràng  xn  >  1. Đt  f n(x) =  xn − x − 1. Khi đó  f n+1(1) = −1 <  0  và f n+1(xn) = xn

n+1 − xn − 1 > xnn − xn − 1 =  f n(xn) = 0.

T đó ta suy ra   1  < xn+1   < xn   . Suy ra dãy  {xn}  có gii hn hu hn   a. Ta chng minh  a = 1. Tht vy, gi s  a >  1. Khi đó  xn ≥  a  vi mi  n  và ta tìm đưc  n đ ln sao cho:  xn

n ≥ an > 3  và  xn + 1 <  3, mâu thun vi  f n(xn) = 0.Đ gii phn cui ca bài toán, ta đt  xn  = 1 + y n  vi  lim y n  = 0. Thay vào

phương trình  f n(xn) = 0, ta đưc  (1 + y n)n = 2 + y n. Ly logarith hai v, ta đưc 

n ln(1 + y n) = ln(2 + y n)

T đó suy ra lim n ln(1 + y n) = ln 2

Nhưng  lim ln(1 + y n)/y n = 1 nên t đây ta suy ra   lim ny n  = ln 2, tc là 

limn→∞n(xn − 1) = ln 2.

Page 138: Dãy số - Giới hạn Tác giả: Trần Nam Dũng- Nguyễn Văn Mậu, 2007

8/13/2019 Dãy số - Giớ i hạn Tác giả: Trần Nam Dũng- Nguyễn Văn Mậu, 2007

http://slidepdf.com/reader/full/day-so-gioi-han-tac-gia-tran-nam-dung-nguyen-van-mau 138/217

5.2. V các dãy s xác đnh bi dãy các phương trình   138

Bài toán 5.4 (VMO 2007). Cho s thc  a >  2  và  f n(x) =  a10xn+10 + xn + · · · +x + 1.

a) Chng minh rng vi mi s nguyên dương  n, phương trình  f n(x) =  a luôn có đúng mt nghim dương duy nht.b) Gi nghim đó là  xn, chng minh rng dãy  {xn} có gii hn hu hn khi 

n dn đn vô cùng.

Li gii.  Kt qu ca câu a) là hin nhiên vì hàm  f n(x) tăng trên  (0, +∞). D dàng nhn thy  0  < xn  <  1. Ta s chng minh dãy  xn   tăng, tc là  xn+1  > xn.Tương t như nhng li gii trên, ta xét 

f n+1(xn) = a10xn+11n   + xn+1

n   + xnn + · · · + x + 1 = xnf n(xn) + 1 = axn + 1

Vì ta đã có  f n+1(1) = a10 + n + 1 > a nên ta ch cn chng minh  axn + 1 < a là 

s suy ra  xn  < xn+1  <  1. Như vy, cn chng minh  xn  <

a

−1

a   . Tht vy, nu 

xn ≥ a − 1

a  thì 

f n(xn) ≥ a10a − 1

a

n+10+

1 −a − 1

a

n+1

1 − a − 1

a

=

= (a − 1)10a − 1

a

n+a − (a − 1)

a − 1

a

n> a

(do  a−

1 >  1). Vy dãy s tăng  {

xn

} tăng và b chn bi 1 nên hi t.

Nhn xét 5.1.  Mt ln na mi liên h  f n+1(x) =  xf n(x) + 1 li giúp chúng ta tìm đưc mi quan h gia   xn   và   xn+1. T li gii trên, ta có th chng minh 

đưc rng  lim xn =a − 1

a  . Tht vy, đt  c =

a − 1

a  < 1, theo tính toán trên thì 

f n(c) − f n(xn) = kcn (vi   k = (a − 1)((a − 1)9 − 1) >  0)

Theo đnh lý Lagrange thì 

f n(c) − f n(xn) =  f (ξ )(c − xn)   vi   ξ   thuc   (xn, c)

Nhưng  f (ξ ) = (n + 10)a10ξ n+9 + nξ n−1 + · · ·  + 1 >  1  nên t đây suy ra 

kcn > c − xn

T đó ta có c − kcn < xn < c

Và có nghĩa làm   lim xn = c.

Page 139: Dãy số - Giới hạn Tác giả: Trần Nam Dũng- Nguyễn Văn Mậu, 2007

8/13/2019 Dãy số - Giớ i hạn Tác giả: Trần Nam Dũng- Nguyễn Văn Mậu, 2007

http://slidepdf.com/reader/full/day-so-gioi-han-tac-gia-tran-nam-dung-nguyen-van-mau 139/217

5.2. V các dãy s xác đnh bi dãy các phương trình   139

Bài toán 4. (VMO 2002) Cho  n là mt s nguyên dương. Chng minh rng phương trình 

1x − 1

 +   14x − 1

 + · · · +   1n2x − 1

 =  12

có mt nghim duy nht   xn   >  1. Chng minh rng khi   n  dn đn vô cùng,  xn

dn đn 4.Bình lun: Vic chng minh phương trình có nghim duy nht   xn   >   1   là 

hin nhiên. Mi liên h   f n+1(x) =   f n(x) +1

((n + 1)2x − 1) cho thy   xn   là dãy 

s tăng ( đây   f n(x) =  1

x − 1 +

  1

4x − 1 + · · · +

  1

n2x − 1−   1

2). Đ bài cho sn 

gii hn ca   xn   là 4 đã làm cho bài toán tr nên d hơn nhiu. Tương t như cách chng minh   lim xn   =   c   nhn xét trên, ta s dùng đnh lý Lagrange đ 

đánh giá khong cách gia   xn   và 4. Đ làm điu này, ta cn tính   f n(4), vi 

f n(x) =  1

x − 1 +

  1

4x − 1 + · · · +

  1

n2x − 1−  1

2. Rt may mn, bài tính  f n(4) này 

liên quan đn 1 dng tng quen thuc.Li gii: Đt  f n(x) như trên và gi  xn  là nghim > 1 duy nht ca phưng trình f n(x) = 0. Ta có 

f n(4) =1

4 − 1 +

1

16 − 1 + · · · +

1

4n2 − 1− 1

2 =

1

1.3+

1

3.5+ · · · +

1

(2n − 1)(2n + 1)− 1

2

=1

21

1− 1

3 +

1

3− 1

5 + · · · +

1

2n

−1

− 1

2n−1

2 = − 1

4n

Áp dng đnh lý Lagrange, ta có 

1

4n = |f n(xn) − f (4)| = |f (c)||xn − 4|

vi  c thuc  (xn, 4)Nhưng do

|f (c)| =1

(c − 1)2 +

1

(4c − 1)2 + · · · >

1

9

Nên t đây  |xn − 4| <

9

4n, suy ra   lim xn  = 4.Trong ví d trên (và trong phn nhn xét bài toán 3) chúng ta đã s dng đnh lý Lagrange đ đánh giá hiu s gia   xn  và giá tr gii hn. ví d cui cùng ca bài vit này, ta tip tc nu ra ng dng dng đnh lý này trong mt tình hung phc tp hơn.

Page 140: Dãy số - Giới hạn Tác giả: Trần Nam Dũng- Nguyễn Văn Mậu, 2007

8/13/2019 Dãy số - Giớ i hạn Tác giả: Trần Nam Dũng- Nguyễn Văn Mậu, 2007

http://slidepdf.com/reader/full/day-so-gioi-han-tac-gia-tran-nam-dung-nguyen-van-mau 140/217

5.2. V các dãy s xác đnh bi dãy các phương trình   140

Bài toán 5. Cho  n   là mt s nguyên dương > 1. Chng minh rng phương trình  xn = x2 + x + 1  có mt nghim dương duy nht, ký hiu là  xn. Hãy tìm s 

thc  a sao cho gii hn  limn→∞ na(xn − xn+1) tn ti, hu hn và khác 0.Bình lun. D thy giá tr  a, nu tn ti, là duy nht. Tương t như bài 

toán 2, có th chng minh đưc rng  xn ≈ 1 +ln(3)

n  . T đó có d đoán là  a = 2.

Đnh lý Lagrange s giúp chúng ta đánh giá hiu   xn − xn+1  và chng minh d đoán này.Li gii.   Đt 

P n(x) =  xn − x2 − x − 1.

Ta có 

P n+1(x) =  xn+1 − x2 − x − 1 =  xn+1 − xn + P n(x) =  xn(x − 1) + P n(x).

T đó P n+1(xn) =  xn

n(xn − 1) + P n(xn) = (x2n + xn + 1)(xn − 1) =  x3

n − 1.

Áp dng đnh lý Lagrange, ta có 

(x2n + xn + 1)(xn − 1) = P n+1(xn) − P n+1(xn+1) = (xn − xn+1)P 

n+1(c)

vi  c thuc  (xn+1, xn), P 

n+1(x) = (n + 1)xn − 2x − 1.T đó 

(n + 1)

xn+1 + 1 +

1

xn+1− 2xn+1 − 1 = P n+1(xn+1) < P n+1(c) <

< P n+1(xn) = (n + 1)(x2n + xn + 1) − 2xn − 1.

T đây, vi lưu ý   lim xn = 1, ta suy ra 

limn→∞

P n+1(c)

n  = 3

Tip tc s dng  lim n(xn − 1) = 3, ta suy ra 

limn→∞nP n+1(c)(xn − xn+1) = lim

n→∞n(x2n + xn + 1)(xn − 1) = 3 ln(3)

⇔   limn→∞n2(xn − xn+1)

P n+1(c)

n  = 3 ln(3)

⇔   limn→∞n2(xn − xn+1) lim

n→∞P n+1(c)

n  = 3 ln(3)

⇔   limn→∞n2(xn − xn+1)3 = 3 ln(3)

⇔   limn→∞n2(xn − xn+1) = ln(3)

Page 141: Dãy số - Giới hạn Tác giả: Trần Nam Dũng- Nguyễn Văn Mậu, 2007

8/13/2019 Dãy số - Giớ i hạn Tác giả: Trần Nam Dũng- Nguyễn Văn Mậu, 2007

http://slidepdf.com/reader/full/day-so-gioi-han-tac-gia-tran-nam-dung-nguyen-van-mau 141/217

5.3. Đnh lý v ba mnh đ tương đương    141

Vy vi   c  = 2   thì gii hn đã cho tn ti, hu hn và khác 0. D thy vi c > 2  thì gii hn đã cho bng vô cùng và vi  c < 2  thì gii hn đã cho bng 0.

Vy  c = 2 là đáp s duy nht ca bài toán.Qua các ví d trên, chúng ta thy công c cơ bn đ kho sát các dãy s chobi dãy các phương trình là các đnh lý cơ bn ca gii tích (v hàm liên tc, hàm đơn điu, đnh lý v s hi t ca dãy s đơn điu và b chn, đnh lý Lagrange)và mi liên h mang tính truy hi gia các phương trình. Hy vng rng vic phân tích các tình hung 5 ví d trên đây s giúp chúng ta có mt cách nhìn tng quát cho các bài toán dng này.

5.3 Đnh lý v ba mnh đ tương đương

Đnh lý 5.1 (V ba mnh đ tương đương).  . Cho dãy s 

 {ck

} vi  0 < ck  <  1,

k = 1, 2, 3, . . .. Xét các dãy s 

X n  =n

i=1

(1 + ci);   Y n  =n

i=1

(1 − ci).

Khi đó ba khng đnh sau là tương đương (i)   lim

n→+∞ X n = +∞,

(ii)   limn→+∞ Y n  = 0,

(iii)   limn→+∞

  ni=1

ci

 = +∞.

Chng minh.Xét khng đnh (i)⇒ (iii).

Gi s   ni=1

ci < M,  vi  0 < M < +∞. Khi đó 

ni=1

(1 + ci) <

1 + 1

n

ni=1

cin

<

1 + M 

n

n< eM ,

vô lý vì rng    limn→+∞ X n = +∞. Do đó    lim

n→+∞

  ni=1

ci

 = +∞.

Xét khng đnh (iii)⇒ (i). Điu này là hin nhiên đúng vì rng 

ni=1

(1 + ci) >

ni=1

ci.

Page 142: Dãy số - Giới hạn Tác giả: Trần Nam Dũng- Nguyễn Văn Mậu, 2007

8/13/2019 Dãy số - Giớ i hạn Tác giả: Trần Nam Dũng- Nguyễn Văn Mậu, 2007

http://slidepdf.com/reader/full/day-so-gioi-han-tac-gia-tran-nam-dung-nguyen-van-mau 142/217

5.4. Mt s bài toán v ưc lưng tng và tích   142

Xét khng đnh (ii)⇒ (iii). Nhn xét rng, ng vi b  n s bt kỳ  a1, a2, . . . , an

vi  0 < ai <  1, thì n

i=1

ai >  1 −n

i=1

(1 − ai).

D dàng kim tra tính đúng đn ca bt đng thc này bng qui np.

Do   limn→+∞

ni=1

(1−ci) = 0 nên ng vi mi  m luôn tn ti  n sao cho  ni=1

(1−ci) <

1

2. T đó ta có 

ni=1

ci  >  1 −n

i=1

(1 − ci) > 1

2.

Suy ra 

+∞i=1 ci  = +∞.

Xét khng đnh (i)⇒ (ii). Ta có 

1 >

ni=1

(1 − c2i ) =n

i=1

(1 + ci)n

i=1

(1 − ci).

Nhưng vì    limn→+∞

ni=1

(1 + ci) = +∞ nên    limn→+∞

ni=1

(1 − ci) = 0 (theo nguyên lý kp).

Do đó    limn→+∞ Y n = 0.

Bây gi ta chuyn sang phn áp dng đnh lý trên đ gii quyt mt s bài toán.

5.4 Mt s bài toán v ưc lưng tng và tích

Bài toán 5.5.  Cho dãy s thc tăng  {un} có tính cht 

limn→+∞ un  = +∞.

Chng minh rng luôn tn ti  k ∈ N sao cho

u1u2

+ u2u3

+ · · · +   ukuk+1

< k − 2007

(ta gi s  u1 >  0).

Page 143: Dãy số - Giới hạn Tác giả: Trần Nam Dũng- Nguyễn Văn Mậu, 2007

8/13/2019 Dãy số - Giớ i hạn Tác giả: Trần Nam Dũng- Nguyễn Văn Mậu, 2007

http://slidepdf.com/reader/full/day-so-gioi-han-tac-gia-tran-nam-dung-nguyen-van-mau 143/217

5.4. Mt s bài toán v ưc lưng tng và tích   143

Gii.Ta s dng bin đi tương đương sau 

k −u1

u2+

 u2

u3+ · · · +

  uk

uk+1

 >  2007 ⇔

ki=1

1 −   ui

ui+1

 >  2007.

Do {un} là dãy tăng nên  0  <  1 −   ui

ui+1< 1.

Đt  ci = 1 −   ui

ui+1. Suy ra  0 < ci < 1. Mt khác, ta có 

ni=1

(1 − ci) =

ni=1

ui

ui+1=

  u1

un+1

tin dn ti  0 khi  n → +∞. Vy nên   ni=1

ci  = +∞ (T 2)⇒ 3). Do đó  ∃k ∈ N đ 

ki=1

1 −   ui

ui+1

 =

ki=1

ci >  2007.

Bài toán 5.6. Cho dãy s {an} dương có tính cht    limn→+∞ an  = +∞. Chng minh 

rng luôn tn ti  k ∈ N sao cho

k

i=1

ai

a1 + a2 + · · · + ai

> 2632007.

Gii.Đt  ci =

  ai

a1 + a2 + · · · + aiVì  ai >  0  nên  0 < ci <  1  và 

1 − ci = a1 + a2 + · · · + ai−1

a1 + a2 + · · · + ai,   vi  i ≥ 2.

Suy ra n

i=2

(1 − ci) =  a1

a1 + a2 + · · · + an

tin dn ti  0 khi  n → +∞.

Vì  ai > 0 và    limn→+∞ an = +∞, nên 

  ni=1

ci = +∞ hay ∃k ∈ N đ   ni=1

ci >  2632007,

điu phi chng minh.

Page 144: Dãy số - Giới hạn Tác giả: Trần Nam Dũng- Nguyễn Văn Mậu, 2007

8/13/2019 Dãy số - Giớ i hạn Tác giả: Trần Nam Dũng- Nguyễn Văn Mậu, 2007

http://slidepdf.com/reader/full/day-so-gioi-han-tac-gia-tran-nam-dung-nguyen-van-mau 144/217

5.5. Bài tp    144

Bài toán 5.7.  Xét dãy tt c các s nguyên t  { pn},  2 =  p1  < p2  < p3  < · · · .Chng minh rng 

+∞n=1

1 pn

= +∞.

Gii.

Bài toán 5.8.  Xét dãy s  {an} xác đnh bi công thc 

a1  = 1

2, an+1  =

 2n − 1

2n + 2an, n = 1, 2, . . . .

Chng minh rng +∞

n=1 an = 1.

Gii.

Bài toán 5.9.  Cho dãy s  {an} dương, tăng và không b chn. Chng minh rng 

+∞n=1

arccos

  an

an+1

2= +∞.

Gii.

5.5 Bài tpBài 5.1.  Cho dãy s  {an} dương, tăng và không b chn. Chng minh rng 

+∞n=1

arccos   an

an+1

 = +∞.

Page 145: Dãy số - Giới hạn Tác giả: Trần Nam Dũng- Nguyễn Văn Mậu, 2007

8/13/2019 Dãy số - Giớ i hạn Tác giả: Trần Nam Dũng- Nguyễn Văn Mậu, 2007

http://slidepdf.com/reader/full/day-so-gioi-han-tac-gia-tran-nam-dung-nguyen-van-mau 145/217

Page 146: Dãy số - Giới hạn Tác giả: Trần Nam Dũng- Nguyễn Văn Mậu, 2007

8/13/2019 Dãy số - Giớ i hạn Tác giả: Trần Nam Dũng- Nguyễn Văn Mậu, 2007

http://slidepdf.com/reader/full/day-so-gioi-han-tac-gia-tran-nam-dung-nguyen-van-mau 146/217

6.2. Dãy s tun hoàn   146

Đnh nghĩa 6.2.  (i) Dãy s  {un} ( un = 0 vi mi  n ∈ N) tho mãn điu kin 

u1

u0 = u2

u1 = · · · = un+1

un = · · ·đưc gi là mt  cp s nhân.

(ii) Khi dãy s  {un} lp thành mt cp s nhân thì thương   q  =  u1

u0đưc gi 

là  công bi ca cp s đã cho.

Nhn xét rng khi cho mt dãy hu hn s khác 0 :  {u0, u1, . . . , us} tho mãn điu kin 

u1

u0=

 u2

u1= · · · =

  us

us−1

thì ta cũng nói rng dãy hu hn đã cho lp thành mt cp s nhân vi công bi 

 p = u

1u0 .

Ta luôn có mi liên h gia cp s cng và cp s nhân sau đây.

Bài toán 6.1.  (i) Nu dãy s  {un} là mt cp s cng thì dãy s  {vn} vi 

vn = aun ,   ∀ n ∈ N, a >  0

s lp thành mt cp s nhân.(ii) Ngưc li, nu dãy s  {un} là mt cp s nhân vi các s hng dương thì 

dãy s  {vn} vi vn = loga un,   ∀ n, ∈ N,   0 < a = 1

s lp thành mt cp s cng.Gii Chng minh đưc suy ra trc tip t Đnh nghĩa 6.1 và 6.2   

Đnh nghĩa 6.3.  Dãy s  {un} ( un = 0 vi mi  n ∈ N) tho mãn điu kin 

un =  2un−1un+1

un−1 + un+1,   ∀ n ∈ N∗

đưc gi là  cp s điu hoà.

6.2 Dãy s tun hoàn

Tương t như đi vi hàm s thông thưng, ta có th coi dãy s  {xn}  như mt hàm   f (n) =  xn  xác đnh trên tp   N và nhn giá tr trong  R. Ta ch quan tâm đn hai loi dãy tun hoàn cơ bn là tun hoàn cng tính và tun hoàn nhân tính.

Page 147: Dãy số - Giới hạn Tác giả: Trần Nam Dũng- Nguyễn Văn Mậu, 2007

8/13/2019 Dãy số - Giớ i hạn Tác giả: Trần Nam Dũng- Nguyễn Văn Mậu, 2007

http://slidepdf.com/reader/full/day-so-gioi-han-tac-gia-tran-nam-dung-nguyen-van-mau 147/217

6.2. Dãy s tun hoàn   147

Đnh nghĩa 6.4.  Dãy s  {un} đưc gi là mt  dãy tun hoàn (cng tính)  nu tn ti s nguyên dương  l sao cho

un+l  = un,   ∀ n ∈ N.   (1)

S nguyên dương   l  nh nht đ dãy  {un} tho mãn  (1) đưc gi là  chu kỳ cơ s ca dãy.

Trong thc hành, đ chng minh mt dãy đã cho là tun hoàn, không nht thit phi xác đnh chu kỳ cơ s ca nó.

Nhn xét 6.1.  Dãy tun hoàn chu kỳ 1 khi và ch khi dãy đó là mt dãy hng.Tương t, ta cũng có đnh nghĩa v dãy tun hoàn nhân tính.

Đnh nghĩa 6.5.  Dãy s  {un} đưc gi là mt  dãy tun hoàn nhân tính nu tn 

ti s nguyên dương  s ( s > 1) sao chousn  = un,   ∀ n ∈ N.   (2)

S nguyên dương  s nh nht đ dãy  {un} tho mãn  (2) đưc gi là  chu kỳ cơ s ca dãy.

Bài toán 6.2.  Chng minh rng dãy  {un}  tun hoàn (cng tính) chu kỳ 2 khi và ch khi dãy có dng 

un = 1

2[α + β  + (α − β )(−1)n+1], α , β   ∈ R.   (3)

Gii Gi s  u0  = α,  u1  =  β  và  un+2  =  un, ∀ n ∈  N. Khi đó ta thy ngay (bng quy np toán hc) dãy  {un} có dng (3). Ngưc li, mi dãy xác đnh theo (3) là mt dãy tun hoàn chu kỳ 2.  

Bài toán 6.3.  Chng minh rng dãy  {un} tun hoàn nhân tính chu kỳ 2 khi và ch khi dãy có dng 

un =

tuỳ ý vi   nl ,u2k+1   vi  n = 2m(2k + 1), m ∈ N∗, k ∈ N.

Gii Chng minh đưc suy trc tip t h thc truy hi.

Bài toán 6.4.  Chng minh rng dãy  {un} tun hoàn chu kỳ 3 khi và ch khi dãy có dng 

un  = 1

3[α +β +γ +(−α−β +2γ )] cos

 2nπ

3  +

√ 3

2  (α−β )sin

 2nπ

3  , α , β , γ   ∈ R.   (4)

Page 148: Dãy số - Giới hạn Tác giả: Trần Nam Dũng- Nguyễn Văn Mậu, 2007

8/13/2019 Dãy số - Giớ i hạn Tác giả: Trần Nam Dũng- Nguyễn Văn Mậu, 2007

http://slidepdf.com/reader/full/day-so-gioi-han-tac-gia-tran-nam-dung-nguyen-van-mau 148/217

6.2. Dãy s tun hoàn   148

Gii. Gi s   u0  =  α,  u1   =  β ,   u2   =  γ  và  un+3   =  un, ∀ n ∈  N. Khi đó, ta thy ngay (bng quy np toán hc) dãy 

 {un

} có dng  (4).

Ngưc li, mi dãy xác đnh theo (4) là mt dãy tun hoàn chu kỳ 3 :

α , β , γ , α , β , γ , . . . .

Bài toán 6.5.  Cho  k ∈ Q \Z. Chng minh rng dãy s  {un} xác đnh theo công thc 

u0 = 1, u1 = −1, un+1 = kun − un−1, n ∈ N∗

không là mt dãy tun hoàn.

Gii. Khi  |k| > 2  thì 

|un+1

|

|k

||un

| − |un

−1

|> 2

|un

| − |un

−1

|.

Nu luôn luôn xy ra  |un| < |un−1| vi mi  n ∈ N∗ thì ta có ngay điu phi chng minh. Nu xy ra  |um| |um−1| > 0  thì suy ra 

|um| < |um+1| < · · ·

và do đó dãy  {un} không là mt dãy s tun hoàn.Xét  |k| 2 vi  k =

  p

q ,  ( p, q ) = 1,  2 q  ∈ Z∗, p ∈ Z. Bng quy np theo  n ta 

thu đưc u j  =

  p jq  j−1

, p j ∈ Z,   ( p j , q ) = 1,   ∀  j ∈ {1, . . . , n}.

T đó suy ra un+1  =

 p

q un − un−1  =

 pn+1

q n  ,

trong đó  pn+1  =  ppn − q 2 pn−1 ∈ Z

và  ( pn+1, q ) = 1. Do  q   2 nên  un = um  khi  n = m  và dãy  {un} không là dãy s tun hoàn.  

Bài toán 6.6.  Xác đnh các giá tr ca  k ∈ Q đ dãy s  {un} xác đnh theo công thc 

u0 = 1, u1 = −1, un+1 = kun − un−1, n ∈ N∗là mt dãy s tun hoàn.

Page 149: Dãy số - Giới hạn Tác giả: Trần Nam Dũng- Nguyễn Văn Mậu, 2007

8/13/2019 Dãy số - Giớ i hạn Tác giả: Trần Nam Dũng- Nguyễn Văn Mậu, 2007

http://slidepdf.com/reader/full/day-so-gioi-han-tac-gia-tran-nam-dung-nguyen-van-mau 149/217

6.2. Dãy s tun hoàn   149

Gii.Theo kt qu ca Bài toán 4, khi 

 |k

| >  2  và 

 |k

|   2,  k  =

  p

  vi   ( p, q ) = 1,

2 q  ∈ Z∗  thì dãy  {un} không là dãy s tun hoàn.Xét  |k| 2 và  k ∈ Z.Vi  k  = 2 thì  {un} là mt cp s cng vi công sai bng  −2 nên hin nhiên 

dãy  {un} không là dãy tun hoàn.Vi  k = 1 thì  {un} là dãy tun hoàn chu kỳ 6 :

u2  = −2, u3 = −1, u4 = 1, u5  = 2, u6  = 1, u7 = −1, . . . .

Vi  k = 0 thì  {un} là dãy tun hoàn chu kỳ 4 :

u0  = 1, u1  = −1, u2  = −1, u3  = 1, u4  = 1, u5 = −1, . . . .

Vi  k = −1 thì  {un} là dãy tun hoàn chu kỳ 3 :

u0  = 1, u1 = −1, u2 = 0, u3 = 1, u4  = −1, . . . .

Vi  k = −2 thì  {un} là dãy tun hoàn chu kỳ 2 :

u0  = 1, u1 = −1, u2 = 1, u3 = −1, u4 = 1, . . . .

Đnh nghĩa 6.6.  a) Dãy s  {un}  đưc gi là mt   dãy phn tun hoàn (cng 

tính) nu tn ti s nguyên dương  l sao choun+l  = −un,   ∀ n ∈ N.   (5)

S nguyên dương   l  nh nht đ dãy  {un} tho mãn  (5) đưc gi là  chu kỳ cơ s ca dãy.

b) Dãy s  {vn} đưc gi là mt  dãy phn tun hoàn nhân tính nu tn ti s nguyên dương  s ( s > 1) sao cho

vsn  = −vn,   ∀ n ∈ N.   (6)

S nguyên dương  s ( s >  1) nh nht đ dãy  {vn} tho mãn   (6) đưc gi là  chukỳ cơ s  ca dãy.

Nhn xét 6.2.  a) Dãy phn tun hoàn vi chu kỳ   l   là mt dãy tun hoàn chu kỳ 2 l.

b) Dãy phn tun hoàn nhân tính chu kỳ  s là mt dãy tun hoàn nhân tính chu kỳ 2 s.

Page 150: Dãy số - Giới hạn Tác giả: Trần Nam Dũng- Nguyễn Văn Mậu, 2007

8/13/2019 Dãy số - Giớ i hạn Tác giả: Trần Nam Dũng- Nguyễn Văn Mậu, 2007

http://slidepdf.com/reader/full/day-so-gioi-han-tac-gia-tran-nam-dung-nguyen-van-mau 150/217

6.2. Dãy s tun hoàn   150

Bài toán 6.7.  Chng minh rng mi dãy  {un} phn tun hoàn chu kỳ  r đu có dng 

un  =  12

(vn − vn+r)   vi  vn+2r  = vn.   (7)

Gii. Gi s   un+r   = −un, ∀ n ∈  N. Khi đó, ta thy ngay rng dãy  {un}   tun hoàn chu kỳ  2r và 

un  = 1

2(un − un+r),

tc là có dng (7).Ngưc li, kim tra trc tip, ta thy mi dãy xác đnh theo (7) đu là dãy 

phn tun hoàn chu kỳ  r.  

Bài toán 6.8.  Cho  f (x)   là mt đa thc vi  deg f   =  k ≥  1, f (x) ∈  Z  ng vi mi   x ∈  Z. Ký hiu   r(k) = min{2s| s ∈  N∗,   2s > k}. Chng minh rng dãy s {(−1)f (k)}   (k = 1, 2, . . .) là dãy tun hoàn vi chu kỳ  r(k).

Gii. Ta có  k!f (x) ∈ Z[x]. Biu din  f (x) dưi dng 

f (x) =  a0 + a1

x

1

+ · · · + ak

x

k

,

trong đó  x

k

 =

 x(x − 1) · · ·(x − k + 1)

k!  .

Ta cn chng minh  f (x + r(k)) − f (x) chia ht cho  2 vi mi  x ∈ Z.Nhn xét rng 

M i =

x + 2s

i

x

i

chia ht cho  2 vi mi   i ∈ N∗,   2s ≥ i, x ∈ Z. Tht vy, ta có 

M i =  1

i!

(2s + x)(2s + x − 1) . . .(2s + x − i + 1) − x(x − 1) . . .(x − i + 1)

.

T s hin nhiên chia ht cho  2s. Mt khác, s mũ ca  2 trong khai trin ca   i!là  ∞

 j=1   i

2 j  <∞

 j=1

i

2 j  = i

≤2s,

nên  M i chia ht cho  2 vi mi   i ∈ N∗, i ≤ 2s, x ∈ Z. T đó suy ra 

T i =

x + r(k)

i

x

i

Page 151: Dãy số - Giới hạn Tác giả: Trần Nam Dũng- Nguyễn Văn Mậu, 2007

8/13/2019 Dãy số - Giớ i hạn Tác giả: Trần Nam Dũng- Nguyễn Văn Mậu, 2007

http://slidepdf.com/reader/full/day-so-gioi-han-tac-gia-tran-nam-dung-nguyen-van-mau 151/217

6.2. Dãy s tun hoàn   151

chia ht cho  2 vi mi   i ∈ Z, i ≤ k,   ∀ x ∈ Z. Do  a j ∈ Z nên 

f (x + r(k)) − f (x) =k

 j=0

a jT  j

chia ht cho 2, điu phi chng minh.  

Bài toán 6.9.  Xác đnh dãy s  {un} tho mãn điu kin 

u2n+1 = 3un,   ∀ n ∈ N.   (8)

Gii. Đt  n + 1 = m,  m = 1, 2, . . .. Khi đó có th vit (8) dưi dng 

u2m−1  = 3um−1,   ∀ m ∈ N∗

hay v2m  = 3vm,   ∀ m ∈ N∗   (9)

vi vm  = um−1, ∀ m ∈ N∗.   (10)

T (9) ta có  v0 = 0. Đt  vm  = mlog2 3y m, m ∈ N∗. Khi đó (9) có dng 

y 2m  =  y m, m ∈ N∗.

Vy  {

y m}

 là mt dãy tun hoàn nhân tính chu kỳ 2. Khi đó theo Bài toán 2 ta có 

y n  =

tuỳ ý vi   nl ,y 2k+1   vi  ncó dng  2m(2k + 1), m ∈ N∗, k ∈ N.

T đó suy ra um  =  vm+1  =  mlog2 3y m+1,

vi 

y n  =

tuỳ ý vi   nl ,y 2k+1   vi  ncó dng  2m(2k + 1), m ∈ N∗, k ∈ N.

Bài toán 6.10.  Xác đnh dãy  {un} tho mãn điu kin 

u2n+1  = −3un + 4,   ∀ n ∈ N.   (11)

Page 152: Dãy số - Giới hạn Tác giả: Trần Nam Dũng- Nguyễn Văn Mậu, 2007

8/13/2019 Dãy số - Giớ i hạn Tác giả: Trần Nam Dũng- Nguyễn Văn Mậu, 2007

http://slidepdf.com/reader/full/day-so-gioi-han-tac-gia-tran-nam-dung-nguyen-van-mau 152/217

6.3. Hàm s chuyn đi cp s cng    152

Gii. Đt  n + 1 = m,  m = 1, 2, . . .. Khi đó có th vit (11) dưi dng 

u2m−1 = −3um−1 + 4,   ∀ m ∈ N∗hay 

v2m = −3vm + 4,   ∀ m ∈ N∗   (12)

vi  vm  = um−1.Đt  vm  = 1 + xm. Khi đó (12) có dng 

x2m = −3xm,   ∀ m ∈ N∗.   (13)

Đt  xm  = mlog2 3y m, m ∈ N∗. Khi đó (13) có dng 

y 2m  = −y m, m ∈ N∗.

Vy  {y m} là mt dãy phn tun hoàn nhân tính chu kỳ 2.Khi đó, theo Bài toán 2, ta có 

y n =

tuỳ ý vi   nl ,−y 2k+1   vi  ncó dng  22m+1(2k + 1), m , k ∈ N,

y 2k+1   vi  ncó dng  22m(2k + 1), m ∈ N∗, k ∈ N.

T đó suy ra um  =  vm+1  = 1 + (m + 1)log2 3y m+1,

vi 

y n = tuỳ ý vi   nl ,

−y 2k+1   vi  ncó dng  22m+1(2k + 1), m , k ∈ N,y 2k+1   vi  ncó dng  22m(2k + 1), m ∈ N∗, k ∈ N.

6.3 Hàm s chuyn đi cp s cng

Bài toán 6.11.  Nu dãy s  {un} là mt cp s cng thì dãy s  {vn}vi  vn  =  aun + b,   ∀ n ∈ N s lp thành mt cp s cng.

Gii. Gi s  {un} là cp s cng vi công sai bng d.Xét dãy s  {vn} vi  vn  = aun + b,   ∀ n ∈ N.Ta có  v0 =  au0 + b, v1 =  au1 + b . . . vn  =  aun + b, vn+1 =  a(n + 1) + b.

Khiđó  v1 − v0  =  v2 − v1  =  v3 − v2 · · · = vn+1 − vn =  ad

Vy dãy  {vn} là cp s nhân vi công sai bng  ad  

Vn đ đt ra là ta đi tìm tt c các hàm s có tính cht chuyn cp s cng thành cp s cng. Xét b đ sau.

Page 153: Dãy số - Giới hạn Tác giả: Trần Nam Dũng- Nguyễn Văn Mậu, 2007

8/13/2019 Dãy số - Giớ i hạn Tác giả: Trần Nam Dũng- Nguyễn Văn Mậu, 2007

http://slidepdf.com/reader/full/day-so-gioi-han-tac-gia-tran-nam-dung-nguyen-van-mau 153/217

6.3. Hàm s chuyn đi cp s cng    153

B đ 6.1.  Cho cp s cng  {an} và hàm s  f   : R → R+ tho mãn điu kin 

f x + y 2

 =  f (x) + f (y )2

  ,   ∀ x, y > 0.

Khi đó dãy  {f (an)} là mt cp s cng.

Chng minh.   T gi thit, ta có các h thc 

a1 − a0 = · · · = an − an−1  = an+1 − an =  . . .

Suy ra 2an = an−1 + an+1,   ∀ n ∈ N∗

Khi đó 

f (an) =  f (an−1

 + an+1

2   ) = f (a

n−1) + f (a

n+1)

2   .

T đó ta có  {f (an)} là mt cp s cng.  

Bài toán 6.12.  Tìm hàm s   f (x)  xác đnh và liên tc trên  R   tho mãn điu kin:

f x + y 

2

 =

 f (x) + f (y )

2  ,   ∀ x, y  ∈ R

Gii.   Đt  f (x) − f (0) = g(x), ta có  g(x) liên tc trên  R, vi  g(0) = 0 và 

gx + y 

2 =

 g(x) + g(y )

2   ,   ∀ x, y  ∈ RLn lưt cho  y  = 0 và  x = 0, thì 

g(x

2) =

 g(x)

2

và 

g(y 

2) =

 g(y )

2  ,   ∀ x, y  ∈ R.

Do đó g

x + y 

2  =  gx

2+ gy 

2,

  ∀ x, y 

 ∈R

Vy g(x + y ) =  g(x) + g(y ),   ∀ x, y  ∈ R

Vì g(x) liên tc trên R,nên phương trình trên là phương trình Cauchy và do đó g(x) =  ax.Suy ra  f (x) = ax + b, (a, b ∈ R).

Page 154: Dãy số - Giới hạn Tác giả: Trần Nam Dũng- Nguyễn Văn Mậu, 2007

8/13/2019 Dãy số - Giớ i hạn Tác giả: Trần Nam Dũng- Nguyễn Văn Mậu, 2007

http://slidepdf.com/reader/full/day-so-gioi-han-tac-gia-tran-nam-dung-nguyen-van-mau 154/217

6.4. Hàm s chuyn đi cp s cng vào cp s nhân   154

6.4 Hàm s chuyn đi cp s cng vào cp s nhân

Bài toán 6.13.  Nu dãy s  {un} là mt cp s cng thì dãy s  {vn}vi  vn  =  aun ,   ∀ n ∈ N, a >  0  s lp thành mt cp s nhân.

Gii.   Gi s  {un} là cp s cng vi công sai bng d.Xét dãy s  {vn} vi  vn  = aun,   ∀ n ∈ N, a >  0.Ta có  v0 =  au0 , v1 =  au1 . . . vn =  an, vn+1 =  an+1.Khiđó 

v1v0

= v2v1

= v3v2

· · · = vn+1

vn= ad

Vy dãy {vn} là cp s nhân vi công bi bng  ad

B đ 6.2.  Cho cp s cng  {an} và hàm s  f   : R → R+ tho mãn điu kin 

f x + y 

2

 = 

f (x)f (y ),   ∀ x, y > 0.

Khi đó dãy  {f (an)} là mt cp s nhân.

Chng minh.   T gi thit, ta có các h thc 

a1 − a0 = · · · = an − an−1  = an+1 − an =  . . .

Suy ra 2an = an−1 + an+1,   ∀ n ∈ N∗

Khi đó 

f (an) =  f ( an−1 + an+12   ) =  f (an−1)f (an+1).

T đó ta có  {f (an)} là mt cp s nhân.  

Như vy ta có hai hàm s trên chuyn cp s cng thành cp s nhân, vn đ đt ra là ta đi tìm tt c các hàm s có tính cht chuyn mt cp s cng bt kỳ thành mt cp s nhân.Trưc ht ta xét bài toán sau.

Bài toán 6.14.  Tìm hàm  f (x) xác đnh và liên tc trên  R tho mãn điu kiên 

f (x + y 

2  ) =

 f (x)f (y ),   ∀ x, y  ∈ R

Gii.   Theo điu kin bài toán ta suy ra f (x) 0,  ∀ 

x∈R.

Nu tn ti  x0  đ f (x0) = 0 thì 

f (x0 + y 

2  ) =

 f (x0)f (y ) = 0,   ∀ y  ∈ R,

tc là  f (x) ≡ 0Xét trưng hp  f (x) >  0,   ∀ x ∈ R. Khi đó ta có 

Page 155: Dãy số - Giới hạn Tác giả: Trần Nam Dũng- Nguyễn Văn Mậu, 2007

8/13/2019 Dãy số - Giớ i hạn Tác giả: Trần Nam Dũng- Nguyễn Văn Mậu, 2007

http://slidepdf.com/reader/full/day-so-gioi-han-tac-gia-tran-nam-dung-nguyen-van-mau 155/217

6.5. Hàm s chuyn đi cp s nhân vào cp s cng    155

ln f (x + y 

2  ) =

 ln f (x) + ln f (y )

2  ,   ∀ x, y  ∈ R

hay 

gx + y 

2

 =

 g(x) + g(y )

2  ,   ∀ x, y  ∈ R

trong đó  g(x) = ln f (x). Theo kt qu Bài toán 4 thì  g(x) =  ax + b.Vy  f (x) =  eax+b, a , b ∈ R tuỳ ý.

6.5 Hàm s chuyn đi cp s nhân vào cp s cng

Bài toán 6.15.  Nu dãy s  {un} là mt cp s nhân vi các s hng dương thì dãy s  {vn}

vi  vn

 = logun

a  ,

  ∀ n

∈N, 0 < a

= 1 s lp thành mt cp s cng.

Gii.   Gi s  {un} là cp s nhân vi công bi bng q. Xét dãy s  {vn} vi vn  =logun

a   ,   ∀ n ∈ N, 0 < a = 1.Ta có 

v0 = logu0a   , v1 = logu1

a   , v2 = logu2a   . . . , vn = logun

a

Khi đó v1 − v0  =  v2 − v1 =  v3 − v2 · · · = vn − vn−1  = logd

a

Vy {vn} là cp s cng vi công sai bng  logda  

B đ 6.3.  Cho cp s nhân  {an} vi  an  > 0∀ n ∈ N  và hàm s  f (x) tho mãn 

điu kin f (

√ xy ) =

 f (x) + f (y 

2  ,   ∀ x, y > 0.

Khi đó dãy  {f (an)} là mt cp s cng.

Chng minh.   T gi thit, ta có các h thc 

a0

a1= · · · =

 an−1

an=

  an

an+1= . . .

Suy ra a2n  =  an−1an+1,   ∀ n ∈ N∗

Khi đó f (an) =  f (√ an−1an+1) =  f ( an−1 + an+1

2  ).

T đó ta có  {f (an)} là mt cp s cng.  

Vn đ đt ra ta đi tìm tt c các hàm s chuyn đi mt cp s nhân bt kỳ thành mt cp s cng. Trưc ht ta xét bài toán sau.

Page 156: Dãy số - Giới hạn Tác giả: Trần Nam Dũng- Nguyễn Văn Mậu, 2007

8/13/2019 Dãy số - Giớ i hạn Tác giả: Trần Nam Dũng- Nguyễn Văn Mậu, 2007

http://slidepdf.com/reader/full/day-so-gioi-han-tac-gia-tran-nam-dung-nguyen-van-mau 156/217

6.6. Hàm s chuyn đi cp s nhân vào cp s điu hoà    156

Bài toán 6.16.  Tìm hàm s  f (x) xác đnh và liên tc trên  R tho mãn điu kin 

f (√ xy ) =  f (x) + f (y )2

  ,   ∀ x, y  ∈ R+

Gii Vì  x > 0, y > o nên có th đt  x =  eu, y  =  ev và  f (eu) =  g(u).Khi đó  g(u) liên tc tên  R và có dng 

gu + v

2

 =

 g(u) + g(v)

2  ,   ∀ u, v ∈ R.

Theo kt qu ca bài toán 8.52 thì g(u) =  au + b.Vy ta có kt qu  f (x) =  a ln x + b,a,b ∈ R tuỳ ý.Theo b đ 6.3 trên ta có hàm s  f (x) = a ln x + b, a, b ∈  R chuyn đi mi 

cp s nhân thành cp s cng.

6.6 Hàm s chuyn đi cp s nhân vào cp s điuhoà

Ta xét bài toán sau.

Bài toán 6.17.  Cho cp s nhân  {an}   vi   an   >   0,   ∀ n ∈   N  và cho hàm s f   : R+ → R+ tho mãn điu kin 

f (√ 

xy ) =  2f (x)f (y )

f (x) + f (y ),   ∀ x, y  ∈ R+

Chng minh rng dãy  {f (an)} là mt cp s điu hoà.

Gii. T gi thit, ta có các h thc 

a1

a0= · · · =

  an

an−1=

 an+1

an= . . .

Suy ra a2n  =  an−1an+1,   ∀ n ∈ N∗

Khi đó 

f (an) =  f (√ 

an−1an+1) =  2f (an−1)f (an+1)

f (an−1) + f (an+1)).

T đó ta có  {f (an)} là mt cp s điu hoà.  

Bây gi ta đi tìm tt c các hàm s có tính cht chuyn mt cp s cng bt kỳ thành cp s điu hoà thông qua vic tìm tt c các hàm s có tính cht sau.

Page 157: Dãy số - Giới hạn Tác giả: Trần Nam Dũng- Nguyễn Văn Mậu, 2007

8/13/2019 Dãy số - Giớ i hạn Tác giả: Trần Nam Dũng- Nguyễn Văn Mậu, 2007

http://slidepdf.com/reader/full/day-so-gioi-han-tac-gia-tran-nam-dung-nguyen-van-mau 157/217

6.6. Hàm s chuyn đi cp s nhân vào cp s điu hoà    157

Bài toán 6.18.  Tìm hàm  f (x) xác đnh và liên tc trên  R+ tho mãn điu kin 

f (√ xy ) =   2f (x)f (y )f (x) + f (y )

,   ∀ x, y  ∈ R+

Gii. Ta có 

f (√ 

xy ) =  2f (x)f (y )

f (x) + f (y ), ∀ x, y ∈ R+

Suy ra 

f (√ 

xy ) =  21

f (x) +

  1

f (y )

, ∀ x, y ∈ R+

Hay 

1f (

√ xy )

 =

1

f (x) +

  1

f (y )2

  , ∀ x, y  ∈ R+

Đt  g(x) =  1

f (x). Khi đó ta có 

g(√ 

xy ) = g(x) + g(y )

2  ,   ∀ x, y  ∈ R+

Theo kt qu ca Bài toán 6.16 thì  g(x) =  a ln x + b. Đ  f (x) liên tc trong  R+

thì  g(x) = 0 vi mi  x ∈ R+. Điu đó tương đương vi  a = 0, b = 0Vy 

f (x) ≡ b ∈ R{0} tuỳ ý.

Page 158: Dãy số - Giới hạn Tác giả: Trần Nam Dũng- Nguyễn Văn Mậu, 2007

8/13/2019 Dãy số - Giớ i hạn Tác giả: Trần Nam Dũng- Nguyễn Văn Mậu, 2007

http://slidepdf.com/reader/full/day-so-gioi-han-tac-gia-tran-nam-dung-nguyen-van-mau 158/217

Chương 7

Mt s lp hàm chuyn đi cáccp s trong tp ri rc

Trong chương này s mô t mt s lp hàm s chuyn đi các cp s trong tp hp  Z,N.

7.1 Hàm s chuyn đi cp s cng thành cp s cng

Trưc ht ta xét bài toán sau.

Bài toán 7.1.  Tìm các hàm s  f (x) xác đnh trên  Z tho mãn tính cht 

f (x + y ) =  f (x) + f (y ),

  ∀ x, y 

 ∈Z

Gii. Trưc ht ta kho sát hàm s  f (x) trong tp hp  NTi  x  = 0, y  = 0, ta đưc  f (0) = 0Ti  x  = 1, y  = 1, ta có  f (2) = 2f (1) đt  f (1) = a  ta có  f (2) = 2aTi  x  = 2, y  = 1, ta có  f (3) = f (2) + f (1) ⇒ f (3) = 3f (1) hay  f (3) = 3aBng phép qui np ta chng minh đưc  f (n) = nf (1) hay  f (n) = na,   ∀ ∈ N∗

Vi x, y ∈ Z

Thay   x  = −y   ta có   f (0) =  f (x) +  f (−x) ⇒  f (x) = −f (−x).  Khi đó ta có hàm  f (x) là hàm l.

Xét  n ∈ Z, n <  0 ⇒ −n > 0, khi đó theo chng minh phn trên ta có f (−n) = −namà  f (n) =

−f (

−n)

⇒f (n) = na

Vy hàm s cn tìm là  f (x) =  ax∀ x ∈ Z.

Bài toán 7.2.  Tìm hàm s  f (x) xác đnh trên  Z tho mãn điu kin:

f x + y 

2

 =

 f (x) + f (y )

2  ,   ∀ x, y  ∈ Z, x + y  = 2k, k ∈ Z

158

Page 159: Dãy số - Giới hạn Tác giả: Trần Nam Dũng- Nguyễn Văn Mậu, 2007

8/13/2019 Dãy số - Giớ i hạn Tác giả: Trần Nam Dũng- Nguyễn Văn Mậu, 2007

http://slidepdf.com/reader/full/day-so-gioi-han-tac-gia-tran-nam-dung-nguyen-van-mau 159/217

7.1. Hàm s chuyn đi cp s cng thành cp s cng    159

Gii.   Đt  f (0) = b, f (x) =  b + g(x) thì  g(0) = 0, thay vào công thc trên ta có 

b + g(x + y 2

  ) =   b + g(x) + b + g(y )2

⇒ g(x + y 

2  ) =

 g(x) + g(y )

2

Ln lưt chn  x = 2k, y  = 0 , hoc  x = 0, y  = 2k ta có 

g(x + y )

2  =

 g(x) + g(y )

2

⇔ g(x + y ) =  g(x) + g(y )∀ x, y ∈ Z

Theo kt qu Bài toán 7.1 ta có  g(x) =  ax,   ∀ x ∈ Z

Vy  f (x) =  ax + bBài toán 7.3.  Chng minh rng điu kin cn và đ đ dãy s  {an} lp thành mt cp s cng là dãy đã cho phi tho mãn h thc 

2am+n  = a2m + a2n,   ∀ m, n ∈ N.   (7.1)

Gii.Điu kin cn.Gi s dãy  {an} là mt cp s cng vi công sai bng d.Khi đó 

an =  ao + (n − 1)d,   ∀ n ∈ N∗.

Vy nên 

a2n + a2m = 2ao + (2m + 2n − 2)d

Và 2am+n = 2 [ao + (m + n − 1)d]

T đó ta có ngay công thc 7.1Điu kin đ.Gi s dãy  {an} tho mãn điu kin 7.1. Ta chng minh dãy  {an} là mt cp

s cng vi công sai bng  d  =  a1 − ao

Thay  m = 0 vào công thc 7.1 ta có 

2an =  ao + a2n

Thay  n = 0 vào công thc 7.1 ta có 

2am  =  ao + a2m

Page 160: Dãy số - Giới hạn Tác giả: Trần Nam Dũng- Nguyễn Văn Mậu, 2007

8/13/2019 Dãy số - Giớ i hạn Tác giả: Trần Nam Dũng- Nguyễn Văn Mậu, 2007

http://slidepdf.com/reader/full/day-so-gioi-han-tac-gia-tran-nam-dung-nguyen-van-mau 160/217

7.1. Hàm s chuyn đi cp s cng thành cp s cng    160

Thay kt qu trên vào công thc 7.1 ta thu đưc 

2am+n  = 2am + 2an − 2ao

am+n  =  am + an − ao   (7.2)

Thay  m = 1 vào công thc 7.2 ,ta có an+1  =  an + d, d = a1 − ao

Vy dãy  {an} là mt cp s cng.  

B đ 7.1.  Điu kin cn và đ đ mt hàm s chuyn mi cp s cng nguyên dương thành cp s cng là hàm đó chuyn tp các s t nhiên thành cp s cng.

Chng minh Điu kin cn.

Nu hàm  f  chuyn mi cp s cng thành cp s cng thì hin nhiên hàm  f chuyn tp các s t nhiên thành mt cp s cng vì tp các s t nhiên là cps cng vi công sai nh nht là 1.

Điu kin đ.Hàm  f  chuyn tp các s t nhiên thành cp s cng,tc là dãy  {f (n)} là cp

s cng  ∀ n ∈ N.Dãy  {an} là cp s cng nguyên dương, vi công bi là  d ∈ N ta phi chng minh dãy  {f (an)} là cp s cng.

Vì dãy  {f (n)} là cp s cng nên theo công thc 7.2 ta có 

f (m + n) =  f (m) + f (n) − f (o),   ∀ m, n ∈ N

Dãy 

 {an

} là cp s cng nguyên dương, vi công bi là  d

 ∈ N  suy ra  an+1  =

an + dKhi đó 

f (an+1) =  f (an + d) =  f (an) + f (d) − f (0)

hay  f (an+1) − f (an + d) =  f (d) − f (0) không đi.Vy dãy  {f (an)} là cp s cng vi công sai là  f (d) − f (0)

Bài toán 7.4.  Xác đnh các hàm s   f   :   Z →   R+ chuyn mi cp s cng {an}, an ∈ Z thành cp s cng.

Gii. Đ gii bài toán này theo B đ 7.1 ta ch cn xác đnh các hàm s chuyn dãy s t nhiên thành cp s cng.Hàm f chuyn dãy s t nhiên thành cp s cng thì ta có:

f (m + n) =  f (m) + f (n) − f (o),   ∀ m, n ∈ N

Page 161: Dãy số - Giới hạn Tác giả: Trần Nam Dũng- Nguyễn Văn Mậu, 2007

8/13/2019 Dãy số - Giớ i hạn Tác giả: Trần Nam Dũng- Nguyễn Văn Mậu, 2007

http://slidepdf.com/reader/full/day-so-gioi-han-tac-gia-tran-nam-dung-nguyen-van-mau 161/217

7.2. Hàm s chuyn đi cp s nhân thành cp s nhân   161

f (m + n) − f (0) = f (m) − f (0) + f (n) − f (0),   ∀ m, n ∈ N.

Đt  g(n) =  f (n)−

f (0) ta có 

g(m + n) = g(m) + g(n)

Khi đó theo bài toán 7.1 ta có  g(x) =  ax, ∀ x ∈ N trong đó  a =  g(1)Do đó  f (x) = g(x) + f (0) .Đt  f (0) = b  thì  f (x) =  ax + b∀ x ∈ N

Kt hp Bài toán 7.2 ta có:Hàm s chuyn đi mi cp s cng thành cp s cng trong tp các s nguyên 

là  f (x) = ax + b, ∀ x ∈ Z.  

Bài toán 7.5.  Xác đnh hàm s  f  chuyn cp s cng nguyên dương  {an}  chotrưc thành cp s cng 

 {bn}

 cho trưc.

Gii. Ta xét hai trưng hp sau:(i) Nu  {an} ≡   N,   theo kt qu Bài toán 7.4 ta có   f (x) =   ax +  b,   ∀ x ∈

N, a , b ∈ R.(ii) Nu  {an} ⊂ N, ta có hàm s  f   : N → R đưc xác đnh như sau 

f (n) =

bn   nu  n ∈ {an}cn   nu  n ∈ {an}

(7.3)

trong đó  cn tuỳ ý trong R. chuyn cp s cng nguyên dương  {an} cho trưc thành cp s cng  {bn} cho trưc.

7.2 Hàm s chuyn đi cp s nhân thành cp s nhân

Trên cơ s các bài toán trên ta tìm các hàm s chuyn các cp s khác trên tp hp s nguyên.Trưc ht ta đi tìm nhng dãy s thc hin phép chuyn tip mt đi lưng trung bình ca cp phn t tương ng ca dãy s.Các bài toán này liên quan cht ch đn vic chuyn tip các cp s, đn s phng đoán các cp s tng quát.

Bài toán 7.6.  Xác đnh dãy s  {un}, sao cho

u(

m + n

2   ) =  u(m)u(n),   ∀ m, n,

 m + n

2   ∈ N∗.   (7.4)

Gii. Ta có 

u(n) =  u(n + n

2  ) =

 u(n)u(n) =

 [u(n)]2 =| u(n) |

Page 162: Dãy số - Giới hạn Tác giả: Trần Nam Dũng- Nguyễn Văn Mậu, 2007

8/13/2019 Dãy số - Giớ i hạn Tác giả: Trần Nam Dũng- Nguyễn Văn Mậu, 2007

http://slidepdf.com/reader/full/day-so-gioi-han-tac-gia-tran-nam-dung-nguyen-van-mau 162/217

7.2. Hàm s chuyn đi cp s nhân thành cp s nhân   162

Đt  u(1) = α, u(2) = β (α ≥ 0, β ≥ o).a) Nu  α = 0 thì 

u(n) = u(1 + 2n − 1

2  ) =

 u(1)u(2n − 1) = 0,   ∀ n ∈ N∗

Vy  u(n) ≡ 0 là nghim duy nht ca phương trình   (7.4)b) Nu  α > 0  và  β  = 0 thì 

u(n) =  u(2 + 2n − 2

2  ) =

 u(2)u(2n − 2) = 0,   ∀ n ≥ 2

Suy ra 

u(n) = α   nu  n = 1

0   nu  n ≥ 2

là nghim ca phương trình  (7.4)c)Xét trưng hp  α > 0  và  β > 0. Gi s tn ti  no ≥ 3 sao cho  u(no) = 0Th thì 

u(no − 1) =  u(no + no − 2

2  ) =

 u(no)u(no − 2) = 0.

Chn  no = 3 thì  u(no − 1) =  u(2) = 0, hay  β  = 0, mâu thun .Do đó, ta có th gi thit rng  u(n) >  0, vi mi  n ∈ N∗. Khi đó 

u(2) = u(

3 + 1

2   ) =  u(3)u(1) = 0.

Suy ra 

u(3) = u2(2)

u(1)  =

 β 2

α .

Mt khác 

u(3) = u(4 + 2

2  ) =

 u(4)u(2).

Suy ra 

u(4) = u2(3)

u(2)

  =(

β 2

α )2

β 

  =  β 3

α2

Bng phương pháp quy np toán hc, ta chng minh đưc rng 

u(n) = β n−1

αn−2,   ∀ n ≥ 3.

Page 163: Dãy số - Giới hạn Tác giả: Trần Nam Dũng- Nguyễn Văn Mậu, 2007

8/13/2019 Dãy số - Giớ i hạn Tác giả: Trần Nam Dũng- Nguyễn Văn Mậu, 2007

http://slidepdf.com/reader/full/day-so-gioi-han-tac-gia-tran-nam-dung-nguyen-van-mau 163/217

7.2. Hàm s chuyn đi cp s nhân thành cp s nhân   163

Ta có β n−1

αn−2  = (

α2

β 

  ).(β 

α

)n

Đt      α =  ab,

β  =  ab2(a >  0, b >  0)

Suy ra 

α2

β   = a,

 β 

α = b.

Vy nghim ca phương trình  (7.4) là 

u(n) =α   nu  n = 1

0   nu  n ≥ 2 (∀ α ≥ 0)

hoc  u(n) =  abn (a > 0, b >  0).

Bài toán 7.7.  Xác đnh dãy s  {un}, sao cho

u(m + n

2  ) =

  2u(m)u(n)

u(m) + u(n),   ∀ m,n,

 m + n

2  ∈ N∗   (7.5)

Gii.

u(m + n

2  ) =

  2u(m)u(n)

u(m) + u(n)

u(m + n

2  ) =

  21

u(m) +

  1

u(n)

Đt   1

u(n), thì phương trình đã cho tương đương vi 

v(m + n

2  ) =

 v(m) + v(n)

2  .

Theo Bài toán 7.2, ta có  v(n) =  an + b  vi  a, b ≥ 0, a + b > 0. Vy nghim ca phương trình  (7.5) là 

u(n) =  1

an + b, a , b ≥ 0, a + b >  0.

Page 164: Dãy số - Giới hạn Tác giả: Trần Nam Dũng- Nguyễn Văn Mậu, 2007

8/13/2019 Dãy số - Giớ i hạn Tác giả: Trần Nam Dũng- Nguyễn Văn Mậu, 2007

http://slidepdf.com/reader/full/day-so-gioi-han-tac-gia-tran-nam-dung-nguyen-van-mau 164/217

7.2. Hàm s chuyn đi cp s nhân thành cp s nhân   164

B đ 7.2.  Chng minh rng điu kin cn và đ đ dãy các s dương  {an} lpthành mt cp s nhân là dãy đã cho phi tho mãn h thc 

a2m+n = a2ma2n,   ∀ x, y  ∈ N   (7.6)

Chng minh. Đt   ln an  = bn vi mi  n ∈ N. Khi đó  an  = ebn  và  (7.6) có dng 

e2bm + n =  eb2m + b2n,   ∀ m, n ∈ N

Hay 2bm+n =  b2m + b2n   (7.7)

Theo bài toán 7.3 thì   (7.7) là điu kin cn và đ đ dãy s  {bn} lp thành mt cp s cng vi công sai  d =  b1 − b0.

T đó theo bài toán 6.13 suy ra điu phi chng minh.

Nhn xét:T công thc  (7.6) ta có Xét  m = 0 ta có  a2

n =  a0a2n  Xét  n = 0 ta có  a2m  = a0a2m

Suy ra  a2ma2n = a2

na2m

a20

Do đó  a2m+n  =

 a2na2

m

a20

Nên  am+n  = anam

a0

Bài toán 7.8.  Xác đnh dãy các s dương  {xn} tho mãn điu kin 

xmn =  xmxn,   ∀ m, n ∈ N∗

Gii. Ta có  x1.n  =  x1xn. Suy ra  x1  = 1.  Gi s  n  =  p là s nguyên t. Khi đó bng qui np ta chng minh đưc  x pk  = (x p)k và nu  n =  pα1

1   pα22   . . . pαs

s   thì:

xn = (x p1)α1(x p2)α2 . . . (x ps)αs

x p  có th nhn giá tr tuỳ ý khi  p là mt s nguyên t.T đó ta có kt lun:x p  có th nhn giá tr tuỳ ý khi  p là mt s nguyên t và 

xn = (x p1)α1(x p2)α2 . . . (x ps)αs

khi  n =  pα11   pα2

2   . . . pαss

Bây gi ta xét tip bài toán sau:

Bài toán 7.9.  Xác đnh hàm s  f  tho mãn tính cht  f (mn) = f (m)f (n) trong đó  m, n   ∈ N.

Page 165: Dãy số - Giới hạn Tác giả: Trần Nam Dũng- Nguyễn Văn Mậu, 2007

8/13/2019 Dãy số - Giớ i hạn Tác giả: Trần Nam Dũng- Nguyễn Văn Mậu, 2007

http://slidepdf.com/reader/full/day-so-gioi-han-tac-gia-tran-nam-dung-nguyen-van-mau 165/217

7.2. Hàm s chuyn đi cp s nhân thành cp s nhân   165

Gii. Ta có  f (1.n) = f (1)f (n). Suy ra  f (1) = 1.  Gi s  n =  p  là s nguyên t.Khi đó bng qui np ta chng minh đưc  f ( pk) =  f ( p)k và nu  n  =  pα1

1   pα22   . . . pαs

s   thì:

f (n) =  f ( p1)α1f ( p2)α2 . . . f  ( ps)αs

f ( p) có th nhn giá tr tuỳ ý khi  p là mt s nguyên t.T đó ta có kt lun:f ( p) có th nhn giá tr tuỳ ý khi  p là mt s nguyên t và 

f (n) =  f ( p1)α1f ( p2)α2 . . . f  ( ps)αs

khi  n =  pα11   pα2

2   . . . pαss

Bài toán 7.10.  Chng minh hàm s   f   :   N →   R   xác đnh như sau vi   n   = pα11   pα2

2   . . . pαss   ,  pi là các s nguyên t thì  f (n) =  f ( p1)α1f ( p2)α2 . . . f  ( ps)αs trong 

đó   f ( pi)  tuỳ ý,   i  = 1, 2, 3, . . .  chuyn cp s nhân có công bi nguyên t thành cp s nhân.

Gii. Gi s có cp s nhân  {n0q k}, k = 0; 1; 2 . . . , n0; q  ∈ N∗ ta phi chng minh {f (n0q k)} cũng là cp s nhân. Tht vy theo kt qa bài toán 7.9 trên ta có ngay điu phi chng minh.

Sau đây ta xét bài toán 

Bài toán 7.11.  Chng minh rng hàm s  f  chuyn mi cp s nhân thành cps nhân khi và ch khi hàm s đó chuyn cp s nhân có công bi nguyên t thành cp s nhân.

Gii. Điu kin cn.Nu hàm s  f  chuyn mi cp s nhân thành cp s nhân thì hin nhiên nó chuyn cp s nhân có công bi nguyên t thành cp s nhân.

Điu kin đ.Nu hàm s  f   : N → R  chuyn cp s nhân có công bi nguyên t thành cp

s nhân. Gi s  {un} là cp s nhân ta phi chng minh  {f (un)} cũng là cp s nhân. Vi  un =  u0q n ta xét hai trưng hp sau:

Nu q là s nguyên t thì bài toán đưc chng minh.Nu q không là s nguyên t thì  q  =  pα1

1   pα22   . . . pαs

s   trong đó  pi, i ∈ N∗ là các s nguyên t. Khi đó ta có 

f (un) =  f (u0q n) =  f (u0( pα11   pα2

2   . . . pαss   )n)

= f (u0( p1)α1.n( p2)α2.n . . . ( ps)αs.n)

= f (u0)f α1.n( p1)

 f α2.n( p2)

  . . . f  αs.n( ps)

Page 166: Dãy số - Giới hạn Tác giả: Trần Nam Dũng- Nguyễn Văn Mậu, 2007

8/13/2019 Dãy số - Giớ i hạn Tác giả: Trần Nam Dũng- Nguyễn Văn Mậu, 2007

http://slidepdf.com/reader/full/day-so-gioi-han-tac-gia-tran-nam-dung-nguyen-van-mau 166/217

7.2. Hàm s chuyn đi cp s nhân thành cp s nhân   166

Theo B đ 7.6 ta chng minh  f 2(um+n) =  f (u2m)f (u2n)

Ta có  f 2(um+n) = (f (u0)f α1.(m+n)( p1)

  f α2.(m+n)( p2)

  . . . f  αs.(m+n)( ps)

  )2

= f 2(u0)f 2α1.(m+n)( p1)

  f 2α2.(m+n)( p2)

  . . . f  2αs.(m+n)( ps)

f (u2m)f (u2n) = f (u0)f α1.2m( p1)

  f α2.2m( p2)

  . . . f  αs.2m( ps)

  f (u0)f α1.2n( p1)

  f α2.2n( p2)

  . . . f  αs.2n( ps)

= f 2(u0)f 2α1.(m+n)( p1)

  f 2α2.(m+n)( p2)

  . . . f  2αs.(m+n)( ps)

Vy ta có điu phi chng minh.  

Page 167: Dãy số - Giới hạn Tác giả: Trần Nam Dũng- Nguyễn Văn Mậu, 2007

8/13/2019 Dãy số - Giớ i hạn Tác giả: Trần Nam Dũng- Nguyễn Văn Mậu, 2007

http://slidepdf.com/reader/full/day-so-gioi-han-tac-gia-tran-nam-dung-nguyen-van-mau 167/217

Chương 8

Mt s bài toán xác đnh dãys trong lp dãy tun hoàn

cng tính và nhân tính.

8.1 Mt s bài toán xác đnh dãy s trong lp dãytun hoàn cng tính

Bài toán 8.1.  Xác đnh dãy  {xn} sao cho  xn+3  = xn + 1, n = 1; 2;3; . . .

Gii. Đt  xn =   n3  + y n. Khi đó ta có 

y n+3 +  n + 33

  =  n3

 + y n + 1

hay  y n+3  = y n, ∀ n ∈ N. Vy nên 

y 0  = y 3  =  y 6  = · · ·y 1  = y 4  =  y 7  = · · ·y 2  = y 5  =  y 8  = · · ·

⇔   y n  =

a   tuỳ ý vi    n = 3k, k ∈ N

b   tuỳ ý vi    n = 3k + 1, k ∈ N

c   tuỳ ý vi    n = 3k + 2, k ∈ N

Bài toán 8.2.  Xác đnh dãy s  {xn} sao cho  xn+3 = 2xn

Gii. Đt  xn = 2

n

3 y n. Suy ra 

2

n + 3

3   y n+3  = 2(2n3 y n) ⇔ y n+3  =  y n ⇔ y n+3  =  y n.

167

Page 168: Dãy số - Giới hạn Tác giả: Trần Nam Dũng- Nguyễn Văn Mậu, 2007

8/13/2019 Dãy số - Giớ i hạn Tác giả: Trần Nam Dũng- Nguyễn Văn Mậu, 2007

http://slidepdf.com/reader/full/day-so-gioi-han-tac-gia-tran-nam-dung-nguyen-van-mau 168/217

8.1. Mt s bài toán xác đnh dãy s trong lp dãy tun hoàn cng tính   168

Vy nên 

y 0  = y 3  =  y 6  = · · ·

y 1  = y 4  =  y 7  = · · ·y 2  = y 5  =  y 8  = · · ·

⇔   y n  =a   tuỳ ý vi    n = 3k, k ∈ N

b   tuỳ ý vi    n = 3k + 1, k ∈ N

c   tuỳ ý vi    n = 3k + 2, k ∈ N

Vy  xn  = 2n3 y n  trong đó 

y n  =

a   tuỳ ý vi    n = 3k, k ∈ N

b   tuỳ ý vi    n = 3k + 1, k ∈ N

c   tuỳ ý vi    n = 3k + 2, k ∈ N

Sau đây ta xây dng bài toán tng quát sau.

Bài toán 8.3.  Xác đnh dãy s  {un} tho mãn điu kin un+b  = un + d

Trong đó  n, b   ∈ N, d   ∈ R

Gii. Đt  un = d

bn + vn. Thay vào công thc  un+b  = un + d, ta có 

d

b(n + b) + vn+b  =

 d

bn + vn + d.

Suy ra  vn+b  = vn do đó  vn  là dãy tun hoàn cng tính chu kỳ  b

Vy  un =

 d

b n + vn  vi  vn  là dãy tun hoàn cng tính chu kỳ  b.Bài toán 8.4.  Xác đnh dãy s  {un} tho mãn điu kin 

un+b  = c.un,

trong đó  n, b   ∈ N∗, c   ∈ R

Gii.   Đt  un = cnb vn  ta có 

cn+bb vn+b  = cc

nb vn.

Suy ra  vn+b  = vn  do đó  vn   là dãy tun hoàn cng tính chu kỳ  b Vy  un  = cnb vn

vi  vn  là dãy tun hoàn cng tính chu kỳ  b.

Bài toán 8.5.  Xác đnh dãy s  {un} tho mãn điu kin 

un+b  =  cun + d,

trong đó  n, b   ∈ N∗, c , d   ∈ R.

Page 169: Dãy số - Giới hạn Tác giả: Trần Nam Dũng- Nguyễn Văn Mậu, 2007

8/13/2019 Dãy số - Giớ i hạn Tác giả: Trần Nam Dũng- Nguyễn Văn Mậu, 2007

http://slidepdf.com/reader/full/day-so-gioi-han-tac-gia-tran-nam-dung-nguyen-van-mau 169/217

8.1. Mt s bài toán xác đnh dãy s trong lp dãy tun hoàn cng tính   169

Gii. Xét trưng hp  c = 1 theo kt qu bài toán  8.3, ta có 

un =  db n + vn

vi  vn  là dãy tun hoàn cng tính chu kỳ  b.

Xét trưng hp  c = 1. Đt  un = vn +  d

1 − c. Khi đó ta có 

vn+b +  d

1 − c = c(vn +

  d

1 − c) + d,   ∀ n ∈ N

hay vn+b  = cvn.

Theo kt qu bài toán  8.4, ta có  vn  = | c |nb

xn, trong đó 

xn+b  =

xn   vi   c >  0

−xn   vi   c <  0

Vy nên 

f (x) =

d

1 − c +  c

nb xn,   vi   xn  tuỳ ý sao cho   xn+b = xn, vi   c >  0

d

1

−c

 + | c |nb xn,   vi   xn   tuỳ ý sao cho   xn+b  = −xn, vi   c <  0

Kt lun:

- Nu  c = 1 thì  un  = d

bn + vn  vi  vn  là dãy tun hoàn cng tính chu kỳ  b.

- Nu  c = 1 thì 

f (x) =

d

1 − c + c

nb xn,   vi   xn   tuỳ ý sao cho   xn+b  =  xn, vi   c > 0

d

1 − c + | c |nb xn,   vi   xn   tuỳ ý sao cho   xn+b  = −xn   vi    c <  0

Bài toán 8.6.

Page 170: Dãy số - Giới hạn Tác giả: Trần Nam Dũng- Nguyễn Văn Mậu, 2007

8/13/2019 Dãy số - Giớ i hạn Tác giả: Trần Nam Dũng- Nguyễn Văn Mậu, 2007

http://slidepdf.com/reader/full/day-so-gioi-han-tac-gia-tran-nam-dung-nguyen-van-mau 170/217

8.2. Hàm s xác đnh trên tp các s nguyên   170

8.2 Hàm s xác đnh trên tp các s nguyên

8.2.1 Hàm s chuyn đi các phép tính s hcKhi xét lp phương trình hàm vi cp ch s t do dng đi xng quen bit, ta 

thưng s dng phép th ch s bng các bin mi đ đưa phương trình hàm đã cho v mt dng phương trình hàm mi đã bit cách gii. Tuy nhiên, trong các trưng hp s dng phép th ch s tng quát, nghim nhn đưc ca phương trình mi, nhìn chung không tho mãn điu kin bài ra. Vì vy, nghim ca phương trình mi cn đưc th li thông qua các d liu ca bài ra. Ta xét mt s ví d minh ho.

Bài toán 8.7.  Xác đnh hàm s  f   :   Z → R tho mãn các điu kin 

f (m + n) =  f (m) + f (n) + mn   (m, n ∈ Z).   (1)

Gii. T phương trình (1) ta nhn đưc 

f (n + 1) = f (1) + f (n) + n,   ∀ n ∈ Z,

hay f (n + 1) − f (n) =  a + n, a =  f (1),   ∀ n ∈ Z.   (2)

Phương trình  f (n + 1) − f (n) = a + n  là mt phương trình sai phân tuyn tính không thun nht cp 1. Do phương trình đc trưng tương ng có nghim  λ = 1,nên ta có nghim tng quát ca phương trình thun nht  f (n + 1) − f (n) = 0 là 

f (n) =  c   (3)

Ta vit n =

 1

2(n + 1)2 −  1

2n2 − 1

2.

Khi đó, nghim riêng ca (2) có dng 

f (n)∗ =  n(dn + e).

Thay  f (n)∗ vào (2) ta đưc 

f (n)∗ = a−

 1

2n.   (4)

Vì  f (n) =  f (n) + f (n)∗  nên t (3) và (4) ta có nghim ca (2) là 

f (n) =  c + 1

2n2 +

a −  1

2

n.   (5)

Page 171: Dãy số - Giới hạn Tác giả: Trần Nam Dũng- Nguyễn Văn Mậu, 2007

8/13/2019 Dãy số - Giớ i hạn Tác giả: Trần Nam Dũng- Nguyễn Văn Mậu, 2007

http://slidepdf.com/reader/full/day-so-gioi-han-tac-gia-tran-nam-dung-nguyen-van-mau 171/217

8.2. Hàm s xác đnh trên tp các s nguyên   171

Do  f (1) = a, t (5) ta có  c = 0.Thay  c = 0 vào (5), ta thu đưc nghim ca (2)

f (n) = 1

2n2 +

a −  1

2

.   (6)

Th li ta thy nghim dng (6) tho mãn điu kin ca đu bài.

Bài toán 8.8.  Tn ti hay không tn ti mt hàm s  f   :   Z → R tho mãn điu kin 

f (m + n) =  f (m) + f (n) + m + n,   (m, n ∈ Z).   (7)

Gii. Lp li cách gii như đi vi Bài toán 1, t phương trình (l) ta suy ra 

f (n + 1) =  f (1) + f (n) + n + 1

hay f (n + 1) − f (n) =  a + n   vi  a =  f (1) + 1.   (8)

Phương trình  f (n + 1) − f (n) = a + n  là mt phương trình sai phân tuyn tính không thun nht cp 1. Do phương trình đc trưng có nghim  λ = 1 nên ta có nghim tng quát ca phương trình thun nht  f (n + 1) − f (n) = 0 là 

f (n) =  c   (9)

Ta vit n =

 1

2(n + 1)2 −  1

2n2 − 1

2.

Khi đó, nghim riêng ca (8) có dng   xn

  =  n(dn +  e). Thay   f (n)∗

 vào (8) ta đưc 

f (n)∗ = 1

2n2 +

a −  1

2

n.   (10)

Vì  f (n) =  f (n) + f (n)∗  nên t (9) và (10) ta có nghim ca (8) là 

f (n) =  c + 1

2n2 +

a −  1

2

n.   (11)

Do  f (1) = a − 1, t (11) ta có  c = 1.Thay  c = 1 vào (11), ta có nghim ca (8)

f (n) = 1

2

n2 + a

− 1

2n + 1.   (12)

Th li ta thy nghim dng (12) không tho mãn điu kin ca đu bài. Vy không tn ti mt hàm s  f   :  Z → R tho mãn điu kin 

f (m + n) = f (m). + f (n) + m + n   (m, n ∈ Z).

Page 172: Dãy số - Giới hạn Tác giả: Trần Nam Dũng- Nguyễn Văn Mậu, 2007

8/13/2019 Dãy số - Giớ i hạn Tác giả: Trần Nam Dũng- Nguyễn Văn Mậu, 2007

http://slidepdf.com/reader/full/day-so-gioi-han-tac-gia-tran-nam-dung-nguyen-van-mau 172/217

8.2. Hàm s xác đnh trên tp các s nguyên   172

Bài toán 8.9.  Xác đnh  f   :   Z → R+ tho mãn điu kin 

f (mn) =  f (m)f (n) (m, n ∈ Z).Gii.

Ta có  x1.n  =  f (1)f (n). Suy ra  f (1) = 1. Gi s  n  =  p  là mt s nguyên t.Khi đó  f ( pk) = (f ( p))k (quy np) và nu  n =  pα1

1   . . . pαss   thì 

f (n) = (f ( p1))α1 · · · (f ( ps))αs.

Vy  f ( p) có th nhn giá tr tuỳ ý khi  p là mt s nguyên t.Kt lun:f ( p) có th nhn giá tr tuỳ ý khi  p là mt s nguyên t và 

f (n) = (f ( p1))α1

· · ·(f ( ps))αs

khi  n =  pα11   . . . t pαs

s   .

Bài toán 8.10.  Xác đnh dãy  f   :   Z → R tho mãn điu kin 

F (m + n) + f (n − m) =  f (3n) (m, n ∈ Z, n ≥ m).

Gii.Cho   m   = 0, ta có   2f (n) =   f (3n). Suy ra   f (0) = 0. Đt   m   =   n   ta đưc 

f (2n) = f (3n). Suy ra, mt mt thì 

f (4n) =  f (6n) = f (9n)

và mt khác thì f (4n) + f (2n) =  f (9n).

T đó suy ra 

f (n) = 1

2f (3n) =

 1

2f (2n) = 0

vi mi  n ∈ Z.

8.2.2 Hàm s chuyn tip các đi lưng trung bình

Trong mc này, ta đi tìm nhng hàm s thc hin phép chuyn tip mt đi lưng trung bình ca cp ch s sang mt đi lưng trung bình ca cp phn t tương ng ca hàm s. Các bài toán này liên quan cht ch đn vic chuyn tip

các cp s; đn s mô phng các cp s tng quát, chng hn, ta có th chuyn mt cp s cng sang mt cp s nhân, cp s điu hoà,...

Dưi đây ta xét mt s bài toán chuyn tip các đi lưng trung bình cơ bn trong chương trình ph thông.1) Phép chuyn các đi lưng trung bình cng

Page 173: Dãy số - Giới hạn Tác giả: Trần Nam Dũng- Nguyễn Văn Mậu, 2007

8/13/2019 Dãy số - Giớ i hạn Tác giả: Trần Nam Dũng- Nguyễn Văn Mậu, 2007

http://slidepdf.com/reader/full/day-so-gioi-han-tac-gia-tran-nam-dung-nguyen-van-mau 173/217

8.2. Hàm s xác đnh trên tp các s nguyên   173

Bài toán 8.11.  Xác đnh hàm s  u(n), sao cho

um + n2 =  u(m) + u(n)2

m,n, m + n2   ∈ ZGii.   Đt  u(1) = α,  u(2) = β . Ta có 

u(2) = u

3 + 1

2

 =

 u(3) + u(1)

2  .

Suy ra u(3) = 2u(2) − u(1) = 2β − α.

Tip tc quá trình như vy, ta đưc 

u(3) = u 4 + 2

2  =

 u(4) + u(2)

2   .

Suy ra u(4) = 2u(3) − u(2) = 2(2β − α) − β  = 3β − 2α.

Bng phương pháp quy np, ta thu đưc 

u(n) = (n − 1)β − (n − 2)α,   ∀ n.

Vy    u(n) = (β − α)n + 2α − β,   ∀ n,

u(1) =   α, u(2) = β.

Đt  α =  a + b;   β  = 2a + b, thì  a =  β − α và  b = 2α − β .Do đó, nghim ca phương trình là  u(n) =  an  + b;  a, b tuỳ ý.2) Phép chuyn đi lưng trung bình cng sang trung bình điu hoà 

Bài toán 8.12.  Xác đnh hàm s  u(n) ∈ Z sao cho

u

m + n

2

 =

  2u(m)u(n)

u(m) + u(n)

m,n,

 m + n

2  ∈ Z

.

Gii.   Ta có 

u

m + n

2

 =

  2u(m)u(n)

u(m) + u(n) ⇔ u

m + n

2

 =

  21

u(m)

 +  1

u(n)

.

Đt    1u(n)  = v(n), thì phương trình đã cho tương đương vi 

v

m + n

2

 =

 v(m) + v(n)

2  .

Page 174: Dãy số - Giới hạn Tác giả: Trần Nam Dũng- Nguyễn Văn Mậu, 2007

8/13/2019 Dãy số - Giớ i hạn Tác giả: Trần Nam Dũng- Nguyễn Văn Mậu, 2007

http://slidepdf.com/reader/full/day-so-gioi-han-tac-gia-tran-nam-dung-nguyen-van-mau 174/217

Page 175: Dãy số - Giới hạn Tác giả: Trần Nam Dũng- Nguyễn Văn Mậu, 2007

8/13/2019 Dãy số - Giớ i hạn Tác giả: Trần Nam Dũng- Nguyễn Văn Mậu, 2007

http://slidepdf.com/reader/full/day-so-gioi-han-tac-gia-tran-nam-dung-nguyen-van-mau 175/217

Page 176: Dãy số - Giới hạn Tác giả: Trần Nam Dũng- Nguyễn Văn Mậu, 2007

8/13/2019 Dãy số - Giớ i hạn Tác giả: Trần Nam Dũng- Nguyễn Văn Mậu, 2007

http://slidepdf.com/reader/full/day-so-gioi-han-tac-gia-tran-nam-dung-nguyen-van-mau 176/217

8.2. Hàm s xác đnh trên tp các s nguyên   176

Gii.   Ta có 

u(n) =  u n + n2 =  u2

(n) + u2

(n)2   =  u2(n) = |u(n)| ≥ 0,   ∀ n ∈ Z.

Đt  u(1) = α ≥ 0 ;   u(2) = β  ≥ 0. Ta có 

u(2) = u

3 + 1

2

 =

 u2(3)+ u2(1)

2  .

Suy ra u2(3) = 2u2(2) − u2(1) = 2β 2 − α2

⇒ u(3) =

 2β 2 − α2 (α ≤ β 

√ 2).

Tương t 

u(3) = u

4 + 2

2

 =

 u2(4)+ u2(2)

2  .

Suy ra u2(4) = 2u2(3) − u2(2) = 2(2β 2 − α2) − β 2 = 3β 2 − 2α2

hay 

u(4) = 

3β 2 − 2α2

α ≤ β 

 3

2

.

Bng quy np toán hc, ta chng minh đưc h thc 

u(n) =  (n

−1)β 2

−(n

−2)α2,

  ∀ n

≥3.

Nhn xét rng, ta luôn có  (n − 1)β 2 − (n − 2)α2 =

 (β 2 − α2)n + 2α2 − β 2.

Đt      α2 = a + b

β 2 = 2a + b.

Suy ra    a =  β 2 − α2

b = 2α2

−β 2.

Vy nghim ca phương trình là  u(n) = √ an + b ;   a ≥ 0,  a + b ≥ 0.

Nhn xét 8.1.  Trong c bn bài toán đã nêu trên, nu ta thay  m bi  (n + 1)và  n bi  (n − 1) thì ta có th đưa chúng đưc v các phương trình sai phân quen bit.

Page 177: Dãy số - Giới hạn Tác giả: Trần Nam Dũng- Nguyễn Văn Mậu, 2007

8/13/2019 Dãy số - Giớ i hạn Tác giả: Trần Nam Dũng- Nguyễn Văn Mậu, 2007

http://slidepdf.com/reader/full/day-so-gioi-han-tac-gia-tran-nam-dung-nguyen-van-mau 177/217

8.2. Hàm s xác đnh trên tp các s nguyên   177

8.2.3 Phương trình trong hàm s vi cp bin t do

Trong mc này, ta đi tìm nhng hàm s thc hin phép chuyn tip mt biu thc đi s ca cp ch s sang mt đi lưng khác ca cp phn t tương ng ca dãy s. Các bài toán này liên quan cht ch đn vic chuyn tip các hàm s; đn s mô phng các hàm s đc bit trong s hc, đi s,...

Bài toán 8.15.  Tìm hàm  f   :  Z → Z tha mãn các điu kin  f (1) = a ∈ Z và 

f (m + n) + f (m − n) = 2f (m)f (n),   ∀ m, n ∈ Z.

Gii.   Cho  m  =  n  = 0  ta đưc  f (0) = 0 hoc  f (0) = 1. Nu  f (0) = 0 thì thay n = 0 ta đưc  2f (m) = 0 vi mi  m ∈ Z. Do vy  f (m) ≡ 0 và ng vi  a = 0.

Nu  f (0) = 1, cho  m =  n  = 1 ta thu đưc  f (2) = 2a2 − 1.Tip tc thay  m = 2; n = 1 vào điu kin bài ra ta đưc  f (3) = 4a3

−3a. T 

đó ta có d đoán  f (n) =  T n(a) vi mi  n ≥ 1.D đoán đó đưc chng minh d dàng bng phương pháp quy np.Mt khác, cho m  = 0 ta đưc  f (n)+f (−n) = 2f (0)f (n) = 2f (n) nên  f (−n) =

f (n). Vy  f (n) là hàm chn. Vy ta đưc 

f (m) =

1   khi  m = 0,

a   khi  m = ±1,

T |m|(a)   khi  |m| ≥ 1, m ∈ Z.

Bài toán 8.16.  Tìm hàm  f   :  Z → R tha mãn các điu kin  f (0) = 0, f (1) = 5

2

và  f (m + n) + f (m − n) = f (m)f (n),   ∀ m, n ∈ Z.

Gii.   Cho  m =  n = 0 ta đưc, do  f (0) = 0,  f (0) = 2. Tip theo, theo quy npta đưc 

f (n) = 2n + 2−n,   ∀ n ∈ Z.

Th li ta thy hàm này tho mãn điu kin bài ra.

Bài toán 8.17.  Tìm hàm  f   :  Z → [0, +∞) tha mãn các điu kin  f (1) = 1 và 

f (m + n) + f (m − n) = 1

2[f (2m) + f (2n)],   ∀ m, n ∈ Z, m ≥ n.

Gii.   Cho  m =  n  = 0 ta đưc  f (0) = 0. Cho  m = 1,  n = 0 thì 

f (1) + f (1) = 1

2[f (2) + f (0)].

Suy ra  f (2) = 4.

Page 178: Dãy số - Giới hạn Tác giả: Trần Nam Dũng- Nguyễn Văn Mậu, 2007

8/13/2019 Dãy số - Giớ i hạn Tác giả: Trần Nam Dũng- Nguyễn Văn Mậu, 2007

http://slidepdf.com/reader/full/day-so-gioi-han-tac-gia-tran-nam-dung-nguyen-van-mau 178/217

8.2. Hàm s xác đnh trên tp các s nguyên   178

Chng minh bng quy np ta đưc  f (n) =  n2.Tht vy, do  f (k) + f (k) =   1

2 [f (2k) + f (0)] nên có ngay  f (2k) = 4k2.

Cũng vy, do  f (k + 1) + f (k − 1) =   12 [f (2k) + f (2)] nên ta có 

f (k + 1) = 1

2f (2k) + 2 − f (k − 1) = (k + 1)2.

Bài toán 8.18.  Tìm các đa thc hai bin   P (m, n)  ( m, n ∈   Z) tho mãn điu kin 

a)   P (am, an) =  a2P (m, n) vi mi  m, n, a ∈ Z,b)   P (b + c, a) + P (c + a, b) + P (a + b, c) = 0 vi mi  a, b, c ∈ Z,c)   P (1, 0) = 1.

Gii.

Trong b) đt  b = 1 − a;  c = 0 ta đưc P (1 − a, a) = −1 − P (a, 1 − a).   (1)

Li đt  c = 1 − a − b và kt hp vi a) ta đưc 

P (a + b, 1 − a − b) =  P (a, 1 − a) + P (b, 1 − b) + 2.   (2)

Đt  f (m) =  P (m, 1 − m) + 2. Khi đó  f (1) = P (1, 0)+ 2 = 3 và (5) tr thành f (m + n) =  f (m) + f (n). Đó là phương trình dãy chuyn đi phép cng 

  f (m + n) =  f (m) + f (n),

f (1) = 3.

(3)

Phương trình (3) có nghim duy nht  f (n) = 3n. Vy nên 

P (n, 1 − n) = 3n − 2.   (4)

Bng phương pháp quy np ta s thu đưc 

P (a, b) = (a + b)2

3  a

a + b − 2

= (a + b)(a − 2b),   ∀ a, b ∈ Z.

Tóm li  P (m, n) = (m + n)2(m − 2n).

Bài toán 8.19.  Cho đa thc Chebyshev   T n(x) = cos(n arccos x). Chng minh rng vi   m, n ∈  Z;  n ≥  m  và  x ∈  R   thì  T n(x)   là nghim ca phương trình dãy sau 

T n+m(x) + T n−m(x) = 2T n(x)T m(x).

Page 179: Dãy số - Giới hạn Tác giả: Trần Nam Dũng- Nguyễn Văn Mậu, 2007

8/13/2019 Dãy số - Giớ i hạn Tác giả: Trần Nam Dũng- Nguyễn Văn Mậu, 2007

http://slidepdf.com/reader/full/day-so-gioi-han-tac-gia-tran-nam-dung-nguyen-van-mau 179/217

8.2. Hàm s xác đnh trên tp các s nguyên   179

Gii.  S đng đnh nghĩa  T n(x) và phương pháp quy np hoc s dng các công thc 

cos(n + m)x + cos(n − m)x = 2 cos nx cos mx

và cosh(n + m)x + cosh(n − m)x = 2 cosh(nx) cosh(mx),

ta có ngay điu phi chng minh.

Bài toán 8.20.  Tìm hàm  f   Z → Z  tha mãn các điu kin 

∃N  ∈ Z : −N < f (n) < N  ∀ n ∈ Z,

f (m + n) + f (m − n) = 2f (m)f (n) ∀ m, n ∈ Z.   (1)

Gii.   Cho  m =  n  = 0 ta đưc  f (0) ∈ {0, 1}. Gi s  f (0) = 0. Cho  n = 0 trong (1) ta đưc  2f (m) = 2f (m)f (0) = 0 và  f  ≡ 0.Gi s   f (0) = 1. Cho   m  = 0   trong (1) ta thu đưc   f (−n) =  f (n)  vi mi 

n ∈ Z. Vy ch cn xét  n ∈ Z. Cho  n = 1 trong (1), ta đưc 

f (m + 1) = 2f (m)f (1) − f (m − 1)

và thu đưc công thc truy hi theo  f (1). Nu  |f (1)| ≥ 2 thì t gi thit ta có 

f (2n) = 2[f (n)]2 − 1

tăng và không gii ni, trái vi gi thit. Vy  f (1) ∈ {−1, 0, 1}.

Vi  f (1) = −1 thì  f (n) = (−1)

n

(quy np).Vi  f (1) = 1 thì  f (n) ≡ 1.Vi  f (1) = 0 ta đưc dãy tun hoàn (quy np)

f (4m) = 1, f (4m + 1) = 0, f (4m + 2) = −1, f (4m + 3) = 0.

Suy ra   f (2) = 4. Chng minh bng quy np ta đưc   f (n) =   n2. Tht vy, dof (k) + f (k) = (1/2)[f (2k) + f (0)] nên có ngay  f (2k) = 4k2. Cũng vy, do  f (k +1) + f (k − 1) = (1/2)[f (2k) + f (2)] nên ta có 

f (k + 1) = 1

2f (2k) + 2 − f (k − 1) = (k + 1)2.

Page 180: Dãy số - Giới hạn Tác giả: Trần Nam Dũng- Nguyễn Văn Mậu, 2007

8/13/2019 Dãy số - Giớ i hạn Tác giả: Trần Nam Dũng- Nguyễn Văn Mậu, 2007

http://slidepdf.com/reader/full/day-so-gioi-han-tac-gia-tran-nam-dung-nguyen-van-mau 180/217

8.2. Hàm s xác đnh trên tp các s nguyên   180

8.2.4 Mt s dng toán liên quan đn dãy truy hi

Bài toán 8.21.  Ký hiu 

un =

   π2

0sinn xdx, n ∈ Z.

Xác đnh hàm s  f   :   Z → R theo công thc 

f (n) = (n + 1)ung(n + 1), n ∈ Z.

Gii.  S dng công thc tích phân tng phn, ta thu đưc 

un = − cos x sinn−1 xπ20

  +    π2

0

(n − 1)sinn−2 x cos2 xdx

=

   π2

0(n − 1) sinn−2 x(1 − sin2 x)dx

= (n − 1)(un−1 − un).

T đây suy ra 

g(n + 2) = n + 1

n + 2un, n ∈ Z.   (1)

T (1) ta nhn đưc 

f (n + 1) = (n + 2)g(n + 1)g(n + 2)

= (n + 2)g(n + 1)n + 1

n + 2un

= (n + 1)g(n + 1)un =  f (n).

Vy nên f (n) = f (0) =

 π

2.

Bài toán 8.22.  Ký hiu 

un =

   π 0

cosn x cos nxdx, n ∈ Z.

Xác đnh hàm s  f   :   Z → R theo công thc 

f (n) = 2nun, n ∈ Z.

Page 181: Dãy số - Giới hạn Tác giả: Trần Nam Dũng- Nguyễn Văn Mậu, 2007

8/13/2019 Dãy số - Giớ i hạn Tác giả: Trần Nam Dũng- Nguyễn Văn Mậu, 2007

http://slidepdf.com/reader/full/day-so-gioi-han-tac-gia-tran-nam-dung-nguyen-van-mau 181/217

8.2. Hàm s xác đnh trên tp các s nguyên   181

Gii.   Đt   cosn x =  u  và  cos nxdx =  dv  thì theo công thc tích phân tng phn,ta thu đưc 

un  =  1

n cosn x sin x

π 0

 +

   π 0

cosn−1 x sin x sin nxdx

= 1

2

   π 0

cosn−1 x[cos(n − 1)x + cos(n + 1)x]dx

= 1

2un−1 −  1

2un +

 1

2

   π 0

cosn−1 x sin x + sin nxdx

= 1

2un−1 −  1

2un +

 1

2un.

Vy nên 

un  =  12

un−1  =  14

un−2  = · · · =   12n−1

u1  =   12n−1

π2

.

Bài toán 8.23.  Xác đnh hàm s  {un} đưc tính theo công thc 

un =

   π4

0tan2n xdx

Gii.   Ta vit  un  dưi dng sau 

un =

   π4

0

tan2n xdx

=

   π4

0

tan2n−2 x[(tan2 x + 1) − 1]dx

=

   π4

0tan2n−2 xd tan x − un−1

= tan2n−1 x

2n − 1

π40 − un−1

=  1

2n − 1 − un−1.

Do vy 

un + un−1  =   12n − 1

,

un−1 + un−2  =  1

2n − 3,

· · ·

Page 182: Dãy số - Giới hạn Tác giả: Trần Nam Dũng- Nguyễn Văn Mậu, 2007

8/13/2019 Dãy số - Giớ i hạn Tác giả: Trần Nam Dũng- Nguyễn Văn Mậu, 2007

http://slidepdf.com/reader/full/day-so-gioi-han-tac-gia-tran-nam-dung-nguyen-van-mau 182/217

8.2. Hàm s xác đnh trên tp các s nguyên   182

u1 + u0  =  1

2 − 1.

Suy ra un = (−1)n

π

4 +

nk=1

(−1)k

2k − 1

.

Bài toán 8.24.  Xác đnh hàm s  f   :   Z → R đưc tính theo công thc 

f (n) =

1 0

xn√ 

1 − xdx, n ∈ Z.

Gii.   Đt  xn = u, √ 

1 − xdx =  dv  thì 

f (n) =

1 0

xn√ 1 − xdx

=− 2

3xn(1 − x)

321

0 +

 2n

3

1 0

xn−1√ 

1 − x(1 − x)dx

= 2n

3

1 0

(xn−1 − xn)√ 

1 − xdx

= 2n

3

1

 0

(xn−1

√ 1 − xdx − 2n

3

1

 0

xn

√ 1 − xdx

2n

3 xn−1 − 2n

3 f (n).

Vy nên 

f (n) =  2n

2n − 3xn−1.

Vì  f (0) = 2

3 nên ta có ngay 

f (n) =

1

 0

x

n√ 

1 − xdx = 2

  (2n)!!

(2n + 3)!!, n ∈ Z.

Bài toán 8.25.  Xác đnh hàm  f   :   Z → Z tho mãn các điu kin 

f (0) = 1, f (f (n) =  f (f (n + 2) + 2) =  n  ∀ n ∈ Z.

Page 183: Dãy số - Giới hạn Tác giả: Trần Nam Dũng- Nguyễn Văn Mậu, 2007

8/13/2019 Dãy số - Giớ i hạn Tác giả: Trần Nam Dũng- Nguyễn Văn Mậu, 2007

http://slidepdf.com/reader/full/day-so-gioi-han-tac-gia-tran-nam-dung-nguyen-van-mau 183/217

8.2. Hàm s xác đnh trên tp các s nguyên   183

Gii.   Nhn xét rng  f   là ánh x 1-1

f (m) =  f (n)   ⇒   f (f (m)) =  f (f (n))   ⇒   m =  n.

Vy nên f (n) = f (n + 2) + 2 ∀ n ∈ Z.

Suy ra f (n + 2) = f (n) − 2, f (0) = 1, f (1) = f (f (0)) = 0.

Vy nên f (2) = f (0) − 2 = −1,

f (3) = f (1) − 2 = −2,

f (n) =

−(n

−1).

Tương t f (−1) =  f (1) + 2 = 2,

f (−2) =  f (0) + 2 = 3,

f (−3) =  f (1) + 2 = 5,

f (n) = −(n − 1).

Bài toán 8.26.  Cho góc  α  vi  0  < α < π. Xác đnh cp s  a, b sao cho dãy hàm {P n(x)} đưc tính theo công thc 

P n(x) =  xn sin α−

x sin(nα) + sin(n−

1)α

luôn luôn chia ht cho  f (x) =  x2 + ax + b.

Gii.   Vi  n = 3 thì 

P 3(x) =  x3 sin α − x sin(3α) + sin 2α = sin α(x + 2 cos α)(x2 − 2x cos α + 1).

T đó suy ra vi  f (x) =  x2 + 2x cos α + 1  thì  P 3(x)...f (x). Vi  n ≥ 3 thì 

P n+1(x) =  xP n(x) + (x2 − 2x cos α + 1) sin nα.

Suy ra  f (x) =  x2 + 2x cos α + 1.

Page 184: Dãy số - Giới hạn Tác giả: Trần Nam Dũng- Nguyễn Văn Mậu, 2007

8/13/2019 Dãy số - Giớ i hạn Tác giả: Trần Nam Dũng- Nguyễn Văn Mậu, 2007

http://slidepdf.com/reader/full/day-so-gioi-han-tac-gia-tran-nam-dung-nguyen-van-mau 184/217

Page 185: Dãy số - Giới hạn Tác giả: Trần Nam Dũng- Nguyễn Văn Mậu, 2007

8/13/2019 Dãy số - Giớ i hạn Tác giả: Trần Nam Dũng- Nguyễn Văn Mậu, 2007

http://slidepdf.com/reader/full/day-so-gioi-han-tac-gia-tran-nam-dung-nguyen-van-mau 185/217

8.3. Hàm s xác đnh trên tp các s hu t    185

Ta vit ν  =

 1

2

(ν  + 1)2

 1

2

ν 2

 1

2

.

Khi đó, nghim riêng ca (2) có dng 

f (ν )∗  =  ν (dν  + e).

Thay  f (ν )∗ vào (2) ta đưc 

f (ν )∗ =

a − 1

2

ν.   (4)

Vì  f (ν ) =  f (ν ) + f (ν )∗  nên t (3) và (4) ta có nghim ca (2) là 

f (ν ) =  c +

 1

2ν 2

+ a − 1

2n.   (5)

Do  f (1) = a, t (5) ta có  c = 0.Thay  c = 0 vào (5), ta thu đưc nghim ca (2)

f (ν ) = 1

2ν 2 +

a −  1

2

.   (6)

Th li ta thy nghim dng (6) tho mãn điu kin ca đu bài.

Bài toán 8.28.  Tn ti hay không tn ti mt hàm s   f   :   Q →  R  tho mãn điu kin 

f (µ + n) =  f (µ) + f (ν ) + µ + n,   (µ, ν  ∈ Q).   (7)Gii.   Lp li cách gii như đi vi Bài toán trên, t phương trình (l) ta suy ra 

f (ν  + 1) = f (1) + f (ν ) + ν  + 1

hay f (ν  + 1) − f (ν ) =  a + ν   vi  a =  f (1) + 1.   (8)

Phương trình  f (ν  + 1) − f (ν ) =  a + ν   là mt phương trình sai phân tuyn tính không thun nht cp 1. Do phương trình đc trưng có nghim  λ = 1 nên ta có nghim tng quát ca phương trình thun nht  f (ν  + 1) − f (ν ) = 0 là 

f (ν ) =  c   (9)

Ta vit ν  =

 1

2(ν  + 1)2 −  1

2ν 2 − 1

2.

Page 186: Dãy số - Giới hạn Tác giả: Trần Nam Dũng- Nguyễn Văn Mậu, 2007

8/13/2019 Dãy số - Giớ i hạn Tác giả: Trần Nam Dũng- Nguyễn Văn Mậu, 2007

http://slidepdf.com/reader/full/day-so-gioi-han-tac-gia-tran-nam-dung-nguyen-van-mau 186/217

8.3. Hàm s xác đnh trên tp các s hu t    186

Khi đó, nghim riêng ca (8) có dng   xn   =   n(dν  +  e). Thay   f (ν )∗  vào (8) ta đưc 

f (ν )∗ =  12

ν 2 + a −  12ν.   (10)

Vì  f (ν ) =  f (ν ) + f (ν )∗  nên t (9) và (10) ta có nghim ca (8) là 

f (ν ) = c + 1

2ν 2 +

a −  1

2

ν.   (11)

Do  f (1) = a − 1, t (11) ta có  c = 1.Thay  c = 1 vào (11), ta có nghim ca (8)

f (ν ) = 1

2ν 2 +

a −  1

2

ν  + 1.   (12)

Th li ta thy nghim dng (12) không tho mãn điu kin ca đu bài. Vy không tn ti mt hàm s  f   :  Q → R tho mãn điu kin 

f (µ + ν ) = f (µ). + f (ν ) + µ + ν   (µ, ν  ∈ Q).

Bài toán 8.29.  Xác đnh  f   :  Q → R+ tho mãn điu kin 

f (µν ) = f (µ)f (ν ) (µ, ν  ∈ Q).

Gii.   Ta có  x1.ν  = f (1)f (ν ). Suy ra  f (1) = 1. Gi s  ν  = p  là mt s nguyên t. Khi đó  f ( pk) = (f ( p))k (quy np) và nu  ν  = pα1

1   . . . pαss   thì 

f (ν ) = (f ( p1))α1 · · · (f ( ps))αs.

Vy  f ( p) có th nhn giá tr tuỳ ý khi  p là mt s nguyên t.Kt lun:f ( p) có th nhn giá tr tuỳ ý khi  p là mt s nguyên t và 

f (ν ) = (f ( p1))α1 · · · (f ( ps))αs

khi  ν  = pα11   . . . t pαs

s   .

Bài toán 8.30.  Xác đnh dãy  f   :  Q → R tho mãn điu kin 

F (µ + ν ) + f (ν − µ) =  f (3ν ) (µ, ν  ∈Q

, ν  ≥ µ).

Gii.   Cho   µ   = 0, ta có   2f (ν ) =  f (3ν ). Suy ra   f (0) = 0. Đt   µ  =  ν   ta đưc f (2ν ) = f (3ν ). Suy ra, mt mt thì 

f (4ν ) =  f (6ν ) = f (9ν )

Page 187: Dãy số - Giới hạn Tác giả: Trần Nam Dũng- Nguyễn Văn Mậu, 2007

8/13/2019 Dãy số - Giớ i hạn Tác giả: Trần Nam Dũng- Nguyễn Văn Mậu, 2007

http://slidepdf.com/reader/full/day-so-gioi-han-tac-gia-tran-nam-dung-nguyen-van-mau 187/217

8.3. Hàm s xác đnh trên tp các s hu t    187

và mt khác thì f (4ν ) + f (2ν ) = f (9ν ).

T đó suy ra f (ν ) =

 1

2f (3ν ) =

 1

2f (2ν ) = 0

vi mi  ν  ∈ Q.Tip theo, ta đi tìm nhng hàm s thc hin phép chuyn tip mt đi lưng 

trung bình ca cp ch s sang mt đi lưng trung bình ca cp phn t tương ng ca hàm s. Các bài toán này liên quan cht ch đn vic chuyn tip các cp s; đn s mô phng các cp s tng quát, chng hn, ta có th chuyn mt cp s cng sang mt cp s nhân, cp s điu hoà,...

Dưi đây ta xét mt s bài toán chuyn tip các đi lưng trung bình cơ bn trong chương trình ph thông.

1) Phép chuyn các đi lưng trung bình cngBài toán 8.31.  Xác đnh hàm s  u(ν ), sao cho

u

µ + ν 

2

 =

 u(µ) + u(ν )

2

µ,ν,

 µ + ν 

2  ∈ Q

Gii.   Đt  u(1) = α,  u(2) = β . Ta có 

u(2) = u

3 + 1

2

 =

 u(3) + u(1)

2  .

Suy ra u(3) = 2u(2)

−u(1) = 2β 

−α.

Tip tc quá trình như vy, ta đưc 

u(3) = u

4 + 2

2

 =

 u(4) + u(2)

2  .

Suy ra u(4) = 2u(3) − u(2) = 2(2β − α) − β  = 3β − 2α.

Bng phương pháp quy np, ta thu đưc 

u(ν ) = (n − 1)β − (n − 2)α,   ∀ n.

Vy 

  u(ν ) = (β −

α)n + 2α−

β,  ∀ 

n,

u(1) =   α, u(2) = β.

Đt  α =  a + b;   β  = 2a + b, thì  a =  β − α và  b = 2α − β .Do đó, nghim ca phương trình là  u(ν ) = an + b;  a, b tuỳ ý.

2) Phép chuyn đi lưng trung bình cng sang trung bình điu hoà 

Page 188: Dãy số - Giới hạn Tác giả: Trần Nam Dũng- Nguyễn Văn Mậu, 2007

8/13/2019 Dãy số - Giớ i hạn Tác giả: Trần Nam Dũng- Nguyễn Văn Mậu, 2007

http://slidepdf.com/reader/full/day-so-gioi-han-tac-gia-tran-nam-dung-nguyen-van-mau 188/217

8.3. Hàm s xác đnh trên tp các s hu t    188

Bài toán 8.32.  Xác đnh hàm s  u(ν ) ∈ Q sao cho

uµ + ν 2

 =   2u(µ)u(ν )u(µ) + u(ν )

µ,ν, µ + ν 2

  ∈ Q .

Gii.   Ta có 

u

µ + ν 

2

 =

  2u(µ)u(ν )

u(µ) + u(ν ) ⇔ u

µ + ν 

2

 =

  21

u(µ) +

  1

u(ν )

.

Đt    1u(ν )  = v(ν ), thì phương trình đã cho tương đương vi 

vµ + ν 

2  = v(µ) + v(ν )

2

  .

Theo Bài toán 1,  v(ν ) =  aν  + b;  a, b ≥ 0, a + b > 0.Vy nghim ca phương trình là 

u(ν ) =  1

aν  + b;   a, b ≥ 0;   a + b > 0.

3) Phép chuyn đi lưng trung bình cng sang trung bìuh nhân

Bài toán 8.33.  Xác đnh hàm s  u(ν ) sao cho

uµ + ν 

2  =  u(µ)u(ν ); µ,ν, µ + ν 

2 ∈Q.

Gii.   Ta có 

u(ν ) =  u

ν  + ν 

2

 = 

u(ν )u(ν ) = 

[u(ν )]2 = |u(ν )| ≥ 0.

Đt  u(1) = α, u(2) = β   ( α ≥ 0,  β  ≥ 0).a) Nu  α = 0 thì 

u(ν ) =  u

1 + 2ν − 1

2

 =

 u(1)u(2n − 1) = 0,   ∀ ν  ∈ Q.

Vy  u(ν ) ≡ 0 là nghim duy nht ca phương trình .b) Nu  α > 0  và  β  = 0 thì 

u(ν ) =  u

2 + 2ν  − 2

2

 = 

u(2)u(2n − 2) = 0,   ∀ ν  ≥ 2.

Page 189: Dãy số - Giới hạn Tác giả: Trần Nam Dũng- Nguyễn Văn Mậu, 2007

8/13/2019 Dãy số - Giớ i hạn Tác giả: Trần Nam Dũng- Nguyễn Văn Mậu, 2007

http://slidepdf.com/reader/full/day-so-gioi-han-tac-gia-tran-nam-dung-nguyen-van-mau 189/217

8.3. Hàm s xác đnh trên tp các s hu t    189

Suy ra 

u(ν ) =   α,   nu  ν  = 1

0   nu  ν  ≥ 2

là nghim ca phương trình.c) Xét trưng hp  α > 0  và  β > 0. Gi s tn ti  n0 ≥ 3  sao cho  u(n0) = 0.

Th thì 

u(n0 − 1) =  u

n0 + n0 − 2

2

 = 

u(n0)u(n0 − 2) = 0.

Chn  n0  = 3 thì  u(n0 − 1) =  u(2) = 0, hay  β  = 0, mâu thun.Do đó, có th gi thit rng  u(ν ) >  0  vi mi  ν  ∈ Q. Ta có 

u(2) = u3 + 1

2  =  u(3)u(1).

Suy ra 

u(3) = u2(2)

u(1)  =

 β 2

α .

Mt khác 

u(3) = u

4 + 2

2

 = 

u(4)u(2).

Suy ra 

u(4) = u2(3)

u(2)  =

β 2

α

2β 

  =  β 3

α2.

Bng phương pháp quy np toán hc, ta chng minh đưc rng 

u(ν ) =  β ν −1

αν −2,   ∀ ν  ≥ 3.

mà 

β ν −1

αν −2  =

α2

β 

β 

α

n

.

Đt 

    α =  ab,

β  =  ab2 (a > 0, b >  0).

Suy ra α2

β   = a,

  β 

α = b.

Page 190: Dãy số - Giới hạn Tác giả: Trần Nam Dũng- Nguyễn Văn Mậu, 2007

8/13/2019 Dãy số - Giớ i hạn Tác giả: Trần Nam Dũng- Nguyễn Văn Mậu, 2007

http://slidepdf.com/reader/full/day-so-gioi-han-tac-gia-tran-nam-dung-nguyen-van-mau 190/217

8.3. Hàm s xác đnh trên tp các s hu t    190

Vy nghim ca phương trình là 

u(ν ) =   α   nu  ν  = 10   nu  ν  ≥ 2

(α ≥ 0)

hoc   u(ν ) = a.bν  (a >  0, b >  0).4) Phép chuyn đi lưng trung bình cng sang trung bình bc hai

Bài toán 8.34.  Xác đnh hàm s  u(ν ), sao cho

u

µ + ν 

2

 =

 u2(µ) + u2(ν )

2

µ,ν,

 µ + ν 

2  ∈ Q

.

Gii.   Ta có 

u(ν ) = u

ν  +  ν 

2

 =

 u2(ν ) + u2(ν )

2  =

 u2(ν ) = |u(ν )| ≥ 0,   ∀ ν  ∈ Q.

Đt  u(1) = α ≥ 0 ;   u(2) = β  ≥ 0. Ta có 

u(2) = u

3 + 1

2

 =

 u2(3)+ u2(1)

2  .

Suy ra u2(3) = 2u2(2) − u2(1) = 2β 2 − α2

⇒u(3) =  2β 2

−α2 (α

≤β √ 

2).

Tương t 

u(3) = u

4 + 2

2

 =

 u2(4)+ u2(2)

2  .

Suy ra u2(4) = 2u2(3) − u2(2) = 2(2β 2 − α2) − β 2 = 3β 2 − 2α2

hay 

u(4) = 

3β 2 − 2α2

α ≤ β 

 3

2

.

Bng quy np toán hc, ta chng minh đưc h thc 

u(ν ) = 

(n − 1)β 2 − (n − 2)α2,   ∀ n ≥ 3.

Nhn xét rng, ta luôn có  (ν − 1)β 2 − (ν  − 2)α2 =

 (β 2 − α2)ν  + 2α2 − β 2.

Page 191: Dãy số - Giới hạn Tác giả: Trần Nam Dũng- Nguyễn Văn Mậu, 2007

8/13/2019 Dãy số - Giớ i hạn Tác giả: Trần Nam Dũng- Nguyễn Văn Mậu, 2007

http://slidepdf.com/reader/full/day-so-gioi-han-tac-gia-tran-nam-dung-nguyen-van-mau 191/217

8.4. Phương trình trong hàm s vi cp bin t do   191

Đt 

    α2 = a + b

β 2 = 2a + b.

Suy ra    a =  β 2 − α2

b = 2α2 − β 2.

Vy nghim ca phương trình là  u(ν ) =√ 

aν  + b ;   a ≥ 0,  a + b ≥ 0.

Nhn xét 8.2.  Trong c bn bài toán đã nêu trên, nu ta thay  m bi  (n + 1)và  n bi  (n − 1) thì ta có th đưa chúng đưc v các phương trình sai phân quen bit.

8.4 Phương trình trong hàm s vi cp bin t do

Trong mc này, ta đi tìm nhng hàm s thc hin phép chuyn tip mt biu thc đi s ca cp ch s sang mt đi lưng khác ca cp phn t tương ng ca dãy s. Các bài toán này liên quan cht ch đn vic chuyn tip các hàm s; đn s mô phng các hàm s đc bit trong s hc, đi s,...

Bài toán 8.35.  Tìm hàm  f   :  Q → Q tha mãn các điu kin  f (1) = a ∈ Q và 

f (µ + n) + f (m − ν ) = 2f (µ)f (ν ),   ∀ m, ν  ∈ Q.

Gii.   Cho  m  =  n  = 0  ta đưc  f (0) = 0 hoc  f (0) = 1. Nu  f (0) = 0 thì thay n = 0 ta đưc  2f (µ) = 0 vi mi  m ∈ Q. Do vy  f (µ) ≡ 0 và ng vi  a = 0.

Nu  f (0) = 1, cho  m =  n  = 1 ta thu đưc  f (2) = 2a2 − 1.Tip tc thay  m = 2; n = 1 vào điu kin bài ra ta đưc  f (3) = 4a3 − 3a. T 

đó ta có d đoán  f (ν ) =  T n(a) vi mi  ν  ≥ 1.D đoán đó đưc chng minh d dàng bng phương pháp quy np.mt khác, cho m  = 0 ta đưc  f (ν )+ f (−ν ) = 2f (0)f (ν ) = 2f (ν ) nên  f (−ν ) =

f (ν ). Vy  f (ν ) là hàm chn. Vy ta đưc 

f (µ) =

1   khi  m = 0,

a   khi  m = ±1,

T |m|(a)   khi 

 |m

| ≥1, m

∈Q.

Bài toán 8.36.  Tìm hàm  f   :  Q → R tha mãn các điu kin  f (0) = 0, f (1) = 5

2và 

f (µ + n) + f (m − ν ) = f (µ)f (ν ),   ∀ m, ν  ∈ Q.

Page 192: Dãy số - Giới hạn Tác giả: Trần Nam Dũng- Nguyễn Văn Mậu, 2007

8/13/2019 Dãy số - Giớ i hạn Tác giả: Trần Nam Dũng- Nguyễn Văn Mậu, 2007

http://slidepdf.com/reader/full/day-so-gioi-han-tac-gia-tran-nam-dung-nguyen-van-mau 192/217

8.4. Phương trình trong hàm s vi cp bin t do   192

Gii.   Cho  m =  n = 0 ta đưc, do  f (0) = 0,  f (0) = 2. Tip theo, theo quy npta đưc 

f (ν ) = 2ν  + 2−ν ,   ∀ ν  ∈ Q.

Th li ta thy hàm này tho mãn điu kin bài ra.

Bài toán 8.37.  Tìm hàm  f   :  Q → [0, +∞) tha mãn các điu kin  f (1) = 1 và 

f (µ + n) + f (m − ν ) = 1

2[f (2m) + f (2n)],   ∀ m, ν  ∈ Q, m ≥ n.

Gii.   Cho  m =  n  = 0 ta đưc  f (0) = 0. Cho  m = 1,  ν  = 0 thì 

f (1) + f (1) = 1

2[f (2) + f (0)].

Suy ra  f (2) = 4.Chng minh bng quy np ta đưc  f (ν ) =  ν 2.Tht vy, do  f (k) + f (k) =   1

2 [f (2k) + f (0)] nên có ngay  f (2k) = 4k2.Cũng vy, do  f (k + 1) + f (k − 1) =   1

2 [f (2k) + f (2)] nên ta có 

f (k + 1) = 1

2f (2k) + 2 − f (k − 1) = (k + 1)2.

Bài toán 8.38.  Tìm các đa thc hai bin  P (m, n)  ( m, ν  ∈  Q) tho mãn điu kin 

a)   P (am, an) =  a2P (m, n) vi mi  m, n, a ∈ Q,b)   P (b + c, a) + P (c + a, b) + P (a + b, c) = 0 vi mi  a, b, c ∈ Q,

c)   P (1, 0) = 1.Gii.  Trong b) đt  b = 1 − a;  c = 0 ta đưc 

P (1 − a, a) = −1 − P (a, 1 − a).   (1)

Li đt  c = 1 − a − b và kt hp vi a) ta đưc 

P (a + b, 1 − a − b) =  P (a, 1 − a) + P (b, 1 − b) + 2.   (2)

Đt  f (µ) = P (m, 1 − µ) + 2. Khi đó  f (1) = P (1, 0) + 2 = 3 và (5) tr thành f (m + n) =  f (µ) + f (ν ). Đó là phương trình dãy chuyn đi phép cng 

  f (m + n) = f (µ) + f (ν ),

f (1) = 3. (3)

Phương trình (3) có nghim duy nht  f (ν ) = 3n. Vy nên 

P (n, 1 − ν ) = 3n − 2.   (4)

Page 193: Dãy số - Giới hạn Tác giả: Trần Nam Dũng- Nguyễn Văn Mậu, 2007

8/13/2019 Dãy số - Giớ i hạn Tác giả: Trần Nam Dũng- Nguyễn Văn Mậu, 2007

http://slidepdf.com/reader/full/day-so-gioi-han-tac-gia-tran-nam-dung-nguyen-van-mau 193/217

8.4. Phương trình trong hàm s vi cp bin t do   193

Bng phương pháp quy np ta s thu đưc 

P (a, b) = (a + b)

23

  a

a + b − 2 = (a + b)(a − 2b),   ∀ a, b ∈ Q.

Tóm li  P (µ, ν ) = (µ + n)2(m − 2n).

Bài toán 8.39.   Cho đa thc Chebyshev   T n(x) = cos(ν  arccos x). Chng minh rng vi  m, n ∈  Q;  ν  ≥  m và   x ∈  R   thì  T n(x)   là nghim ca phương trình dãy sau 

T n+m(x) + T n−m(x) = 2T n(x)T m(x).

Gii.  S đng đnh nghĩa  T n(x) và phương pháp quy np hoc s dng các công thc 

cos(n + m)x + cos(n − m)x = 2 cos nx cos mx

và cosh(n + m)x + cosh(n − m)x = 2 cosh(nx) cosh(mx),

ta có ngay điu phi chng minh.

Bài toán 8.40.  Tìm hàm  f  Q → Q tha mãn các điu kin 

∃ν  ∈ Q : −ν < f (ν ) < ν  ∀ ν  ∈ Q,

f (µ + n) + f (m − ν ) = 2f (µ)f (ν ) ∀ m, ν  ∈ Q.   (1)

Gii.   Cho  m =  n  = 0 ta đưc  f (0) ∈ {0, 1}. Gi s  f (0) = 0. Cho  ν  = 0 trong (1) ta đưc  2f (µ) = 2f (µ)f (0) = 0 và  f  ≡ 0.

Gi s   f (0) = 1. Cho   m  = 0   trong (1) ta thu đưc   f (−ν ) =   f (ν )  vi mi ν 

 ∈Q. Vy ch cn xét  ν 

 ∈Q. Cho  ν  = 1 trong (1), ta đưc 

f (µ + 1) = 2f (µ)f (1) − f (m − 1)

và thu đưc công thc truy hi theo  f (1). Nu  |f (1)| ≥ 2 thì t gi thit ta có 

f (2n) = 2[f (ν )]2 − 1

tăng và không gii ni, trái vi gi thit. Vy  f (1) ∈ {−1, 0, 1}.Vi  f (1) = −1 thì  f (ν ) = (−1)n (quy np).Vi  f (1) = 1 thì  f (ν ) ≡ 1.Vi  f (1) = 0 ta đưc dãy tun hoàn (quy np)

f (4m) = 1, f (4µ + 1) = 0, f (4µ + 2) = −1, f (4µ + 3) = 0.

Suy ra   f (2) = 4. Chng minh bng quy np ta đưc   f (ν ) =   ν 2

. Tht vy, dof (k) + f (k) = (1/2)[f (2k) + f (0)] nên có ngay  f (2k) = 4k2. Cũng vy, do  f (k +1) + f (k − 1) = (1/2)[f (2k) + f (2)] nên ta có 

f (k + 1) = 1

2f (2k) + 2 − f (k − 1) = (k + 1)2.

Page 194: Dãy số - Giới hạn Tác giả: Trần Nam Dũng- Nguyễn Văn Mậu, 2007

8/13/2019 Dãy số - Giớ i hạn Tác giả: Trần Nam Dũng- Nguyễn Văn Mậu, 2007

http://slidepdf.com/reader/full/day-so-gioi-han-tac-gia-tran-nam-dung-nguyen-van-mau 194/217

8.4. Phương trình trong hàm s vi cp bin t do   194

Bài toán 8.41.  Ký hiu 

uν  =   π20

sinn xdx, ν  ∈ Q.

Xác đnh hàm s  f   :  Q → R theo công thc 

f (ν ) = (ν  + 1)ung(ν  + 1), ν  ∈ Q.

Gii.  S dng công thc tích phân tng phn, ta thu đưc 

uν  = − cos x sinν −1 xπ20

  +

   π2

0(ν − 1) sinν −2 x cos2 xdx

=    π20

(ν  − 1) sinν −2 x(1 − sin2 x)dx

= (ν − 1)(uν −1 − un).

T đây suy ra 

g(ν  + 2) = ν  + 1

ν  + 2un, ν  ∈ Q.   (1)

T (1) ta nhn đưc 

f (ν  + 1) = (ν  + 2)g(ν  + 1)g(ν  + 2)

= (ν  + 2)g(ν  + 1)ν  + 1

ν  + 2uν 

= (ν  + 1)g(ν  + 1)uν  = f (ν ).

Vy nên f (ν ) = f (0) =

 π

2.

Bài toán 8.42.  Ký hiu 

uν  =

   π 0

cosν x cos nxdx, ν  ∈ Q.

Xác đnh hàm s  f   :  Q

→R theo công thc 

f (ν ) = 2ν uν , ν  ∈ Q.

Page 195: Dãy số - Giới hạn Tác giả: Trần Nam Dũng- Nguyễn Văn Mậu, 2007

8/13/2019 Dãy số - Giớ i hạn Tác giả: Trần Nam Dũng- Nguyễn Văn Mậu, 2007

http://slidepdf.com/reader/full/day-so-gioi-han-tac-gia-tran-nam-dung-nguyen-van-mau 195/217

8.4. Phương trình trong hàm s vi cp bin t do   195

Gii.   Đt   cosn x =  u  và  cos nxdx =  dv  thì theo công thc tích phân tng phn,ta thu đưc 

uν  =  1

ν  cosn x sin x

π 0

 +

   π 0

cosν −1 x sin x sin nxdx

= 1

2

   π 0

cosν −1 x[cos(ν − 1)x + cos(ν  + 1)x]dx

= 1

2uν −1 −  1

2uν  +

 1

2

   π 0

cosν −1 x sin x + sin nxdx

= 1

2uν −1 −  1

2uν  +

 1

2uν .

Vy nên 

uν  = 12

uν −1  =  14

uν −2  = · · · =   12ν −1

u1  =   12ν −1

π2

.

Bài toán 8.43.  3. Xác đnh hàm s  {uν } đưc tính theo công thc 

uν  =

   π4

0tan2ν xdx

Gii.   Ta vit  un  dưi dng sau 

uν  =

   π4

0

tan2ν xdx

=

   π4

0

tan2ν −2 x[(tan2 x + 1) − 1]dx

=

   π4

0tan2ν −2 xd tan x − uν −1

= tan2ν −1 x

2ν − 1

π40 − uν −1

=  1

2ν − 1 − uν −1.

Do vy 

uν  + uν −1  =   12ν  − 1

,

uν −1 + uν −2  =  1

2ν − 3,

· · ·

Page 196: Dãy số - Giới hạn Tác giả: Trần Nam Dũng- Nguyễn Văn Mậu, 2007

8/13/2019 Dãy số - Giớ i hạn Tác giả: Trần Nam Dũng- Nguyễn Văn Mậu, 2007

http://slidepdf.com/reader/full/day-so-gioi-han-tac-gia-tran-nam-dung-nguyen-van-mau 196/217

Page 197: Dãy số - Giới hạn Tác giả: Trần Nam Dũng- Nguyễn Văn Mậu, 2007

8/13/2019 Dãy số - Giớ i hạn Tác giả: Trần Nam Dũng- Nguyễn Văn Mậu, 2007

http://slidepdf.com/reader/full/day-so-gioi-han-tac-gia-tran-nam-dung-nguyen-van-mau 197/217

8.4. Phương trình trong hàm s vi cp bin t do   197

Gii.   Nhn xét rng  f   là ánh x 1-1

f (µ) = f (ν )   ⇒   f (f (µ)) =  f (f (ν ))   ⇒   m =  n.

Vy nên f (ν ) =  f (ν  + 2) + 2 ∀ ν  ∈ Q.

Suy ra f (ν  + 2) = f (ν ) − 2, f (0) = 1, f (1) = f (f (0)) = 0.

Vy nên f (2) = f (0) − 2 = −1,

f (3) = f (1) − 2 = −2,

f (ν ) =

−(ν 

 −1).

Tương t f (−1) =  f (1) + 2 = 2,

f (−2) =  f (0) + 2 = 3,

f (−3) =  f (1) + 2 = 5,

f (ν ) = −(ν  − 1).

Bài toán 8.46.  Cho góc  α  vi  0  < α < π. Xác đnh cp s  a, b sao cho dãy hàm {P n(x)} đưc tính theo công thc 

P n(x) =  xν sin α−

x sin(να) + sin(ν −

1)α

luôn luôn chia ht cho  f (x) =  x2 + ax + b.

Gii.   Vi  ν  = 3 thì 

P 3(x) =  x3 sin α − x sin(3α) + sin 2α = sin α(x + 2 cos α)(x2 − 2x cos α + 1).

T đó suy ra vi  f (x) =  x2 + 2x cos α + 1  thì  P 3(x)...f (x). Vi  ν  ≥ 3 thì 

P ν +1(x) =  xP n(x) + (x2 − 2x cos α + 1) sin να.

Suy ra  f (x) =  x2 + 2x cos α + 1.

Page 198: Dãy số - Giới hạn Tác giả: Trần Nam Dũng- Nguyễn Văn Mậu, 2007

8/13/2019 Dãy số - Giớ i hạn Tác giả: Trần Nam Dũng- Nguyễn Văn Mậu, 2007

http://slidepdf.com/reader/full/day-so-gioi-han-tac-gia-tran-nam-dung-nguyen-van-mau 198/217

8.5. S dng gii hn đ gii phương trình hàm   198

Bài tp

Bài 1. Xác đnh hàm s  f   :  Q→R nu bit:  f (1) = a,  f (µ + n) =  f (ν ) + f (µ);

Bài 2. Xác đnh hàm s  f   :  Q → R nu bit:  f (1) = a,  f (m − ν ) =  f (ν ) + f (µ)(µ,ν,m − ν  ∈ Q).Bài 3. Xác đnh hàm s  f   :  Q → R nu bit  f 

µν 

 =  f (ν ) + f (µ) (µ,ν,  mν  ∈ Q).

Bài 4. Xác đnh hàm s   f   :   Q →  R   tho mãn điu kin  f 

µν 

 =  f (µ) − f (ν )

(µ, ν, mν  ∈ Q).Bài 5. Xác đnh hàm s  f   :   Q →  R   tho mãn điu kin  f (µ + n) =  f (µ)f (ν )(m, ν  ∈ Q).Bài 6. Xác đnh hàm s  f   :  Q → R nu bit  f (µ+n)+f (m−ν ) =   1

2(f (2m)+f (2n),  (µ,ν,m − ν  ∈ Q).Bài 7. Xác đnh hàm s   f   :   Q

 →  R+ tho mãn điu kin   f (µ  +  ν ) =   f (µ)

f (ν )(m, ν  ∈ Q).Bài 8. Xác đnh hàm s  f   :  Q → R+ tho mãn điu kin  f 

µ+ν 2

 = 

f (µ)f (ν )

(µ, ν, µ+ν 2   ∈ Q).

Bài 9   . Xác đnh hàm s   f   :   Q →  R+ tho mãn điu kin   xµ+ν 2

=   2f (µ)f (ν )f (µ)+f (ν )

(µ, ν, µ+ν 2   ∈ Q).

Bài 10   . Xác đnh hàm s   f    :   Q  →   R+ tho mãn điu kin   f 

µ+ν 2

  = 

f (µ)2+f (ν )2

2   (µ,ν, µ+ν 2   ∈ Q).

Bài 11. Xác đnh các hàm  f   :  Q → Q tho mãn các điu kin  f (1) = 2 và 

f (xy ) =  f (x)f (y ) − f (x + y ) + 1, x, y ∈ Q.

8.5 S dng gii hn đ gii phương trình hàm

Mt trong nhng tính cht cn thit đ s dng gii hn là tính liên tc ca hàm s. Khi s dng gii hn đ gii phương trình hàm ngưi ta thưng làm như sau.

1. Xây dng mt đng thc đúng vi mi giá tr ca  n  sau đó ly gii hn hai v nh s dng tính cht liên tc ca hàm s.

2. Tính liên tc không có tác dng đi vi phương trình hàm trong tp hu t 

Q. Tuy nhiên nu bit chc chn là hàm liên tuc, ta có th thit lp công thc cho hàm trong  Q và suy ra công thc phi tìm tương t trong tp  R.

Bài toán 8.47.  Tìm tt c các hàm  f   : R → R liên tc, tho mãn điu kin

f (x + y ) =  f (x) + f (y ),   ∀ x, y ∈ R.

Page 199: Dãy số - Giới hạn Tác giả: Trần Nam Dũng- Nguyễn Văn Mậu, 2007

8/13/2019 Dãy số - Giớ i hạn Tác giả: Trần Nam Dũng- Nguyễn Văn Mậu, 2007

http://slidepdf.com/reader/full/day-so-gioi-han-tac-gia-tran-nam-dung-nguyen-van-mau 199/217

8.5. S dng gii hn đ gii phương trình hàm   199

Gii.   Cho  x =  y  = 0, suy ra  f (0) = 2f (0, suy ra  f (0) = 0. Cho  x =  y  = 1, thì đưc  f (2) = 2f (1). Cho  x = 2,  y  = 1 thì đưc  f (3) = f (2) + f (1) = 3f (1). Quy 

np ta đưc  f (n) =  nf (1), vi mi  n ∈ N∗.Ký hiu   f (1) =  a, suy ra   f (n) =  na, vi mi   n ∈  N∗. Cho  x  =  n,   y  = −nta đưc   0 =   f (0) =   f (n) +  f (−n). Suy ra   f (n) = −f (n),   f (−n) =  a(−n), và f (n) = an  vi mi  n ∈ Z.

Đt  x =  y , ta có  f (2x) = 2f (x),  f (3x) f (2x + x) = 2f (x) + f (x),  f (3x) =3f (x). Suy ra 

f (mx) =  mf (x),   ∀ m ∈ N, x ∈ R.

T an =  f (n) =  f 

m −   n

m

 =  mf 

 n

m

suy ra  f (n/m) =  an/m. Suy ra  f (x) =  ax, vi mi  x

∈Q. T đó suy ra vi mi 

x ∈ R luôn tn ti  {xn}∞1  ,  xn ∈ Q sao cho   limn→∞xn =  x. Ta có 

f (xn) =  axn.

Ly gii hn ta có limn→∞ f (xn) = lim

n→∞(axn).

T đó  f (x) =  ax, vi mi  x ∈ R.

Bài toán 8.48.  Tìm tt c các hàm liên tc  f   : R → R tho mãn điu kin

f (x + y ) + f (x − y ) = 2f (x) + 2f (y ).

Gii.   Cho  x  =  y  = 0, suy ra  f (0) = 0. Cho  x  = 0 suy ra  f (y ) + f (−y ) = 2f (y ).Do đó  f (−y ) =  f (y ), tc là  f (x) là hàm s chn trên  R.

Ký hiu   f (1) =   a. Đt   x   =   y   = 1, suy ra   f (2) + f (0) = 4f (1) = 4a. T đó   f (2) = 4a. Đt   x   = 2, y   = 1  suy ra   f (3) + f (1) = 2f (2) + 2f (1),   f (3) =2f (2)+ f (1) = 9a.

Ta chng minh quy np  f (n) =  an2. Ta gi s  f (n) = an2, phi chng minh f (n +1) = a(n+ 1)2. Cho x  =  n, y  = 1, ta có  f (n +1)+f (n−1) = 2f (n) + 2f (1).Suy ra  f (n + 1) = 2an2 − a(n − 1)2 + 2a =  a(2n2 − (n − 1)2 + 2). Tip tc khai trin cho ta 

f (n + 1) = a(n + 1)2.

Do  f  là hàm s chn nên  f (−n) = f (n) = an2 = a(−n)2. Ta có  f (n) = an2 vi mi  n ∈ Z.Bây gi ta chng minh công thc  f (nx) = n2f (x), vi mi  n ∈ N. Vi  x =  y 

ta có  f (2x) + f (0) = 2f (x) + 2f (x) = 22f (x). Suy ra  f (2x) = 22f (x).

Page 200: Dãy số - Giới hạn Tác giả: Trần Nam Dũng- Nguyễn Văn Mậu, 2007

8/13/2019 Dãy số - Giớ i hạn Tác giả: Trần Nam Dũng- Nguyễn Văn Mậu, 2007

http://slidepdf.com/reader/full/day-so-gioi-han-tac-gia-tran-nam-dung-nguyen-van-mau 200/217

8.5. S dng gii hn đ gii phương trình hàm   200

Gi s  f (nx) =  n2f (x), ta phi chng minh  f ((n + 1)x) = (n + 1)2f (x). Tht vy  f ((n + 1)x) + f ((n

−1)x) = 2f (nx) + 2f (x). Suy ra 

f ((n + 1)x) = −(n − 1)2f (x) + 2n2f (x) + 2f (x)

T đây tip tc khai trin cho ta   f ((n + 1)x) = (n + 1)2f (x).  Khi đó   an2 =f (n) = f (m. nm) = m2f (n/m). Suy ra  f (n/m) = a(n/m)2, vi mi  m, n ∈ N. Suy ra ta có  f (x) =  ax2, vi mi  x ∈ Q.

Vy vi mi  x ∈ R  luôn tn ti  {xn}∞n ,  xn ∈  Q  sao cho   limn→∞ xn  = x. T đó 

f (xn) =  ax2n, hay là 

limn→∞ f (xn) = lim

n→∞ ax2n.

T đó  f (x) =  ax2, vi mi  x ∈ R.

Gii (2).  Ta có  f (0) = 0, cho  x =  y  ta đưc  f (2x) = 4f (x). Suy ra 

f (2x)

(2x)2  =

 f (x)

x2  ,   ∀ x ∈ R− {0}.

Suy ra 

g(2x) =  g(x), g(x) = f (x)

x2  .   (8.1)

Suy ra g(x) =  g

x

2

= g

 x

22

= · · · = g

 x

2n

,

Do f  liên tc nên  g  liên tc trên R

−{0

}. Suy ra  g(x) = lim

n→∞

g(x/2n) = g(0) = a.

Suy ra g(x) =  a,   ∀ x ∈ R− {0}.

Vy   f (x) =  ax2, vi mi   x ∈  R − {0}. Do   f (0) = 0  nên  f (x) =  ax2, vi mi x ∈ R.

Bài toán 8.49.  Gi s  f   : R → R và tha mãn các điu kin

1.   f (1) = 1,

2.   f (x + y ) = f (x) + f (y ), x , y   ∈ R,

3.   f (x).f (1/x) = 1, x = 0.

Gii.   Ta chng t rng hàm cn tìm là  f (x) =  x, vi mi  x ∈ R. Theo bài toán (8.47), t điu kin th nht và th hai ta suy ra  f (x) =  x, vi mi  x ∈ Q.

Bây gi ta chng t  f (x là hàm liên tc. Cn chng minh   limh→∞

f (x+h) =  f (x)

hay là   limh→0

(f (x) + f (h)) =  f (x). Tc là cn chng t   limh→0

f (h) = 0.

Page 201: Dãy số - Giới hạn Tác giả: Trần Nam Dũng- Nguyễn Văn Mậu, 2007

8/13/2019 Dãy số - Giớ i hạn Tác giả: Trần Nam Dũng- Nguyễn Văn Mậu, 2007

http://slidepdf.com/reader/full/day-so-gioi-han-tac-gia-tran-nam-dung-nguyen-van-mau 201/217

8.5. S dng gii hn đ gii phương trình hàm   201

Nu  a, b > 0  thì  |a + b| = |a| + |b|, t điu kin th ba suy ra rng vi  x = 0 ta có  f (x) và  f (1/x) cùng du. Chú ý rng theo bt đng thc gia trung bình cng 

và trung bình nhân, ta có f 

x + 1

x

=

f (x) + f  1

x

= |f (x)| +

f  1

x

2

 f (x).f 

1

x

 = 2.

Suy ra f (y ) 2  vi mi  y   2.   (8.2)

Vy nu  |y |   12   hay  |1/y | 2 thì theo (8.2) ta có  f (1/y ) 2.

T điu kin th ba ta có 

1 = |f (y ).f (1/y )| |f (y )|.2.

Suy ra 

|f (y )|  1

2.   (8.3)

Nu  |y |   14 , suy ra  |2y |   1

2 , suy ra  |f (2y )|   12. Do đó 

|f (y )|  1

4.   (8.4)

Bng quy np, ta chng minh đưc 

|f (y )|   12n

,   vi mi  |y |   12n−1 .

Tht vy, gi s  |f (y )|   12n−1   vi  |y |  1/2n−1. Khi đó nu  |y |   1/2n thì theo

gi thit quy np ta có 1

2n−1   |2y |.

Suy ra  1/2n−1 |f (2y )|, hay là 

1

2n−1

2f (y ).

Tc là  |f (y )| 1/2n vi  |y |  1/2n. Vy   limh→0

f (h) = 0, suy ra  f (x) là hàm liên 

tc trên  R, mà  f (x) =  x, vi mi  x ∈ Q. Vy 

f (x) = x,   vi mi  x ∈ R.

Page 202: Dãy số - Giới hạn Tác giả: Trần Nam Dũng- Nguyễn Văn Mậu, 2007

8/13/2019 Dãy số - Giớ i hạn Tác giả: Trần Nam Dũng- Nguyễn Văn Mậu, 2007

http://slidepdf.com/reader/full/day-so-gioi-han-tac-gia-tran-nam-dung-nguyen-van-mau 202/217

8.5. S dng gii hn đ gii phương trình hàm   202

Bài toán 8.50.  Cho f   : R → R liên tc tha mãn điu kin  f (1) = 1, f (x + y ) +f (xy ) =  f (x) + f (y ) + f (x).f (y ).

(a) Chng minh rng  f (x) cng tính:  f (x + y ) =  f (x) + f (y ).

(b) Hãy tìm tt c các hàm  f  tha mãn điu kin

f (x + y ) + f (xy ) =  f (x) + f (y ) + f (x).f (y ).

Gii. Câu (a). Cho  y  = 1, thì ta có 

f (x + 1) = f (x) + 1.   (8.5)

Thay  x bi  x + 1, ta có 

f (x + 1 + y ) + f ((x + 1)y ) =  f (x + 1) + f (y ) + f (x + 1)f (y ).

Vì  f (x + 1) = f (x) + 1  nên ta có 

f (x + y ) + 1 + f (xy  + y ) =  f (x) + 1 + f (y ) + f (x + 1).f (y )

= f (x) + f (y ) + f (x)f (y ) + 1 + f (y )

= f (x + y ) + f (xy ) + f (y ) + 1.

T đó suy ra f (xy  + y ) = f (xy ) + f (y )   (8.6)

Vy vi  u, v bt kỳ, tn ti  x sao cho  u =  vx, suy ra  x =

  u

v = 0. Suy ra f (u + v) =  f (vx + v) =  f (vx) + f (v) =  f (u) + f (v).   (8.7)

Nu  v = 0 thì  f (u) = f (u) + f (0), suy ra  f (0) = 0. Thay  x =  y  = 0 vào

f (x + y ) + f (xy ) =  f (x) + f (y ) + f (x).f (y ),

ta đưc f (0) + f (0) = f (0) + f (0) + f (0).f (0).

Suy ra f (0) = 0   (8.8)

T (8.7) và (8.8) suy ra  f  cng tính.

Câu (b). T câu (a) suy ra   f (x +  y ) =  f (x) +  f (y ), và  f (xy ) =  f (x).f (y ).Tc là hàm   f (x) va cng tính va nhân tính. Suy ra  f (x) =  x,  x =   m

n , thành 

Page 203: Dãy số - Giới hạn Tác giả: Trần Nam Dũng- Nguyễn Văn Mậu, 2007

8/13/2019 Dãy số - Giớ i hạn Tác giả: Trần Nam Dũng- Nguyễn Văn Mậu, 2007

http://slidepdf.com/reader/full/day-so-gioi-han-tac-gia-tran-nam-dung-nguyen-van-mau 203/217

8.5. S dng gii hn đ gii phương trình hàm   203

th  x ∈  Q. Suy ra vi mi  x ∈ R, luôn tn ti  xn ∈  Q  :  xn →  x  khi  n → ∞. Dof x) liên tc trên  R nên  limn→∞ f (xn) = f (x), hay là  x =  f (x).

Gi s hàm s không gim. Cho   x   =   y , ta có   f (x2) =   f 2(x)     0, suy ra f (u) 0 vi mi  u 0. Nu  x > y  thì  f (x) = f (y  + x − y ) =  f (y ) + f (x − y ) f (y ), vì  f (x − y ) 0 vi  x − y   0.

Vi mi  x ∈ R−Q, tn ti  rn  :  rn ∈ Q sao cho

rn > x : limn→∞ rn =  x.   (8.9)

Tn ti  sn  :  sn ∈ Q sao cho

sn < x : limn→∞ sn = x.   (8.10)

Suy ra  sn  < x < rn. Do  f   là hàm s không gim nên  f (sn)

f (x)

f (rn), hay là sn   f (x) rn.   (8.11)

Ly gii hn cho ta limn→∞ sn   f (x)   lim

n→∞ rn.

T (8.9), (8.10) và (8.11), theo nguyên lý kp ta có   x     f (x)     x. Suy ra f (x) = x. Cho  x =  y  = 1, t đó ta có  f (2) = 2. Thay  x =  x − 1 và  y  = y − 1 ta có 

f (x + y ) + f ((x − y )(y  + 1)) = f (x − 1) + f (y  + 1) + f (x − 1).f (y  + 1).   (8.12)

Cho  y  = 1, ta có 

f (x + 1) + f (x) =  f (x) + f (1) + f (x).f (1).

Đng thc này tương đương vi 

f (x + 1) = f (x) + 1.   (8.13)

Cho  x = 2, t (8.12) cho ta 

f (2 + y ) + f (y  + 1) = f (1) + f (y  + 1) + f (1).f (y  + 1).

Vì (8.13) nên t đây ta có f (2 + y ) =  f (1)(1 + f (y  + 1)) = 1 + f (y  + 1),

hay f (2 + y ) =  f (2) + f (y ).   (8.14)

Page 204: Dãy số - Giới hạn Tác giả: Trần Nam Dũng- Nguyễn Văn Mậu, 2007

8/13/2019 Dãy số - Giớ i hạn Tác giả: Trần Nam Dũng- Nguyễn Văn Mậu, 2007

http://slidepdf.com/reader/full/day-so-gioi-han-tac-gia-tran-nam-dung-nguyen-van-mau 204/217

8.5. S dng gii hn đ gii phương trình hàm   204

Cho  x = 2, t đng thc 

f (x + y ) + f (xy ) =  f (x) + f (y ) + f (x).f (y ),

ta đưc f (2 + y ) + f (2y ) =  f (2) + f (y ) + f (2).f (y ).

Phương trình này tương đương vi 

f (2y ) = 2f (y ),   (8.15)

hay là f (2y )

2y   =

 f (y )

y   .   (8.16)

Đt  g(x) = f (x)/x,  x = 0, ta có (8.16) tương đương vi 

g(x) =  g(2x).

Suy ra g(x) =  g

x

2

= g

x

4

= · · · = g

 x

2n

.   (8.17)

Suy ra tn ti  (xn)∞0   vi mi  x ∈ R sao cho  xn → x, g(x) là hàm s liên tc trên R− {0}. Thành ra,   lim

n→∞ g(xn) = g(x). Tc là 

limn→∞ g

 1

2n

= g(x).

Vì (8.17) nên t đây ta có  g(1) = g(x), hay   f (1)1   = g(x),  f (1) = g(x) =  f (x)/x.Cui cùng ta đưc  f (x) =  x.

Suy ra  f  cng tính.Chú ý rng trong ý (b) nu cn xét riêng  x ∈ Q thì ta có 

f (n) = f (1 + 1 + · · · + 1   n  s  1

) =  nf (1) = n.

Ta có f (n.

m

n ) = f (n).f (m/n)

tương đương vi mi 

f (m) =  n.f (n/m)

f (m) =  nf (m/n)Suy ra 

f m

n

=

 f (m)

n  =

 mf (1)

n  =

 m

n .

Vy  f (m/n) =  m/n, hay  f (x) =  x, vi mi  x ∈ Q.

Page 205: Dãy số - Giới hạn Tác giả: Trần Nam Dũng- Nguyễn Văn Mậu, 2007

8/13/2019 Dãy số - Giớ i hạn Tác giả: Trần Nam Dũng- Nguyễn Văn Mậu, 2007

http://slidepdf.com/reader/full/day-so-gioi-han-tac-gia-tran-nam-dung-nguyen-van-mau 205/217

Page 206: Dãy số - Giới hạn Tác giả: Trần Nam Dũng- Nguyễn Văn Mậu, 2007

8/13/2019 Dãy số - Giớ i hạn Tác giả: Trần Nam Dũng- Nguyễn Văn Mậu, 2007

http://slidepdf.com/reader/full/day-so-gioi-han-tac-gia-tran-nam-dung-nguyen-van-mau 206/217

8.5. S dng gii hn đ gii phương trình hàm   206

Cách 2. Khi  0 x < 2, ta có  2 − x >  0,

f ((2 − x).f (x)).f (x) = f (2 − x + x) =  f (2) = 0.

Do  f (x) = 0 nên  f ((2 − x)f (x)) = 0, suy ra  (2 − x)f (x) 2. Thành th,

f (x)   2

2 − x  (8.22)

Mt khác,  f ((y − x)f (x)) = 0 nên  (y − x)f (x) <  2. Ta c đnh  x và cho  y  → 2,do tính liên tc ca  f  nên  (y − x)f (x) → (2 − x)f (x) 2. Suy ra 

f (x)   2

2 − x.   (8.23)

T (8.22) và (8.23) suy ra  f (x) =   22−x . Tóm li 

f (x) =

  22−x   khi  0 x <  2

0   khi  x 2

Bài toán 8.52.  Tìm hàm  f (x) xác đnh và liên tc vi mi  x > 0  và tho mãnđiu kin

1.   f (uv) =  f (v) + f (u),   ∀ u, v > 0

2.   limx→1

f (x) = 0,

3.   f (x) = 0, ∀ x > 0.

Gii.   Gi s  f (x) ≡ 0, vi mi  x > 0.

f (x) =  f (1.x) =  f (1) + f (x),

suy ra  f (1) = 0. Ly  x > 0,  xn → x0  khi  n → ∞. Suy ra  xn/x0 → 1 khi  n → ∞.Suy ra 

lim f 

xn

x0

 = 0.

Vy  f (xn) =  f (x0,  xnx0 ) =  f (x0) + f (xnx0

) → f (x0) + 0. Tc là 

limxn→x0

f (xn) =  f (x0).

f (uv) =  f (u) + f (v),   ∀ u, v > 0.   (8.24)

Page 207: Dãy số - Giới hạn Tác giả: Trần Nam Dũng- Nguyễn Văn Mậu, 2007

8/13/2019 Dãy số - Giớ i hạn Tác giả: Trần Nam Dũng- Nguyễn Văn Mậu, 2007

http://slidepdf.com/reader/full/day-so-gioi-han-tac-gia-tran-nam-dung-nguyen-van-mau 207/217

8.5. S dng gii hn đ gii phương trình hàm   207

Suy ra   f (xn) =   nf (x), vi   x >   0,   n ∈  N∗. Do   f (1) = 0  nên   f (xn) =   nf (x)đúng khi  n = 0. Hơn na, vi mi  n

 ∈ N∗   thì  0 = f (1) = f (xn, x−n) = f (xn) +

f (x−1) = nf (x) + f (x−n). Suy ra  f (x−n) = −nf (x), và 

xmn

= f 

x

1n

m =  mf (x1/n) =  m

1

nf (x) =

 m

n f (x).

Suy ra  f (xm/n) =   mn f (x), m , n ∈ Z. Do đó 

f (xr) = rf (x),   ∀ x > 0   r ∈ Q.

Đc bit  f (2r) =  r.f (2) =  r.A, r ∈  Q, A =  f (2). Nu  x >  0 thì  x  = 2log2 x suy ra vi mi  x > 0  thì tn ti mt dãy s hu t  {rn}∞0   sao cho

limr→∞

rn = log2

x.

Suy ra limr→∞ 2rn = 2log2 x = x

dn đn   limn→∞ f (2rn) =  f (x), mà  f (2rn) = A.rn → A. log2 x khi  n → ∞.

Suy ra limn→∞ f (2rn) =  A. log2 x =  f (x).

Do  f (x) ≡ 0  vi  x > 0  nên vi mi  x > 0  ta có  A = 0, đt  A = loga 2,  a > 0  và a = 1,  f (x) − A log2 x = loga 2. log2 x = loga x. Vy 

f (x) = loga x,   ∀ x > 0,   0 < a = 1.

Bài toán 8.53.  Tìm f (x) xác đnh và liên tc vi mi  x > 0  và tho mãn điukin

1.   f (uv) =  f (u).f (v),   ∀ u, v > 0

2.   limx→1

f (x) = 1.

Gii.  Ta có vi mi  x >  0

f (x) =  f (√ 

x.√ 

x) =  f 2(√ 

x) 0.

Suy ra  f (x) x, ∀ x > 0.Nu tn ti  x0  > 0 đ  f (x0) = 0 thì vi mi  x > 0  ta có  f (x) = f (x0.  xx0 ) =

f (x0).f  xx0

 = 0. Theo gi thit ta cũng có   lim

x→1f (x) = 1. Vy  f (x) >  0, vi mi 

x > 0.

Page 208: Dãy số - Giới hạn Tác giả: Trần Nam Dũng- Nguyễn Văn Mậu, 2007

8/13/2019 Dãy số - Giớ i hạn Tác giả: Trần Nam Dũng- Nguyễn Văn Mậu, 2007

http://slidepdf.com/reader/full/day-so-gioi-han-tac-gia-tran-nam-dung-nguyen-van-mau 208/217

8.5. S dng gii hn đ gii phương trình hàm   208

Xét hàm s  g(x) = lim f (x), vi mi  x > 0  tho mãn điu kin 

g(u, v) = ln f (uv) = ln(f (u).f (v)) == ln f (u) + ln f (v)

= g(u) + g(v)

và   limx→1

g(x) = 0.

Vy  g(x) tha mãn điu kin bài toán 8.52 trên. Suy ra 

g(x) = loga x,   0 < a = 1, x >  0.

Suy ra  ln f (x) = logα x =  logαe. ln x. Do đó 

f (x) =  xα

vi  α = loga e,  (0 < a = 1)Bài toán 8.54.  Cho f   : (−1, 1) → R liên tc và

f (x) =  f 

  2x

1 + x2

,   ∀ x ∈ [−1, 1].

Chng minh rng  f (x) là hàm hng.

Gii.  Ta c đnh  x, xét dãy s  (xn)∞1   xác đnh bi 

x0  = x > 0, xn+1  =  2x

1 + x2.   (8.25)

Dãy  (xn) là dãy s tăng. Suy ra  xn+1   xn, hay  2xn/(1 + x2

n) xn. T đây ta có −1 xn   1. Theo bt đng thc gia trung bình cng và trung bình nhân, ta có 

xn+1  =  2xn

1 + x2n

1.

Do dãy s  (xn) tăng và b chn dưi bi  1 nên tn ti gii hn  limn→∞ xn = l > 0.Ta có (8.25) tương đương vi 

l =  2l

1 + l2,

t đó ta có  l = 1.Dãy s 

f (xn+1) =  f    2xn

1 + x2n =  f (xn).

Ly   f (x) =  f (x0) = · · ·  =  f (xn). Do   f (x)   liên tc trên   [−1, 1] nên    limn→∞ f (xn)

= f (1), hay  f (x) =  f (1) = c, vi  c là hng s.(trưng hp  x0 =  x < 0  xét tương t)

Page 209: Dãy số - Giới hạn Tác giả: Trần Nam Dũng- Nguyễn Văn Mậu, 2007

8/13/2019 Dãy số - Giớ i hạn Tác giả: Trần Nam Dũng- Nguyễn Văn Mậu, 2007

http://slidepdf.com/reader/full/day-so-gioi-han-tac-gia-tran-nam-dung-nguyen-van-mau 209/217

8.5. S dng gii hn đ gii phương trình hàm   209

Bài toán 8.55.  Tìm tt c các hàm  f (x) xác đnh và liên tc trên  R − {0} vàtha mãn điu kin sau

(f (x2) − x2)(f (x) − x) =  1

x3,   ∀ x = 0.   (8.26)

Gii.  Phương trình (8.26) tương đương vi 

(x2f (x2) − x4)(xf (x) − x2) = 1.

Đt  xf (x) − x2 = g(x), ta thu đưc 

g(x2)g(x) = 1.   (8.27)

Suy ra  g(x)

= 0, vi mi  x

∈ R. Ta có  g(0) =

 ±1 và  g(1) =

 ±1. Thay  x bi 

 −x

vào trong (8.27) ta thu đưc 

g(x2)g(−x) = 1 = g(x2)g(x).

Suy ra  g(−x) =  g(x) trên tp đi xng qua gc to đ  R. Suy ra  g(x) là hàm s chn trên  R, nên ta ch cn xét  g(x) trên tp  x 0 là đ.

Xét  0 x 1, ta có 

g(x) =  1

g(x2) =

  11

g(x4)

= g(x4).

Suy ra  g(x) =  g(x4).

Li có g(x4) =

  1

g((x4)2) =

  11

g((x4)2)

= g((x4)4) =  g(x42).

Suy ra  g(x4) =  g(x42). Vy ta thu đưc 

g(x) =  g(x4) =  g((x4)4) = · · · = g((· · ·(x4)4)4 · · · )4) =  g(x4n), n → +∞.

Qua gii hn ta thu đưc 

g(x) = limn→+∞ g(x(1/4)n),

do   lim0   x   1 x4n

= 0. Suy ra 

g(x) = limn→+∞ g(x4n) =  g(0),

mà  g(0) = ±1. Suy ra  g(x) = ±1 = c.

Page 210: Dãy số - Giới hạn Tác giả: Trần Nam Dũng- Nguyễn Văn Mậu, 2007

8/13/2019 Dãy số - Giớ i hạn Tác giả: Trần Nam Dũng- Nguyễn Văn Mậu, 2007

http://slidepdf.com/reader/full/day-so-gioi-han-tac-gia-tran-nam-dung-nguyen-van-mau 210/217

8.5. S dng gii hn đ gii phương trình hàm   210

Xét  x > 1, ta có 

g(x) =  1

g(x12 )

=  1

1

g(x14 )

= g(x1

4 ) = · · · = g(x(1

4 )n

).

Qua gii hn, ta thu đưc  g(x) = limn→+∞ x(1

4)n = g(1) vì vi  x >  1  thì  limn→+∞  =

1, suy ra  g(x) =  g(1) = ±1 = c.Vy  c =  xf (x) − x2, hay  f (x) =  c/x + x,  c là mt hng s.

Bài toán 8.56.  Cho f   : (−1, 1) → R liên tc tho mãn điu kin

f (x) =  f 

  2x

1 + x2

,   ∀ x ∈ (−1, 1).   (8.28)

Chng minh rng  f (x) là hàm hng.Gii.   Xét  0 < x < 1. Ta c đnh  x, xét dãy s  (xn)+∞1   như sau 

x0  =  x, xn+1  = 1 − 

1 − x2n

xn.   (8.29)

Dãy này đưc suy ra t vic xét dãy s 

xn =  2xn + 1

1 + x2n+1

.

Ta chng minh rng  (xn)∞1  xác đnh vi mi  n và 

limn→∞ xn  = 0.   (8.30)

T (8.28) suy ra   f (x) =   f (x0) =   f (x1) = · · ·  =  f (xn). Do   f (x)   liên tc trên (−1, 1) nên  f (x) = lim

n→∞ f (xn) =  f (0).

Ta chng minh dãy s  (xn)∞1   b chn.D thy   (xn)∞1   luôn dương vi mi  n ∈ N∗. Ta chng minh  xn   1, vi mi 

n ∈ N∗.Nu  n = 0 thì  x0 =  x < 1, đúng theo gi thit.Gi s  xk  < 1, ta có 

xk+1  =  1 − 1 − x2n

xk 1.

Bt đng thc này tương đương vi  1−xk  

 1 − x2

k. T đây ta có  xk(xk−1) 0,

điu này luôn đúng vi  xk  < 1.

Page 211: Dãy số - Giới hạn Tác giả: Trần Nam Dũng- Nguyễn Văn Mậu, 2007

8/13/2019 Dãy số - Giớ i hạn Tác giả: Trần Nam Dũng- Nguyễn Văn Mậu, 2007

http://slidepdf.com/reader/full/day-so-gioi-han-tac-gia-tran-nam-dung-nguyen-van-mau 211/217

Page 212: Dãy số - Giới hạn Tác giả: Trần Nam Dũng- Nguyễn Văn Mậu, 2007

8/13/2019 Dãy số - Giớ i hạn Tác giả: Trần Nam Dũng- Nguyễn Văn Mậu, 2007

http://slidepdf.com/reader/full/day-so-gioi-han-tac-gia-tran-nam-dung-nguyen-van-mau 212/217

8.5. S dng gii hn đ gii phương trình hàm   212

Ta chng minh   (xn)∞1   : limn→∞ xn   =  α. Suy ra   lim f (xn) =  f (α)  vì  f (x)   liên 

tc trên  R, hay là  f (x) =  f (α), vi mi   x ∈

  [0, α], hay   f (x) =   c, vi   c  là mt hng s.

a) Chng minh   limn→∞ = α  vi  (xn)∞1   xác đnh bi (8.32).

Ta có  (xn)∞1   là dãy s tăng. Xét  g (x) =  x2 + c,  g (x) = 2x > 0  vi  x ∈ [0, α].Suy ra  g(x) đng bin trên  [0, α]. Do đó,  g(x1) > g(x0), tương t vi  x2  > x1   ta có  g(x2) > g(x1). Vy  (xn)∞1   là dãy s tăng.

b) Chng minh   (xn)∞1   b chn bi  α (bng phương pháp quy np). Vi  x0  =x < α. Gi s (8.32) đúng vi  n  =  k:  xk  < α. Suy ra  xk+1  =  x2

k + c < α2 + c =  αvì  α là nghim ca (8.31). Suy ra (8.32) đúng vi  n =  k + 1.

T a) và b) suy ra   limn→∞xn  = α.

Trưng hp 2. Xét  x ∈ [α, β ], xét dãy s 

x0 =  x, xn+1  =  x2n + c.   (8.33)

Chng minh tương t như trưng hp 1,   (xn)∞1   là dãy s gim.,  xn   α, suy ra limn→∞ xn =  α.

Suy ra f (x) =  f (α), ∀ x ∈ [α, β ].

Trưng hp 3.  x ∈ [β, +∞), xét dãy s xác đnh bi 

x =  x0, xn+1  =√ 

xn − c  vi  xn = x2n+1 + c.   (8.34)

Chng minh rng   limn→∞

xn = β .

Xét  g(x) = √ x − c. Tính đo hàm cho ta 

g(x) =  1

2√ 

x − c > 0,   vi  x ∈ [β, +∞).

Suy ra  g(x) đng bin trên  [β, +∞).Ta có  x1  =

 √ x0 − c < x0, hay  x2

0 − x0 +  c >  0  luôn đúng do  x0 ∈  [β, +∞).Nu  x1 < x0  thì dãy s  (xn)∞1   gim.

Ta chng minh   (xn)∞1   b chn dưi bi   β  bng phương pháp quy np. Nu x0  = x β , gi s  xk  > β ,  xk+1  =

 √ xk + c

√ β − c =  β . Điu này luôn đúng 

vì  β  là mt nghim ca  β 2 − β + c = 0. Suy ra tn ti   limn→∞ xn =  l  (l β ). T đó 

limn→∞ xn  =  β.   (8.35)

T dãy s (8.32) suy ra  f (xn) = f (x2n+1 + c) =  f (xn+1). Ly 

f (x0) = f (x1) = · · · = f (xn).

Page 213: Dãy số - Giới hạn Tác giả: Trần Nam Dũng- Nguyễn Văn Mậu, 2007

8/13/2019 Dãy số - Giớ i hạn Tác giả: Trần Nam Dũng- Nguyễn Văn Mậu, 2007

http://slidepdf.com/reader/full/day-so-gioi-han-tac-gia-tran-nam-dung-nguyen-van-mau 213/217

Page 214: Dãy số - Giới hạn Tác giả: Trần Nam Dũng- Nguyễn Văn Mậu, 2007

8/13/2019 Dãy số - Giớ i hạn Tác giả: Trần Nam Dũng- Nguyễn Văn Mậu, 2007

http://slidepdf.com/reader/full/day-so-gioi-han-tac-gia-tran-nam-dung-nguyen-van-mau 214/217

8.5. S dng gii hn đ gii phương trình hàm   214

T (8.38) suy ra 1

f (xk) 

  1

2kf (x0).

Suy ra 

xn   x0 +n

k=0

1

2kf (x0) = x0 +

  1

f (x0).

nk=0

1

2k  = x0 +

  2

f (x0).

Vì n

k=0

1

2k  =

  1

1 −   12

nên 

xn   a = x0 +  2

f (x0) ,   ∀ n.

Do  f  là hàm s tăng nên  2nf (x0)    f (x0) < f (a), vi mi  a. T đây và (8.38)ta suy ra  f (a) >  2nf (x0)  vi mi  n. Suy ra  f (a) > ∞, mâu thun vi gi thit rng  f (x) tn ti. Điu mâu thun này cho ta điu phi chng minh.

Bài toán 8.59.  Tìm tt c các hàm  f   :  R → R  liên tc ti  x  = 0 và tho mãnđiu kin nf (nx) = f (x) + nx, trong đó  n > 1  là s t nhiên c đnh nào đó.

Gii.   Cho  n  = 0, t đó thay giá tr này vào biu thc đã cho, ta có   nf (0) =f (0) + 0, hay   (n − 1)f (0) = 0. Suy ra  f (0) = 0, vì  n > 1. Cũng t biu thc đã cho, thay  x bi  x/n thì 

nf n xn= f x

n+n x

n,

hay nf (x) = f 

x

n

+x.

Suy ra 

f (x) =  1

n.f x

n

+

x

n.   (8.39)

Trong (8.39) thay  x bi  x/n, ta có 

x

n=

  1

n  x

n2+

  x

n2.

Suy ra 

f (x) =  1

n

1

nf   x

n2

+

 x

n2

+

 x

n =

  1

n2f   x

n2

+

  x

n3 +

 x

n.   (8.40)

Page 215: Dãy số - Giới hạn Tác giả: Trần Nam Dũng- Nguyễn Văn Mậu, 2007

8/13/2019 Dãy số - Giớ i hạn Tác giả: Trần Nam Dũng- Nguyễn Văn Mậu, 2007

http://slidepdf.com/reader/full/day-so-gioi-han-tac-gia-tran-nam-dung-nguyen-van-mau 215/217

8.5. S dng gii hn đ gii phương trình hàm   215

Trong (8.39) li thay  x bi  x/n2 thì ta có 

f   xn2=   1n f   xn3

+  xn3 .   (8.41)

T (8.40) suy ra 

f (x) =  1

n3f   x

n3

+ f racxn5 +

  x

n3 +

 x

n.

T đó, ta có th chng minh quy np theo  k rng 

f (x) =  1

nkf   x

nk

+

  x

n2k−1 +

  x

n2k−3 + · · · +

 x

n.   (8.42)

Ta có  S k  =   xn2k−1

 +   xn2k−3

 + · · · +  xn

là tng cp s nhân hu han. Suy ra 

S k  =  x. 1n1 −   1

n2

=  nx

n2 − 1.

Suy ra limk→∞

S k  =  nx

n2 + 1,

và 

limk→∞1

nk f   x

nk = 0.f (0),

vì  f (x) liên tc ti  x = 0 suy ra  f (x) =  nx/(n2 − 1).Th li, ta đưc kt qu đúng. Vy 

f (x) =  nx

n2 − 1, n > 1, n ∈ N∗, x ∈ R.

Bài tp

Bài 8.1.  Tìm tt c các hàm  f  xác đnh và liên tc trên  R tha mãn điu kin

f (x3) − x2f (x) =   1x3 − x, ∀ x = 0.

Bài 8.2.  Gi s  f   : R → R liên tc và f (x+ y ).f (x−y ) = f 2(x) vi mi x, y ∈ R.Chng minh rng  f  = 0 hoc f  không có không đim.

Page 216: Dãy số - Giới hạn Tác giả: Trần Nam Dũng- Nguyễn Văn Mậu, 2007

8/13/2019 Dãy số - Giớ i hạn Tác giả: Trần Nam Dũng- Nguyễn Văn Mậu, 2007

http://slidepdf.com/reader/full/day-so-gioi-han-tac-gia-tran-nam-dung-nguyen-van-mau 216/217

Page 217: Dãy số - Giới hạn Tác giả: Trần Nam Dũng- Nguyễn Văn Mậu, 2007

8/13/2019 Dãy số - Giớ i hạn Tác giả: Trần Nam Dũng- Nguyễn Văn Mậu, 2007

http://slidepdf.com/reader/full/day-so-gioi-han-tac-gia-tran-nam-dung-nguyen-van-mau 217/217

Tài liu tham kho

[1] N Vă M "Đ th đ i à hâ th h t" NXB Giá d